You are on page 1of 415

I H C QU C GIA H N I TR NG I H C KHOA H C T NHIN

=============================

Nguy n Vn M u (Ch bin), Tr n Nam Dng inh Cng H ng, Nguy n ng Ph t T Duy Ph ng, Nguy n Th y Thanh

BI N PH C NH L V P D NG

H N I 2009

I H C QU C GIA H N I TR NG I H C KHOA H C T NHIN


=============================

Nguy n Vn M u (Ch bin), Tr n Nam Dng inh Cng H ng, Nguy n ng Ph t T Duy Ph ng, Nguy n Th y Thanh

BI N PH C NH L V P D NG

H N I 2009

M cl c
L i ni u 1 S ph c, bi n ph c l ch s 1.1 1.2 v cc d ng bi u di n 8 11

L ch s hnh thnh khi ni m s ph c

. . . . . . . . . . . . . . 11

Cc d ng bi u di n s ph c . . . . . . . . . . . . . . . . . . . . 17 1.2.1 1.2.2 1.2.3 1.2.4 1.2.5 1.2.6 1.2.7 Bi u di n s ph c d i d ng c p . . . . . . . . . . . . . 17 Bi u di n s ph c d i d ng i s . . . . . . . . . . . . 21

Bi u di n hnh h c c a s ph c . . . . . . . . . . . . . . 22 Bi u di n s ph c nh ma tr n . . . . . . . . . . . . . . 24 D ng l ng gic v d ng m c a s ph c . . . . . . . . . 25 Bi u di n cc s ph c trn m t c u Riemann . . . . . . . 27 Kho ng cch trn C . . . . . . . . . . . . . . . . . . . . 30

1.3 2

Bi t p . . . . . . . . . . . . . . . . . . . . . . . . . . . . . . . . 33 36

S ph c v bi n ph c trong l ng gic 2.1 2.2 2.3 2.4

Tnh ton v bi u di n m t s bi u th c . . . . . . . . . . . . . 36 Tnh gi tr c a m t s bi u th c l ng gic . . . . . . . . . . . 43 D ng ph c c a b t ng th c Cauchy . . . . . . . . . . . . . . . 51 T ng v tch sinh b i cc a th c l ng gic . . . . . . . . . . . 54 2.4.1 2.4.2 Ch ng minh cng th c l ng gic . . . . . . . . . . . . . 56 T ng v tch cc phn th c c a bi u th c l ng gic . . 64
4

M CL C

2.5 2.6 2.7 3

B t ng th c l ng gic . . . . . . . . . . . . . . . . . . . . . . 68 c trng hm c a hm s l ng gic . . . . . . . . . . . . . . 76

Bi t p . . . . . . . . . . . . . . . . . . . . . . . . . . . . . . . . 83 ng d ng c a s ph c trong i s 88

M ts 3.1

Phng trnh v h phng trnh i s . . . . . . . . . . . . . . 88 3.1.1 3.1.2 3.1.3 3.1.4 3.1.5 Phng trnh b c hai . . . . . . . . . . . . . . . . . . . . 88 Phng trnh b c ba . . . . . . . . . . . . . . . . . . . . 92 Phng trnh b c b n . . . . . . . . . . . . . . . . . . . 98 Phng trnh b c cao . . . . . . . . . . . . . . . . . . . . 103 Cc bi ton v phng trnh, h phng trnh i s . . 109

3.2

Cc bi ton v a th c . . . . . . . . . . . . . . . . . . . . . . 111 3.2.1 3.2.2 3.2.3 3.2.4 Phng trnh hm trong a th c . . . . . . . . . . . . . 111 Cc bi ton v a th c b t kh quy . . . . . . . . . . . 120 Bi ton v s chia h t c a a th c . . . . . . . . . . . . 135 Quy t c d u Descartes trong ng d ng . . . . . . . . . . 136

3.3

Phng trnh hm v i bi n i phn tuy n tnh . . . . . . . . . 144 3.3.1 3.3.2 3.3.3 M t s tnh ch t c a hm phn tuy n tnh . . . . . . . . 145 ng c u phn tuy n tnh. . . . . . . . . . . . . . . . . . 146 Phng trnh hm sinh b i hm phn tuy n tnh . . . . 160

3.4 4

Bi t p . . . . . . . . . . . . . . . . . . . . . . . . . . . . . . . . 163 166

S ph c trong cc bi ton s h c v t h p 4.1 4.2 4.3 4.4

Gi i phng trnh Diophant . . . . . . . . . . . . . . . . . . . . 166 Rt g n m t s t ng t h p . . . . . . . . . . . . . . . . . . . . 167 Cc bi ton m . . . . . . . . . . . . . . . . . . . . . . . . . . 169 S ph c nguyn v ng d ng trong l thuy t s . . . . . . . . . . 172 4.4.1 Tnh ch t chia h t trong t p cc s ph c nguyn . . . . 174

M CL C

4.4.2 4.4.3 4.5 5

S nguyn t Gauss . . . . . . . . . . . . . . . . . . . . . 177 M t s p d ng s ph c nguyn . . . . . . . . . . . . . . 185

Bi t p . . . . . . . . . . . . . . . . . . . . . . . . . . . . . . . . 189 ng d ng c a s ph c trong hnh h c 192

M ts 5.1

M t m t s k t qu c a hnh h c ph ng b ng ngn ng s ph c193 5.1.1 5.1.2 5.1.3 5.1.4 5.1.5 5.1.6 Gc gi a hai ng th ng . . . . . . . . . . . . . . . . . 194 Tch v h ng c a hai s ph c . . . . . . . . . . . . . . . 194 Tch ngoi c a hai s ph c. Di n tch tam gic . . . . . . 195 ng trn . . . . . . . . . . . . . . . . . . . . . . . . . 196 M t cc php bi n hnh ph ng b ng ngn ng s ph c 196 i u ki n ng quy, th ng hng, vung gc v cng n m trn m t ng trn ( ng vin) . . . . . . . . . . . . . 198

5.2 5.3 5.4

M t s v d p d ng . . . . . . . . . . . . . . . . . . . . . . . . 198 Ch ng minh b t ng th c hnh h c . . . . . . . . . . . . . . . 212 Cc bi ton hnh h c ch ng minh v tnh ton . . . . . . . . . 214 5.4.1 5.4.2 S ph c v a gic u . . . . . . . . . . . . . . . . . . . 221 ng th c l ng gic trong tam gic . . . . . . . . . . . 222

5.5

B ng cc cng th c c b n ng d ng s ph c vo gi i ton hnh h c . . . . . . . . . . . . . . . . . . . . . . . . . . . . . . . 223

5.6 6

Bi t p . . . . . . . . . . . . . . . . . . . . . . . . . . . . . . . . 227 231

Kh o st dy s v phng trnh sai phn 6.1 6.2 6.3 6.4 6.5

M t s khi ni m c b n v tnh ch t c a sai phn . . . . . . . 231 Tnh t ng b ng phng php sai phn . . . . . . . . . . . . . . 239 Phng trnh sai phn tuy n tnh v i h s h ng . . . . . . . . . 257 H phng trnh sai phn tuy n tnh thu n nh t v i h s h ng 271 H phng trnh sai phn tuy n tnh v i h s h ng . . . . . . . 279

M CL C

6.6 7

M t s l p phng trnh sai phn phi tuy n c ch m . . . . . . 291 376

Kh o st cc phng trnh i s 7.1 7.2 7.3 7.4

Nh c l i cc ki n th c c b n v s ph c v hm ph c . . . . . 375 S nghi m c a phng trnh a th c trn m t kho ng . . . . . . 409 nh gi kho ng nghi m . . . . . . . . . . . . . . . . . . . . . . 442 Gi i g n ng phng trnh a th c . . . . . . . . . . . . . . . 481

Ph l c A. Hm sinh v p d ng . . . . . . . . . . . . . . . . . . 517 P-1 V d minh h a . . . . . . . . . . . . . . . . . . . . . . . . . . . 517 P-2 Khi ni m v hm sinh . . . . . . . . . . . . . . . . . . . . . . 518

P-3 M t s v d p d ng . . . . . . . . . . . . . . . . . . . . . . . . 525 Ph l c B. H h i quy v h tu n hon . . . . . . . . . . . . . 538 Q-1 Ma tr n ly linh . . . . . . . . . . . . . . . . . . . . . . . . . . 539 Q-2 Ma tr n tu n hon . . . . . . . . . . . . . . . . . . . . . . . . . 542 Ti li u tham kh o 551

L i ni u
Chuyn "Bi n ph c, nh l v p d ng" ng vai tr nh l m t cng c c l c nh m gi i quy t hi u qu nhi u bi ton c a hnh h c, gi i tch, i s , s h c v ton t h p. Ngoi ra, cc tnh ch t c b n c a s ph c v hm bi n ph c cn c s d ng nhi u trong ton hi n i, cc m hnh ton ng d ng, ... Trong cc k thi Olympic ton sinh vin qu c t v qu c gia, th cc bi ton lin quan n bi n ph c th ng c c p d i nhi u d ng phong ph thng qua cc c trng v cc bi n i khc nhau c a phng php gi i, v a mang tnh t ng h p cao v a mang tnh c th su s c. Chng trnh ton h c b c Trung h c ph thng c a h u h t cc n c n c ta, sau nhi u l n c i cch, n i dung

u c ph n ki n th c s ph c.

s ph c cu i cng cng c a vo chng trnh Gi i tch 12, tuy nhin cn r t n gi n. V nhi u l do khc nhau, r t nhi u h c sinh, th m ch l h c sinh kh, gi i sau khi h c xong ph n s ph c cng ch hi u m t cch r t n s: s d ng s ph c, c th gi i c m i phng trnh b c hai, tnh m t vi t ng c bi t, ... Vi c s d ng s ph c v bi n ph c trong nghin c u, kh o st hnh h c (ph ng v khng gian) t ra c nhi u u vi t, nh t l trong vi c xem xt cc v n lin quan n cc php bi n hnh, qu tch v cc d ng mi n b o gic. Nhn chung, hi n nay, chuyn s ph c v bi n ph c (cho b c trung h c ph thng v i h c) c trnh by
8

d ng gio trnh, trnh by l thuy t

L i ni u

c b n v c c p n cc p d ng tr c ti p theo cch phn lo i phng php v theo c th c th c a cc d ng v d minh h a. p ng nhu c u b i d ng nghi p v sau i h c cho i ng gio vin, cc h c vin cao h c, nghin c u sinh chuyn ngnh Gi i tch, Phng trnh vi phn v tch phn, Phng php ton s c p v b i d ng h c sinh gi i v chuyn s ph c, bi n ph c v p d ng, chng ti vi t cu n chuyn nh ny nh m trnh by y cc ki n th c t ng quan, cc k thu t c b n v phng php s d ng s ph c v bi n ph c ti p c n cc d ng ton khc nhau c a hnh h c, s h c, ton r i r c v cc lnh v c lin quan. y l chuyn b i d ng nghi p v sau i h c m cc tc gi gi ng d y cho cc l p cao h c, cho i tuy n thi olympc ton sinh vin qu c gia v qu c t v l n i dung b i d ng gio vin cc tr ng i h c, cao ng v tr ng chuyn trong c n c t nhi u nm nay. Trong ti li u ny, chng ti s d ng m t s n i dung v l thuy t cng nh bi t p mang tnh h th ng c cc Th c s v h c vin cao h c th c hi n theo m t h th ng lgc nh t nh d i d ng cc chuyn nghi p v b c sau i h c. Nh ng d ng bi t p khc l m t s thi c a cc k thi h c sinh gi i v cc bi ton trong cc t p ch Ton h c v tu i tr , Kvant, Mathematica, cc sch gio khoa, chuyn v chuyn kh o, ... hi n hnh trong n c. Cu n sch c chia thnh 5 chng. Chng 1. S ph c v bi n ph c, l ch s v cc d ng bi u di n Chng 2. Tnh ton trn s ph c v bi n ph c Chng 3. M t s ng d ng c a s ph c trong i s

Chng 4. S ph c trong cc bi ton s h c v t h p

10

L i ni u

Chng 5. S ph c v ng d ng trong hnh h c Chng 6. S ph c v l i gi i c a phng trnh sai phn Cc tc gi xin chn thnh c m n lnh o B Gio D c v o t o, tr ng HKHTN, HQGHN ng h v ng vin cc tr ng h b i d ng nng cao ki n th c chuyn mn nghi p v sau i h c cc nm t 2002 n 2009 thnh cng t t p. C m n cc gio vin t 64 t nh thnh trong c n c nghe gi ng, trao i semina v c b n th o, g i nhi u ki n ng gp quan tr ng cho n i dung cng nh cch trnh by th t cc chuyn . Cu n sch c hon thnh v i s gip nhi t tnh v m t n i dung c a cc thnh vin trong semina lin tr ng-vi n Gi i tch - i s c a Tr ng i H c Khoa H c T Nhin, HQGHN. Cc tc gi xin by t lng bi t n t i ng nghi p v c gi c ki n ng gp cu n sch chuyn ny c hon thi n.

H N i ngy 02 thng 06 nm 2009 Cc tc gi

Chng 1

S ph c, bi n ph c l ch s cc d ng bi u di n
1.1 L ch s hnh thnh khi ni m s ph c

L ch s s ph c b t u t th k XVI. l th i k Ph c hng c a ton h c chu u sau m di trung c . Cc i l ng o1 1, b 1, a + b 1 xu t hi n u tin t th k XVI trong cc cng trnh c a cc nh ton h c Italy "Ngh thu t v i hay l v cc quy t c c a i s " (1545) c a G.Cardano (1501 - 1576) v " i s " (1572) c a R.Bombelli (1530 - 1572). Nh ton h c c Felix Klein (1849 - 1925) nh gi cng trnh c a G.Cardano nh sau: "tc ph m qu gi n t t nh ny ch a ng nh ng m m m ng c a i s hi n i v n v t xa t m c a ton h c th i c i". Khi gi i phng trnh b c hai Cardano v Bombelli a vo xt k hi u 1 l l i gi i hnh th c c a phng trnh x2 + 1 = 0, xt bi u th c b 1 l nghi m hnh th c c a phng trnh x2 + b2 = 0. Khi bi u th c t ng qut hn d ng (x a)2 + b2 = 0
1

Tn g i " o" l d ch t ti ng Php "imaginaire" do R.Descates xu t nm 1637.

11

12

Chng 1. S ph c, bi n ph c l ch s v cc d ng bi u di n

c th xem l nghi m hnh th c c a phng trnh (x a)2 + b2 = 0. V sau bi u th c d ng a + b 1, b = 0 xu t hi n trong qu trnh gi i phng trnh b c hai v b c ba (cng th c Cardano) c g i l i l ng " o" v sau c Gauss g i l s ph c2 v th ng c k hi u l a + bi, trong k hi u i := 1 c L.Euler3 a vo 1777 g i l n v " o". Qu trnh th a nh n s ph c nh m t cng c qu gi c a ton h c di n ra r t ch m ch p. Ngay tn g i v k hi u i := 1 l n v " o" cng gy nn nhi u n i bn khon, th c m c t d n n kh ng ho ng ni m tin v n khng c g chung v i s - m t cng c c a php m, m c d ng i ta v n xem l m t k hi u tr u t ng tho mn nh ngha i2 = 1. S kh ng ho ng ni m tin cng tr nn su s c hn b i vi c chuy n m t cch thi u cn nh c v thi u th n tr ng m t s quy t c c a i s thng th ng cho cc s ph c s n sinh ra nh ng ngh ch l kh ch u. Ch ng h n nh ngh ch l sau y: v i = 1 nn i2 = 1, nhng ng th i b ng cch s d ng cc quy t c thng th ng c a php ton khai cn b c hai l i thu c i2 = Ha ra 1 = 1! Ta nh n m nh l i r ng h th c i2 = 1
2 3

1 1 =

(1)(1) = (1)2 = 1 = 1.

Thu t ng "s ph c" l do nh ton h c Php N.Carnot (1753-1823) a vo u tin (1803) L. Euler (1707-1783) l nh ton h c Th y s

1.1. L ch s hnh thnh khi ni m s ph c

13

l nh ngha s m i i cho php ta a vo xt s ph c. i u c ngha r ng h th c khng th ch ng minh, n ch l quy c. Tuy v y, cng c ng i mu n ch ng minh h th c . Trong cu n sch "Phng php to " c a mnh, Vi n s L.S. Pointriagin m t l i ch ng minh nh sau: u tin ng i ta l y n a ng trn v i ng knh AB. T i m R tu c a n a ng trn h ng vung gc RS. Theo m t nh l c a hnh h c s c p, di ng vung gc RS l trung bnh nhn gi a cc di c a cc o n th ng AS v SB. V ni n di nn s khng sai st l n khi ni r ng bnh phng o n th ng RS b ng tch cc o n th ng AS v BS. By gi , tr v v i m t ph ng ph c. k hi u i m 1 l A ; i m +1 l B v i m i l R. Khi S s l i m 0. Tc gi c a php ch ng minh l p lu n nh sau: o n th ng RS l i, o n th ng AS l 1 v SB l +1. Nh v y, theo nh l v a nh c l i trn ta c i2 = (1)(+1) = 1. Th t ng ti c l php ch ng minh k l ny v n c vi t trong sch v gi ng d y m t s tr ng ph thng tr c th chi n th II.

L ch s ton h c cng ghi l i r ng Cardano cng nh c n cc nghi m ph c nhng l i g i chng l cc nghi m "ngu bi n". Ch ng h n, khi gi i h phng trnh x + y = 10 xy = 40 Cardano tm c nghi m 5 + 5 v 5 + 5 v ng g i nghi m ny l "m thu n tu" v th m ch cn g i l "nghi m m ngu bi n". C l tn g i " o" l di s n vnh c u c a "m t th i ngy th ng trn tr ng c a s h c".

14

Chng 1. S ph c, bi n ph c l ch s v cc d ng bi u di n

Th m ch i v i nhi u nh bc h c l n th k XVIII b n ch t i s v b n ch t hnh h c c a cc i l ng o khng c hnh dung m t cch r rng m cn y b n. Ch ng h n, l ch s cng ghi l i r ng I.Newton khng th a nh n cc i l ng o v khng xem cc i l ng o thu c vo cc khi ni m s , cn G.Leibniz th th t ln r ng: "Cc i l ng o - l ni n nu p huy n di u i v i tinh th n c a ng t i cao, d ng nh m t gi ng l ng c s ng m t ch n no y gi a ci c th t v khng c th t".

Ng i u tin nhn th y l i ch do a s ph c vo ton h c mang l i chnh l nh ton h c Italy R. Bombelli. Trong cu n " i s " (1572) ng nh ngha cc php tnh s h c trn cc i l ng o v do ng sng t o nn l thuy t cc s " o". Thu t ng s ph c c dng u tin b i K.Gauss4 (nm 1831). Vo th k XVII - XVIII nhi u nh ton h c khc cng nghin c u cc tnh ch t c a i l ng o (s ph c!) v kh o st cc ng d ng c a chng. Ch ng h n L.Euler m r ng khi ni m logarit cho s ph c b t k (1738), cn A.Moivre5 nghin c u v gi i bi ton cn b c t nhin i v i s ph c (1736). S nghi ng i v i s o (s ph c!) ch tiu tan khi nh ton h c ng i Nauy

l C.Wessel a ra s minh ho hnh h c v s ph c v cc php ton trn chng trong cng trnh cng b nm 1799. i khi php bi u di n minh ho s ph c cng c g i l "s Argand" ghi nh n cng lao c a nh ton h c Thu S R.Argand - ng i thu c k t qu nh c a Wessel m t cch c l p. L thuy t thu n tu s h c i v i cc s ph c v i t cch l cc c p s th c c th t (a; b), a R, b R c xy d ng b i nh ton h c Ailen l W.Hamilton (1837). y n v " o" i ch n gi n l m t c p s th c c

th t - c p (0; 1), t c l n v " o" c l gi i m t cch hi n th c.


4 5

C.Gauss (1777-1855) l nh ton h c c A.Moivre (1667-1754) l nh ton h c Anh

1.1. L ch s hnh thnh khi ni m s ph c

15

Cho n th k XIX, Gauss m i thnh cng trong vi c lu n ch ng m t cch v ng ch c khi ni m s ph c. Tn tu i c a Gauss cng g n li n v i php ch ng minh chnh xc u tin i v i nh l c b n c a i s kh ng nh r ng trong tr ng s ph c C m i phng trnh a th c u c nghi m. B n ch t i s c a s ph c th hi n ch s ph c l ph n t c a tr ng m

r ng ( i s ) C c a tr ng s th c R thu c b ng php ghp i s cho R nghi m i c a phng trnh x2 + 1 = 0. V i nh l c b n c a i s , Gauss ch ng minh c tr ng C tr thnh tr ng ng i s . i u c ngha l khi xt cc nghi m c a phng trnh i s trong tr ng ny ta khng thu c thm s m i. ng nhin tr ng s th c R (v do c tr ng s h u t Q) khng c tnh ch t ng i s . Ch ng h n, phng trnh v i h s th c c th khng c nghi m th c. Nhn l i hn 2500 nm t th i Pythagor n gi , con ng pht tri n khi ni m v s c th tm t t b i N Z Q R C v i cc bao hm th c: N Z Q R C. B ng cc k t qu su s c trong cc cng trnh c a cc nh ton h c K.Weierstrass, G.Frobenius, B.Peirce ng i ta m i nh n ra r ng m i c g ng m r ng t p s ph c theo con ng trn u khng c k t qu kh quan. K.Weierstrass ch ng minh t p h p s ph c C khng th m r ng thnh t p h p r ng hn b ng cch ghp thm s m i trong t p h p s r ng hn thu c v n b o ton m i php tnh v m i quy lu t c a cc php ton ng trong t p h p s ph c. Nh v y, cc t p h p s m i ch a t p s ph c ch c th thu c b ng vi c t b m t s tnh ch t thng th ng no c a cc s ph c. Ch ng h n nh ton h c Ailen l W.Hamilton (1805 - 1865) b t ph ra kh i ph m vi s ph c v thu c cc quatenion l tr ng h p n gi n nh t c a h siu

16

Chng 1. S ph c, bi n ph c l ch s v cc d ng bi u di n

ph c nhng nh ph i t b tnh ch t giao hon c a php nhn. H th ng cc quatenion l h khng giao hon v cc quatenion th hi n c trong khng gian b n chi u R4. D ng t ng qut c a quatenion l a + bi + cj + dk; a, b, c, d R, trong 1; i; j; k c Hamilton ch ra l cc n v siu ph c v c Hamilton g i l cc quatenion. y

i2 = j 2 = k 2 = ijk = 1 v chnh Hamilton l p ra b ng nhn sau y: x i j k i 1 k j j k 1 i k j i 1 d nh b ng nhn ny ta lu hnh v b tr sau. Ta bi u di n cc quatenion i, j, k b i ba i m trn ng trn theo th t cng chi u kim ng h . Tch c a hai s b t k trong b ba i, j, k b ng s th ba n u php vng quanh t th a s th nh t n th a s th hai l theo chi u kim ng h v b ng s th ba v v i d u tr n u php vng quanh ng c chi u kim ng h . R rng l php nhn khng c tnh ch t giao hon. i v i ton h c ngy nay cc s ph c v siu ph c l nh ng ch nh th hon ton t nhin, n khng " o" hn cht no so v i chnh cc s th c. Nhn l i l ch s lu di c a s pht tri n khi ni m s ta th y r ng c m i l n khi a vo nh ng s m i cc nh ton h c cng ng th i a vo cc quy t c th c hi n cc php ton trn chng. ng th i v i i u cc nh ton h c lun lun c g ng b o ton cc quy lu t s h c c b n (lu t giao hon c a

1.2. Cc d ng bi u di n s ph c

17

php c ng v php nhn, lu t k t h p v lu t phn b , lu t s p x p tuy n tnh c a t p h p s ). Tuy nhin s b o ton khng ph i khi no cng th c hi n c. V nh khi xy d ng tr ng s ph c ng i ta khng b o ton c lu t s p x p tuy n tnh v n c trong tr ng s th c, hay khi xy d ng t p h p cc s quatenion ta cng khng b o ton c lu t giao hon c a php nhn. T ng k t l ch s ton b qu trnh pht tri n khi ni m s , nh ton h c c L.Kronecker (1823 - 1891) vi t: "Th ng t o ra s t nhin, cn t t c cc lo i s cn l i u l cng trnh sng t o c a con ng i". C th ni r ng v i kh ng nh b t h ny L.Kronecker xc nh n n mng v ng ch c cho to lu i ton h c trng l m con ng i ang s h u.

1.2
1.2.1

Cc d ng bi u di n s ph c
Bi u di n s ph c d i d ng c p

M i s ph c a + bi hon ton c xc nh b i vi c cho hai s th c a v b thng th ng (a, b R) g i l cc thnh ph n c a chng. Ng i u tin c g ng nu r c trng quy lu t c a cc php tnh b ng ngn ng cc thnh ph n khng c n nh c n k hi u "nghi v n" i l Hamilton. C th , ng di n t m i s ph c b i m t c p s th c (c th t ) thng th ng. V t p h p s th c l t p h p con c a t p h p s ph c C nn khi xc nh cc php tnh s h c c b n trn cc s ph c ta c n i h i r ng khi p d ng cho cc s th c cc php ton a l i k t qu nh k t qu thu c trong s h c cc s th c. M t khc, n u ta mong mu n cc s ph c c nh ng ng d ng trong cc v n c a gi i tch th ta c n i h i r ng cc php ton c b n c a vo ph i tho mn cc tin thng th ng c a s h c cc s th c. nh ngha 1.1. M t c p s th c c th t (a; b), a R, b R, c g i

18

Chng 1. S ph c, bi n ph c l ch s v cc d ng bi u di n

l m t s ph c n u trn t p h p cc c p quan h b ng nhau, php c ng v php nhn c a vo theo cc nh ngha (tin ) sau y: a=c b = d. Ch r ng i v i hai s ph c b ng nhau (a; b) v (c; d) ta c th vi t i) Quan h ng nh t trong t p s ph c: (a; b) = (c; d) (a; b) (c; d) (n u mu n nh n m nh y l quan h ng nh t gi a hai c p s th c s p th t ) ho c (a; b) = (c; d) (n u mu n ni r ng y l quan h b ng nhau gi a hai s ph c). ii) Php c ng trong t p s ph c: (a; b) + (c; d) := (a + c; b + d) v c p (a + c; b + d) c g i l t ng c a cc c p (a; b) v (c; d). iii) Php nhn trong t p s ph c: (a; b)(c; d) := (ac bd; ad + bc) v c p (ac bd; ad + bc) c g i l tch c a cc c p (a; b) v (c; d). iv) S th c trong t p s ph c: C p (a; 0) c ng nh t v i s th c a, ngha l (a; 0) := a hay l (a; 0) a. T p h p cc s ph c c k hi u l C. Nh v y, m i ph n c a nh ngha s ph c u c pht bi u b ng ngn ng s th c v cc php ton trn chng. Trong nh ngha ny ba tin u th c ch t l nh ngha khi ni m b ng nhau, khi ni m t ng v khi ni m tch c a cc s ph c. Do vi c i chi u cc tin v i nhau s khng d n n b t c mu thu n no. i u duy nh t c th gy ra i cht lo ng i l tin iv). V n l ch v n d cc

khi ni m b ng nhau, t ng v tch cc s th c c ngha hon ton xc nh v do n u cc khi ni m ny khng tng thch v i nh ng khi ni m c c p n trong cc tin i) - iii) khi xt cc s th c v i t cch l cc c p d ng c bi t th bu c ph i lo i tr tin iv). Do ta c n i chi u tin iv) v i cc tin i), ii) v iii).

1.2. Cc d ng bi u di n s ph c

19

1) i) - iv). Gi s hai s th c a v b b ng nhau nh nh ng c p d ng c bi t ng nh t v i chng: (a; 0) = (b; 0). Khi theo tin i), ta c (a; 0) = (b; 0) a = b, t c l chng b ng nhau theo ngha thng th ng. 2) ii) - iv). Theo tin ii), t ng hai s th c a v c c xt nh nh ng c p (a; 0) v (c; 0) l b ng c p (a + c; 0 + 0) = (a + c; 0). Nhng theo tin iv) th (a + c; 0) a + c. Nh v y (a; 0) + (c; 0) = (a + c; 0 + 0) = (a + c; 0) a + c, t c l ng nh t b ng t ng a + c theo ngha thng th ng. 3) iii) - iv). Theo tin iii), tch cc s th c a v b c xt nh nh ng c p (a; 0) v (c; 0) l b ng c p (ac 0 0; a 0 + 0 c) = (ac; 0) v theo tin iv) ta c (ac; 0) ac. Nh v y (a; 0)(c; 0) = (ac; 0) ac, t c l ng nh t b ng tch a v i c theo ngha thng th ng. Nh v y tin iv) tng thch v i cc tin i), ii) v iii). Ta cng lu cc cng th c sau y c suy tr c ti p t iii) v iv): (a; b) = (a; b), R. Th t v y, t iv) v iii) ta c: (a; b) = (; 0)(a; b) = (a 0 b; b + 0 a) = (a; b). N u = m N th theo ii) ta c (a; b) + (a; b) = (2a; 2b);

20

Chng 1. S ph c, bi n ph c l ch s v cc d ng bi u di n

(2a; 2b) + (a; b) = (3a; 3b), . . . t c l (ma; mb) l k t qu php c ng lin ti p m s h ng b ng (a; b). i u ph h p v i bi u t ng thng th ng l php nhn v i s t nhin tng ng v i php c ng m s h ng b ng nhau. D dng th y r ng cc tin ii) v iii) l tng thch v i nhau v cc quy lu t thng th ng c a cc php tnh th c hi n trn cc s v n c b o ton khi chuy n sang s ph c (ng nhin ph i c t b cc quy lu t c quan h t i tnh ch t s p c tuy n tnh). T nh ngha suy ra trong t p h p C php c ng v php nhn c tnh ch t k t h p v giao hon ; php nhn lin h v i php c ng theo lu t phn b ; php c ng c php tnh ng c l php tr v do t n t i ph n t 0 l c p (0 ; 0) v (a; b) + (0; 0) = (a; b), a, b R. Vai tr n v trong t p h p s ph c C l c p (1; 0) v theo tin iii) (a; b)(1; 0) = (a; b). Hai s ph c z = (a; b) v z = (a; b) c g i l lin h p v i nhau. Ta c z z = (a; b)(a; b) = a2 + b2 0. T tnh ch t ny suy ra r ng v i m i (a; b) = (0; 0) t n t i c p ngh ch o (a; b)1, c th l c p a2 1 a b (a; b) = 2 , 2 . 2 2 +b a +b a + b2

Nh v y ta ch ng minh r ng t p h p cc s ph c C l p thnh m t tr ng. Tr ng c tnh ch t : (a) R C. (b) Phng trnh x2 + 1 = 0 c nghi m trong C. l c p (0 ; 1) v (0 ; -1). D i d ng c p cc php ton trn C c th c hi n theo cc quy t c

1.2. Cc d ng bi u di n s ph c

21

(i). (a1; b1)+(a2 ; b2 ) = (a1 +a2; b1 +b2 ) ; (a1; b1)(a2 ; b2) = (a1 a2; b1 b2) ; (ii). (a1 ; b1)(a2; b2) = (a1a2 b1 b2; a1 b2 + a2b1 ); (a1; b1) a1a2 + b1 b2 a1b2 aq2b1 , trong (a2 ; b2) = (0; 0). = ; (iii). (a2; b2) a2 + b2 a2 + b2 2 2 2 2 1.2.2 Bi u di n s ph c d i d ng i s

Nh v y, ta nh ngha v di n t m i quy t c tnh th c hi n trn cc s ph c b ng ngn ng cc thnh ph n t c l b ng ngn ng cc s th c. i u ny r t quan tr ng v v i cch ng i ta khng b m nh b i "ci o"c a k hi u i mang l i (m c d n r t th c v i l c p (0 ; 1).) By gi ta tr v v i cch vi t thng th ng (hay d i d ng Descartes) i v i s ph c. R rng l m i s ph c (a; b) C u bi u di n c d i d ng (a; b) = (a; 0) + (0; b) = (a; 0) + (b; 0)(0; 1) = a + bi, trong c p (0; 1) c k hi u b i ch i. T tin iii), suy r ng i2 = (0; 1)(0; 1) = (0 0 1 1; 0 1 + 1 0) = (1; 0) = 1. Nh v y ta tr v v i cch vi t thng th ng i v i s ph c (a; b) d i d ng a + bi nhng gi y n v o i c ngha hon ton hi n th c v n l

m t trong cc c p s th c m cc php tnh trn chng c nh ngha b i cc tin i), ii), iii) v iv), chnh l c p (0; 1). Th m ch, c th xem nhn t i bn c nh s th c b nh m t d u hi u ch r s th c b l thnh ph n th hai c a s ph c (a; b). Thnh ph n th nh t c a s ph c z = a + bi c g i l ph n th c c a s v c k hi u Re z, thnh ph n th hai c g i l ph n o v c k hi u l Im z. C n nh n m nh r ng ph n o cng nh ph n th c c a s ph c l nh ng s th c.

22

Chng 1. S ph c, bi n ph c l ch s v cc d ng bi u di n

Bi u th c (a; b) = a + bi c g i l d ng i s hay d ng Descartes c a s ph c. H th c (a; b) = a + bi ch ng t r ng gi a cc c p s th c c th t (a; b) v cc bi u th c d ng a + bi t n t i php tng ng n tr m t - m t v php tng ng c m t b i h th c v a nu. Nh php tng ng , thay v xt cc c p ta c th xt cc bi u th c a + bi bi u di n chng. Cc php ton (i)-(iii) i v i cc s ph c vi t d i d ng i s z1 := a1 + b1i ; z2 := a2 + b2i c nh ngha nh sau (i*) z1 + z2 = (a1 + b1i) + (a2 + b2i) = (a1 + a2) + (b1 + b2 )i, z1 z2 = (a1 + b1i) (a2 + b2i) = (a1 a2) + (b1 b2 )i, (ii*) z1z2 = (a1 + b1i)(a2 + b2i) = (a1a2 b1 b2) + (a1b2 + a2b1 )i, (iii*) a1 a2 + b1b2 a1b2 a2b1 a1 + b1i z1 = = + i, trong a2 + b2 = 0. 2 2 z2 a2 + b2i a2 + b2 a2 + b2 1 2 1 2

N u z = a + bi th s ph c lin h p z = a bi. Do z + z =2 Re z, z z =2 Im z, z z =|z|2 , trong |z| = r = S |z| = r = z z = a2 + b2.

z z = a2 + b2 c g i l mun c a s ph c z. i v i s

ph c z1 , z2 C, ta lun c ||z1| |z2|| |z1 + z2 | |z1| + |z2|. 1.2.3 Bi u di n hnh h c c a s ph c

Ta bi t r ng gi a t p h p m i c p s th c c th t v t p h p m i i m c a m t ph ng Euclide v i cc t a Descartes vung gc R2 c th xc l p php tng ng n tr m t-m t. c i u m i c p s th c c th t (a; b) c n c t tng ng v i i m M(a; b) c honh x = a v tung y = b.

1.2. Cc d ng bi u di n s ph c

23

V m i s ph c c nh ngha nh l m t c p s th c c th t nn m i s ph c (a; b) = a + bi c th t tng ng v i i m M(a; b) v ng c l i, m i i m M(a; b) c a m t ph ng s tng ng v i s ph c (a; b) = a + bi. l php tng ng n tr m t-m t. Nh php tng ng (a; b) a + bi ta xem cc s ph c nh l m t i m c a m t ph ng t a hay vect v i i m u t i g c t a O(0; 0) v i m mt t i M(a; b). nh ngha 1.2. M t ph ng t a v i php tng ng n tr m t-m t (a; b) a + bi c g i l m t ph ng ph c hay m t ph ng Gauss v cng c k hi u l C v z = a + bi l m t i m c a m t ph ng . M t cch ng n g n, m t ph ng R2 m cc i m c a n c ng nh t v i cc ph n t c a tr ng C c g i l m t ph ng ph c. Tr c honh c a m t ph ng t a c g i l tr c th c (do cc i m c a n tng ng v i cc s (a; 0) a R) cn tr c tung c g i l tr c o (do cc i m c a n tng ng v i cc s thu n o (0; b) = bi). S ph c z = a + bi cng c th bi u di n c b i m t vect i t g c t a v i cc hnh chi u a v b trn cc tr c t a . Nh v y, vect z = a + bi b ng bn knh vect c a i m z. V i cch bi u di n s ph c d i d ng vect i t g c t a , cc php c ng v tr cc s ph c c th c hi n theo cc quy t c c ng v tr cc vect. Tuy nhin php nhn v php chia c n th c hi n theo quy t c (ii*) v (iii*) do trong i s vect khng c cc quy t c tng t tr c ti p nh v y. Thng th ng, cc thu t ng "s ph c" ; "vect " ; "i m" c xem l ng ngha.

24

Chng 1. S ph c, bi n ph c l ch s v cc d ng bi u di n

1.2.4

Bi u di n s ph c nh ma tr n

Trong m c 1.1 v m c 1.2 , ta xy d ng tr ng s ph c nh cc c p s th c c th t z = (a; b), a, b R. u i m l n nh t c a phng php ny l n "ha gi i" c ci ph n th n b do k hi u "nghi v n" i mang l i. Bn c nh cch xy d ng , cn t n t i nhi u cch xy d ng khc n a. Sau y ta s trnh by cch xy d ng d a trn php c ng v nhn ma tr n trn tr ng s th c. Ta xt t p h p cc ma tr n c p hai d ng c bi t trn tr ng s th c M := a b b a a, b R

m trn cc php ton c ng v nhn c th c hi n theo cc quy t c thng th ng c a i s ma tr n. C th ch ng minh r ng t p h p M l p thnh m t tr ng. Ti p , m i s ph c z = a + bi ta t tng ng v i ma tr n a b b a (1.1)

l nh x tng ng n tr m t-m t. Qua nh x ny ton b tr ng s ph c c nh x ln t p h p M cc ma tr n d ng (1.1). Ta c a b c d b a + d c a b c d b a d c = = a+c b+d (b + d) a + c ac bd ad + bc (ad + bc) ac bd (1.2) (1.3)

T (1.2) v (1.3) suy ra r ng nh x xy d ng l ng c u gi a C v M v ma tr n v ph i c a (1.2) l tng ng v i cc s ph c (a + c) + (b + d)i = (a + bi) + (c + di) v ma tr n v ph i c a (1.3) l tng ng v i cc s ph c (ac bd) + (ad + bc)i = (a + bi)(c + di).

1.2. Cc d ng bi u di n s ph c

25

T suy r ng t ng v tch hai s ph c trong C tng ng v i t ng v tch cc nh c a chng trong M. ng th i ta cng thu c t p h p M0 cc ma tr n c p 2 d ng M0 := a 0 0 a aR

l ng c u v i t p h p cc s th c trong R. Trong php ng c u ny m i s th c a tng ng v i ma tr n a 0 0 a . a 0 0 a N u ta xt m t ma tr n ty c a M th T c th ng nh t ma tr n ; z= trong j= 0 1 1 0 . a b b a = v i s th c a.

a 0 0 1 0 a + b 1 0

= a + bj,

D th y r ng j 2 = 1. Th t v y, ta c j2 = 0 1 1 0 0 1 1 0 = o. 1 0 0 1 = 1.

T , ma tr n j c vai tr nh n v 1.2.5

D ng l ng gic v d ng m c a s ph c

B ng cch s d ng t a c c trn m t ph ng ph c C: (a) di bn knh vect r := |z| = z z = a2 + b2 ; (b) gc c c = Arg c g i l acgumen c a z, ta thu c h th c

z = a + bi = r(cos + i sin ),

(1.4)

26

Chng 1. S ph c, bi n ph c l ch s v cc d ng bi u di n

Re z = a = r cos , Im z = b = r sin . Bi u th c (1.4) c g i l d ng l ng gic hay d ng c c c a s ph c z = a+bi. Argumen = Arg z l hm th c a tr c a bi n ph c z = 0 v i v i z cho, cc gi tr c a hm sai khc nhau m t b i nguyn c a 2. Hm acgumen khng xc nh t i z = 0. Thng th ng ng i ta s d ng gi tr chnh c a acgumen = arg z xc nh v i i u ki n b sung < arg z ho c 0 arg z < 2. n gi n cch vi t cc s ph c ta t cos i sin = ei . d ng l ng gic (1.4) c bi n i thnh d ng m z = rei . l d ng s m c a s ph c z = 0. Cc cng th c (1.4) v (1.5) c bi t ti n l i khi th c hi n php nhn v chia cc s ph c. D dng ch ng minh r ng n u z1 = r1 ei1 v z2 = r2 ei2 th 1. z1 z2 = r1 r2 ei(1 +2 ) ; ; z1 r1 2. = ei1 2 ) , r2 = 0. z2 r2 Php nng s ph c z = a + bi = r(cos + i sin ) ln ly th a b c n i v i s ph c c th c hi n theo cng th c Moivre. (1.5)

1.2. Cc d ng bi u di n s ph c

27

z n = rn ein .

(1.6)

wk = Cng th c

+2k n z = n rei n ,

k = 0; 1; . . . ; n 1.

(1.7)

[r(cos + i sin )]n = rn (cos n + i sin n).

(1.8)

c g i l cng th c Moivre. N u r = 1 th cng th c Moivre c d ng c bi t (cos + i sin )n = cos n + i sin n. (1.9)

T cng th c (1.6) suy r ng cn b c n c a s ph c c ng n gi tr . Ta cng c n lu r ng khi ni m "gi tr s h c" trong php khai cn s ph c khng c a vo v khng th a khi ni m vo theo cch t nhin no . n y, quay l i tm hi u "tr chi ng y bi n" trong ngh ch l nu ta th y sai l m ch y u ph m ph i hai c a s ph c. T (1.6) v (1.8) v b ng cch thay b i ta c 1 i i cos = (e + ie ), 2 1 sin = (ei ei ). 2i Cc cng th c (1.10) c g i l cng th c Euler. 1.2.6 Bi u di n cc s ph c trn m t c u Riemann trn,

l vi c ch n cc gi tr c a cn b c

(1.10)

Cng v i cch bi u di n s ph c nh l vect hay i m trn m t ph ng cn c cch bi u di n hnh h c khc n a. l bi u di n s ph c b i i m c a m t c u g i l m t c u Riemann.

28

Chng 1. S ph c, bi n ph c l ch s v cc d ng bi u di n

Trong khng gian Euclide ba chi u v i h t a Descartes vung gc (; ; ) 1 1 v i bn knh b ng ta xt m t c u v i tm t i i m 0; 0; 2 2 1 2 1 = S = (; ; ) : 2 + 2 + 2 4 sao cho n ti p xc v i m t ph ng z t i g c t a v tr c th c c a m t ph ng z trng v i tr c { = 0; = 0}, cn tr c o th trng v i tr c { = 0; = 0}. Ta xt php chi u v i c c b c t i i m P (0; 0; 1). Gi s z C l i m ty . N i i m z C v i c c b c P b ng o n th ng. o n th ng ny c t m t c u S t i i m A(z). V ng c l i, gi s A S l m t i m ty c a m t c u. Khi tia P A s c t m t ph ng ph c t i i m z. Hi n nhin r ng l m t php tng ng n tr m t-m t. nh ngha 1.3. Php tng ng :C nh m t z A(z) S

trn c g i l php chi u n i v i c c t i i m P . i m

A(z) S c g i l nh n i hay l nh c u c a i m z. nh l 1.1. Trong php chi u n i :C z A(z) S

i m x = x + iy C s tng ng v i i m A(z) S c t a l = x , 1 + |z|2 = y , 1 + |z|2 = |z|2 1 + |z|2 (1.11)

Cng th c (1.11) c g i l cng th c c a php chi u n i. Ch ng minh. Th t v y, v ba i m P (0; 0; 1), A(z) = (; ; ) v z = (x; y; 0) cng n m trn m t ng th ng nn cc t a c a chng ph i th a mn h th c 0 0 1 = = , x0 y0 01

1.2. Cc d ng bi u di n s ph c

29

hay l x= r ng 2 + 2 |z| = (1 )2
2

, 1

y=

, 1

z=

+ i 1

(1.12)

2 + 2 +

1 2

1 4

ta thu c |z|2 = v do = , 1

|z|2 . Th gi tr vo (1.12) ta tm c 1 + |z|2 = x , 1 + |z|2 = y 1 + |z|2

Hi n nhin trong php bi n i , i m P (0; 0; 1) khng tng ng v i i m z no c a m t ph ng C. By gi ta xt s ph c l t ng" z = v b sung" cho m t ph ng ph c C b ng cch thm cho n i m xa v cng duy nh t (g i t t l i m v cng) tng ng v i s ph c z = . nh ngha 1.4. T p h p l p nn t m t ph ng ph c C v i m v cng (k hi u l ) c g i l m t ph ng ph c m r ng v k hi u l C. Nh v y C = C {} v C khng ph i l m t tr ng. T nh l 1.1 suy r ng php chi u n i xc l p s tng ng n tr m t-m t gi a cc i m c a C v cc i m c a S \ {P }. Hi n nhin khi |z| th i m A(z) s d n n i m P (0; 0; 1). Th t v y, t tnh ng d ng c a hai tam gic zOP v AP O suy r ng AP = 1 1 + |z|2

30

Chng 1. S ph c, bi n ph c l ch s v cc d ng bi u di n

v do AP 0 khi |z| . T s l lu n ta rt ra k t lu n r ng php chi u n i : C S \ {P } c th thc tri n vo C thnh : C S b ng cch t v () = P (0; 0; 1). Do , m t cch t nhin ta c th cho r ng i m z = tng ng v i c c b c" P c a m t c u S v m i i m trn m t c u S c th xem nh l m t i m tng ng c a m t ph ng C. Phng php bi u di n hnh h c cc s ph c nh trn c g i l phng php bi u di n c u c a cc s ph c. M t c u S, v l do , c g i l m t c u s ph c Riemann6. Tnh u vi t c a m t c u Riemann l ch trn m t c u Riemann i m v cng duy nh t c a = , zC

m t ph ng ph c c m t m t cch kh tr c quan. Sau ny khi nghin c u m t v n no n u mu n xt c i m z = th ta s ti n hnh cc l p lu n trn m t c u Riemann. 1.2.7 Kho ng cch trn C

Tng ng v i hai phng php bi u di n hnh h c s ph c c m t , ta s a vo trong C hai mtric. Trong mtric th nh t kho ng cch gi a hai i m z1 , z2 C c gi thi t b ng dC = dC (z1 ; z2) := |z1 z2 | =
6

(x1 x2 )2 + (y1 y2)2 .

B.Riemann (1826-1866) l nh ton h c c.

1.2. Cc d ng bi u di n s ph c

31

Mtric ny l mtric Euclide thng th ng trong m t ph ng R2 . Trong mtric th hai (g i l mtric c u) kho ng cch gi a hai i m z1 v z2 C c hi u l kho ng cch (trong khng gian ; ; ) gi a cc nh c u c a chng. Kho ng cch ny c g i l kho ng cch c u hay kho ng cch Jordan7 gi a hai i m z1 v z2 C: dC = dC (z1 ; z2). nh l 1.2. Gi s dC = dC (z1; z2) l kho ng cch c u gi a cc i m z1 = x1 + iy1 v z2 = x2 + iy2. Khi dC (z1 ; z2) = n u z2 = th dC (z1 ; ) = 1 1 + |z1|2 (1.14) (1 + |z1 |z1 z2| 2 )1/2 (1 + | |z2|2 )1/2 , (1.13)
def

v kho ng cch c u th a mn cc tin thng th ng c a m t mtric. Ch ng minh. 1. Th t v y, t cng th c (1.11) ta c dC (z1 ; z2) = [(1 2 )2 + (1 2 )2 + (1 2 )2 ]1/2 = [1 + 2 2(1 2 + 1 2 + 1 2 )]1/2 = |z2|2 |z1|2 x1 x2 + 2 2| 2 1 + |z1| 1 + |z2 | (1 + |z1|2 )(1 + |z2 |2)
1/2

|z1|2 |z2|2 y1y2 + + (1 + |z1|2)(1 + |z2|2 ) (1 + |z1|2 )(1 + |z2|2 ) =

{|z1 |2(1 + |z2)2) + |z2|2(1 + |z1|2) 2(x1 x2 + y1y2 ) 2|z1 |2 |z2|2} 1 + |z1|2 |z1 |2 + |z2|2 2x1 x2 2y1 y2 1 + |z1|2 1 + |z2|2
1/2

1/2

1 + |z2|2 = (x1 x2)2 + (y1 y2 )2 1 + |z1|2 1 + |z2|2


1/2

=
7

C.Jordan (1838-1922) l nh ton h c Php.

32

Chng 1. S ph c, bi n ph c l ch s v cc d ng bi u di n

|z1 z2| 1 + |z1|2 1 + |z2|2

Cng th c (1.13) c ch ng minh. Trong tr ng h p khi z2 = , ta c dC (z1 ; ) =


2 2 1 + 1 + (1 1 )2 =

1 1 =

1 (1 + |z + 1|2 )1/2

v z2 = nn 2 = 1. Hi n nhin r ng dC (z1 ; z2) 0 v dC (z1; z2) = 0 z2 = z2 . Cng d th y

dC (z1 ; z2) = dC (z2; z1 ). Ta cn ph i ch ng minh r ng dC (z1 ; z3) dC (z1; z2) + dC (z2; z3).

i v i z1, z2 v z3 , ta c ng nh t sau (z1 z2)(1 + z3z 3 ) = (z1 z3)(1 + z2 z 3) + (z3 z2)(1 + z1 z 3). T |z1 z2 |(1 + |z3|2 ) Nhng r ng (1 + uv)(1 + u v) cho nn |1 + z2z 3 |2 = (1 + z2 z 3)(1 + z 2z3 ) v |1 + z1 z3|2 (1 + |z1|2 )(1 + |z3|2 ). (1.17) (1 + |z2|2)(1 + |z3|2) (1.16) (1 + |u|2)(1 + |v|2) |z1 z3 |(|1 + z2z 3 |) + |z3 z2|(|1 + z1 z 3|). (1.15)

T cc h th c (1.13) v (1.15) - (1.17) ta thu c i u ph i ch ng minh.

1.3. Bi t p

33

Ta nh n xt r ng trn cc t p h p b ch n M C (t c l nh ng t p h p c ch a trong hnh trn c nh no {|z| mtric - c u l tng ng v i nhau. Th t v y, n u M {|z| |z1 z1 | 1 + R2 R} th t (1.13) ta c dC (z1 ; z2) |z1 z2 |, z1 , z2 M. R, R < }) hai mtric Euclide v

Do mtric c u th ng c p d ng khi xt cc t p h p khng b ch n. V ni chung, khi ti n hnh cc l p lu n trn C ta s d ng mtric Euclide dC , cn trn C th s d ng mtric - c u dC . T i u v a ch ng minh trn y cng suy ra r ng vi c a vo m i mtric trn y u bi n C thnh khng gian mtric.

1.3

Bi t p

Bi 1.1. Cho a, b C, ch ng minh cc b t ng th c sau 1. |ab| = |a||b|; |a| a = ; b |b|

2. |a b| |a| + |b|; |a b| ||a| |b||; 3. N u Re a 0 th |1 + a| 1 + |a| ; 2

a b 1 + . 4. |a + b| (|a| + |b|) 2 |a| |b| Bi 1.2. Gi s a, b C, ch ng minh cc ng nh t th c sau 1. |a + b|2 + |a b|2 = 2(|a|2 + |b|2). 2. |1 b|2 |a b|2 = (1 + |ab|)2 (|a| + |b|)2. a 3. |a + b|2 = (|a| + |b|)2 2[|a Re (a b| b)]. b| 4. |1 + a 2 + |a b|2 = (|a|2 + 1)(|b| + 1).

34

Chng 1. S ph c, bi n ph c l ch s v cc d ng bi u di n

5. (1 + ab)(1 + a (1 + |a|2)(1 + |b|2). b) Bi 1.3. Gi s z = a + bi, z = 1. Ch ng minh r ng w = o khi v ch khi a2 + b2 = 1. Bi 1.4. Gi s s ph c z = 1 v |z| = 1. Khi ta c th bi u di n z d i 1 + it d ng z = , t R. 1 it Bi 1.5. Ch ng minh r ng n u gi tr chnh arg z= arg (a + ib) th a mn i u ki n < arg z th n c tnh theo cng th c b n u a > 0, arctan a b arg (a + ib) = arctan a + n u a < 0, b 0, b arctan a n u a < 0, b < 0. Bi 1.6. Ch ng minh r ng n u gi tr chnh arg z= arg (a + ib) th a mn i u ki n 0 arg z < 2 th n c tnh theo cng th c b n u a > 0, b > 0 arctan a b arg (a + ib) = arctan a + 2 n u a > 0, b < 0, b arctan a + n u a < 0. Bi 1.7. Ch ng minh cc b t ng th c sau z 1 |arg z|. |z| (ii) |z 1| ||z| 1| + |z|arg z|. (iii) |z| |Re z + |Im z| 2|z|. (i) Bi 1.8. Gi s a C, |a| < 1. Ch ng minh r ng cc b t ng th c |z| < 1 za < 1 l tng ng. v 1 z a Bi 1.9. Gi s |z1 + z2 | = |z1 z2|, arg z1 = , z1 = 0, z2 = 0. Tm arg z2. Bi 1.10. Gi s |z1| = a, |z2| = b, |z1 + z2 | = c. Tnh |z1 z2 |. Bi 1.11. Gi s |z |z|| = |z|. Tm arg z. Bi 1.12. Gi s |z + z | = |z|. Tm arg z. z1 l s thu n z+1

1.3. Bi t p

35

Bi 1.13. Gi s |z + |z|| = |z|. Tm arg z. Bi 1.14. Gi s |z| = |z |z|i|. Tm arg z. Bi 1.15. Gi s z z = |z|i. Tm arg z.

Chng 2

S ph c v bi n ph c trong l ng gic
2.1 Tnh ton v bi u di n m t s bi u th c

Trong ph n ny, ta xt m t s tnh ton trn cc s ph c c th . V d 2.1. Tch ph n th c v ph n o c a s z= L i gi i. Ta c z= 3+i 3+i 3+i 9 + 6i 1 4 3 = = = +i , 1 2i + i + 2 3i 3+i 9+1 5 5 3+i (1 + i)(1 2i)

v zz = a2 + b2. 4 3 T Re z = , Im z = 5 5 V d 2.2. Tch ph n th c v ph n o c a 1 w = , z = x + iy. z L i gi i. Ta c u + iv = 1 1 x iy x iy = = 2 x + iy x + iy x iy x + y2


36

2.1. Tnh ton v bi u di n m t s bi u th c

37

T suy ra u= V d x2 x y , v= 2 2 +y x + y2

2.3. Tm argumen c a s ph c z = 1 3i.

L i gi i. Trong tr ng h p ny ta c a = 1, b = 3. Ta nh n c h d ng cos = 1 2 sin = 3 2 4 + 2k, k Z. Do 3 arg z = V d 4 + 2k, 3 k Z.

T suy ra k =

2.4. Tm Arg ( 3 + i).

L i gi i. M i gi tr argumen c a s 3 + i u th a mn phng trnh 1 tan = 3 T suy ra k = V z = 3 + i thu c gc ph n t th hai nn k l argumen n u k = 2n + 1 (s l ). Do 5 + 2n, n Z. Arg ( 3 + i) = + (2n + 1) = 6 6 V d 2.5. Bi u di n s ph c sau y d i d ng l ng gic cos z= i sin ( 3 + i) 3 3 i1 + k, 6 k Z.

38

Chng 2. S ph c v bi n ph c trong l ng gic

L i gi i. Ta c , z1 = cos i sin = cos( ) + i sin 3 3 3 3 , z2 = 3 + i = 2 cos + i sin 6 6 3 3 z3 = i 1 = 2 cos + i sin . 4 4 T suy ra r ng |z| = z= 2 cos 2, = 11 3 + = . Do 3 6 4 12 + i sin 11 12 .

11 12

V d 2.6. Hy bi u di n tan 5 qua tan . L i gi i. Ta c h th c cos 5 + i sin 5 = (cos + i sin )5 . S d ng khai tri n nh th c Newton cho v ph i v tch ph n th c v ph n o, ta c cos 5 = cos5 10 cos3 sin2 + 5 cos sin4 sin 5 = 5 cos4 sin 10 cos 2 sin3 + sin5 . T suy ra tan 5 = 5 tan 10 tan 3 + tan5 1 10 tan 2 + 5 tan4

V d 2.7. Bi u di n tuy n tnh sin5 qua cc hm l ng gic c a gc b i. L i gi i. t z = cos + i sin . Khi z 1 = cos i sin . z k = cos k + i sin k, z k = cos k i sin k. T suy ra cos = z + z 1 , 2 sin = z z 1 . 2i

2.1. Tnh ton v bi u di n m t s bi u th c

39

z k + z k = 2 cos k, z k z k = 2i sin k. s d ng cc h th c ny ta thu c sin = = = = V d 2.8. Tm


5

z z 1 z 5 5z 3 + 10z 10z 1 + 5z 3 z 5 = 2i 32i 5 5 3 3 (z z ) 5(z z ) + 10(z z 1 ) 32i 2i sin 5 5 2i sin 3 + 10 2i sin 32i sin 5 5 sin 3 + 10 sin 16 3 2 + 2i. 8(cos 45 + i sin 45 ).

L i gi i. Ta c 2 + 2i = T ta thu c

1 45 + k360 45 + k360 3 3 cos + i sin 2 + 2i = 8 3 3 = 2 (cos(15 + k120 ) + i sin(15 + k120 )) . Do v y, g i cc gi tr cn b c ba c a s ph c 2 + 2i l w0, w1 , w2 ta c w0 = w1 = 2 (cos 15 + i sin 15 ) ; 2 (cos 135 + i sin 135 ) =

2( cos 45 + i sin 45 ) ; w2 = 2 (cos 255 + i sin 255 ) = 2( sin 15 i cos 15 ). r ng 1 cos 45 = sin 45 = , 2 ta c w1 = 1 + i.

40

Chng 2. S ph c v bi n ph c trong l ng gic

tnh w0 v w2, ta lu 15 = 45 30 . Do 1 cos 15 = cos 45 cos 30 + sin 45 sin 30 = 2 1 3 1 . sin 15 = 2 2 2 T thu c 3+1 31 +i , w0 = 2 2 31 3+1 i . w2 = 2 2 V d 2.9. Tnh A = in + in+1 + in+2 + in+3 v B = i i2 i99 i100. L i gi i. Ta vi t n theo mod 4 v c ngay n u n = 4k, 1 i n u n = 4k + 1, in = n u n = 4k + 2, 1 i n u n = 4k + 3, T y suy ra A = in + in+1 + in+2 + in+3 = 0 v B = i i2 i99 i100 = 1. S d ng cng th c bi n i l ng gic quen thu c ho c s d ng d ng l ng gic c a s ph c, ta d dng thu c cc tnh ch t sau. Tnh ch t 2.1. i v i m i a th c l ng gic An (x) = a0 + a1 cos x + b1 sin x + + an cos nx + bn sin nx lun tm c cc a th c i s Pn (t) v Qn1 (t) l n l t c b c khng qu n v n 1 i v i t sao cho An (x) = Pn (cos x) + sin xQn1 (cos x). 3 1 + 2 2

2.1. Tnh ton v bi u di n m t s bi u th c

41

Tnh ch t 2.2. i v i m i a th c l ng gic theo sin b c n (n Sn (x) = b0 + b1 sin x + b2 sin 2x + + bn sin nx lun tm c a th c i s Qn1 (t) sao cho Sn (x) = b0 + sin xQn1 (cos x). Tnh ch t 2.3. V i m i a th c l ng gic theo cosin d ng Cn (x) = a0 + a1 cos x + a2 cos 2x + + an cos nx

1) d ng

lun tm c a th c i s Pn (t) v i h s b c cao nh t l 2n1 an sao cho Cn (x) = Pn (cos x). Ng c l i, v i m i a th c i s Pn (t) v i h s b c cao nh t b ng 1, qua php t n ph t = cos x u bi n i c v d ng Cn (x) v i an = 21n . V d 2.10. Vi t cng th c bi u di n c a cos nx v sin nx theo cc lu th a

c a cos x v sin x. L i gi i. Theo cng th c Moivre th


n

cos nx + i sin nx = (cos x + i sin x)n .

Theo cng th c khai tri n nh th c Newton, ta c (cos x + i sin x) =


k=0 n k Cn cosnk x(i sin x)k

= cos x + trong

1 iCn cosn1

2 x sin x Cn cosn2 x sin2 x + := A + iB,

A= B= Vy

(1) 2 sinn x n u n ch n n n1 (1) 2 Cn cos x sinn1 x n u n l ;


2 (1) 2 Cn cos x sinn1 x n u n ch n n1 (1) 2 sinn x n u n l .
n2

2 4 cos nx = cosn x Cn cosn2 x sin2 x + Cn cosn4 x sin4 x + A, 1 3 sin nx = Cn cosn1 x sin x Cn cosn3 x sin3 x + + B.

42

Chng 2. S ph c v bi n ph c trong l ng gic

V d

2.11. Bi u di n cc hm s sinn x v cosn x d i d ng cc a th c

l ng gic. L i gi i. Gi s z = cos t + i sin t. Khi z 1 = (cos t + i sin t)1 = cos t i sin t. Do cos t = Ta c
1 2 n1 (z + z 1 )n = z n + Cn z n1 z 1 + Cn z n2 z 2 + + Cn zz n+1 + z n 1 2 (z n + z n ) + Cn (z n2 + z (n2) ) + + Cn n u n ch n, 1 (z n + z n ) + Cn (z n2 + z (n2) ) + + Cn 2 (z + z 1 ) n u n l
n1 n

z + z 1 z + z 1 v sin t = . 2 2i

1 2 v (z z 1 )n = z n Cn z n1 z 1 + Cn z n2 z 2 + + (1)n z n 1 2 (z n + z n ) Cn (z n2 + z (n2) ) + + (1) 2 Cn n u n ch n, 1 (z n z n ) Cn (z n2 z (n2) ) + + (1)


n1 2 n n

= Vy

Cn 2 (z z 1 ) n u n l .

n1

1 n 1 1 2 cos nx + Cn cos(n 2)x + + Cn n u n ch n n 2n1 2 cos x = n1 1 1 cos nx + Cn cos(n 2)x + + Cn 2 cos x n u n l , 2n1 n (1) 2 2 cos nx 2C 1 cos(n 2)x + + (1) n C n n u n ch n 2 2 n n n 2 n1 n1 sinn x = (1) 2 n1 1 2 sin nx 2iCn sin(n 2)x + + (1) 2 Cn 2 2 sin x 2n n unl . V d 2.12. Ch ng minh ng th c 2 (m 1) m sin sin = m1 v i m N . sin 2m 2m 2m 2 L i gi i. G i P l v tri c a ng th c.

2.2. Tnh gi tr c a m t s bi u th c l ng gic

43

Xt phng trnh

x2m 1 = 0. Ta th y phng trnh ny c hai nghi m th c

l x = 1 v (2m 2) nghi m ph c. K hi u k l nghi m ph c c a phng 2k 2k trnh v i k = 0, 1, . . . , 2m 1, ta c k = cos + i sin (xem [4]). 2m 2m Khi 2mk = cos 2(2m k) 2(2m k) + i sin 2m 2m 2k 2k = cos 2 + i sin 2 2m 2m 2k 2k i sin = k . = cos 2m 2m

Vy x2m 1 = (x2 1)(x 1 )(x 1) (x m1 )(x m1 ) = (x2 1)[x2 (1 + 1 )x + 1] [x2 (m1 + m1 )x + 1]


m1

= (x 1)
k=1

x2 2x cos

2k +1 . 2m

Do v i x = 1, ta c x2m 1 = x2 1
m1

x2 2x cos
k=1

k +1 . m

Chuy n qua gi i h n khi x 1, ta c


m1

m = 22(m1)
k=1

sin2

k = 22(m1) p2 . 2m

m V y nn P = m1 . 2

2.2

Tnh gi tr c a m t s bi u th c l ng gic

Tr c h t ta tnh gi tr c a m t s bi u th c l ng gic t i cc i m cho tr c c cc c th c bi t b ng cc cng th c bi n i l ng gic c b n.

44

Chng 2. S ph c v bi n ph c trong l ng gic

V d 2.13. Ch ng minh cc cng th c 51 5+1 o o sin 18 = ; cos 36 = . 4 4 L i gi i. Ta c cos 54o = sin 36o cos (3 18o ) = sin (2 18o ) 4 cos3 18o 3 cos 18o = 2 sin 18o cos 18o 4 sin2 18o + 2 sin 18o 1 = 0. Suy ra sin o l nghi m dng c a phng trnh 4t2 2t 1 = 0. Do 18 + 51 5+1 sin 18o = v suy ra cos 36o = 1 sin2 18o = . 4 4 V d 2.14. Ch ng minh cng th c sin a sin(60o a) sin(60o + a) = L i gi i. Th t v y, ta c sin a sin(60o a) sin(60o + a) = = sin a(sin 60o cos a sin a cos 60o )(sin 60o cos a + sin a cos 60o ) 1 1 3 3 = sin a cos a sin a cos a + sin a 2 2 2 2 = sin a 3 1 cos2 a sin2 a 4 4 = 1 1 sin a[3(1 sin2 a) sin2 a] = sin 3a. 4 4 1 sin 3a. 4

V d 2.15. Ch ng minh cng th c 51 sin 2o sin 18o sin 22o sin 38o sin 42o sin 58o sin 62o sin 78o sin 82o = . 1024 1 L i gi i. S d ng cng th c sin a sin(60o a) sin(60o + a) = sin 3a, ta c 4 sin 2o sin 18o sin 22o sin 38o sin 42o sin 58o sin 62o sin 78o sin 82o = (sin 2o sin 58o sin 62o )(sin 18o sin 42o sin 78o )(sin 22o sin 38o sin 82o ) 1 1 = (sin 6o sin 54o sin 66o ) = sin 18o . 64 256

2.2. Tnh gi tr c a m t s bi u th c l ng gic

45

T ng th c sin 18o =
o o o

51 , ta suy ra 4
o o o o o o

sin 2 sin 18 sin 22 sin 38 sin 42 sin 58 sin 62 sin 78 sin 82 = V d 2.16. Ch ng minh sin 1o l m t s v t .

51 . 1024

L i gi i. Ta ch ng minh b ng phng php ph n ch ng. Gi s r ng sin 1o l m t s h u t . S d ng cng th c sin 3a = 3 sin a 4 sin3 a, ta thu c sin 3o cng l m t s h u t . Tng t , ta suy ra sin 9o , sin 27o , sin 81o l cc s h u t . M sin 81o = cos 9o v sin 18o = 2 sin 9o cos 9o nn sin 18o cng l m t s h u t. p Gi s sin 18o = , (p, q N, (p; q) = 1). M t khc ta c sin 18o = q p 4p + q 51 nn = . Suy ra 5 = l m t s h u t (mu thu n). 4 q q V y sin 1o l s v t . V d 2.17. Ch ng minh ng th c cos L i gi i. Ta c 2 3 cos cos + cos 7 7 7 7 2 3 + 2 sin cos = 2 sin cos 2 sin cos 7 7 7 7 7 7 2 3 4 2 = sin sin sin + sin sin 7 7 7 7 7 = sin . 7 2 sin 2 3 1 cos + cos = 7 7 7 2 51 4

46

Chng 2. S ph c v bi n ph c trong l ng gic

V y nn

sin 2 3 7 = 1. + cos = cos cos 7 7 7 2 2 sin 7

V d 2.18. Tnh gi tr c a bi u th c S = (sin x + sin 2x + sin 4x)5 ( sin x + sin 2x + sin 4x)5 (sin x sin 2x + sin 4x)5 (sin x + sin 2x sin 4x)5 , ng v i x = 20o . L i gi i. V i x = 20o th x x 3x cos = cos = sin 4x, 2 2 2 1 1 sin x sin 2x = cos x , 2 2 1 x x 3 cos + . cos x cos = 2 2 2 2 sin x + sin 2x = 2 sin Do S = 25 [(sin x + sin 2x)5 sin5 2x sin5 x] = 25 .5[sin x sin 2x(sin3 2x + sin3 x) + 2 sin2 x sin2 2x(sin x + sin 2x)] = 25 .5 sin x sin 2x(sin 2x + sin x)(sin2 x + sin 2x sin x + sin2 2x) x x = 25 .5 sin x sin 2x cos cos2 sin 2x sin x 2 2 x x 1 1 x 4 cos cos2 cos + = 2 .5 cos x 2 2 2 2 4 x 3 1 cos = 24 .5 cos x 2 2 4 x x 1 3 1 = 22 3 5 cos + cos = 15 3. 2 2 4 2 2 V d 2.19. Cho bi u th c 1 S = 32x(x2 1)(2x2 1)2 + x

2.2. Tnh gi tr c a m t s bi u th c l ng gic

47

Tnh S cos

2 . 9 2 , ta c 9 1 cos

L i gi i. Xt x = cos v i =

S = 32 cos (cos2 1)(2 cos2 1)2 + =

1 32 cos 2 sin2 (2 cos 2 1)2 cos 2 1 8 cos 2 sin2 2 = cos cos 8 cos 1 2 sin2 4 = = = 1. = cos cos cos V y S cos V d 2 9 = 1.

2.20. Tnh cc t ng sau : S1 = cos 3 5 7 9 + cos + cos + cos + cos , 10 10 10 10 10 3 3 5 5 7 7 9 S2 = cos cos + cos cos + cos cos + cos cos . 10 10 10 10 10 10 10 10

L i gi i. Nh n xt r ng cos 5 = 16 cos 5 20 cos3 + 5 cos . Th t v y cos 5 = cos(2 + 3) = cos 2 cos 3 sin 2 sin 3 = (2 cos2 1)(4 cos3 3 cos ) 2 sin cos (3 sin 4 sin3 ) = 8 cos5 4 cos3 6 cos3 + 3 cos 6(1 cos2 ) cos 8(1 cos2 )2 cos = 16 cos5 20 cos 3 + 5 cos . V i cc gi tr = 3 5 7 9 , = , = , = , = , 10 10 10 10 10

48

Chng 2. S ph c v bi n ph c trong l ng gic

th cos 5 = 0. Do v y cos 3 5 7 9 , cos , cos , cos , cos 10 10 10 10 10

l cc nghi m c a a th c f (x) = 16x5 20x3 + 5x. Theo nh l Vite, ta thu c 5 S1 = 0 , S2 = . 4 V d 2.21. Tnh t ng S = cos 5o + cos 77o + cos 149o + cos 221o + cos 293o . L i gi i. Ta c cos 5 = 16 cos5 20 cos3 + 5 cos . V i cc gi tr = 5o , = 77o , = 149o , = 221o , = 293o , th cos 5 u b ng cos 25o . Do cos 5o , cos 77o , cos 149o , cos 221o , cos 293o ,

l cc nghi m c a a th c P (x) = 16x5 20x3 + 5x cos 25o . Theo nh l Vite, ta c S = 0. V d 2.22. Ch ng minh r ng
3

cos

2 + 7

cos

4 + 7

cos

8 = 7

1 3 (5 3 7). 2

L i gi i. Nh n xt r ng xk = cos 2k 2k + i sin (k = 0, 1, . . . , 6) 7 7

l cc nghi m c a phng trnh x7 = 1. T suy ra xk = cos 2k 2k + i sin (k = 0, 1, . . . , 6) 7 7

2.2. Tnh gi tr c a m t s bi u th c l ng gic

49

l cc nghi m c a phng trnh x6 + x5 + + x + 1 = 0 v ng th i cng l nghi m c a phng trnh 1 x+ x T suy ra yk = xk + 1 2k (k = 1, 2, 3) = xk + xk = 2 cos xk 7


3

1 + x+ x

2 x+

1 x

1 = 0.

l cc nghi m c a phng trnh y 3 + y 2 2y 1 = 0. Nhng v cos nn ta c th thay cos 8 6 = cos 7 7

6 8 b i cos . 7 7 L p phng trnh b c ba c cc nghi m l


3

2 cos

2 , 7

2 cos

4 , 7

2 cos

8 7

r i d a theo cc h th c gi a cc nghi m v cc h s c a phng trnh m tnh c t ng c n thi t. M t cch t ng qut, n u , , l cc nghi m c a phng trnh x3 + ax2 + bx + c = 0 cn 3 , 3 , 3 l cc nghi m c a phng trnh x3 + Ax2 + Bx + C = 0 th (A)3 = =+++3 3+ 3 = a 3AB 3 C, 3 +
3

hay 3 A3 = a + 3AB 3 C.

50

Chng 2. S ph c v bi n ph c trong l ng gic

Tng t ta cng tm c B 3 = b + 3ABC 3C 2. Trong tr ng h p c a bi ton cho th a = 1, b = 2, c = 1, C = 1. Do , ta c A3 = 3AB + 4 B 3 = 3AB 5. t AB = z v nhn v v i v c a hai ng th c ny, ta c z 3 + 9z 2 + 27z + 20 = 0. Suy ra z = Vy
3

3 7 3. Do A = 2 + 7

3 3 7 5. 2 = 7 1 3 53 7 . 2

2 cos

4 cos

2 + 7

8 cos

Nh n xt. B ng phng php tng t , ta c i) 2 4 8 3 3 3 1 3 + cos + cos = 3 96 , 9 9 9 2 cos n 2 4 ii) = 1 Cn tan2 + Cn tan4 + A, cos
3

cos

trong A= (1) 2 tann v i n ch n, n1 n1 (1) 2 Cn tann1 v i n l .


n

iii) trong

sin n 1 3 5 = Cn tan Cn tan3 + Cn tan5 + + A, cos


n

A=

n1 (1) 2 +1 Cn tann1 , v i n ch n n+1 (1) 2 tann , v i n l .

2.3. D ng ph c c a b t ng th c Cauchy

51

2.3

D ng ph c c a b t ng th c Cauchy

Ta c nh n xt r ng t m t ng th c cho i v i b s th c ta u c th m r ng (theo nhi u cch th c khc nhau) thnh m t ng th c m i cho b s ph c tng ng. Ch ng h n, ta c th coi m i s th c a cho nh l ph n th c c a m t s ph c z = a + ib v i b R. Ta nu m t s ng nh t th c v sau c n s d ng. V d 2.23. V i m i b s (aj ; bj ; uj ; vj ), ta lun c ng th c sau:
n n n n

aj uj
j=1 j=1

bj vj
j=1

aj bj
j=1

uj vj (2.1)

=
1 j<k n

(aj bk bj ak )(uj vk uk vj ).

Nh n xt r ng, t ng nh t th c ny ta thu c ng nh t th c Lagrange sau y i v i b s ph c. V d 2.24. V i m i b s ph c (aj ; bj ), ta lun c ng th c sau


n n n

|aj |
j=1

2 j=1

|bj |
j=1

aj bj =
1 j<k n

|aj bk ak bj |.

(2.2)

Ch ng minh. T ng th c (2.1), b ng cch thay aj b i aj , vj b i bj v uj b i aj , ta s thu c (2.2). H th c (2.2) cho ta b t ng th c Cauchy sau y i v i b s ph c. H qu 2.1. V i m i b s ph c (aj ; bj ), ta lun c b t ng th c sau
n n n

|aj |
j=1

2 j=1

|bj |

2 j=1

aj bj .

(2.3)

Gi s ta c b n c p s dng (ak ; bk ) sao cho ak [; ], > 0, k = 1, 2, . . . , n. bk

52

Chng 2. S ph c v bi n ph c trong l ng gic

Khi , theo nh l o v d u c a tam th c b c hai th hay a2 + b2 k k T y suy ra


n n n

ak bk

ak bk

0,

( + )ak bk , k = 1, 2, . . . , n.

a2 k
k=1

+
k=1

b2 k

( + )
k=1

ak bk .

Theo b t ng th c Cauchy, th
n
1 2

1 2

a2 k
k=1

k=1

b2 k

1 2

a2 + k
k=1 k=1

b2 k

V y nn
n
1 2

1 2

a2 k
k=1

k=1

b2 k

1 ( + ) 2

ak bk .
k=1

T y, ta thu c b t ng th c o Cauchy. nh l 2.1. Gi s ta c b n c p s dng (ak ; bk ) sao cho ak [; ], > 0, k = 1, 2, . . . , n. bk Khi


n
1 2

1 2

a2 k
k=1 k=1

b2 k

A G

ak bk ,
k=1

trong A= + , G= 2 .

Nhn chung, c r t nhi u b t ng th c nh n c t cc ng nh t th c. V v y, vi c thi t l p c cc ng nh t th c c coi nh m t phng php h u hi u sng tc v ch ng minh b t ng th c.

2.3. D ng ph c c a b t ng th c Cauchy

53

V d 2.25. Ch ng minh r ng v i m i b ba s (x; y; z), ta lun c ng th c sau (2x + 2y z)2 + (2y + 2z x)2 + (2z + 2x y)2 = 9(x2 + y 2 + z 2 ). Hy t ng qut ho? V d 2.26. Ch ng minh r ng v i m i b b n s (x; y; z; t), ta lun c ng

th c sau (x+y+zt)2+(y+z+tx)2+(z+t+xy)2 +(t+x+yz)2 = 4(x2 +y 2 +z 2 +t2 ). Hy t ng qut ho? V d 2.27. Ch ng minh r ng v i m i b s (uk ; vk ; pk ), ta lun c ng th c sau
n n n

(uk vj + uj vk )pj pk = 2
j,k=1 k=1

uk pk
k=1

vk pk

V d 2.28. Ch ng minh r ng v i m i b s (uk ; vk ; pk ), ta lun c ng th c sau


n n

(uj vj + uk vk )pj pk = 2
j,k=1 k=1

uk vk pk

Ti p theo, ta xt m t s m r ng khc (d ng ph c) c a b t ng th c Cauchy. nh l 2.2 (N.G.de Bruijn). V i b s th c a1, . . . , an v b s ph c (ho c th c) z1, . . . , zn , ta u c


n 2

ak zk
k=1

1 2

n 2 zk

|zk | +
k=1 k=1

a2 k
k=1

ng th c x y ra khi v ch khi ak = Re (zk ) (k = 1, . . . , n), trong l s


n

ph c v
k=1

2 2 zk l s th c khng m.

54

Chng 2. S ph c v bi n ph c trong l ng gic

Ch ng minh. B ng cch th c hi n ng th i php quay quanh g c to i v i cc zk cng m t gc, ta thu c


n

ak zk
k=1

0.

R rng php quay ny khng nh h ng n gi tr c a modul cc s .


n n

ak zk ,
k=1 k=1

2 zk ,

|zk | (k = 1, . . . , n).

V y ch c n ch ng minh cho tr ng h p
n

ak zk
k=1

0.

N u ta t zk = xk + iyk
n 2

(k = 1, . . . , n), th
n 2 n n

ak zk
k=1

=
k=1

ak xk
k=1

a2 k
k=1

x2 k

V
2 2x2 = |zk |2 + Re zk , k

ta nh n c
n 2

ak zk
k=1

1 2

a2 k
k=1 k=1

|zk | +
k=1

2 Re zk

T b t ng th c ny v
n 2 Re zk k=1 n n 2 zk k=1 k=1 2 zk

= Re

ta thu c i u c n ch ng minh.

2.4

T ng v tch sinh b i cc a th c l ng gic


n

V d 2.29. Tnh t ng cos(kx).


k=0

2.4. T ng v tch sinh b i cc a th c l ng gic

55

L i gi i. Xt cc t ng A = cos x + cos 2x + + cos nx, B = sin x + sin 2x + + sin nx. Ta c 1 + A + iB = 1 + (cos x + i sin x) + (cos 2x + i sin 2x) + + (cos nx + i sin nx) = 1 + (cos x + i sin x) + (cos x + i sin x)2 + + (cos x + i sin x)n 1 cos(n + 1)x + i sin(n + 1)x 1 (cos x + i sin x)n+1 = 1 (cos x + i sin x) 1 cos x i sin x =
n+1 sin n+1 x sin n+1 x i cos n+1 x x 2i sin n+1 x cos n+1 x 2 2 2 2 2 2 = sin x sin x i cos x 2 sin2 x 2i sin x cos x 2 2 2 2 2 2 n+1 sin 2 x n+1 x x n+1 x i cos x sin + i cos = sin x sin 2 2 2 2 2 sin n+1 x cos n x sin n+1 x sin n x sin n+1 x n n 2 2 2 2 2 cos x + i sin x = +i sin x 2 2 sin x sin x 2 2 2

2 sin2

= Vy

A= V d 2.30. Rt g n A = sin v i m N .

sin n+1 x cos n x 2 2 1. x sin 2

2 (m 1) sin sin 2m 2m 2m

L i gi i. Xt phng trnh x2m 1 = 0. Phng trnh ny c nghi m th c x = 1 v (2m 2) nghi m ph c. G i xk l nghi m ph c c a phng trnh v i 2k 2k k = 1, 2, . . . , 2m 2, t c xk = cos + i sin . 2m 2m 2k 2k i sin = xk , k = 1, 2, . . . , m 1. Nh n xt r ng x2mk = cos 2m 2m V y nn m1 2k 2m 2 x 1 = (x 1) + 1). (x2 2x cos 2m
k=1

56

Chng 2. S ph c v bi n ph c trong l ng gic

Do x2m 1 = x2 1 Cho x 0, ta thu c

m1

(x2 2x cos
k=1

2k + 1). 2m

m1

m=2 V y A= 2.4.1 V d r ng m

m1 k=1

sin2

k = 22(m1) A2. 2m

2m1

Ch ng minh cng th c l ng gic 2.31. Cho tam gic ABC c A = (i) OH = OIa = R 2; (ii) R = 2ra ; (iii) a2 + b2 + c2 = 7R2 . 2 4 ,B= ,C = . Ch ng minh 7 7 7

2 4 L i gi i. Ta c a = 2R sin , b = 2R sin , c = 2R sin . 7 7 7 Ti p theo ta tnh OH. OH 2 = 9R2 (a2 + b2 + c2) = 9R2 4R2 sin2 = 9R2 4R2 = 9R2 4R2 Xt z = cos 3 1 2 2 3 1 + 2 2 cos cos 2 4 + sin2 + sin2 7 7 7

2 4 8 + cos + cos 7 7 7 3 5 + cos + cos 7 7 7 .

+ i sin , ta thu c 7 7 z + z3 + z5 = 1 z 1 z7 z = 2 = 21 z z 1 1z

Tch ph n th c hai v , ta c cos 3 5 1 + cos + cos = 7 7 7 2

2.4. T ng v tch sinh b i cc a th c l ng gic

57

V y nn OH 2 = 9R2 7R2 = 2R2 hay OH = OIa = R 2. Ti p theo, tnh OIa . Ta c 2 4 sin sin abc 2 7 7 7 OIa = R2 + = R2 + 4R2 2 4 b+ca + sin sin sin 7 7 7 sin Do 2 3 5 7 4 + sin sin = sin + sin sin + sin 7 7 7 7 7 7 7 4 4 3 2 4 2 sin cos 2 sin cos = 4 sin sin sin , 7 7 7 7 7 7 7 2 2 2 2 nn OIa = R + R = 2R hay OIa = R 2. S abc , ra = , suy ra Ta s d ng cc cng th c R = 4S pa sin Rra = hay R = 2ra . Ti p theo, theo cu (i) ta c a2 + b2 + c2 = 4R2 sin2 V d 2.32. Ch ng minh r ng
m

abc R2 abc = = 4(p a) 2(b + c a) 2

2 4 + sin2 + sin2 7 7 7

= 4R2

7 = 7R2 . 4

22m cos2m x =
k=0

k C2m cos 2(m k)x.

L i gi i. Ta c cos x =

e +e 2

ix

ix

Do

22m cos2m x = (eix + eix )2m


2m 2m k C2m (eix )k (eix )2mk = k=0 m1 2m k C2m e2(km)ix + k=0 m1 k=m+1 m k C2m k=0 m C2m k m C2m e2(km)ix + C2m k=0 k C2m e2(km)ix

cos 2(m k)x +

cos 2(m m)x =


k=0

k C2m cos 2(m k)x.

58

Chng 2. S ph c v bi n ph c trong l ng gic

V d 2.33. Cho c p s c ng {an } v i cng sai d. Tnh cc t ng


n n

Sn =
k=1

sin ak , Tn =
k=1

cos ak .

L i gi i. - N u d = 2k (k Z) th Sn = n sin a1 ; d - N u d = 2k (k Z) th sin = 0. ta c 2 2 sin an sin d d = 2 sin[a1 + (n 1)d] sin 2 2 3 1 = cos a1 + n d cos a1 + n 2 2 3 2 d , ta c d = g(n) g(n + 1). 2

d .

Xt g(n) = cos a1 + n

2 sin an . sin Vy

2 sin a . sin d = g(1) g(2), 1 2 d 2 sin a2. sin = g(2) g(3), . . . . . . . . . 2 2 sin an . sin d = g(n) g(n + 1). 2 C ng cc ng nh t th c theo v , ta c 2Sn sin d d = g(1) g(n + 1) = cos a1 2 2 = 2 sin a1 + cos a1 + n 1 2 d

n n1 d sin d . 2 2

Do sin a1 + Sn = Theo cch gi i nh trn, ta thu c

n n1 d sin d 2 2 . d sin 2

N u d = 2k (k Z) th Tn = n cos a1 ;

2.4. T ng v tch sinh b i cc a th c l ng gic

59

N u d = 2k (k Z) th cos a1 + Tn = n n1 d sin d 2 2 . d sin 2

Ch 2.1. Nh v y, v i m i m t c p s c ng, ta tm c m t cng th c tnh t ng tng ng. Ch ng h n, v i x = l (l Z) ta c


n

Tn =
k=1

cos(2k 1)x cos x + n n1 .2x . sin .2x cos nx. sin nx sin 2nx 2 2 = = . 2x sin x 2 sin x sin 2

Nh n xt r ng, v i nh ng gi tr c a x sao cho sin 2nx = sin x (sin x = 0) th 1 ta lun c Tn = . T , ta thu c m t s k t qu sau : 2 V i n = 2, ch n x = , ta c 5 cos V i n = 3, ch n x = , ta c 7 cos V i n = 4, ch n x = 3 5 1 + cos + cos = . 7 7 7 2 3 1 + cos = . 5 5 2

, ta c 9 3 5 7 1 + cos + cos + cos = . 9 9 9 9 2

cos

V d

2.34. Tnh t ng
n n

Sn =
k=1

k sin kx,

Tn =
k=1

k cos kx v i x = 2k (l Z).

60

Chng 2. S ph c v bi n ph c trong l ng gic

L i gi i. Tr c h t, ta nh c l i r ng (Bi ton 2.40)


n

sin sin kx =

k=1

n n+1 x sin x 2 2 x sin 2

cos cos kx =

k=1

n n+1 x sin x 2 2 . x sin 2


n

Ta c
n n

Sn =
k=1 n

k. sin kx =
k=1 n

[(cos kx) ] =
k=1 n

cos kx

Tn =
k=1

k cos kx =
k=1

[(sin kx) ] =
k=1

sin kx

T suy ra cc cng th c c n tm. V d 2.35. Ch ng minh r ng v i m i s t nhin n u t n t i a th c P (x) b c n tho mn h th c sin(n + 1)t = sin t P (cos t) t R. Tnh t ng cc h s c a a th c ny. L i gi i. Ta ch ng minh b ng quy n p theo n d a vo h th c truy ton sin(n + 1)x sin(n 1)x = 2 cos nx sin x. r ng, t ng cc h s c a P (x) b ng P (1). L y o hm hai v c a (2.4) ta c (n + 1) cos(n + 1)t = cos t P (cos t) + sin t P (cos t)( sin t). V khi cos x = 1 th sin x = 0 v cos mx = 1 v i m i m N nn P (1) = n + 1. (2.4)

2.4. T ng v tch sinh b i cc a th c l ng gic

61

V d

2.36. Tnh t ng
n

Sn =
k=1

3k1 . sin3

x . 3k

L i gi i. Xu t pht t h th c 1 sin3 a = (3 sin a sin 3a), 4 ta tnh c Sn = V d 2.37. Tnh t ng


n

1 n x 3 . sin n sin x . 4 3

Sn =
k=1

arctan

2k . 2 + k2 + k4

L i gi i. t 1 + n2 + n4 = xy, 2n = x + y. Khi x = n2 + n + 1, y = (n2 n + 1). Ta c arctan 2n x+y = arctan x + arctan y = arctan 2 + n2 + n4 1 xy = arctan(n2 + n + 1) arctan(n2 n + 1), v xy < 1.

Vy Sn = arctan 3 arctan 1 + arctan 7 arctan 3 + + arctan(n2 + n + 1) arctan(n2 n + 1) = arctan(n2 + n + 1) . 4 V d 2.38. Cho c p s c ng {an } v i cng sai d. Tnh t ng
n

Sn =
k=1

k sin ak .

62
n

Chng 2. S ph c v bi n ph c trong l ng gic

L i gi i. Xt Bn =
k=1 n1

sin ak . Ta c
n1

Sn =
k=1

[k (k + 1)]Bk + n.Bn =
k=1

Bk + n.Bn

Ta c N u d = 2k (k Z), th Sn = N u d = 2k (k Z) th n1 cos a d cos a + k 1 d ( 1 2) [ 1 ( 2) ] Sn = 2 sin d +


2 k=1 n sin(a1 + n1 d) sin( n d) 2 2

n(n 1) sin a1. 2

sin

d 2

1 = 2 sin d
2

n1

cos a1
k=1

d 2

n1

k=1

cos a1 + k

1 2

2n sin a1 + = 1 d sin 2

n n1 d . sin d 2 2
n1

d (n 1) cos a1 2

k=1

cos a1 + k

1 2

2n sin a1 + trong

n n1 d sin d 2 2

d n2 + d sin d 2 2 cos a1 + kd = d 2 k=1 sin 2 Nh n xt 2.1. Tng t ta cng tnh c cc t ng


n1

cos a1

n1 d 2

Tn =
k=1

k cos ak , Un =
k=1

ak sin bk , Vn =
k=1

ak cos bk .

trong {an } v {bn } l hai c p s c ng. V d 2.39. Tnh t ng


n n

Sn =
k=1

q sin( + k), Tn =
k=1

q k cos( + k),

2.4. T ng v tch sinh b i cc a th c l ng gic

63

trong q, , l cc s th c cho tr c. L i gi i. Ta c Tn + iSn = (cos + i sin ) + q[cos( + ) + i sin( + )] + + q n [cos( + n) + i sin( + n)] = (cos + i sin )[1 + q(cos + i. sin ) + + q n (cos n + i. sin n)] = (cos + i sin )[1 + q + + (q )n], v i = cos + i sin . T suy ra (q )n+1 1 q 1 n+1 1)(q 1) ((q ) = (cos + i sin ) (q 1)(q 1) n+2 q [cos(n + ) + i. sin(n + )] q[cos(n ) + i. sin(n )] = 1 2q cos + q 2 q n+1 {cos[(n + 1) + ] + i. sin[(n + 1) + ]} + cos + i. sin + 1 2q cos + q 2 cos q cos(n ) q n+1 cos[(n + 1) + ] + q n+2 cos(n + ) = 1 2q cos + q 2 sin q sin(n ) q n+1 sin[(n + 1) + ] + q n+2 sin(n + ) +i . 1 2q cos + q 2 Tn + iSn = (cos + i sin ) Vy Sn = v Tn = cos q cos(n ) q n+1 cos[(n + 1) + ] + q n+2 cos(n + ) . 1 2q cos + q 2
n

sin q sin(n ) q n+1 sin[(n + 1) + ] + q n+2 sin(n + ) 1 2q cos + q 2

Nh n xt 2.2. B ng phng php tng t , ta tnh c cc t ng sau : i) Vn =


k=1

ak sin( + k),

64
n

Chng 2. S ph c v bi n ph c trong l ng gic

ii) Un =
k=1 n

ak cos( + k),
n

iii) wn =
k=1

ak sin bk , Rn =
k=1

ak cos bk ,

trong {an } l c p s nhn v i cng b i q = 1 v {bn } l c p s c ng v i cng sai d. 2.4.2 T ng v tch cc phn th c c a bi u th c l ng gic

Ch r ng, trong m t s tr ng h p, tnh t ng h u h n cc phn th c l ng gic, ng i ta th ng s d ng m t s tnh ch t c a a th c, c bi t l cng th c n i suy Lagrange. D i y l m t s nh l v p d ng. nh l 2.3 (Cng th c n i suy Lagrange). N u x1 , x2, . . . , xm l m gi tr tu i m t khc nhau v f (x) l a th c b c nh hn m th ta c ng nh t th c f (x) = f (x1 ) (x x2)(x x3) . . . (x xm ) (x1 x2)(x1 x3) . . . (x1 xm ) (x x1)(x x3) . . . (x xm ) + + f (x2 ) (x2 x1)(x2 x3) . . . (x2 xm ) (x x1)(x x2) . . . (x xm1 ) . + f (xm ) (xm x1 )(xm x2 ) . . . (xm xm1 )

Ch ng minh.

Ta c n ch ng minh (x x2 )(x x3 ) . . . (x xm ) (x1 x2 )(x1 x3) . . . (x1 xm ) (x x1 )(x x3 ) . . . (x xm ) f (x2 ) (x2 x1 )(x2 x3) . . . (x2 xm ) (x x1)(x x2) . . . (x xm1 ) 0. f (xm ) (xm x1 )(xm x2) . . . (xm xm1 )

f (x) f (x1 )

V tri c a ng th c l m t a th c b c khng v t qu m 1 v c m nghi m x1 , x2, . . . , xm . V y a th c ng nh t b ng 0. T suy ra i u ph i ch ng minh.

2.4. T ng v tch sinh b i cc a th c l ng gic

65

H qu 2.2. c cc ng nh t th c sau y : Ta (x 3)(x 5)(x 7) (x 2)(x 5)(x 7) + i) ( 2 3)( 2 5)( 2 7) ( 3 2)( 3 5)( 3 7) (x 2)(x 3)(x 5) (x 2)(x 3)(x 7) + 1, + ( 5 2)( 5 3)( 5 7) ( 7 2)( 7 3)( 7 5) ii) a2 (x b)(x c) (x c)(x a) (x a)(x b) + b2 + c2 = x2 . (a b)(a c) (b c)(b a) (c a)(c b)

nh l 2.4. N u f (x) l a th c b c khng v t qu m2 v x1, x2, . . . , xm l m gi tr i m t khc nhau tu , th ta c ng nh t th c f (x1 ) f (x2 ) + (x1 x2 )(x1 x3) . . . (x1 xm ) (x2 x1)(x2 x3) . . . (x2 xm ) + + Ch ng minh. f (xm ) = 0. (xm x1)(xm x2) . . . (xm xm1 )

V tri c a ng th c chnh l h s c a h ng t b c m 1

trong a th c f (x) cho. ng nh t cc h s c a cc ly th a cng b c ta c ngay i u ph i ch ng minh. V d S= 2.40. Tnh t ng cos 1o cos 2o + (cos 1o cos 2o )(cos 1o cos 3o ) (cos 2o cos 1o )(cos 2o cos 3o ) cos 3o . + (cos 3o cos 1o )(cos 3o cos 2o ) L i gi i. S d ng nh l 1, v i f (x) = x, x1 = cos 1o , x2 = cos 2o , x3 = cos 3o th S = 0. V d 2.41. Cho c p s c ng {an } v i cng sai d, v i d, a1, a2, . . . , an khc
n

b i c a . Tnh t ng Sn =
k=1

1 . sin ak sin ak+1

66

Chng 2. S ph c v bi n ph c trong l ng gic

L i gi i. Ta c cot an cot an+1 = Suy ra 1 1 (cot an cot an+1 ). = sin an sin an+1 sin d Vy Sn = 1 (cot a1 cot a2 + cot a2 cot a3 + . . . + cot an cot an+1 ) sin d sin(an+1 a1) 1 1 (cot a1 cot an+1 ) = = sin d sin d sin a1 sin an+1 sin nd 1 . = sin d sin a1 sin(a1 + nd) sin nd . sin d sin a1 sin(a1 + nd) sin(an+1 an ) sin d = . sin an sin an+1 sin an sin an+1

Vy Sn =

V d 2.42. Cho c p s c ng {an } v i cng sai d, trong d = l ; a1 , a2, . . . , an = + l (l Z). Tnh t ng 2


n

Tn =
k=1

1 cos ak cos ak+1

L i gi i. Ta c tan an+1 tan an = Suy ra 1 1 (tan an tan an+1 ). = cos ak cos ak+1 sin d Vy Tn = 1 ( tan a1 + tan a2 tan a2 + tan a3 . . . tan an + tan an+1 ) sin d 1 = ( tan a1 + tan an+1 ). sin d sin(an+1 an ) sin d = . cos an cos an+1 cos an cos an+1

2.4. T ng v tch sinh b i cc a th c l ng gic

67

V d

2.43. Tnh t ng
n

Sn =
k=1

2k1 tan2

x x tan k1 k 2 2

L i gi i. Ta c tan 2a = Suy ra tan2 a tan 2a = tan 2a 2 tan a. Vy Sn = 20 tan2 x x x x x x tan 0 + 21 tan2 2 tan 1 + + 2n1 tan2 n tan n1 1 2 2 2 2 2 2 x x x x x x 0 1 n1 tan n1 2 tan n = 2 tan 0 2 tan 1 + 2 tan 1 2 tan 2 + + 2 2 2 2 2 2 2 x n = tan x 2 tan n . 2 2.44. Tnh t ng
n

2 tan a . 1 tan2 a

V d

Tn =
k=1

1 2k1

tan

2k

x . 1

L i gi i. Ta c tan a = cot a 2 cot 2a. Suy ra tan2 a. tan 2a = tan 2a 2 tan a. Vy Tn = 20 tan2 x x x x x x tan 0 + 21 tan2 2 tan 1 + + 2n1 tan2 n tan n1 21 2 2 2 2 2 x x x x x x 0 1 n1 tan n1 2 tan n = 2 tan 0 2 tan 1 + 2 tan 1 2 tan 2 + + 2 2 2 2 2 2 2 x n = tan x 2 tan n . 2

68

Chng 2. S ph c v bi n ph c trong l ng gic

2.5

B t ng th c l ng gic

Trong ph n ny, ta xt m t s b t ng th c lin quan n bi u th c (hm s ) l ng gic. V d (n 2.45. Ch ng minh r ng t p gi tr c a m i a th c l ng gic b c n 1), khng ch a s h ng t do (t c a0 = 0) An (x) = a1 cos x + b1 sin x + + an cos nx + bn sin nx v i a2 + b2 > 0 n n ch a c gi tr dng v gi tr m. L i gi i. V a2 + b2 > 0 nn t n t i m t gi tr x0 sao cho An (x0 ) = 0. n n M t khc v i x = x0 ta c (xem m c 1.1 Chng 1) An (x0 ) + An x0 + 2 n+1 + + An x0 + 2n n+1 = 0.

Do An (x0) = 0 nn t ng trn ph i ch a t nh t m t s h ng dng v m t s h ng m. H qu 2.3. T p gi tr c a m i a th c l ng gic b c n (n 1) d ng

An (x) = a0 + a1 cos x + b1 sin x + + an cos nx + bn sin nx (a2 + b2 > 0) n n ch a c gi tr l n hn a0 v gi tr nh hn a0. H qu 2.4. M i a th c l ng gic b c n (n do An (x) = a1 cos x + b1 sin x + + an cos nx + bn sin nx lun c t nh t m t nghi m th c. Ch ng minh. Ta th y An (x) lun nh n c gi tr dng v gi tr m. Hn n a, An (x) l m t hm s lin t c trn R. T suy ra t n t i t nh t m t gi tr x0 An (x0) = 0. 1), khng ch a s h ng t

2.5. B t ng th c l ng gic

69

V d

2.46. V i n l m t s t nhin v x

0;

. 2(n + 1)

Ch ng minh r ng (1 cosn x)(1 + cosn x) < tan nx tan x. L i gi i. T 0 < x(n + 1) < 0 < nx < , suy ra 2 (2.5)

, 0<x< v cos nx > 0, cos x > 0. 2 2

Ta c (1 cosn x)(1 + cosn x) < tan nx tan x (2.5) 1 cos2n x < hay cos(n + 1)x < cos2n+1 x cos nx. D dng ch ng minh c (2.6) b ng phng php quy n p theo n. Vy (1 cosn x)(1 + cosn x) < tan nx tan x v i x 0; V d 2.47. V i n l m t s t nhin v x 0; , n N. 2(n + 1) (2.6) sin nx sin x , cos nx cos x

. 2(n + 1)

Ch ng minh r ng (1 cosn x)(1 + cosn x) < tan nx sin x. L i gi i. Ta c (2.7) 1 cos2n x < tan nx sin x tan nx sin x + cos2n x > 1. K hi u f (n) = tan nx sin x + cos2n x. Ta ch ng minh f (k + 1) > f (k), v i k = 0; . . . ; n 1. (2.7)

70

Chng 2. S ph c v bi n ph c trong l ng gic

Th t v y, ta c tan kx sin x + cos2k x < tan(k + 1)x sin x + cos2n+2 x cos2k x cos2n+2 x < sin x[tan(k + 1)x tan kx] sin x cos2k x sin2 x < sin x . cos(k + 1)x cos kx , nn cos(k + 1)x.coskx > 0. 2(n + 1 V v y cos2k x cos(k + 1)x cos kx < 1, i u ny lun ng. Do x 0; Vy f (n) > f (n 1) > > f (1) > f(0) = 1. Do tan nx sin x + cos2n x > 1. Vy (1 cosn x)(1 + cosn x) < tan nx sin x v i x V d 2.48. Ch ng minh r ng (n + 1) cos L i gi i. V i m i n n cos > 1, v i m i n n+1 n 2. 0; 2(n + 1) , n N.

2, ta c

n cos > 1 n+1 n n cos cos > 1 cos n+1 n n+1 (2n + 1) n sin sin > sin2 2n(n + 1) 2n(n + 1) 2(n + 1) (n + 1) cos v sin 2n (2n + 1) > sin > sin . 2n(n + 1) 2n(n + 1) 2(n + 1) (2.8)

2.5. B t ng th c l ng gic

71

M t khc, b ng phng php quy n p theo n, ta d dng ch ng minh c sin T (2.8) v (2.9), ta suy ra n sin Vy (n + 1) cos V d n cos > 1, v i n n+1 n 2. (2n + 1) sin > sin2 . 2n(n + 1) 2n(n + 1) 2(n + 1) > sin . 2n(n + 1) 2(n + 1) (2.9)

2.49. Ch ng minh r ng 2| sin x| + 2| cos x| 3, x R. . Khi b t ng 4

L i gi i. Khng m t tnh t ng qut, c th coi x 0; th c cho c d ng 2sin x + 2cos x 3, x 0; . 4

Theo b t ng th c gi a trung bnh c ng v trung bnh nhn, ta c 2sin x + 2cos x = 2sin x + Do x 0; nn 4 sin x + 2 cos x 2 v v v y 2sin x + 2cos x ng th c x y ra, ch ng h n, khi x = V d 3, x 0; k , k Z. 2 . 4 0, 1 cos x 1 cos x 2 + 2 2 2 3 3 2sin x+2 cos x2 .

2.50. Xc nh s dng a sao cho acos 2x 2 cos2 x, x R. (2.10)

72

Chng 2. S ph c v bi n ph c trong l ng gic

L i gi i. t cos 2x = t th t [1; 1] v (2.10) c d ng at V i0<t 1 + t, t [1; 1]. (2.11)

1 th (2.11) c d ng a (1 + t) t , t (0; 1]. e.


1

(2.12)

Trong (2.12) cho t 0, ta thu c a Tng t , v i 1

t < 0 th (2.11) c d ng a (1 + t) t , t [1; 0). e.


1

(2.13)

Trong (2.13) cho t 0, ta thu c a K t h p, ta thu c a = e. V i a = e th ta lun c et nn ecos 2x

1 + t, t R,

2 cos2 x, x R.

V d 2.51. Cho a th c l ng gic f (x) = b1 sin x + b2 sin 2x + + bn sin nx tho mn i u ki n |f (x)| Ch ng minh r ng |b1 + 2b2 + 3b3 + + nbn | L i gi i. Ta c f (x) = b1 cos x + 2b2 cos 2x + 3b3 cos 3x + + nbn cos nx. 1. | sin x|, v i m i x R, bi R, i = 1, 2, ..., n.

2.5. B t ng th c l ng gic

73

V y nn f (0) = b1 + 2b2 + 3b3 + + nbn . Theo nh ngha c a o hm t i i m x = 0 th f (0) = lim Suy ra |f (0)| = lim Vy |b1 + 2b2 + 3b3 + + nbn | V d 1.
x0

f (x) f (0) f (x) = lim . x0 x0 x x

f (x) x

x0

lim

f (x) x

x0

lim

sin x = 1. x

2.52. Cho cc s th c a, b, c, d. Ch ng minh r ng n u v i m i x R, c2 + d2

ta u c a cos x + b sin x + c cos 2x + d sin 2x th a = b = 0. L i gi i. N u c2 + d2 = 0 th k t qu l hi n nhin. V v y ta gi thi t r ng r = c2 + d2 > 0. d c L y gc (0 < ) sao cho cos 2 = , sin 2 = . Khi ta c r r a cos x + b sin x + c cos 2x + d sin 2x = a cos x + b sin x + r cos 2(x ) r , x R.

Thay x b i x + vo b t ng th c trn, ta thu c r cos 2x + A cos x + B sin x v i A = a cos + b sin v B = b cos a sin . Trong (2.14) cho x = 0 v x = , ta c A = 0. r , x R, (2.14)

74

Chng 2. S ph c v bi n ph c trong l ng gic

T suy ra r cos 2x + B sin x r, x R r, x R

r(1 2 sin2 x) + B sin x sin x(2r sin x B) N u B = 0, ta ch n x0 sao cho 0 < sin x0 <

0, x R. B . Khi 2r

sin x0 (2r sin x0 B) < 0 (mu thu n). V y B = 0, nn a cos + b sin = 0 b cos a sin = 0 a=0 b = 0.

V d 2.53. Cho cc s th c a, b, A, B. Xt a th c l ng gic f (x) = 1 a cos x b sin x A cos 2x B sin 2x. Ch ng minh r ng n u f (x) 0, x R th a2 + b2 2 v A2 + B 2 1.

L i gi i. t r = a2 + b2 , R = A2 + B 2. Ch n , sao cho a= A= Suy ra f (x) = 1 Ta c 2 =1 r R cos 2 + f + 4 2 2 f =1 r R cos 2 4 2 4 . 4 r cos(x ) R cos 2(x ) 0, x R. r cos ; b = r sin ; a cos x + b sin x = r cos(x );

R cos 2 ; B = R sin 2 ; A cos 2x + B sin 2x = R cos 2(x ).

2.5. B t ng th c l ng gic

75

2 < 0. M t khc, th 2 = cos 2 + = cos 2 , cos 2 + 4 4 4 nn m t trong hai s f + ho c f ph i c m t s m (mu 4 4 thu n). V y r 2. N u r > 2 th 1 r Ta l i c r cos( ) R, f ( + ) = 1 r cos( + ) R. f () = 1 N u R > 1 th 1 R < 0. M cos( ) = cos( + ), nn m t trong hai s f () ho c f ( + ) ph i c m t s m (mu thu n). VyR V d 1. 2.54. Cho a th c l ng gic f (x) = 1 + a cos x + b cos 2x + cos 3x. Ch ng minh r ng n u f (x) 0, x R th a = b = 0.

L i gi i. Cho x = ta c f () = 1 a + b 1 = a + b 0. a b 1 Cho x = ta c f = 1 + 1 = (a b) 0 a = b. 3 3 2 2 2 Ta c f (x) = 1 + cos 3x + a(cos 2x + cos x) = 1 + 4 cos3 x 3 cos x + a(2 cos2 x 1 + cos x) = (cos x + 1)(4 cos2 x 4 cos x + 1) + a(cos x + 1)(2 cos x 1) = (cos x + 1)(2 cos x 1)(2 cos x 1 + a) 0 , x R.

76

Chng 2. S ph c v bi n ph c trong l ng gic

N u a = 0 th f (x) = (cos x + 1)(2 cos x 1)2 N u a < 0, ch n x sao cho 0 , x R, i u ny ng.

1a 1 < cos x < th f (x) < 0 (mu thu n). 2 2 1a 1 th f (x) < 0 (mu thu n). N u a > 0, ch n x sao cho > cos x > 2 2 V y a = b = 0.

2.6
V d

c trng hm c a hm s l ng gic
2.55. Tm hm f : Z R tho mn cc i u ki n f (1) = a v i

a [1; 1] cho tr c v f (x + y) + f (x y) = 2f (x)f (y), x, y Z. L i gi i. Cho x = y = 0 ta c 2f (0) = 2f 2 (0) nn f (0) = 0 ho c f (0) = 1. N u f (0) = 0 th cho y = 0, x Z ta c 2f (x) = 0 nn f (x) 0. N u f (0) = 1 th cho x = y = 1, ta c f (2) + 1 = 2f 2 (1) nn f (2) = 2f 2 (1) 1 = 2a2 1 = cos(2 arccos a) (xem m c 5.1). Cho x = 2, y = 1 ta c f (3) + f (1) = 2f (2)f (1) f (3) = 2(2a2 1)a a = 4a3 3a = cos(3 arccos a).

Ch ng minh quy n p theo n ta c f (n) = cos(n arccos a), n N. Cho x = 0, y Z ta c f (y) + f (y) = 2f (y) nn f (y) = f (y). Do f (x) = cos(x arccos a), x Z. Th l i ta th y f (x) = cos(x arccos a) tho mn i u ki n bi ton. V y f (x) = cos(x arccos a), x Z. V d 2.56. Cho a, a1, a2, . . . , an l cc s th c. T n t i hay khng t n t i Pn (x) = xn + a1 xn1 + + an1 x + an

m t a th c

2.6. c trng hm c a hm s l ng gic

77

tho mn i u ki n |Pn (x)| a, x [a; a]?

x , x [a; a]. a Ch ng minh quy n p theo n ta th y Pn (x) tho mn yu c u bi ton. L i gi i. Xt a th c Pn (x) = a cos n arccos V d 2.57. Tm a th c P (x) = a0xn + a1xn1 + + an v i a0 = 0 tho

mn i u ki n (1 x2)[P (x)]2 = n2 [1 P 2 (x)] x R. trong P (x) l o hm c a P (x). L i gi i. D th y hai a th c d ng P (x) = cos(n arccos x) v Q(x) = cos(n arccos x) tho mn i u ki n bi ton. Ta ch ng minh khng cn a th c no khc tho mn bi ra. Th t v y P 2 (1) = 1, [(1 x2)(P (x))2] = n2 [1 P 2 (x)] , x R P 2 (1) = 1, P (x)[n2P (x) xP (x) + (1 x2)P (x)] = 0, x R. Nhng P (x) ch c h u h n nghi m nn (2.15) n2 P (x) xP (x) + (1 x2)P (x) 0, x R. So snh cc h s trong (2.16) ta c an1 = 0 v k(k 2n)ank = (n k + 2)(n k + 1)ank+2 (v i 2 k n). (2.16) (2.15)

Suy ra an3 = an5 = = 0, cn an2 , an4 , ... c xc nh duy nh t theo an . M t khc, ta c


n 2

aj
j = 0

= 1

= P 2 (1)

78

Chng 2. S ph c v bi n ph c trong l ng gic

nn an ch nh n hai gi tr i nhau. V y ch c khng qu hai a th c P (x) v Q(x) tho mn yu c u bi ton. V d 2.58. Cho cj C, j = 0, 1, . . . , n ; c0 = 0, cn = 0 ; z = eit, t R. Ch ng minh r ng n u h(z) = c0 + c1 z + c2 z 2 + + cn z n th |h(z)|2 l m t a th c l ng gic b c n theo t. L i gi i. Ta c eit = cos t + i sin t nn eikt = cos kt + i sin kt, t R. t ck = ak + ibk v i ak , bk R ; k = 0, 1, . . . , n . Khi
n

h(z) =
k=0 n

(ak + ibk )(cos kt + i sin kt) [(ak cos kt bk sin kt) + i(bk cos kt + ak sin kt)],
k=0 n 2 n 2

=
2

|h(z)| =
k=0

(ak cos kt bk sin kt)


n

+
k=0

(bk cos kt + ak sin kt)

= 0 +
j=1

(j cos jt + j sin jt),

v i
n n

0 =
k=0

|ck | ; k + ik = 2
j=0

cj ckj , (k = 1, . . . , n 1).

n + in = 2c0 cn = 0 (do c0 v cn cng khc 0). V y |h(z)|2 l m t a th c l ng gic b c n theo t. V d 2.59. Ch ng minh r ng hm s f (x) = sin2p x (p l m t s t nhin) l m t a th c l ng gic theo hm s cosin.

2.6. c trng hm c a hm s l ng gic

79

L i gi i. T cng th c eix = cos x + i sin x d dng suy ra sin x = V y nn sin x = Suy ra (1)p f (x) = 2p 2 = (1) 22p +
k=p+1 p 2p k (1)k C2p eikx ei(2pk)x k=0 p1 k (1)k C2p e2ikx2ipx k=0 k (1)k C2p e2i(kp)x p1 p C2p 22p p C2p ) + 2p 2 2p

eix + eix eix eix , cos x = . 2i 2 eix eix 2i


2p

2p

(1)p = 2p 2 = (1) 22p1


p

(1)
k=0 p1

k C2p (e2i(kp)x

+e

2i(kp)x

k (1)k C2p cos 2(k p)x + k=0

p C2p . 22p

V y f (x) l m t a th c l ng gic theo csin b c 2p. V d 2.60. Tm cc hm f (x) xc nh v i m i x R v tho mn i u ki n f (0) = 2003, f = 2004, 2 f (x + y) + f (x y) = 2f (x) cos y, x, y R.

L i gi i. Trong i u ki n f (x + y) + f (x y) = 2f (x) cos y, x, y R thay x = t , y = , t R, ta thu c 2 2 f (t) + f (t ) = 2f t cos = 0. (2.17) 2 2 Ti p t c thay x = , y = t , t R, ta c 2 2 f (t) + f ( t) = 2f cos t = 2 2004 sin t. (2.18) 2 2

80

Chng 2. S ph c v bi n ph c trong l ng gic

Thay ti p x = 0, y = t , t R, ta c f (t ) + f ( t) = 2f (0) cos(t ) = 2 2003 cos t. T (2.17) v (2.18) ta suy ra 2f (t) + f (t ) + f ( t) = 2 2004 sin t. T (2.19) v (2.20) ta suy ra 2f (t) 2 2003 cos t = 2 2004 sin t nn f (t) = 2003 cos t + 2004 sin t. Th l i ta th y f (0) = 2003 cos 0 + 2004 sin 0 = 2003, f = 2003 cos + 2004 sin = 2004, 2 2 2 f (x + y) + f (x y) = = 2003 cos(x + y) + 2004 sin(x + y) + 2003 cos(x y) + 2004 sin(x y) = 2003 2 cos x cos y + 2004 2 sin x cos y = 2 (2003 cos x + 2004 sin x) cos y = 2 f (x) cos y. V y f (x) = 2003 cos x + 2004 sin x. V d 2.61. Tm cc hm s lin t c f : R [1; 1] tho mn i u ki n f (0) = 1, f (x + y) + f (x y) = 2f (x)f (y) v i m i x, y R. L i gi i. V f (x) lin t c trn R v f (0) = 1 > 0 nn > 0 sao cho f (x) > 0, x (; ). Trong phng trnh cho i ch x v y, ta c f (y + x) + f (y x) = 2f (y)f (x) v i m i x, y R. T suy ra f (x y) = f (y x) v i m i x, y R. Cho y = 0 ta thu c f (x) = f (x), x R. Ni cch khc, f (x) l hm s ch n. (2.21) (2.20) (2.19)

2.6. c trng hm c a hm s l ng gic

81

Ta xt cc tr ng h p sau : Tr ng h p 1. | f (x) |= 1, x R. Khi , do f (x) lin t c trn R v f (0) = 1 nn f (x) = 1, x R. Hm s ny tho mn i u ki n bi ton. Tr ng h p 2. x0 R, | f (x0 ) |< 1. Nh n xt r ng v i n0 l n v nh trn, th x0 x0 ( ; ) f n0 n0 2 2 N u n N : f x0 2n > 0.

= 1 th trong phng trnh cho ta t y = x c f (2x) = 2[f (x)]2 1, x R. (2.22)

T y c f x0 =2 f 2n1 x0 f n2 = 2 f 2 x0 f n3 = 2 f 2 x0 2 1 = 1, 2n 2 x0 1 = 1, 2n1 2 x0 1 = 1, 2n2 x0 2 f (x0 ) = 2 f 1 = 1, tri v i gi thi t. 2 x0 < 1, n N. Ch n x1 = n th x1 = 0 v 2 0 . T (7) ta c 2

V y nn f

x0 2n

0 < f (x1) < 1 v f (x) > 0, x (|x1|; |x1|). t f (x1) = cos , 0 < <

f (2x1 ) = 2[f (x1 )]2 1 = 2 cos2 1 = cos 2. Gi s f (kx1 ) = cos k, k n, n N. Khi , ta c

f ((n + 1)x1 ) = f (nx1 + x1 ) = 2f (nx1 )f (x1) f ((n 1)x1) = 2 cos n cos cos(n 1) = cos(n + 1).

82

Chng 2. S ph c v bi n ph c trong l ng gic

V y theo nguyn l quy n p, ta c f (mx1 ) = cos m, m N . Do f (x) l hm s ch n nn t k t qu trn, k t h p v i gi thi t f (0) = 1 = cos 0. Ta c f (mx1) = cos m, m Z. Trong (2.22) thay x b i x1 ta c 2 f (x1 ) = 2 f T suy ra f Gi s f x1 2k x1 2 = , k 2k 1 + f (x1 ) = 2 1 + cos = cos . 2 2 x1 2
2

(2.23)

(2.24)

1.

= cos

n, n N , ta c x1 2n 1 + cos = 2n = cos . 2 2n+1

x1 f n+1 2

1+f = 2

V y theo nguyn l quy n p th f T (2.24) v (2.25), ta c f mx1 m = cos n , n N , m Z. n 2 2 (2.26) x1 2n = cos , n N . 2n (2.25)

T (2.26) v do hm s f (x) lin t c trn R, ta c f (x1 t) = cos t, t R, hay b ng cch t x1t = x ; = a , ta thu c x1 f (x) = cos ax, x R. Th l i, ta th y hm s ny tho mn i u ki n bi ton.

2.7. Bi t p

83

2.7

Bi t p

Bi 2.1. Tm s nguyn n n u (1 + i)n = (1 i)n . Bi 2.2. Bi u di n cc s ph c sau d i d ng l ng gic 1. a = 1 + i 3l ; 3. c = 1 + cos + i sin ; 2. b =2 + 3 + i ; 1 + cos + i sin 4. d = 1 + cos i sin

Bi 2.3. Tm i u ki n Re (z1z2 ) = (Re z1)(Re z2). Bi 2.4. p d ng cng th c Moivre hy: 1. Bi u di n tan 5 qua tan . 2. Bi u di n tuy n tnh sin5 qua cc hm sin c a gc b i c a . 3. Bi u di n cos4 v sin4 cos3 qua hm cosin c a cc gc b i. Bi 2.5. Ch ng minh r ng 1. sin +sin(+)+sin(+2)+ +sin(+n) = 2. cos +cos(+)+cos(+2)+ +cos(+n) = sin (n+1) sin + 2 sin 2 sin (n+1) cos + 2 sin 2
(n) 2 (n) 2

3. 1 + a cos + a2 cos 2 + + an cos n an+2 cos n an+1 cos(n + 1) a cos + 1 = a2 2a cos + 1 4. a sin + a2 sin 2 + + an sin n an+2 sin n an+1 sin(n + 1) + a sin + 1 = a2 2a cos + 1 Bi 2.6. Ch ng minh r ng

sin(n + 1) cos n n 1 + 2 sin 2 n + 1 sin(n + 1) cos n 2. sin2 + sin2 2 + + sin2 n = 2 2 sin 1. cos2 + cos2 2 + + cos2 n =

84

Chng 2. S ph c v bi n ph c trong l ng gic

Bi 2.7. Ch ng minh r ng 4 6 2n 1 2 + cos + cos + + cos = 2n + 1 2n + 1 2n + 1 2n + 1 2 2+ 2+2 2. cos = 16 2 2 10 + 2 5 3. sin = 5 2 1. cos Bi 2.8. Ch ng minh r ng v i m i n ch n (n = 2m), ta u c
2 4 n cos n = cosn Cn cosn2 sin2 +Cn cosn4 sin4 +(1)m Cn sinn .

Bi 2.9. Ch ng minh r ng v i m i n l (n = 2m + 1), ta u c


1 3 n1 sin n = Cn cosn1 sin Cn cosn3 sin3 + +(1)m1 Cn sinn1 cos .

Bi 2.10. Ch ng minh r ng v i m i n Z, ta u c n+2 cos , 2 2 n+2 1 n1 . 2. sin + Cn sin 2 + + Cn sin n + sin n + 1 = 2n cosn sin 2 2
1 n1 1. cos +Cn cos 2+ +Cn cos n+cos n + 1 = 2n cosn

Bi 2.11. L p phng trnh m nghi m c a n l cc s 1. sin2 2 3 n , sin2 , sin2 , , sin2 ; 2n + 1 2n + 1 2n + 1 2n + 1 2 3 n 2. cot2 , cot2 , cot2 , , cot2 ; 2n + 1 2n + 1 2n + 1 2n + 1

Bi 2.12. Ch ng minh cc ng th c sau 2 n 2n + 1 1. sin sin sin = , 2n + 1 2n + 1 2n + 1 2n 2 n 1 2. cos cos cos = n. 2n + 1 2n + 1 2n + 1 2 Bi 2.13. Ch ng minh cc ng th c sau 2 (n 1) n 1. sin sin sin = n1 , 2n 2n 2n 2 2 (n 1) n 2. cos cos cos = n1 , 2n 2n 2n 2

2.7. Bi t p

85

Bi 2.14. Tm t t c cc gi tr c a 4 3i ; 1. cn b c hai 2. cn b c n c a n ; v 3i . 3. cn b c 5 c a z = 8 + 8i Bi 2.15. Ch ng minh r ng n u l m t gi tr c a cn b c n c a s ph c z C th m i gi tr khc c a cn thu c b ng php nhn v i t ng gi tr k , k = 0, 1, . . . , n 1 l cn b c n c a n v , t c l , 1 , 2 , . . . , n1 . Bi 2.16. 1. Tnh t ng S m i cn b c n c a 1. 2. Tnh t ng cc ly th a b c k c a m i cn b c n c a s ph c . Bi 2.17. Tnh t ng S = 1 + 2 + 32 + + nn1 . Bi 2.18. Cho 1 < n N, 0 = c R, gi i cc phng trnh sau: 1. (x + c)n (x c)n = 0; 2. (x + ci)n (x ci)n = 0; 3. (x + ci)n + i(x ci)n = 0; 4. (x + ci)n (cos + i sin )(x ci)n = 0, = 2k. Bi 2.19. Ch ng minh r ng n u n l b i c a 3 th 1 + i 3 2 v n u n khng chia h t cho 3 th 1 + i 3 2
n n

1 i 3 2

=2

1 i 3 2

= 1.

86

Chng 2. S ph c v bi n ph c trong l ng gic

Bi 2.20. Tnh cc bi u th c 1. 1 = 1 + 2. 2 = 1 +
1+i 2 1i 2 n

1+
n

1+i 2 1i 2

1+
2

1+i 2 1i 2

22

1 +
22

1+i 2 1i 2

2n

.
2n

1+

1+

1 +

Bi 2.21. Ch ng minh r ng cos 1. cos x + cos 2x + + cos nx = ; sin 2. sin x + sin 2x + + sin nx = ; (n + 1)x nx sin 2 2 v i x = k (k Z) x 2 sin 2

(n + 1)x nx sin 2 2 v i x = k (k Z) x 2 sin 2

3. cos x + cos 3x + + cos(2n 1) x =

sin 2nx v i x = k (k Z) ; 2 sin x sin2 nx 4. sin x + sin 3x + + sin(2n 1) x = v i x = k (k Z) ; sin x sin(2n + 1)x 5. 1 + cos 2x + cos 4x + + cos 2nx = v i x = k (k Z) 2 sin x sin nx sin(n + 1)x v i x = k (k Z). sin x

; 6. sin 2x + sin 4x + + sin 2nx = Bi 2.22. Ch ng minh r ng 1. sin x + 3 sin 3x + + (2n 1) sin(2n 1)x sin 2nx cos x 2n cos 2nx sin x = v i x = k (k Z) ; 2 sin2 x 2. cos x + 22 cos 2x + 32 cos 3x + + n2 cos nx (2n + 1)x x x x x + cos sin nx 2n sin cos nx + 2n2 sin2 sin 2 2 2 2 = 3 x 4 sin 2 k (k Z) ; v ix= 2 3. cos x + 32 cos 3x + 52 cos 5x + + (2n 1)2 cos(2n 1)x (4n2 + 1) sin 2nx sin2 x 2 sin 2nx + 2n sin 2x cos 2nx = 2 sin3 x v i x = k (k Z).

2.7. Bi t p

87

Bi 2.23. T n t i hay khng t n t i a th c d ng Pn (x) = xn + a1xn1 + + an1 x + an v tho mn |Pn (x)| 2, x [2; 2] ?

Bi 2.24. Ch ng minh hm s f (x) = cos2p x (v i p l m t s t nhin) l m t a th c l ng gic. Bi 2.25. Tm t t c cc hm s lin t c trn R v tho mn f (x)f (y) f (x + y) = sin x sin y x, y R. Bi 2.26. Ch ng minh r ng n u v i m i x R tho mn a cos x + b sin x + c cos 2x + d sin 2x th a = b = 0. Bi 2.27. Cho a th c l ng gic f (x) = 1 + a1 cos x + a2 cos 2x + a3 cos 3x. Ch ng minh r ng n u f (x) 0 x R th a1 + a2 + a3 3. c2 + d2 .

Bi 2.28. Tm gi tr l n nh t c a hm s y = 4 sin 3x 4 cos 2x 5 sin x + 5.

Chng 3

M ts ng d ng c a s ph c trong i s
3.1
3.1.1

Phng trnh v h phng trnh i s


Phng trnh b c hai

Ta nh c l i tnh ch t nghi m c a tam th c b c hai v i h s th c f (x) := ax2 + bx + c = 0, a = 0, = b2 4ac. N u < 0, phng trnh khng c nghi m th c. N u 0 th phng trnh c hai nghi m th c : x1,2

b . = 2a

Nh n xt r ng trong tr ng h p < 0, phng trnh tuy khng c nghi m b i th c nhng v n c hai nghi m ph c l cc s ph c lin h p x1,2 = . 2a Khi cc h s c a phng trnh b c hai f (x) = 0 l cc s ph c th ta v n s d ng cc php bi n i ng nh t th c nh trong tr ng h p s th c. Ta vi t af (x) = a x + 2 hay b ax + 2
2 2 2

b 2

(ax) +

b 2

b2 4ac = 0, 4

b2 4ac , = 4

88

3.1. Phng trnh v h phng trnh i s

89

V d

3.1. Cho c p s dng a, b v i a b v a + b = 1. G i un , vn l cc

nghi m c a tam th c b c hai fn (x) = x2 bn x an , n N. Ch ng minh r ng un , vn (1; 1), n N . L i gi i. Ta c = b2n + 4a2n > 0 nn cc phng trnh tng ng u c nghi m phn bi t. Theo gi thi t th fn (1) = 1 + bn an > 0, n N v fn (1) = 1 bn an > 0, n N . V y 1 v 1 n m ngoi kho ng nghi m. M t khc bn un + vn 1 < = < 1, 2 2 nn un , vn (1; 1), n N .

V d

3.2. Cho 0 < p < q. Gi thi t cc s t1, t2 , . . . , tn

tho mn b t phng trnh b c hai f (t) := t2 (p + q)t + pq 0. K hi u 1 (t1 + t2 + + tn ), n 1 B = (t2 + t2 + + t2 ). 2 n n 1 A=

90

Chng 3. M t s

ng d ng c a s ph c trong i s

Ch ng minh r ng 4pq A2 . B (p + q)2 L i gi i. T gi thi t ta thu c


n

(tk p)(tk q) 0
k=1

hay
n n

t2 k
k=1

(p + q)
k=1

tk + npq 0.

S d ng k hi u c a bi ton, ta c B (p + q)A + pq 0. V y nn pq p + q B 2 + = pq A2 A A2 p+q 1 A 2pq


2

(p + q)2 (p + q)2 , 4pq 4pq

i u ph i ch ng minh. ng th c x y ra khi v ch khi (tk p)(tk q) = 0, A = 2pq , k = 1, . . . , n. p+q

V d 3.3. Cho a, b, c R, q > 0 v cho tam th c b c hai g(x) = a(q 2 + q 2 )x2 + b(q
2

+ q 2 )x + c(q 0 + q 0)

khng c nghi m u th c. Ch ng minh r ng tam th c b c hai f (x) = ax2 + bx + c khng c nghi m th c.

3.1. Phng trnh v h phng trnh i s

91

L i gi i. V i q = 1, i u c n ch ng minh l hi n nhin. Khng m t tnh t ng qut ta c th coi q > 1 v vai tr c a q v p = ng. Gi s f = b2 4ac 0. Khi
2

1 l bnh q

g = b2(q

+ q 2 )2 8ac(q 2 + q 2 )

b2(q Ta s ch ng minh

+ q 2 )2 2b2(q 2 + q 2).

(q hay

+ q 2 )2 2(q 2 + q 2 )

2(q 1 q 1 )

(3.1)

v t ta suy ra i u ph i ch ng minh. Vi t (3.1) d i d ng


2

h(q) := q Khi ta c h (q) =

1 2(q q 1 ) 0.

1 2 2 (q q) 1 q qq 2

0 q > 1.

V y nn h(q) l hm ng bi n trong [1; +) v v v y h(q) h(1) = 0 q 1. T ta c (3.1) ng, suy ra i u ph i ch ng minh. V d 3.4. Cho bi t tan x, tan y l hai nghi m c a phng trnh b c hai at2 + vd + c = 0. Tnh gi tr c a bi u th c M = a sin2(x + y) + b sin(x + y) cos(x + y) + c cos2(x + y).

92

Chng 3. M t s

ng d ng c a s ph c trong i s

L i gi i. Theo nh l Vite th b c tan x + tan y = , tan x tan y = . a a Khi cos(x + y) = 0 th tan x tan y = 1 v khi a = c nn M = c. Khi cos(x + y) = 0 th tan x tan y = 1 v khi a = c v tan(x + y) = V y nn M = [a tan2 (x + y) + b tan(x + y) + c]cos2 (x + y) a tan2(x + y) + b tan(x + y) + c = = c. 1 + tan2 (x + y) tan x + tan y b = . 1 tan x tan y ca

3.1.2

Phng trnh b c ba

Xt phng trnh b c ba v i h s th c ho c ph c. ax3 + bx2 + cx + d = 0 (a = 0). Trong tr ng h p khi cc h s l cc s th c, th ta quan tm nhi u hn n cc nghi m th c c a phng trnh. V d 3.5. Gi i phng trnh ax3 + bx2 + cx + d = 0 (a = 0) bi t x = x0 l m t nghi m c a phng trnh. L i gi i. V x0 l m t nghi m c a phng trnh (3.2) nn ax3 + bx2 + cx0 + d = 0. 0 0 (3.2)

3.1. Phng trnh v h phng trnh i s

93

Do c th vi t (1) d i d ng ax3 + bx2 + cx + d = ax3 + bx2 + cx0 + d = 0. 0 0 T ta nh n c (x x0 )[ax2 + (ax0 + b)x + ax2 + bx0 + c] = 0. 0 Xt phng trnh ax2 + (ax0 + b)x + ax2 + bx0 + c = 0. 0 Ta c = (ax0 + b)2 4a(ax2 + bx0 + c). 0 N u < 0 th phng trnh (3.3) v nghi m v nh v y th phng trnh (3.2) c nghi m duy nh t x = x0. N u 0 th phng trnh (3.3) c hai nghi m (ax0 + b) x1,2 = . 2a V y phng trnh (1) c 3 nghi m l (ax0 + b) x0 ; x1,2 = 2a V d 3.6. Gi i phng trnh 4x3 3x = m, |m| 1. L i gi i. t m = cos (= cos( 2)). V cos = 4 cos3 phng trnh (3.4) c 3 nghi m l x1 = cos 2 ; x2,3 = cos . 3 3 (3.4) 3 cos , nn 3 3 . (3.3)

94

Chng 3. M t s

ng d ng c a s ph c trong i s

V d 3.7. Gi i phng trnh 4x3 3x = m, |m| > 1. L i gi i. Nh n xt r ng khi |x| 1 th tr tuy t i c a bi u th c c a phng trnh khng v t qu 1 nn = m. V v y, ta c th t x= 1 2 a+ 1 a , a = 0. (3.5) v tri

Ta d dng ch ng minh c r ng 4x3 3x = 1 2 a3 + 1 a3 .

T ta c cch gi i i v i phng trnh (3.5) nh sau t m= Khi a= v phng trnh (3.5) c d ng 4x3 3x = Phng trnh ny c nghi m x= 1 2 a+ 1 . a 1 2 a3 + 1 a3 .
3

1 2

a3 +

1 a3

m2 1

Ta ch ng minh r ng phng trnh (3.5) c nghi m duy nh t. Gi s phng trnh (3.5) c nghi m x0 th x0 [1; 1]. Do |x0| > 1. Khi (3.5) c d ng 4x3 3x = 4x3 3x0 0 hay (x x0)[4x2 + 4xx0 + 4x2 3] = 0. 0

3.1. Phng trnh v h phng trnh i s

95

Xt phng trnh 4x2 + 4x0 x + 4x2 3 = 0. 0 Ta c = 12 12x2 < 0 v v v y phng trnh (3.6) v nghi m. 0 V y phng trnh (3.5) c m t nghi m duy nh t l x= 1 2
3

(3.6)

m+

m2 1 +

m2 1 .

V d

3.8. Gi i v bi n lu n phng trnh 4x3 + 3x = m, m R. (3.7)

L i gi i. N u phng trnh (3.7) c nghi m x = x0 th cng chnh l nghi m duy nh t c a phng trnh. Th t v y, v i x > x0 th 4x3 + 3x > 4x3 + 3x0 = m v v i x < x0 th ta c 0 4x3 +3x < 4x3 +3x0 = m. Do phng trnh (3.7) c khng qu m t nghi m. 0 t x= 1 2 a 1 a (a = 0).

Ta d dng ch ng minh ng th c 4x3 + 3x = Do , n u t m= th a3 = m m2 + 1 1 2 a3 1 a3 1 2 a3 1 a3 ,

v khi nghi m duy nh t c a phng trnh (3.7) l x= 1 2 a 1 a = 1 2


3

m+

m2 + 1 +

m2 + 1 .

96

Chng 3. M t s

ng d ng c a s ph c trong i s

V d 3.9. Gi i v bi n lu n phng trnh t3 + at2 + vd + c = 0. (3.8)

a L i gi i. t t = y . Khi phng trnh (3.8) c th vi t c d i 3 d ng a 3 a 2 a y + c = 0. +a y +b y 3 3 3 a3 ab a2 b; q = + + c. (3.9) y 3 py = q, p = 3 27 3 N u p = 0 th phng trnh (3.8) c nghi m duy nh t y = 3 q. N u p > 0 th ta a phng trnh cho v d ng cc bi ton xt b ng p cch t y = 2 x ta thu c phng trnh d ng 3 3 3q 3 4x 3x = m, m = . (3.10) 2p p N u |m| 1 th ta t m = cos v phng trnh (3.10) c 3 nghi m x1 = cos N u |m| 1 th t m= 1 2 d3 + 1 d3 2 ; x2,3 = cos . 3 3

ta thu c nghi m duy nh t c a phng trnh cho l x= 1 2 d+ 1 d = 1 2


3

m+ m2 + 1 +

m m2 + 1 .

N u p < 0 th t y = 2 t

p x ta s c phng trnh 4x3 + 3x = m. 3 m= 1 2 d3 1 d3 ,

v i d3 = m m2 + 1.

3.1. Phng trnh v h phng trnh i s

97

Khi phng trnh c nghi m duy nh t x= 1 2


3

m+

m2 1 +

m2 1 .

T nghi m x ta tnh c nghi m y v t suy ra nghi m t t=2 p1 32


3

m+ m2 1 +

a m2 1 . 3

V d

3.10. Gi i phng trnh 8x3 + 24x2 + 6x 10 3 6 = 0.

L i gi i. Phng trnh cho tng ng v i phng trnh sau 10 + 3 6 3 3 2 x + 3x + x = 0. 4 8 t x = y 1. Ta thu c phng trnh 3 6 9 3 = 0. y y 4 8 L i t y = t 3 ta thu c phng trnh 2 . 4t3 3t = 2 Phng trnh ny c cc nghi m l t1 = cos 3 7 , t2 = cos , t3 = cos . 12 4 12

Tr l i v i n x ta c cc nghi m x1 = cos 3 7 1, x2 = cos 1, x3 = cos 1. 12 4 12

98

Chng 3. M t s

ng d ng c a s ph c trong i s

3.1.3

Phng trnh b c b n

Xt phng trnh b c b n v i h s th c ho c ph c ax4 + bx3 + cx2 + dx + e = 0 (a = 0). V d 3.11. Gi i phng trnh trng phng ax4 + bx2 + c = 0 (a, b, c R, a = 0, c = 0) L i gi i. t x2 = y, y 0. Khi phng trnh (3.11) tr thnh ay 2 + by + c = 0, y 0. Gi i phng trnh b c hai ny ta tm c y, t (v i y 0) tnh c x. N u ac < 0 th x= N u ac 0 v b2 4ac 0 th x= b b2 4ac . 2a b + b2 4ac . 2a (3.11)

N u ac 0 v b2 4ac < 0 th phng trnh (3.11) v nghi m. V d 3.12. Gi i phng trnh (x + a)(x + b)(x + c)(x + d) = m v i gi thi t r ng a + b = c + d. L i gi i. t x2 + (a + b)x = x2 + (c + d)x = t, ta thu c phng trnh (t + ab)(t + cd) = m. Gi i v bi n lu n phng trnh ny ta thu c t v t tnh c x.

3.1. Phng trnh v h phng trnh i s

99

V d

3.13. Gi i phng trnh (x + a)4 + (x + b)4 = m. (3.12)

L i gi i. t x = t trng phng d ng (9)

a+b . Phng trnh (3.12) tr thnh phng trnh 2

2t4 + 12t2 + 2 m = 0. Gi i phng trnh ny tm c t, t tnh c x. V d 3.14. Gi i phng trnh ax4 + bx3 + cx2 + dx + e = 0 v i i u ki n ad2 = eb2 (a, e = 0). (3.14) (3.13)

Phng trnh (3.13) v i i u ki n (3.14) c g i l phng trnh h i quy. L i gi i. Vi t i u ki n (3.14) d i d ng e = a d b


2

d = . b

Ta c d = b, e = a2 v sau khi th vo phng trnh (3.13), ta thu c ax4 + bx3 + cx2 + bx + a2 = 0. (3.15)

Nh n xt r ng x = 0 khng ph i l nghi m c a phng trnh (3.15). Chia hai v c a phng trnh cho x2 ta thu c at2 + vd + c 2a = 0, t = x + . x (3.16)

Gi i v bi n lu n phng trnh (3.16), t tm c t, sau tnh x.

100

Chng 3. M t s

ng d ng c a s ph c trong i s

V d 3.15. Gi i phng trnh x4 = ax2 + bx + c, bi t r ng b2 = 4(a + 2)(c + 1), b = 0. L i gi i. Xt tam th c b c hai ng v i bi t th c = b2 4(a + 2)(c + 1) = 0 d ng f (x) = (a + 2)x2 + bx + c + 1. Vi t phng trnh (3.18) d i d ng tng ng sau (x2 + 1)2 = (a + 2)x2 + bx + c + 1. N u a + 2 < 0 t c a < 2 th (x2 + 1)2 > 0, (a + 2)x2 + bx + c + 1 0 nn phng trnh cho v nghi m. N u a + 2 > 0 t c a > 2 th ta c th vi t (3.19) d i d ng (x2 + 1)2 = a + 2x c + 1
2

(3.17)

(3.18)

(3.19)

( d u (+) ng v i tr ng h p b > 0, d u () ng v i tr ng h p b < 0). Ta thu c cc phng trnh b c hai x2 + 1 = a + 2x c + 1 .

V d 3.16. Gi i phng trnh x4 = ax2 + bx + c (b = 0). (3.20)

3.1. Phng trnh v h phng trnh i s

101

L i gi i. G i l s th c tho mn h th c b2 = 4(a + 2)(c + 2 ). (3.21)

Nh n xt r ng lun lun t n t i s nh trn v phng trnh (3.21) l m t phng trnh b c 3 i v i . Khi tam th c b c hai f (x) := (a + 2)x2 + bx + (c + 2 ) c nghi m kp v b (a + 2) x + f (x) = 2(a + 2) c + 2 Vi t phng trnh cho d i d ng x4 + 2x2 + 2 = f (x) hay (x2 + )2 = f (x). N u a + 2 = 0 th (3.22) c d ng (x2 + )2 = c + 2 . N u a + 2 < 0 th phng trnh (3.22) v nghi m. N u a + 2 > 0 th phng trnh (3.22) tr thnh c p phng trnh b c hai x2 + = a + 2 x b 2(a + 2) . (3.22)
2

n u a + 2 = 0, n u a + 2 = 0.

V d

3.17. Gi i phng trnh t4 + t3 + t2 + t + = 0. (3.23)

a L i gi i. t t = x . Khi phng trnh (3.23) c d ng 4 x4 = ax2 + bx + c, (3.24)

102

Chng 3. M t s

ng d ng c a s ph c trong i s

trong a= 62 , 16 23 1 + , b= 16 2 1 4 c = (3 162 + 64 256). 16

Ti p theo ta gi i (3.24) theo cch gi i c a bi ton tr c. V d 3.18. Cho a > 0. Khai tri n bi u th c 1a x
8

+ 1+a x

ta thu c a th c (b c 4) P (x). Gi i phng trnh P (x) = 0. L i gi i. t a2 x = t ta thu c phng trnh t4 + 28t3 + 70t2 + 28t + 1 = 0. y l phng trnh h i quy nn d dng a v d ng phng trnh b c hai y 2 + 28y + 68 = 0 v i y = t + 1. t Phng trnh b c hai ny c hai nghi m y1,2 = 14 c t v tnh c phng trnh c 4 nghi m th c m. V d 3.19. Gi i phng trnh t4 + 4t3 + 3t2 12t 16 = 0. L i gi i. t t = x 1. Ta c phng trnh x4 = 3x2 + 10x + 4.

128. T ta tm

3.1. Phng trnh v h phng trnh i s

103

Ta xc nh a sao cho 102 = 4(3 + 2a)(4 + a2) hay 2a3 + 3a2 + 8a 13 = 0. Ta th y a = 1 tho mn phng trnh. V y c th vi t phng trnh cho d i d ng (x2 + 1)2 = 5x2 + 10x + 5, hay (x2 + 1)2 = 5(x + 1)2 . Gi i phng trnh ny ta thu c cc nghi m l 5 1+4 5 x1,2 = 2 3.1.4 Phng trnh b c cao

Ta xt m t s tr ng h p c bi t c a phng trnh b c cao gi i c b ng cch s d ng cc ng nh t th c i s v l ng gic. V d 3.20. Cho b s m, n, p R. Gi i phng trnh x3 + n 3 3 3 xm xn xp + = 0. 3 3 x +p 2 2 x+m x+n x+p (x + n)3 + (x + p)3

x3 + m 3 + (x + m)3

L i gi i. Nh n xt r ng 1 3 (x m)2 x3 + m 3 = + . (x + m)3 4 4 (x + m)2 V v y phng trnh cho tng ng v i phng trnh sau 1 3 (x m)2 1 3 (x n)2 1 3 (x p)2 + + + + + 4 4 (x + m)2 4 4 (x + n)2 4 4 (x + p)2 3 3 xm xn xp = 0. + 2 2 x+m x+n x+p (3.25)

104

Chng 3. M t s

ng d ng c a s ph c trong i s

t xn xp xm = a, = b, =c x+m x+n x+p v r ng 1 3 2 1 3 2 1 3 2 3 3 + a + + b + + c + abc = 0 4 4 4 4 4 4 4 2 c th bi n i c v d ng (ab + c)2 = (1 a2 )(1 b2). Thay cc gi tr a, b, c theo bi n x, m, n, p ta c (ab + c)2 = 1 a2 = V y (3.26) c d ng x2[x2 + 2(x + p) mn + mn mp np][x2 2(x + p) mn + mn mp np] = 0. Gi i ra ta c cc nghi m c a phng trnh l x1 = x2 = 0, x3,4 = ( mp np) mn, x5,6 = mn ( mp + np. 4[x3 + (mn mp np)x]2 , (x + m)2(x + n)2 (x + p)2 (3.26)

4mx 4nx , 1 b2 = . 2 (x + m) (x + n)2

. n+2 Ch ng minh r ng v i m i a th c Q(x) R[x] b c n th a th c V d 3.21. Cho 0 < < P (x) = (x2 2x cos + 1)Q(x) khng th c t t c cc h s u khng m.

3.1. Phng trnh v h phng trnh i s

105

L i gi i. Gi s Q(x) = a0xn + a1 xn1 + + an1 x + an v P (x) = b0 xn+2 + b1 xn+1 + + bn+1 x + bn+2 . Khi b0 = a0, b1 = a1 2a0 cos , b2 = a2 + a0 2a1 cos , ......... bn+1 = an1 2an cos , bn+2 = an .

Suy ra bk = ak + ak2 2ak1 cos , an+2 = an+1 = 0, a1 = a2 = 0 v


n+2

bk sin k = 0.
k=0

M sin k > 0 v

0;

n+2

nn t n t i h s bj < 0.

V d 3.22. Cho a0 , a1, . . . , an l n + 1 s i m t khc nhau. Gi i h phng trnh sau x0 + x1a0 + x2a2 + + xn an 0 0 x0 + x1a1 + x2a2 + + xn an 1 1 . . . . . . . . . x0 + x1an + x2 a2 + + xn an n n =0 =0 = 0.

(1)

L i gi i. Xt a th c f (y) = xn y n + xn1 y n1 + + x1 y + x0 . Ta c deg f n. T h (1) ta c f (a0 ) = f (a1) = = f (an ) = 0,

106

Chng 3. M t s

ng d ng c a s ph c trong i s

nn f (y) c n + 1 nghi m phn bi t, do f (y) 0. T suy ra x0 = x1 = = xn = 0. Th l i ta th y x0 = x1 = = xn = 0 tho mn h cho. V y h c nghi m duy nh t (x0 ; x1; ; xn ) = (0; 0; ; 0). V d 3.23. Cho a1, a2, . . . , an l n s th c i m t khc nhau. Gi i h phng trnh n1 a1 x1 + an2 x2 + + xn + an 1 1 n1 a2 x1 + an2 x2 + + xn + an 2 2 . . . . . . . . . n1 an x1 + an2 x2 + + xn + an n n =0 =0 = 0.

L i gi i. Xt a th c f (u) = un + x1 un1 + + xn1 u + xn . T h trn ta c f (a1) = f (a2 ) = = f (an ) = 0. Xt g(u) = (u a1)(u a2 ) (u an ) = un + A1un1 + + An1 u + An , trong do g(u) c n nghi m a1, a2, . . . , an v deg g = n c h s b c cao nh t b ng 1 nn theo nh A1 A2 . . . An Xt a th c h(u) = f (u) g(u) = (x1 A1)un1 + (x2 A2)un2 + + (xn An ). l Vite th = (1)1 (a1 + a2 + + an ) = (1)2 (a1a2 + a1 a3 + + an1 an ) ......... = a1a2 an

(2)

3.1. Phng trnh v h phng trnh i s

107

Ta c deg h n 1 v h(u) cng c n nghi m l a1, a2, . . . , an phn bi t nn h(u) 0. Do ta c x1 = A1, x2 = A2, . . . , xn = An , (3.27)

trong cc s Ai c xc nh t (2). V y h c nghi m c xc nh nh (3) v (2). V d 3.24. T n t i hay khng t n t i cc s a1, a2, . . . , an R l cc nghi m c a a th c P (x) = xn +
k=1 n k (1)k Cn ak xnk . k

L i gi i. Gi s t n t i cc s nh v y. Khi theo nh l Vite th


k Cn ak = k i1 <i2 <<ik k (t ng ny c Cn s h ng).

ai1 ai2 aik (k = 1, . . . , n)

Gi s |ak | = max{|a1|, |a2|, . . . , |an|}. Khi ta c


k Cn |ak |k = i1 <i2 <<ik k |ai1 ||ai2 | |aik | Cn |ak |k .

Vy |a1| = |a2| = = |an |. M t khc |a1 + a2 + + an | = n|a1| nn a1, a2, . . . , an cng d u v do chng b ng nhau v t b ng a. Ta c P (x) = (x a)n l a th c tho mn i u ki n bi ra.

108

Chng 3. M t s

ng d ng c a s ph c trong i s

V d

3.25. Ch ng minh r ng khng t n t i m t t p h u h n cc s th c

dng M sao cho ng v i m i n nguyn dng u t n t i a th c b c n thu c M[x] c ng n nghi m u thu c M. L i gi i. Gi s t n t i t p M = {a1, a2, . . . , an } (a = a1 < < an = b) tho mn yu c u c a bi ton. Khi theo gi thi t th t n t i a th c P (x) = b0 + b1x + + bn xn M[x] sao cho P (x) c n nghi m x1, x2, . . . , xn thu c M. Theo nh l Vite th
n

x2 j
j=1

bn1 bn
n

bn2 2 < bn

bn1 bn

+2

|bn2 | . |bn |

Suy ra na
j=1 2

x2 j

b a

+2

b n N , a

i u ny l khng th x y ra. V y khng t n t i m t t p h u h n cc s th c dng M sao cho ng v i m i n nguyn dng u t n t i a th c b c n thu c M[x] c ng n nghi m u thu c M. V d 3.26. Cho a th c f (x) R[x] c t nh t 2 nghi m th c. Ch ng minh r ng a th c P (x) = f (x) f (x) cng c t nh t 2 nghi m th c. L i gi i. Gi s x1, x2 (x1 x2 ) l nghi m c a f (x). Xt hm s g(x) = ex f (x). Ta c g (x) = ex [f (x) f (x)].

3.1. Phng trnh v h phng trnh i s

109

Theo nh l Rolle th g (x) c t nh t m t nghi m th c trong (x1 , x2) n u x1 < x2 v c nghi m b ng x1 n u x2 = x1. Suy ra a th c P (x) c t nh t m t nghi m th c. V deg f (x) = deg P (x) = n nn n u n l th hi n nhin f (x) c t nh t 3 nghi m th c v v v y theo l p lu n trn th P (x) s c t nh t 2 nghi m th c. N u n ch n th do P (x) c nghi m th c nn n ph i c t nh t 2 nghi m th c.

3.1.5

Cc bi ton v phng trnh, h phng trnh i s

M t phng trnh v i n ph c f (z) = 0 v v i nghi m z = x + iy, c th gi i b ng cch tch ph n th c v ph n o, ta lun c th a v d ng h phng trnh h(x, y) = 0 g(x, y) = 0. Ch ng h n, tm cn b c ba c a s ph c 1 + i, ta tm s ph c z = x + iy sao cho z 3 = 1 + i. B ng cch tch ph n th c v ph n o trong ng th c (x + iy)3 = 1 + i, ta thu c h phng trnh x3 3xy 2 = 1 3x2 y y 3 = 1. (3.28)

Gi i h ny, tm c (x, y), t , ta s tm c z. Tuy nhin, r rng z c th tm c b ng cch khai cn 1 + i, c th l z= = 6 2 cos 3 1+i=
3

2 cos

+ i sin 4 4 , k = 0, 1, 2.

2k + 12 3

+ i sin

2k + 12 3

T , ng c l i ta tm c nghi m c a h phng trnh (3.28) l (x, y) = 6 2 cos 2k + 12 3 , 6 2 sin 2k + 12 3 , k = 0, 1, 2.

110

Chng 3. M t s

ng d ng c a s ph c trong i s

Nh th , m t s h phng trnh c th c "xu t x " t cc phng trnh nghi m ph c. B ng cch i ng c l i qu trnh t phng trnh nghi m ph c v h phng trnh, t h phng trnh cho ta thu c phng trnh

nghi m ph c g c. Gi i phng trnh nghi m ph c ny, so snh ph n th c v ph n o, ta c nghi m c a h phng trnh. Ti p theo, ta xt v d sau y. V d 3.27. (Vi t Nam 1996) Gi i h phng trnh 3x 1 + 1 =2 x+y 7y 1 1 = 4 2. x+y L i gi i. Tr c h t, ta nh n th y x, y > 0. t x = u, y = v. H phng trnh cho tr thnh u 1 + 1 u2 + v 2 1 v 1 2 + v2 u 2 = 3 4 2 = . 7

V u2 + v 2 l bnh phng c a m-un s ph c z = u + iv, b ng cch c ng phng trnh th nh t v i phng trnh th hai (sau khi nhn v i i), ta c u iv 2 4 2 u + iv + 2 = +i u + v2 3 7 V z 1 u iv = 2 = , nn phng trnh trn u c vi t l i d i d ng 2 + v2 u |z| z 1 4 2 2 z+ = +i z 3 7 4 2 2 2 z +i z+1=0 3 7 1 2 2 2 z = +i 2 3 21 7

3.2. Cc bi ton v a th c

111

T suy ra (u, v) =

2 2 2 1 , 2 3 21 7

Do h phng trnh cho c hai nghi m 2 2 2 1 2 2 (x, y) = , 2 = 3 21 7

4 22 8 11 , 21 3 7 7 7

3.2
3.2.1

Cc bi ton v a th c
Phng trnh hm trong a th c

Nghi m c a a th c ng vai tr quan tr ng trong vi c xc nh m t a th c. C th n u a th c P (x) b c n c n nghi m x1 , x2, . . . , xn th P (x) c d ng P (x) = c(x x1 )(x x2 ) (x xn ). Tuy nhin, n u ch xt cc nghi m th c c a a th c th trong nhi u tr ng h p s khng c s nghi m. Hn n a, trong cc bi ton phng trnh hm a th c, n u ch xt cc nghi m th c th l i gi i s l khng hon ch nh. nh l c b n c a i s v v y ng m t vai tr h t s c quan tr ng trong d ng ton ny. Ta s d ng cch pht bi u n gi n nh t c a n: m t a th c v i h s ph c (bao g m c s th c) lun c t nh t m t nghi m ph c (bao g m c nghi m th c). D i y ta xem xt m t s p d ng. V d 3.28. Cho 0 < < 1. Tm t t c cc a th c f (x) b c n (n 2) sao

cho t n t i dy s r1, r2 , . . . , rn (r1 < r2 < . . . < rn ) tho mn cc i u ki n sau f (ri ) = 0, (i = 1, 2, . . . , n). f (ri + (1 )ri+1 ) = 0

112

Chng 3. M t s

ng d ng c a s ph c trong i s

L i gi i. Nh n xt r ng v i a < b v x = a + (1 )b th x (a, b). Khi p= 1 1 2 1 + = . xa xb (1 )(b a)

1 1 Do v y p > 0 khi v ch khi > , p < 0 khi v ch khi < v p = 0 khi 2 2 1 v ch khi = . Theo gi thi t th 2
n

f (x) = c
i=1

(x ri )

nn f (x) = f (x) V i n 3 v 0 <

i=1

1 . x ri

1 ta t x = r1 + (1 )r2 . Khi theo gi thi t th 2 f (x) = 0 v ng th i ta l i c 1 1 f (x) = + + f (x) x r1 x r2


n

i=3

1 < 0, x ri

1 mu thu n. Tng t v i n 3 v < < 1 ta cng nh n c i u v l. 2 1 N u n = 2 v = th tng t nh trn cng d n n i u mu thu n. Do 2 1 v y ch cn tr ng h p n = 2 v = xt. Khi m i tam th c b c hai 2 c 2 nghi m phn bi t u tho mn bi. V d 3.29. Tm t t c cc a th c P (x) khc h ng sao cho P (x)P (x + 1) = P (x2 + x + 1), x R. (3.29)

L i gi i. Gi s x0 l nghi m c a P (x) = 0. Khi x2 + x0 + 1 cng l 0 nghi m. Thay x b i x 1 trong (3.29), ta th y r ng P (x 1)P (x) = P (x2 x + 1). V P (x0 ) = 0 nn x2 + x0 + 1 cng l nghi m c a P (x) = 0. 0 Ch n l nghi m c m-un l n nh t (n u t n t i vi nghi m v i m-un l n

3.2. Cc bi ton v a th c

113

nh t, ta ch n m t trong s cc nghi m ). T cch ch n nh v y ta suy ra |2 + + 1| || v |2 + 1| || v c 2 + + 1 v 2 + 1 u l nghi m c a P (x) = 0. Ta nh n xt r ng = 0. Ti p theo, ta c 2|| = |(2 ++1)(2 +1)| |2 ++1|+|2 +1| ||+|| 2||. V y ph i x y ra d u ng th c nn t y suy ra 2 + + 1 = (2 + 1) v i m t h ng s dng no . Hn n a t tnh l n nh t c a || ta cn suy ra |2 + + 1| = |2 + 1| = ||. Nh v y = 1 v ta c 2 + + 1 = (2 + 1), suy ra 2 + 1 = 0. T = i v x2 + 1 l th a s c a P (x). Nh v y ta c th vi t P (x) d i d ng P (x) = (x2 + 1)m Q(x), trong Q(x) l a th c khng chia h t cho x2 + 1. Th ng c tr l i vo phng trnh (3.29), ta th y Q(x) cng tho mn i u ki n Q(x)Q(x + 1) = Q(x2 + x + 1), x R. (3.30)

N u phng trnh Q(x) = 0 l i c nghi m th l lu n trn y suy ra nghi m c m-un l n nh t c a n ph i l i. i u ny khng th x y ra v x2 + 1 khng chia h t Q(x). Ta suy ra r ng Q(x) l m t h ng s , gi s l c. Thay vo phng trnh (3.30) c a Q(x), ta c c = 1. Nh v y l p cc a th c tho mn phng trnh (3.29) l P (x) = (x2 + 1)m v i m l m t s nguyn dng no .

114

Chng 3. M t s

ng d ng c a s ph c trong i s

V d 3.30. Tm t t c cc a th c P (x) tho mn i u ki n P (x)P (x + 1) = P (x2), x R. (3.31)

L i gi i. Gi s l nghi m c a phng trnh P (x) = 0. Khi t phng trnh suy ra 2 , 4 , 8 , . . . cng l nghi m c a P (x) = 0. T y suy ra

r ng || = 0 ho c || = 1, v n u ng c l i ta s thu c dy v h n cc nghi m phn bi t c a P (x). Tng t 1 l nghi m c a P (x) v l lu n tng t , ta cng c | 1| = 0 ho c | 1| = 1. Gi s r ng || = 1 v | 1| = 1. Ta vi t = cos + isin, ta th y r ng 2cos = 1. T y 1 suy ra cos = hay = ho c = 5 . Gi s = . Xt 2 cng 2 3 3 3 l nghi m c a P (x) = 0. Nh v y 2 1 cng l nghi m c a P (x) = 0 v 2 2 2 2 1 + sin2 = 3. Mu thu n v m i nghi m c a | 1| = cos 3 3 P (x) = 0 c m-un b ng 0 ho c 1. Tng t v i tr ng h p = 5 . 3 Nh v y ta c th k t lu n r ng = 1, ho c 1 = 0. T y P (x) c d ng P (x) = cxm (1 x)n , v i c l m t h ng s no v m, n l cc s nguyn khng m. Thay vo phng trnh cho, ta d dng ki m tra c r ng c = 1 v m = n. Nh v y l p cc a th c tho mn i u ki n cho l P (x) = xm (1 x)m , trong m l m t s t nhin. V d 3.31. Tm t t c cc a th c P (x) th a mn phng trnh P 2 (x) P (x2 ) = 2x4 , x R. L i gi i. N u t P (x) = axk + R(x) v i deg R(x) = r < k th ta c P 2 (x) P (x2 ) = (a2 a)x2k + 2axk R(x) + R2 (x) R( x2). T suy ra deg(P 2 (x) P (x2)) ho c b ng 2k n u a = 1, ho c b ng k + r n u a = 1 v r 0, ho c b ng khi a = 1 v r = (t c l ng nh t (3.32)

3.2. Cc bi ton v a th c

115

b ng 0). T , suy ra k 4. n y ta d dng tm c cc nghi m c a (3.32) l x4 + 1, x3 + x, 2x2 v x2 . V d 3.32. Tm t t c cc a th c P (x) th a mn phng trnh P (x2 2) = P 2 (x) 2, x R. (3.33)

L i gi i. C hai a th c h ng th a mn phng trnh l a th c ng nh t 1 v a th c ng nh t 2. V i cc a th c b c l n hn hay b ng 1, p d ng h qu c a nh l ta suy ra v i m i s nguyn dng n, t n t i khng qu m t a th c P (x) th a mn (3.33). i m kh y l ta khng c c ch n gi n

xy d ng cc nghi m. Dng phng php ng nh t h s , ta tm c cc nghi m b c 1, 2, 3, 4 l n l t l: x, x2 2, x3 3x, x4 4x2 + 2. T y, c th d on c quy lu t c a dy nghi m nh sau: Po = 2, P1 = x, Pn+1 = xPn Pn1 , n = 1, 2, 3, . . . (3.34)

Cu i cng, hon t t l i gi i bi ton, ta ch c n ch ng minh cc a th c c xc nh b i (3.34) th a mn phng trnh (3.33). Ta c th th c hi n i u ny b ng cch s d ng quy n p ton h c ho c b ng cch sau: Xt x b t k thu c [2; 2], t x = 2 cos t th t cng th c (3.34), ta suy ra P2 (x) = 4 cos 2t 2 = 2 cos 2t, P3 (x) = 2 cos t.2 cos 2t 2 cos t = 2 cos 3t, v ni chung Pn (x) = 2 cos(nt). T , ta c Pn (x22) = Pn (4 cos2 t2) = Pn (2 cos 2t) = 2 cos(2nt) = 4 cos2 (nt)2 = P 2(x)2. ng th c ny ng v i m i x [2; 2] do ng v i m i x. Bi ton c gi i quy t hon ton.

116

Chng 3. M t s

ng d ng c a s ph c trong i s

V d

3.33. Tm t t c cc b (a; P ; Q) trong a l h ng s th c, P, Q l P 2(x) P (x2 ) = + a. Q2 (x) Q(x2)

cc a th c sao cho: (3.35)

L i gi i. N u (a; P ; Q) l nghi m th (a; P R; Q R) cng l nghi m. Khng m t tnh t ng qut, ta gi s (P ; Q) = 1. Phng trnh c th vi t l i thnh P 2 (x)Q(x2) = Q2(x)(P (x2 ) + aQ(x2)). (3.36)

Do (P 2 (x); Q2(x)) = 1 nn t y ta suy ra Q2(x) = cQ(x2). T gi i ra c Q(x) = cxn , v i n l s t nhin no . Thay vo phng trnh (3.36), ta c P 2 (x) = cP (x2) + ac2 x2n . t R(x) = P (x) , ta c phng trnh c R2 (x) = R(x2) + ax2n. (3.37)

Thay x = 0 vo phng trnh (3.37), ta c R2 (0) = R(0). Do (P ; Q) = 1 nn a = 0 v R(0) = 0. T R(0) = 1. t R(x) = 1 + xk S(x) v i S(0) = 0. Thay vo (3.37), ta c: 1 + 2xk S(x) + x2k S 2 (x) = 1 + x2k S(x2) + a2n 2xk S(x) + x2k (S 2(x) S(x2)) = ax2n . N u k > 2n th chia c hai v cho x2n , ta c 2xk2n S(x) + x2k2n (S 2(x) S(x2)) = a. Thay x = 0 vo suy ra 0 = a, mu thu n. N u k < 2n th chia hai v cho k, ta c 2S(x) + xk (S 2 (x) S(x2)) = ax2nk .

3.2. Cc bi ton v a th c

117

Thay x = 0 vo, suy ra S(0) = 0, mu thu n. V y ch cn m t kh nng c th x y ra l k = 2n. Lc ta c phng trnh 2S(x) + x2n(S 2 (x) S(x2)) = a. l lu n tng t nh trong l i gi i c a v d 3.31, ta suy ra S 2 (x) S(x2) ho c ng nh t b ng 0, ho c c b c b c c a S(x). Nh v y, n u S 2(x) S(x2) khng ng nh t 0 th v tri s c b c l 2n + s, mu thu n. a V y S 2 (x) S(x2 ) = 0, suy ra S(x) = , v thay l i vo ng th c 2 S 2(x) S(x2) = 0 ta suy ra a = 2. Ta c k t qu a = 2, Q(x) = cxn , P (x) = c(1 + x2n ). V d 3.34 (Vi t Nam 2006). Hy xc nh t t c cc a th c P (x) v i h

s th c, th a mn h th c sau: P (x2) + x(3P (x) + P (x)) = (P (x))2 + 2x2 , x R. L i gi i. Thay x = x vo (3.38), ta c P (x2 ) x(3P (x) + P (x)) = (P (x))2 + 2x2 . Tr (3.38) cho (3.39), ta c 4x(P (x) + P (x)) = P 2 (x) P 2 (x) (P (x) + P (x))(P (x) P (x) 4x) = 0. (3.40) (3.39) (3.38)

(3.40) ng v i m i x thu c R, do ta ph i c ho c l P (x) + P (x) = 0 ng v i v s cc gi tr x ho c P (x) P (x) 4x = 0 ng v i v s cc gi tr x. Do P (x) l a th c nn t y ta suy ra ho c P (x) + P (x) = 0 ng v i m i x ho c P (x) P (x) 4x = 0 ng v i m i x.

118

Chng 3. M t s

ng d ng c a s ph c trong i s

Ta xt cc tr ng h p sau: Tr ng h p 1. P (x) + P (x) = 0. Khi ta c phng trnh P (x2 ) + 2xP (x) = (P (x))2 + 2x2 P (x2 ) x2 = (P (x) x)2 . t Q(x) = P (x) x th Q(x2) = Q2(x), ta c Q 0, Q 1, Q(x) = xn . T P (x) = x, P (x) = x + 1, P (x) = xn + x. So snh v i i u ki n

P (x) + P (x) = 0, ta ch nh n c cc nghi m: P (x) = x v P (x) = x2k+1 + x, k = 0, 1, 2, . . . Tr ng h p 2. P (x) P (x) 4x = 0. Khi ta c phng trnh P (x2 ) + x(4P (x) 4x) = P 2 (x) + 2x2 P (x2 ) 2x2 = (P (x) 2x)2 . t Q(x) = P (x) 2x th Q(x2) = Q2(x), ta c Q 0, Q 1, Q(x) = xn . T P (x) = 2x, P (x) = 2x + 1, P (x) = xn + 2x. So snh v i i u ki n P (x) P (x) 4x = 0, ta ch nh n c cc nghi m: P (x) = 2x, P (x) = 2x + 1 v P (x) = x2k + 2x, k = 1, 2, 3, . . . T ng h p hai tr ng h p, ta c t t c nghi m c a (3.38) l cc a th c P (x) = x, P (x) = 2x, P (x) = 2x + 1, P (x) = x2k+1 + x, P (x) = x2k + 2x v i k = 2, 3, . . . V d 3.35. Tm t t c cc a th c v i h s th c P (x) th a mn ng th c sau v i m i s th c x P (x)(2x2) = P (2x3 + x). (3.41)

L i gi i. Ta s i tm nghi m khng ng nh t h ng s b c nh nh t c a (3.41). Xt tr ng h p P (x) c b c nh t, P (x) = ax + b. Thay vo (3.41), ta c (ax + b)(2ax2 + b) = a(2x3 + x) + b

3.2. Cc bi ton v a th c

119

So snh h s c a cc n th c

hai v , ta c h

a3 = 2a, 2ba2 = 0, ab = a, b2 = b H ny v nghi m (do a = 0) nn ta c th k t lu n: khng t n t i a th c b c nh t th a mn (3.41). Xt tr ng h p P (x) c b c 2, P (x) = ax2 + bx + c. Thay vo (3.41), ta c (ax2 + bx + c)(4ax4 + 2bx2 + c) = a(2x3 + x)2 + b(2x3 + x) + c

4a2 x6 + 4abx5 + (4ac + 2ab)x4 + 2b2 x3 + (ac + 2bc)x2 + bcx + c2 = 4ax6 + 4ax4 + 2bx3 + ax2 + bx + c So snh h s cc n th c hai v , ta c h

4a2 = 4a, 4ab = 0, 4ac + 2ab = 4a, 2b2 = 2b, ac + 2bc = a, bc = b, c2 = c H ny c nghi m a = c = 1, b = 0. Nh v y, P (x) = x2 + 1 l a th c b c hai th a mn (3.41). Ta suy ra (x2 + 1)k l t t c cc a th c b c ch n (khng ng nh t h ng s ) th a mn (3.41). V y cn cc nghi m c a (3.41) c b c l th sao? R rng a th c x2 + 1 khng "sinh" ra c cc nghi m b c l . R t may m n, ta c th ch ng minh a th c b c l khng th l nghi m c a (3.41). ch ng minh i u ny, d a vo tnh ch t m i a th c b c l u c t nh t m t nghi m th c, ta ch c n ch ng minh n u P (x) l m t a th c khc h ng s th a mn (3.41) th P (x) khng c nghi m th c (y chnh l n i dung bi thi HSG Vi t Nam 1990). Th t v y, gi s l nghi m th c c a P (x), khi 23 + cng l nghi m c a P (x). N u > 0 th ta c , + 23 , + 23 + 2( + 23 )3 , . . . dy tng v t t c u l nghi m c a P (x), mu thu n. Tng t , n u < 0 th dy ni trn l dy gi m v ta cng c P (x) c v s nghi m. N u = 0, t

120

Chng 3. M t s

ng d ng c a s ph c trong i s

P (x) = xk Q(x) v i Q(0) = 0, thay vo phng trnh ta c: xk Q(x)(2x2)k Q(2x2 ) = (2x3 + x)k Q(2x3 + x) Q(x)(2x2)k Q(2x2 ) = (2x2 + 1)k Q(2x3 + x). Thay x = 0 vo ta c 0 = Q(0), mu thu n. V y P (x) khng c nghi m th c, c ngha l P (x) khng th c b c l . Ni cch khc, bi ton c gi i quy t hon ton. 3.2.2 Cc bi ton v a th c b t kh quy

Nghi m ph c c a a th c v i h s nguyn, trong nhi u tr ng h p l cha kho ch ng minh tnh b t kh quy (trn Z v Q) c a a th c . Chng ta tm hi u cc l lu n m u trong v n ny thng qua cc tnh ch t v v d sau y. Tr c h t xt m t s tiu chu n xt tnh b t kh quy c a a th c v i h s nguyn. nh l 3.1. (Tiu chu n Eisenstein1) Cho P (x) = an xn + an1 xn1 + + a1x + a0 l m t a th c h s nguyn. Gi s t n t i s nguyn t p th a mn nh ng i u ki n sau: 1) an khng chia h t cho p. 2) T t c nh ng h s cn l i u chia h t cho p. 3) a0 khng chia h t cho p2 . Khi a th c P (x) b t kh quy trn Q[x]. Ch ng minh. Gi s a th c P (x) kh quy trn Z[x] v P (x) = g(x)h(x), trong g(x) = b0 + b1x + + br xr , bi Z, 0 < r < n h(x) = c0 + c1x + + cs xs , ci Z, 0 < s < n.
1

M.Eisenstein (1823-1852) l nh ton h c c.

3.2. Cc bi ton v a th c

121

ng nh t h s trong ng th c P (x) = g(x)h(x) ta c a0 = b0c0 a1 = b1c0 + b0c1 ...... ak = bk c0 + bk1 c1 + + b0 ck . . . . . . . . . an = br cs . Theo gi thi t p chia h t a0 = b0 c0 , m p l s nguyn t nn ho c p chia h t b0 ho c p chia h t c0 . Gi s p chia h t b0, th th p khng chia h t c0,v n u khng th p2 chia h t a0 = b0c0 , tri gi thi t. p khng th chia h t m i h s c a g(x), v n u th th p s chia h t an = br cs , tri gi thi t. Gi s bt (0 < t < n) l h s u tin trong dy h s b0, b1, . . . , br c a g(x) khng chia h t cho p. Xt at = bt c0 + bt1c1 + + b0 ct, trong at, bt1 , . . . , b0 u chia h t cho p. V y btc0 ph i chia h t cho p, m p nguyn t , ta suy ra ho c c0 chia h t cho p, ho c bt chia h t cho p, mu thu n gi thi t v bt v c0 . V d 3.36. Ch ng minh r ng n u p l m t s nguyn t , th a th c P (x) = xp1 + xp2 + + x + 1 b t kh quy trn Q[x]. L i gi i. Ta bi n i a th c P (x) P (x) = xp 1 , v i x = 1. x1

t x 1 = y, ta nh n c: (y + 1)p 1 1 p2 p1 P (x) = = y p1 + Cp y p2 + + Cp y + Cp . y 1 p2 p1 t Q(y) = y p1 + Cp y p2 + + Cp y + Cp . Do Q(y) l a th c Eisenstein, Q(y) b t kh quy, suy ra c P (x) cng b t kh quy, v l. Qua v d trn ta th y r ng b ng cch i bi n ta c th ch ng minh s b t kh quy c a a th c cha th a mn tiu chu n Eisenstein thnh a th c th a mn

122

Chng 3. M t s

ng d ng c a s ph c trong i s

tiu chu n Eisenstein. i u ny pht huy tc d ng c a tiu chu n Eisenstein. C r t nhi u v d th c hi n phng php ny nh P (x) = x4 2x + 3 b t kh quy v ta thay th x = y + 1 vo a th c P (x) th nh n c a th c th a mn tiu chu n Eisenstein y 4 + 4y 3 + 6y 2 + 2y + 2. Nhng khng ph i lc no cng thay bi n a m t a th c cha th a mn tiu chu n Eisenstein thnh a th c th a mn tiu chu n Eisenstein. Ta xt ti p v d sau V d 3.37. T n t i a th c b t kh quy, m n khng c m t cch bi n

i tuy n tnh c a n no c th chuy n n thnh a th c Eisenstein (v i s nguyn t p no ). L i gi i. Xt a th c P (x) = 2x2 + 1, a th c ny b t kh quy trn Q[x]. i bi n x = ay + b (a, b l nh ng s nguyn b t k). Khi ta c Q(y) = 2a2 y 2 + 4aby + 2b2 + 1. Gi s Q(y) l a th c th a mn tiu chu n Eisenstein, ngha l t n t i s nguyn t p sao cho 2a2 khng chia h t cho p, v 4ab ; (2b2 + 1) cng chia h t cho p. Suy ra, b khng chia h t cho p, nhng khi 4a chia h t cho p, t y c ho c a chia h t cho p ho c 4 chia h t cho p. T tr ng h p th nh t ta nh n c h s cao nh t c a a th c Q(y) chia h t cho p, i u ny v l. T tr ng h p th hai 4 chia h t cho p, suy ra p = 2. Nhng khi 2b2 + 1 khng chia h t cho p ta cng thu c i u v l. nh l 3.2. (Tiu chu n Eisenstein m r ng) Cho P (x) = an xn + an1 xn1 + + a1x + a0 l m t a th c h s nguyn c b c n. Gi s t n t i s nguyn t p th a mn nh ng i u ki n sau: 1) an khng chia h t cho p. 2) T n t i k, 0 k < n sao cho a0, a1, . . . , ak chia h t cho p.

3.2. Cc bi ton v a th c

123

3) a0 khng chia h t cho p2 . Khi a th c P (x) c a th c c b t kh quy trn Z[x] m b c c a n khng nh thua k + 1. V i k = n 1 ta c tiu chu n Eisenstein quen thu c. Ch ng minh N u a th c P (x) b t kh quy trn Z[x] th nh l c ch ng minh. N u a th c P (x) khng b t kh quy trn Z[x] v c bi u di n thnh tch c a nh ng a th c v i h s nguyn b t kh quy P (x) = g(x)f (x) . . . . Do a0 . p, nn h s t do c a m t a th c no cng chia h t cho p. . Khng m t tnh ch t t ng qut ta coi l h s t do c a a th c g(x). Khi ta c th vi t P (x) d i d ng sau P (x) = g(x)h(x), trong g(x) = b0 + b1 x + + br xr , bi Z, 0 < r < n h(x) = c0 + c1 x + + cs xs , ci Z, 0 < s < n. . . V a th c g(x) c ch n sao cho b0 . p, m a0 = b0 c0 . . Cng theo gi thi t th an = br cs . p br . p. . . Gi s bt (0 < t . 2 . p nn c0 . . . p. .

r) l h s u tin c a g(x) trong dy b0, b1, . . . , br khng

chia h t cho p. Ta xt at = btc0 + bt1 c1 + + ctb0 . v ph i c a ng th c ny c s h ng u tin c a t ng khng chia h t cho p, cn l i cc chia h t cho p. Nn at khng chia h t cho p, v theo gi thi t th 2 c a nh l th t Ta c r k + 1, suy ra k + 1 t k + 1. 0.

r + k + 1 t = (k + 1) + r t

124

Chng 3. M t s

ng d ng c a s ph c trong i s

V d 3.38. Ch ng minh a th c sau b t kh quy P (x) = x4 + x3 3x2 3x 3. L i gi i. Gi s P (x) khng b t kh quy trn Z[x]. p d ng nh l trn v i k = 2, p = 3, suy ra a th c c th c s c a P (x) l G(x) c b c 2+1 = 3

b t kh quy. T deg G(x) = 3, suy ra P (x) c nghi m h u t , nhng nghi m h u t c a P (x) n u c l c c a 3, ki m tra th y cc c ny u khng l nghi m c a P (x). V y P (x) b t kh quy trn Z[x] nn b t kh quy trn Q[x]. 1 B 3.1. Cho l m t s ph c sao cho Re < b . Khi 2 b > b1 . Ch ng minh. t = + i v i , l cc s th c, v k hi u Re l ph n th c c a s ph c . 1 Theo gi thi t th Re = v < b . Ta c 2 b > b1 b
2

> b1

b2 2b + 2 + 2 = b2 + 1 + 2 2b 2b + 2 + 2 1 < b . 2 B c ch ng minh. nh l 3.3 (Tiu chu n Perron). Cho P (x) l a th c v i h s nguyn. Gi thi t t n t i s nguyn b v s nguyn t p, sao cho chng th a mn nh ng i u ki n sau: 1. P (b) = p, 2. P (b 1) = 0, 3. T t c cc nghi m i (i = 1, 2, . . . , n) c a a th c P (x) th a mn b t 1 ng th c Re i < b . 2 Khi a th c P (x) b t kh quy trn Z[x].

3.2. Cc bi ton v a th c

125

Ch ng minh. Gi s P (x) = G(x)H(x), khng ph i l h ng s .

y G(x), H(x) l a th c nguyn

T ng th c P (b) = p (do p l s nguyn t ) suy ra G(b) = 1 v H(b) = p ho c l G(b) = p v H(b) = 1. Xt G(b) = 1 v gi s nghi m c a G(x) l 1 , 2 , . . . , k (1 G(x) = c(x 1 )(x 2 ) . . . (x n ), y c l h s cao nh t c a G(x). V G(b) = 1 v G(b 1) Z , ta c b t ng th c G(b) G(b 1) 1, 1 hay l () k < n). Khi

b 1 b 2 b k b 1 1 b 1 2 b 1 k i u ny v l, v theo v i b 3.2 th c a th c P (x) b t kh quy trn Z[x]. V d 3.39. Cho b

b i > 1 v i m i i. Nh v y, b 1 i

3 l s nguyn v p l m t s nguyn t .

Ta vi t p trong h s c s b, ngha l p c d ng p = a0bn + a1bn1 + + an , y n l s t nhin, a0 = 0 v 0 ai < b (i = 0, 1, . . . , n). Xt a th c

P (x) = a0xn + a1 xn1 + + an . Ch ng minh r ng P (x) b t kh quy trn Z[x] (gi thi t p a th c l h ng s ). L i gi i. Ta dng tiu chu n Perron. Hi n nhin P (b) = p v P (b 1) = 0, v b 1 > 1, v t t c h s ai khng m v t nh t m t trong chng l s dng. Ta ch c n ch ng minh m i nghi m c a P (x) th a mn b t ng th c 1 Re < b . 2 b v v i p < b th

126

Chng 3. M t s

ng d ng c a s ph c trong i s

1 l hi n nhin. 2 N u Re > 0. Ta k hi u r l m un c a , ngha l = r. N u Re 0 b t ng th c Re < b N ur 1 r 1 < b suy Re 1, t b t ng th c hi n nhin Re 2 1 ra Re < b l hi n nhin v b 2. 2 N u r > 1, v i cc i u trn th = 0, khi chia c hai v c a ng th c P () = 0 cho n ta nh n c a0 + T y ta c a0 + a2 a1 an = 2 + + n . a1 a2 an + 2 + + n = 0.

Theo b t ng th c v mun cho v ph i, ta c a2 an + + n 2 m Re a0 + a1 a0 + a1 a1 a2


2

+ +

an a2 an n = 2 + + n, r r

nn a2 an + + n. 2 r r

Re a0 + Ta bi t r ng ai

b 1, ta c th nh gi v ph i c a b t ng th c trn 1 1 + + n 2 r r 1 1 n1 b1 r . = 2 . 1 r 1 r

an a2 + + n 2 r r

(b 1)

B i v r > 1, t b t ng th c cu i cng ta nh n c an 1 a2 + + n < (b 1) 2 . 2 r r r t suy ra Re a0 + a1 < (b 1) 1 . r(r 1) 1 1 1 r = (b 1) 1 , r(r 1)

3.2. Cc bi ton v a th c

127

a1 > 0 (v a1 l s khng m) v v a0 a1 1. Suy ra l m t s nguyn dng, nn Re a0 + a1 a1 Re a0 + = Re a0 + 1 Nhng ta gi thi t Re > 0 suy ra Re v cu i cng ta nh n c b t ng th c 1 < (b 1) 1 . r(r 1)

V r > 1 nn t b t ng th c cu i cng ta suy ra r(r 1) < b 1. 1 1 3 Gi s Re b , hay l r b r 1 b . 2 2 2 3 Nhn hai v b t ng th c trn ta c r(r 1) b2 2b + , t c 4 3 7 2 2 b 2b + < b 1 ho c b 3b < . 4 4 V b 3 nn b t ng th c cu i khng x y ra. Ta c i u c n ch ng minh. Sau y ta ti p t c xt thm m t s l p a th c b t kh quy khc n a V d 3.40. Cho P (x) l a th c c b c l n = 2m + 1 v P (ai ) = 1 v i n

s nguyn khc nhau a1, a2, . . . , an. Khi P (x) b t kh quy. L i gi i. Gi s a th c P (x) phn tch c thnh tch P (x) = G(x)H(x) v i G(x), H(x) l nh ng a th c h s nguyn khc h ng s . Khng m t tnh t ng qut ta c th gi s deg G(x) m.

T ng th c P (ai ) = 1, suy ra G(ai )H(ai) = 1. Do nh ng s G(ai ), H(ai) l nh ng s nguyn, ta nh n c G(ai ) = 1 v i m i i = 1, n. Nhng n = 2m + 1 > m, suy ra t nh t trong cc ng th c G(ai ) = 1 ho c G(ai ) = 1 th a mn v i hn m gi tr c a i. M t khc, deg G(x) = m nn G(x) l a th c h ng s 1 ho c 1, i u ny tri v i gi s . V d 3.41. Cho P (x) l a th c v i h s nguyn b c n 8. Gi s t n t i k n nh ng s nguyn khc nhau a1, a2 , . . . , ak , y k > , sao cho P (ak ) = 1. 2 Khi a th c P (x) b t kh quy.

128

Chng 3. M t s

ng d ng c a s ph c trong i s

Gi i. Gi s P (x) = G(x)H(x), khi t nh t m t trong nh ng a th c G(x) n sao cho phng trnh G(x) = 1 s c v H(x), cho l G(x), c b c m 2 n n , nn k > m. Nhng theo gi thi t th n 8, k nghi m ak . Do k > , m 2 2 khi k 5. Ta c ho c l G(ai ) = 1, ho c l G(ai ) = 1. M b c c a G(x) nh hn k nn trong c hai tr ng h p ta nh n c G(x) l h ng s +1 ho c 1, l i u v l. V d 3.42. Ch ng minh r ng n u a th c f (x) Z[x] b c n m nh n gi tr n b ng 1 t i nhi u hn 2 i m nguyn phn bi t th f (x) l a th c b t 2 kh quy. L i gi i. t v 0 < deg g(x) < deg h(x) < n. V n = 2m ho c n = 2m + 1 nn deg g(x) t i l s x1, x2, . . . , xl (l > 2m) sao cho f (xi ) = g(xi )h(xi ) = 1, i = 1, 2, . . . , l (l > 2m). Suy ra g(xi ) = 1 v i m i i = 1, 2, . . . , l. V y trong l s xi ph i c t nh t m + 1 s g(xi ) = 1 ho c g(xi ) = 1 m g(x) ch l a th c b c nh thua ho c b ng m nn g(x) 1 ho c g(x) 1, tri v i gi s . V y f (x) l a th c b t kh quy. V d 3.43. Cho a1 , a2, . . . , an l n s nguyn phn bi t. Ch ng minh r ng P (x) = (x a1)2 (x a2 )2 . . . (x an )2 + 1 l b t kh quy. L i gi i. Gi s P (x) l kh quy. Khi P (x) = f (x)g(x), trong f (x), g(x) l cc a th c c h s nguyn v c b c l n hn 0. C th coi h s cao nh t m. M t khc, theo gi thi t t n n = m. Gi s f (x) = g(x)h(x), trong g(x), f(x) Z[x] 2

3.2. Cc bi ton v a th c

129

c a f (x) dng. Do P (x) v nghi m nn f (x), g(x) cng v nghi m, nn f (x), g(x) l a th c dng trn R v f (ak )g(ak ) = 1 v i m i k = 1, 2, . . . , n. Gi s f (x) = xr + , g(x) = xs + , v i s + r = 2n N u x y ra r < s th r < n nn g(x) 1, v l. N u x y ra r = s = n th f (x) g(x) c b c ak nn f (x) g(x) = 0 hay f (x) = g(x), x R. Do ta c P (x) = (x a1)2 (x a2 )2 . . . (x an )2 + 1 = g(x) v i g(x) 1 = (x a1)(x a2 ) . . . (x an ) v v v y (x a1)2 (x a2)2 . . . (x an )2 + 1 = (x a1)(x a2) . . . (x an ) + 1 , v i m i x R. i u ny khng th x y ra. V y P (x) b t kh quy. V d 3.44. Cho a1 , a2, . . . , an l n nh ng s nguyn khc nhau, p l s
2 2

n 1 v tri t tiu t i n i m

nguyn b t k v P (x) = p(x a1 )(x a2 ) . . . (x an ) + 1. a th c ny b t kh quy, lo i tr cc tr ng h p sau y: 1 . p = 4, n = 2 v a1, a2 l hai s lin ti p, ngha l a1 = a2 1. C th P (x) = 4(x a1)(x a1 1) + 1 = 2(x a1) 1 . 2 . p = 1, n = 4 v a1, a2, a3, a4 (s p x p thch h p) l b n s lin ti p. Khi P (x) = (x a1)(x a1 1)(x a1 2)(x a1 3) + 1 = (x a1 1)(x a1 2) 1 .
2 2

3 . p = 1, n = 2 v a1, a2 khc nhau 2 n v , ngha l a1 = a2 2. Khi P (x) = (x a1 )(x a1 2) + 1 = (x a1 1)2 .

130

Chng 3. M t s

ng d ng c a s ph c trong i s

L i gi i. Gi s

P (x) phn tch c d i d ng P (x) = G(x).H(x) v i

G(x), H(x) l nh ng a th c c h s nguyn v c b c nh hn n. V P (ai ) = 1, v i i = 1, 2, . . . , n, nn ta c G(ai )H(ai ) = 1. M t khc, G(ai ), H(ai ) l nh ng s nguyn, suy ra v i m i i th G(ai ) = H(ai ) = 1 G(ai ) = H(ai ) = 1. Khi G(ai ) H(ai) = 0, suy ra G(x) H(x) c n nghi m. M deg (G(x) H(x)) < n nn ta c G(x) H(x) = 0 G(x) = H(x), x R. Ta c P (x) = G(x)2 , n l s ch n, v h s cao nh t c a P (x) l p ph i l s chnh phng Ngha l P (x) b t kh quy khi ta ch ra n l m t s l , ho c n l ch n nhng p khng l s chnh phng, trong tr ng h p ring p l m t s m. Ta t p = q 2 v n = 2m, y q l h s cao nh t c a G(x) cn m l b c

c a n. Khng m t tnh t ng qut gi s q > 0 (tr ng h p ng c l i c th thay ton b d u c a G(x)). Ta xt a th c G(x), v i m i i ta c G(ai ) = 1 ho c l G(ai ) = 1. Ta s ch ng minh m i ng th c ny th a mn v i ng m gi tr c a i. Th t v y, n u G(ai ) = 1 ng v i nhi u hn m gi tr c a i, m b c c a G(x) l m nn G(x) trng v i h ng s 1, v l. Suy ra s l ng k c a ch s i sao cho G(ai ) = 1 khng l n hn m. N u k < m, Ta s nh n c G(ai ) = 1 v i n k > m ch s c a i, t y ta l i suy ra G(x) trng v i h ng s 1, v l. Nh v y, ng th c G(ai ) = 1 ch th a mn v i ng m gi tr c a i. Khng m t tnh t ng qut ta c th cho G(a1 ) = G(a2) = = G(am ) = 1 v khi G(am+1 ) = G(am+2 ) = = G(an ) = 1

3.2. Cc bi ton v a th c

131

Ta xt a th c G(x) 1. a th c ny c b c m, h s cao nh t l q v bi t m nghi m khc nhau a1 , a2, . . . , am . Suy ra G(x) 1 = q(x a1)(x a2 ) . . . (x am ). t 1 (x) = (x a1)(x a2 ) . . . (x am ), 2 (x) = (x am+1 )(x am+2 ) . . . (x an ), (x) = 1 (x)2(x). T P (x) = G(x)2 ta c: p(x) + 1 = (q1(x) + 1)2 , suy ra p(x) = q 22 (x) + 2q1(x). 1 Ta bi t r ng p = q 2 v (x) = 1 (x)2(x), n = 2m, chia c hai v ng th c trn cho q1(x), ta nh n c q 2(x) 1 (x) = 2. Ta so snh h s t do hai v tri ng th c, l (1)m q am+1 am+2 . . . an a1a2 . . . am = 2. Suy ra q = 1 ho c q = 2, v tng ng p = 1 ho c p = 4. Nh v y ta ch ng minh c a th c P (x) b t kh quy khi p = 1 v p = 4. Ta xt hai tr ng h p c th ny Tr ng h p p = 4 (q = 2) : Trong tr ng h p ny () c d ng (x am+1 )(x am+2 ) . . . (x an ) (x a1)(x a2) . . . (x am ) = 1. () ()

Ta ch ng minh () ch ng v i m = 1. Th t v y, cho x = a1 ta nh n c (a1 am+1 )(a1 am+2 ) . . . (a1 an ) = 1. V cc th a s N um ()

v tri l nh ng s nguyn, nn cc th a s thu c {+1; 1}.

3, t nh t hai trong nh ng th a s ny trng nhau, v l.

132

Chng 3. M t s

ng d ng c a s ph c trong i s

N u m = 2 ta ch c hai th a s v v tch c a chng dng nn chng cng d u, suy ra chng trng nhau, v l. N u m = 1 th () c d ng (x a2) (x a1) = 1. Suy ra a1 = a2 + 1. Nh v y ph n 1 c ch ng minh. Tr ng h p p = 1 (q = 1) : Trong tr ng h p ny () c d ng (x am+1 )(x am+2 ) . . . (x an ) (x a1 )(x a2) . . . (x am ) = 2. ( ) Ta ch ng minh m 3. Th t v y, thay x = a1 vo ( ) nh n c (a1 am+1)(a1 am+2 ) . . . (a1 an ) = 2. N um 4 th ch ng minh tng t ph n trn c t nh t hai th a s b ng 3.

nhau. Suy ra m

N u m = 3, th ( ) c d ng (a1 a4)(a1 a5 )(a1 a6) = 2. T ng th c ny suy ra hai trong nh ng th a s v tri b ng 1, nh v y

chng ph i c d u tri nhau, v ng c l i th chng trng nhau. Ta cho l a1 a5 = 1, a1 a6 = 1, khi a1 a4 = 2 hay a1 = a4 2. Ta l p l i cng l lu n nh i v i a2 thay vo ch a1 . Ta nh n c a2 = ai 2, y i l s no y trong cc s 4, 5, 6. ng th c a2 = a4 2 khng x y ra v khi y th a1 = a2 . Suy ra a2 = a5 2 ho c l a2 = a6 2. Khng m t tnh ch t t ng qut ta cho a2 = a5 2. Cu i cng b ng cch thay x = a3 vo ( ) v l lu n tng t c a3 = a6 2. So snh h s c a x2 ( ) c a4 a5 a6 + a1 + a2 + a3 = 6 = 0, v l.

Nh v y m = 3 v m ch cn l i hai kh nng m = 1 v m = 2. N u m = 2 : l n l t thay vo ( ) x = a1 v x = a2 ta nh n c (a1 a3)(a1 a4) = 2, (a2 a3 )(a2 a4) = 2.

3.2. Cc bi ton v a th c

133

T ng th c th nh t suy ra b n kh nng sau: a1 a3 = 1, a1 a4 = 2; a1 a3 = 1, a1 a4 = 2; a1 a3 = 2, a1 a4 = 1; a1 a3 = 2, a1 a4 = 1. V vai tr c a a3 v a4 l i x ng, nn ta ch xt hai tr ng h p u l . Tr ng h p a3 = a1 + 1 v a4 = a1 + 2. Thay vo ng th c (a2 a3)(a2 a4) = 2, ta nh n c (a2 a1 1)(a2 a1 2) = 2 a2 a1 2 = 2 a2 a1 1 = 1 a2 a1 2 a2 a1 1 =1 = 2.

Trong tr ng h p th nh t nh n c a1 = a2, tri gi thi t. T tr ng h p th hai suy ra a2 = a1 + 3. Nh v y a1; a3 = a1 + 1, a4 = a1 + 2, a2 = a1 + 3 l b n s nguyn lin ti p, i u ny ch ng t ph n 2 c ch ng minh. N u m = 1, th ( ) c d ng (x a2) (x a1) = 2, t y suy ra a1 = a2 +2, t c l ph n 3 c ch ng minh. V d 3.45 (IMO 1993). Ch ng minh r ng v i m i n N, n > 1, a th c

P (x) = xn + 5xn1 + 3 khng th phn tch thnh tch c a hai a th c c b c khng nh hn 1 v i h s nguyn. L i gi i. Gi s x1 , x2, . . . , xn l t t c cc nghi m c a a th c P (x). Khi ta c P (x) = (x x1)(x x2) (x xn ). Suy ra 3 = (1)n x1 x2 xn . Ta c v i m i i th xn + 5xn1 + 3 = 0 3 = |xn1 ||xi + 5|, i = 1, 2, . . . , n. i i i (3.42)

Gi s ng c l i r ng a th c P (x) kh quy, t c l P (x) = Q(x).S(x) v i Q(x), S(x) l cc a th c khc h ng v i h s nguyn. Th th r rng Q(x) s l tch c a m t s th a s x xi v S(x) l tch c a cc th a s cn l i.

134

Chng 3. M t s

ng d ng c a s ph c trong i s

Khng m t tnh t ng qut, gi s Q(x) = (x x1)(x x2) (x xk ), S(x) = (x xk+1 ) . . . (x xn ). Suy ra |x1x2 xk | v |xk+1 xn | l cc s nguyn c tch l 3. Nh v y m t s b ng 1 v m t s b ng 3. Khng m t tnh t ng qut, gi s |x1 x2 xk | = 3 v |xk+1 xn | = 1. Trong (3.42) cho i ch y t 1 n k r i nhn v theo v , ta c 3k = |x1 xk |n1 |(x1 + 5) (xk + 5)| = 3n1 .|Q(5)|. Suy ra k n 1. Nh v y S(x) l nh th c b c nh t v P (x) c nghi m nguyn. Nhng nghi m nguyn c a P (x) ch c th l -1, 1, -3, 3. Ki m tra l i th chng u khng l nghi m. Mu thu n ny ch ng t i u gi s trn l sai, t c a th c P (x) l b t kh quy. V d 3.46 (Nh t B n 1999). Ch ng minh r ng khng th phn tch a th c f (x) = (x2 + 12 )(x2 + 22 ) (x2 + n2 ) + 1 thnh tch c a hai a th c h s nguyn b c l n hn hay b ng 1. L i gi i. Gi s ng c l i, ta c f (x) = g(x).h(x) v i g(x), h(x) l cc a th c v i h s nguyn c b c l n hn hay b ng 1. Khi , r ng f (ki) = 1 v i k = 1, 2, . . . , n, ta c 1 = g(ki)h(ki), k = 1, 2, , n. V s 1 ch c 4 cch phn tch thnh tch c a cc s nguyn trong Z[i] l 1.1, (1).(1), i.(i) v (i).i nn ta c v i m i k {1, 2, . . . , n} th (g(ki); h(ki)) {(1; 1); (1; 1); (i; i); (i; i)}. Nh v y trong m i tr ng h p ta u c g(ki) = h(ki) = h(ki). Nh th a th c g(x) h(x) c 2n nghi m phn bi t, trong khi b c c a n nh thua 2n.

3.2. Cc bi ton v a th c

135

V y ta ph i c g(x) h(x) l a th c h ng 0, t c l g(x) = h(x). T deg g(x) = deg h(x) = n. V f (x) l a th c n kh i (h s b c cao nh t b ng 1) nn ta c th gi s g(x), h(x) n kh i. Khi a th c g 2 (x) h2(x) c b c nh hn 2n. a th c ny c t nh t 2n nghi m ki v i k {1; 2; . . . ; n}. Suy ra g 2 (x)h2 (x) = 0. Ta khng th c g(x) = h(x) v g v h n kh i. V y ta ph i c g(x) = h(x). Nh th f (x) = g 2 (x). T y suy ra (g(0))2 = f (0) = (n!)2 + 1. i u ny l khng th v g(0) l s nguyn v n 1. 3.2.3 Bi ton v s chia h t c a a th c

Nh n xt r ng n u a th c P (x) chia h t cho a th c Q(x) th m i nghi m c a Q(x) u l nghi m c a P (x). Tnh ch t n gi n ny l cha kho gi i nhi u bi ton v s chia h t c a a th c. Chng ta xem xt m t s v d . V d 3.47. V i gi tr no c a n th x2n + xn + 1 chia h t cho a th c

3 1 = cos + i sin l nghi m c a L i gi i. Nh n xt r ng = + i 2 2 3 3 phng trnh Q(x) = x2 + x + 1 = 0. a th c P (x) = x2n + xn + 1 chia h t cho Q(x) khi v ch khi P () = 0. i u ny tng ng v i 4n 2n 2n 4n + i sin + cos + i sin +1=0 3 3 3 3 2n 2 cos + 1 = 0 n = 3k + 1 ho c n = 3k + 2. 3 V y v i n = 3k + 1 ho c n = 3k + 2 (k Z) th P (x) chia h t cho Q(x). cos Trong v d d i y, m t l n n a, cn c a n v l i ng vai tr then ch t. V d cho P (x5 ) + xQ(x5) + x2R(x5 ) = (x4 + x3 + x2 + x + 1)S(x). Ch ng minh r ng P (x) chia h t cho x 1. 3.48 (USA MO 1976). Cho P (x), Q(x), R(x), S(x) l cc a th c sao

x2 + x + 1.

136

Chng 3. M t s
2i 5

ng d ng c a s ph c trong i s

L i gi i. t = e cc phng trnh

th 5 = 1. Thay x l n l t b i , 2, 3 , 4, ta c

P (1) + Q(1) + 2R(1) = 0 P (1) + 2 Q(1) + 4R(1) = 0 P (1) + 3 Q(1) + 6R(1) = 0 P (1) + 4 Q(1) + 8R(1) = 0 Nhn cc phng trnh t th nh t n th t l n l t v i , 2, 3, 4, ta c P (1) 2Q(1) 3 R(1) = 0 2 P (1) 4Q(1) R (1) = 0 3P (1) Q(1) 4 R(1) = 0 4P (1) 3Q(1) 2 R(1) = 0 S d ng ng th c 1 + + 2 + 3 + 4 = 0, ta c 5P (1) = 0, t c l P (x) chia h t cho x 1. 3.2.4 Quy t c d u Descartes trong ng d ng

Ti p theo, ta xt m t s bi ton kh o st s nghi m th c, s nghi m ph c c a a th c thng qua s l n i d u ho c s l n gi nguyn d u c a dy h s tng ng b ng cch s d ng quy t c d u Descartes. Trn c s m r ng ph m vi ng d ng c a quy t c ny trong i s . B 3.2. K hi u s v tr i d u c a dy cc h s c a cc a th c b c n f (x) = a0 + a1x + a2x2 + + an xn f (x) = a0 a1x + a2 x2 + + (1)n an xn

3.2. Cc bi ton v a th c

137

l n l t l W + v W . Gi s a l s h ng u tin khc 0 trong dy cc h s a0, a1, a2 , . . . , a1 , a , . . . , an c a a th c f (x). Khi n (W + + W ) 0.

Ch ng minh. K hi u ak xk v al xl l hai s h ng tho mn i u ki n ak = 0, ak+1 = ak+2 = = al1 = 0, al = 0. Khi f (x) = a x + + ak xk + alxl + + an xn , an = 0. Suy ra l m t gi tr ring c a k. N u (n k) l th v i m i c p ak xk + alxl s ng gp m t n v vo cho W + ho c cho W . N u (n k) ch n th t c p ak xk + al xl hai i l ng W + v W nh n thm m t n v cho m i i l ng ho c khng nh n g c . M t khc ta c
L C

(n ) =
L

(l k) +

(l k),
C

trong t ng

c l y theo cc hi u (l k) l ,

c l y theo cc hi u

(l k) ch n. T suy ra
L C

(n ) (W + W ) =

(l k 1) +
L C

{l k [1 sign (ak al )]}.

Nh n xt r ng, cc s h ng trong

u khng m. Do 0.

(n ) (W + + W )

nh l 3.4. Gi s a th c f (x) = a0 + a1x + a2x2 + + an xn (an = 0) c cc nghi m u th c, g i W l s v tr i d u c a dy h s a0, a1 , . . . , an v N l s khng i m dng c a a th c f (x) th W = N.

138

Chng 3. M t s

ng d ng c a s ph c trong i s

Ch ng minh. Gi s N + l s khng i m dng, N l s khng i m m c a f (x). Khi N l s khng i m dng c a f (x). Do a th c f (x) ch c cc nghi m th c nn v i cc k hi u nh b 3.2, ta c n = N + + + N hay n = N + + N v 0 (n)(W + +W ) = (N + +N )(W + +W ) = (N + W + )+(N W ).

Theo quy t c d u Descartes, ta c (N + W + ) Do (N + W + ) = 0 v (N W ) = 0. Suy ra N + = W + t c l N = W. V d 3.49. Cho a th c f (x) = a0 + a1x + a2x2 + + an xn tho mn cc i u ki n a0 = 0, an = 0 v 2m h s lin ti p u b ng 0, v i m Z+ , m < Ch ng minh r ng a th c f (x) c t nh t 2m nghi m ph c. L i gi i. V i cc k hi u nh b 3.2 v k hi u N + , N l n l t l s nghi m dng v m c a f (x). Theo quy t c d u Descartes, ta c W + W+ + + W N +, W N+ + + N N . Suy ra n . 2 0 v (N W ) 0.

3.2. Cc bi ton v a th c

139

Do N + + + N l s nghi m th c c a f (x) nn s nghi m ph c c a f (x) l n (N + + + N ) Theo l i gi i b 3.2 ta c


L C

n (W + + + W ).

n (W + + + W ) =

(l k 1) +

{l k [1 sign (ak al )]}.

Do n u ak = 0, ak+1 = ak+2 = = ak+2m = 0 th Ho c l k l v l k Ho c l k ch n v l k 2m + 1 ; 2m + 2. 2m.

V y s nghi m ph c c a a th c f (x) l n (N + + + N ) V d

3.50. K hi u H2 l t p h p t t c cc a th c h s th c P2 (x) b c

hai v i h s t do b ng 1 (t c P2 (0) = 1) v c nghi m th c. Gi s 1 = 1 + p1 x + p2 x2 + + pm xm + P2 (x) Ch ng minh r ng v i m i P2 (x) H2, cc a th c P2m (x) := 1 + p1 x + p2 x2 + + p2m x2m, m N u khng c nghi m th c. L i gi i. Khng m t tnh t ng qut, xt a th c P2 (x) = (1 1x)(1 2 x), trong 1 2 = 0. Khi 1 1 = P2 (x) (1 1x)(1 2 x) 1 2 1 1 = 1 2 2 (1 1 x) 1 (1 2 x) 1 2 1 = 1 2 2

k xk 1
k=0

1 1

k xk 2
k=0

140

Chng 3. M t s

ng d ng c a s ph c trong i s

1 2 1 2 2 2 3 3 2 k k k 1 2 1 2 2 = + x+ x + + 1 x + 1 2 1 2 1 2 1 2 1 2 =1 + (1 + 2 )x + (2 + 1 2 + 2 )x2 + + 1 2
i+j=k

i j xk + 1 2

Suy ra P2 (x) =1 + (1 + 2 )x + (2 + 1 2 + 2 )x2 + + 1 2


i+j=k

i j xk 1 2 (i N, j N).

+ +
i+j=2m1

i j x2m1 1 2

+
i+j=2m

i j x2m 1 2

Xt tch P (x)P2 (x) = = 1 (1 + 2 )x + 12 x2 1 + (1 + 2)x + (2 + 1 2 + 2 )x2 1 2 + +


i+j=k

i j xk + + 1 2
i+j=2m1

i j x2m1 + 1 2
i+j=2m

i j x2m . 1 2

Ta th c hi n php nhn a th c, r i thu g n cc n th c ng d ng. Sau y ta ti n hnh thu g n cc n th c b c k (k trn. i j xk (1 + 2 ) 1 2


i+j=k i+j=k1

2m) sinh ra t tch hai a th c

i j xk + 1 2 1 2
i+j=k2

i j xk 1 2 i+1 j+1 xk = 0. 1 2

=
i+j=k

i j 1 2
i+j=k1

i+1 j 1 2
i+j=k1

i j+1 + 1 2
i+j=k2

Ti p theo, ta thu g n cc n th c b c 2m + 1 (1 + 2 )
i+j=2m

i j x2m+1 + 1 2 1 2
i+j=2m1

i j x2m+1 1 2 i+1 j+1 x2m+1 2 1


i+j=2m1

=
i+j=2m

i+1 j 2 1

i+j=2m

i j+1 1 2

=
i+j=2m+1

i j x2m+1 . 1 2

3.2. Cc bi ton v a th c

141

M t khc, khi nhn hai a th c trn ta th y ch xu t hi n m t h ng t b c 2m + 2 l 1 2 x2


i+j=2m

i j x2m = 1 2
i+j=2m

i+1 j+1 x2m+2 . 1 2

Ta thu c P (x)P2 (x) = 1


i+j=2m+1

i j x2m+1 + 1 2
i+j=2m

i+1 j+1 x2m+2. 2 1

Nh v y trong s cc h s c a a th c H(x) = P (x)P2 (x) c 2m h s lin ti p b ng 0. Theo bi ton 3.49 th H(x) c t nh t 2m nghi m ph c nhng deg H(x) = 2m + 2 nn H(x) c khng qu hai nghi m th c. Theo bi, a th c P (x) c hai nghi m th c nn P2 (x) khng c nghi m th c. Ti p theo, ta ch ng minh nh l quan tr ng sau y m t m t s l p a th c khng c nghi m th c. nh l 3.5 (Laguerre).
2

K hi u Hn l t p h p a th c h s th c Pn (x)

b c n, (n > 0) v i h s t do b ng 1 v c t t c cc nghi m u th c. Gi s 1 = 1 + p1 x + p2 x2 + + pk xk + . Pn (x) Khi , ng v i m i Pn (x) Hn , cc a th c d ng P2m (x) := 1 + p1 x + p2 x2 + + p2m x2m, m N u khng c nghi m th c. Ch ng minh. Gi s Pn (x) = 1 + a1x + + an xn , v i an = 0 l a th c thu c Hn .
2

N.Laguerre (1834-1886) l nh ton h c Php.

142

Chng 3. M t s

ng d ng c a s ph c trong i s

T gi thi t, ta c 1 = Pn (x)(1 + p1 x + p2 x2 + + p2m x2m + p2m+1 x2m+1 + ) hay 1 = Pn (x)[P2m(x) + p2m+1 x2m+1 + ]. T h th c cu i ny, ta thu c Pn (x)P2m (x) = 1 p2m+1 x2m+1 + So snh ( ng nh t th c theo lu th a) hai v , ta c ng th c Pn (x)P2m (x) = 1 + q2m+1 x2m+1 + q2m+2x2m+2 + + q2m+n x2m+n , trong q2m+1 = p2m+1 , q2m+n = p2m an . Theo bi ton 3.49, th a th c H(x) := Pn (x)P2m (x) c t nh t 2m nghi m ph c, t c l H(x) c khng qu n nghi m th c (do deg H(x) = 2m + n). Nhng theo gi thi t th a th c Pn (x) c n nghi m th c nn a th c P2m (x) khng c nghi m th c. Ti p theo, ta pht bi u k t qu quan tr ng sau y nh l m t h qu tr c ti p suy ra t nh l Laguerre. H qu 3.1. Gi s Pn (x) l a th c th c b c n (n > 0) v i h s t do b ng 1 (Pn (0) = 1) v c cc nghi m u th c. Khi , ng v i m i m (m N ) cho tr c, lun t n t i a th c P2m (x) b c 2m khng c nghi m th c sao cho Pn (x)P2m (x) = 1 + q1x + q2x2 + + q2m+n x2m+n , trong q1 = q2 = = q2m = 0.

3.2. Cc bi ton v a th c

143

Ch ng minh. Do Pn (x) l a th c b c n (n > 0) v i h s t do b ng 1, c cc nghi m u th c nn 1 = 1 + p1 x + p2 x2 + + p2m x2m + p2m+1 x2m+1 . Pn (x) Theo nh l 3.5 v i m i m N lun t n t i a th c P2m (x) = 1 + p1 x + p2 x2 + + p2m x2m khng c nghi m th c. Khi , 1 = P2m (x) + p2m+1 x2m+1 . Pn (x) Suy ra Pn (x)P2m (x) = 1 p2m+1 x2m+1 + . ng nh t h s theo t ng b c lu th a hai v , ta thu c

q2m+1 = p2m+1 , , q2m+n = p2m an . Vy Pn (x)P2m (x) = 1 + q1x + q2x2 + + q2m+n x2m+n , trong q1 = q2 = = q2m = 0. Sau y ta m t m t l p a th c b c l c duy nh t m t nghi m th c (n). V d 3.51. K hi u Hn l t p h p a th c th c Pn (x) b c n, (n > 0) v i h

s t do b ng 1 (t c Pn (0) = 1) v c cc nghi m u th c. Gi s 1 = 1 + p1 x + p2 x2 + + pk xk + Pn (x) Ch ng minh r ng ng v i m i Pn (x) Hn , cc a th c P2m+1 (x) := 1 + p1 x + p2 x2 + + p2m+1 x2m+1 , m N u c duy nh t m t nghi m th c (n).

144

Chng 3. M t s

ng d ng c a s ph c trong i s

L i gi i. Tng t nh cch ch ng minh nh l trn, ta gi s Pn (x) = 1 + a1 x + + an xn l a th c thu c Hn v 1 = Pn (x)(1 + p1 x + p2 x2 + + p2m+1 x2m+1 + p2m+2 x2m+2 + ). Do 1 = Pn (x)[P2m+1 (x) + p2m+2 x2m+2 + ]. T h th c ny, ta suy ra Pn (x)P2m+1 (x) = 1 p2m+2 x2m+2 + . So snh ng nh t th c theo lu th a hai v , ta thu c ng th c

Pn (x)P2m+1 (x) = 1 + q2m+2x2m+2 + q2m+3 x2m+3 + + q2m+n+1 x2m+n+1 , trong q2m+2 = p2m+2 , q2m+n+1 = p2m+1 an . Theo bi ton 3.49 th a th c H(x) := Pn (x)P2m+1 (x) c t nh t 2m nghi m ph c, t c l H(x) c khng qu n + 1 nghi m th c. V a th c Pn (x) c n nghi m th c, nn a th c P2m+1 (x) s c khng qu m t nghi m th c. M t khc, P2m+1 (x) l a th c b c l nn n c t nh t m t nghi m th c. Do a th c P2m+1 (x) c duy nh t m t nghi m th c.

3.3

Phng trnh hm v i bi n i phn tuy n tnh

Ta kh o st l p cc phng trnh hm v i acgumen bi n i sinh b i hm phn tuy n tnh th c d ng f ((x)) = af (x) + b,

3.3. Phng trnh hm v i bi n i phn tuy n tnh

145

trong (x) = x + , = 0. x+

3.3.1

M t s tnh ch t c a hm phn tuy n tnh

Tr c h t, ta kh o st phng trnh i s v i h s th c d ng m = x, m = 0. x+ Phng trnh (3.43) tng ng v i phng trnh b c 2 x2 + x m = 0. m = 0. Phng trnh (3.44) c nghi m th c khi v ch khi := 2 + 4m 0. (i) N u = 0 th phng trnh (3.44) c nghi m kp x0 = 2 (ii) N u > 0 th phng trnh (3.44) c 2 nghi m th c phn bi t x1,2 = 2 2 Trong tr ng h p khi < 0 th phng trnh (3.44) c 2 nghi m ph c lin h p z1,2 = 2 i 2 (3.44) (3.43)

Ti p theo, ta ch ra cch t n s ph a phng trnh i s t ng qut sinh b i hm phn tuy n tnh (x) d ng x + = x, = 0 x+ v phng trnh d ng (3.43). Ta s d ng cc ng nh t th c sau x + =+ x+ x+ (3.45)

146

Chng 3. M t s

ng d ng c a s ph c trong i s

v vi t phng trnh d ng (3.45) d i d ng + hay = t, t + ( + ) trong t = x . R rng phng trnh (3.46) c d ng (3.43). Tr ng h p c bi t khi + = 0 th phng trnh (3.46) c d ng n gi n + 2 =t t v hm phn tuy n tnh tng ng (x) = c tnh ch t c bi t ((x)) x, t c hm (x) l php bi n i i h p. 3.3.2 ng c u phn tuy n tnh. ph n trn, y ta s trnh by cc x + x+ (3.47) (3.46) =x+ = (x ) + , x+ (x ) + ( + )

nh x phn tuy n tnh c c p tnh ch t c b n nh t c a nh x .

nh x phn tuy n tnh c xc nh b i h th c w= az + b , cz + d ad bc = 0, (3.48)

trong a, b, c, d l cc s ph c. V i i u ki n ad bc = 0 ta c w const. Trong cng th c (3.48) n u c = 0 cn d = 0 th a b b. w = z + = az + d d l m t hm nguyn.

3.3. Phng trnh hm v i bi n i phn tuy n tnh

147

nh l 3.6. nh x phn tuy n tnh (3.48) l m t php ng phi t C ln C. Ch ng minh 1. Tr ng h p c = 0 l hi n nhin. 2. Ta xt tr ng h p c = 0. Gi i phng trnh (3.48) i v i z ta c z= dw b , cw + a ad bc = 0. (3.49)

l hm ng c c a (3.48). nh x (3.49) n tr trong m t ph ng C v l nh x phn tuy n tnh. Do (3.48) n tr m t - m t trn C. d Tnh lin t c c a (3.48) t i cc i m z = , l hi n nhin. B ng cch t c w() = a , c w d c =

ta th y r ng (3.48) lin t c trn C. nh l c ch ng minh. nh l 3.7. nh x phn tuy n tnh b o gic kh p ni trn C. d Ch ng minh. i v i tr ng h p z = , tnh b o gic suy ra t nh n c xt r ng t i cc i m ad bc dw = = 0. dz (cz + d)2 d v l gc c gi a 1 v 2 t i i m y. Suy ra r ng gc gi a cc nh 1 v 2 c a 1 v 2 d tng ng qua nh x (3.48) t i i m w = (tng ng v i z = ) l b ng c v By gi gi s hai ng cong 1 v 2 i qua i m z = lim d az + b z c cz + d 1 z+ d c c = 0. = lim d az + b z c

Tr ng h p z = cng c ch ng minh tng t .

148

Chng 3. M t s

ng d ng c a s ph c trong i s

nh ngha 3.1. nh x phn tuy n tnh bi n mi n D ln mi n D c g i l ng c u phn tuy n tnh, cn cc mi n D v D c g i l nh ng mi n ng c u phn tuy n tnh v i nhau. nh l 3.8. T p h p m i ng c u phn tuy n tnh l p thnh m t nhm v i php ton l p hm h p, ngha l 1) H p (tch) cc ng c u phn tuy n tnh l ng c u phn tuy n tnh. 2) nh x ng c c a ng c u phn tuy n tnh l ng c u phn tuy n tnh. Ch ng minh. Kh ng nh 2) l hi n nhin. Ta ch ng minh 1). Gi s a1z + b1 , c1z + d1 a2 + b2 , w= c2 + d2 = Khi a1z + b1 + b2 az + b (a1 a2 + c1 b2)z + (b1 a2 + d1 b2) c1z + d1 w= = , = a1z + b1 (a1 c2 + c1 d2 )z + (b1 c2 + d1 d2 ) cz + d c2 + d2 c1 z + d1 a2 trong ad bc = (a1d1 b1 c1)(a2 d2 b2c2 ) = 0. Nh n xt 3.1. Hi n nhin r ng nhm cc ng c u phn tuy n tnh l nhm 1 khng giao hon. Th t v y, gi s w(z) = , (z) = z + 1. z Khi w((z)) = Do w((z)) = (w(z)). 1 , z+1 (w(z)) = 1 + 1. z a1 d1 b1c1 = 0, a2d2 b2c2 = 0.

3.3. Phng trnh hm v i bi n i phn tuy n tnh

149

V qua php chi u n i c ng th ng l n ng trn trn C u tng ng v i ng trn trn m t c u Riemann nn ta c th quy c g i ng th ng hay ng trn trn m t ph ng ph c u l " ng trn" trn C (ta xem ng th ng trn C l ng trn trn C i qua i m ), v g i hnh trn, ph n ngoi hnh trn v n a m t ph ng (hnh trn v i bn knh v cng) u l "hnh trn" trn C. S(a, R) = {|z a| < R} hnh trn, S (a, R) = {|z a| > R} ph n ngoi hnh trn, P (R, ) = z C : Re(ei z) > R l n a m t ph ng.

Th t v y, t e+i = cos + i sin , z = x + iy, ta c P (R, ) = {(x, y) R2 : x cos + y sin > R}. l n a m t ph ng. nh l 3.9. ng c u phn tuy n tnh b t k bi n "hnh trn" (" ng trn") thnh "hnh trn" (tng ng thnh " ng trn"). Ni cch khc: "hnh trn" v " ng trn" u l b t bi n c a nhm cc ng c u phn tuy n tnh. Ch ng minh. nh x phn tuy n tnh c th bi u di n d i d ng h p c a cc nh x : w= a bc ad + ; c c2 = 1 ; =z+ d , c

1 trong c hai nh x tuy n tnh v nh x = . i v i cc nh x tuy n 1 tnh nh l 3.9 l hi n nhin. Ta ch c n xt php ngh ch o w = . z 1. Ta xt tr ng h p hnh trn S(a, R). nh c a n s l 1 a < R, w |1 aw| < R|w|, |1 aw|2 < R2 |w|2

150

Chng 3. M t s

ng d ng c a s ph c trong i s

1 2Re(aw) + |a2||w|2 < R2 |w|2 . Ti p theo ta xt ba tr ng h p sau a) |a| > R. Ta c (|a|2 R2 )|w|2 2Re(aw) + 1 < 0 aw |a|2 |a|2 1 2 |w| 2Re 2 + < 2 |a| R2 (|a|2 R2 )2 (|a|2 R2 )2 |a| R2 2 R2 a w 2 < |a| R2 (|a|2 R2 )2 R a < 2 w 2 2 |a| R |a| R2 l hnh trn. b) Gi s |a| < R. Tng t nh trn ta c w |a|2 a R > 2 2 R R |a|2

c) Gi s |a| = R. t a = |a|ei, = arg a, ta c: Re(aw) > l n a m t ph ng. 2. i v i ph n ngoi hnh trn A(a, R) nh l c xt tng t . 3. By gi ta xt php nh x n a m t ph ng Re(ei z) > R, R > 0. nh c a n s l Re ei v do 2R|w|2 + 2Re(ei w) > 0 |w|2 + 2Re w+ ei 2R
2

1 1 Re(ei w) > 2 2|a|

1 w

> R Re ei

w |w|2

> R Re(eiw) > R|w|2 ,

1 1 ei w + > 2 2R 4R 4R2 1 1 ei > > , w+ 2 4R 2R 2R

l ph n ngoi hnh trn. Php nh x n a m t ph ng Re(ei z) > R > 0 c xt tng t .

3.3. Phng trnh hm v i bi n i phn tuy n tnh

151

1 . a i m ny thu c nh hnh trn S(a, R) cng v i m t ln c n no c a n. Nh n xt 3.2. Trong m i tr ng h p, i m a c nh x thnh i m nh l 3.10. nh x phn tuy n tnh bi n mi n thnh mi n. Ch ng minh. D = (B). 1. Ch ng minh D l t p h p m . V i m i w0 D, t n t i duy nh t i m z0 B sao cho (z0) = w0. Gi s U (z0 ) B l ln c n c a i m z0 (hnh trn v i tm z0 n u z0 = ho c ph n ngoi hnh trn n u z0 = ). Khi theo nh l 3.9 ta c (U (z0 )) l hnh trn" ch a i m w0 cng v i m t ln c n no c a n. Nh v y w0 l i m trong c a D v do D l t p h p m . 2. Ch ng minh D l t p h p lin thng. V B l t p lin thng nn t nh l 3.6 suy ra r ng D l t p h p lin thng. Nh v y D l t p h p m lin thng, ngha l: D l m t mi n. nh l 3.6, 3.7 v 3.9 l nh ng tnh ch t c trng c a nh x phn tuy n tnh. Ngoi tnh b o gic v b o ton ng trn, nhm cc ng c u phn tuy n tnh cn c nh ng b t bi n khc n a. ng c u phn tuy n tnh (3.48) ch a ba tham s ph c lt s c a ba trong b n h s a, b, c, d v i h s th t (= 0). Cc tham s ny c xc nh n tr b i i u ki n: ba i m cho tr c z1, z2, z3 c a m t ph ng ph c (z) bi n thnh ba i m w1 , w2, w3 c a m t ph ng ph c (w). i u c suy ra t nh l sau y. nh l 3.11. T n t i ng c u phn tuy n tnh duy nh t bi n ba i m khc nhau z1, z2, z3 C thnh ba i m khc nhau w1 , w2, w3 C tng ng. ng Gi s B l mi n, w = (z) l nh x phn tuy n tnh,

152

Chng 3. M t s

ng d ng c a s ph c trong i s

c u c xc nh theo cng th c z z1 z3 z2 w w1 w3 w2 = w w2 w3 w1 z z2 z3 z1 Ch ng minh 1.Tnh duy nh t. Gi s ta c hai ng c u w1 (z) v w2(z) th a mn cc i u ki n c a nh l. Gi s 2 (w) l nh x ng c c a w2(z). Ta xt nh x 2 [w1(z)]. l m t ng c u phn tuy n tnh. ng c u ny c ba i m b t ng z1 , z2 v z3 v w1(zk ) = wk , 2 (wk ) = zk , Do n u t 2 [w1 (z)] = az + b th cz + d azk + b = zk , czk + d k = 1, 2, 3, k = 1, 2, 3. (3.50)

k = 1, 2, 3,

hay l
2 czk + (d a)zk b = 0,

k = 1, 2, 3.

a th c b c hai

v tri ch c th c ba nghi m khc nhau (z1 = x2 = z3 )

khi m i h s c a n u b ng 0, t c l a = d, b = c = 0 v 2 [w1(z)] z hay l w1 (z) w2 (z). 2. S t n t i. ng c u phn tuy n tnh th a mn i u ki n c a nh l c xc nh theo cng th c (3.50). Th t v y, gi i phng trnh (3.50) i v i w ta thu c hm phn tuy n tnh. Ngoi ra khi th c p z = z1 v w = w1 vo eq3.50 th c hai v c a (3.50) u b ng 0. Th c p z = z3 v w = w3 vo (3.50) ta thu c c hai v u b ng 1 v cu i cng, th c p z = z2 v w = w2 ta thu c c hai v u b ng . Trong hnh h c, bi u th c = z z1 z3 z1 : z z2 z3 z2

3.3. Phng trnh hm v i bi n i phn tuy n tnh

153

c g i l t s phi i u ha c a b n i m z, z1, z2 v z3. N u b n i m z1 , z2, z, z3 n m trn m t ng trn (ho c ng th ng) tht s phi i u ha l m t s th c. Th t v y a) N u cc i m z1, z2, z, z3 n m trn ng th ng = 0 + tei, < t < ta c: z1 = 0 + t1 ei, z2 = 0 + t2ei , z = 0 + t0ei , z3 = 0 + t3 ei v t (z1 , z2, z, z3) = z z1 z3 z1 t0 t1 t3 t1 : = : R. z z2 z3 z2 t0 t2 t3 t2

b) N u cc i m z, z1, z2, z3 n m trn ng trn = 0 + reit , r > 0, 0 t 2, ta c z1 = 0 + rei1 , z2 = 0 + rei2 , z3 = 0 + rei3 v t ta c ei0 ei1 ei3 ei1 : ei0 ei2 ei3 ei2 ei = ei
0 +1 2 0 +2 2

(z1 , z2, z, z3) =

ei ei

0 1 2 0 1 2

ei ei

0 1 2 0 1 2

ei : ei

2 +1 2 1 +3 2

ei ei

3 1 2 3 2 2

ei ei

3 1 2 3 2 2

0 1 0 1 sin 2 2 = 0 2 : 3 2 R. sin sin 2 2 sin T nh l 3.11 ta rt ra m t tnh ch t quan tr ng n a c a ng c u phn tuy n tnh. H qu 3.2. T s phi i u ha l m t b t bi n c a nhm cc ng c u phn tuy n tnh. nh ngha 3.2. 1. Hai i m z v z c g i l i x ng v i nhau qua ng trn = {|z z0 | = R} C n u chng c cc tnh ch t sau: a) z v z cng n m trn m t tia i t z0; b) |z z0 | |z z0| = R2 .

154

Chng 3. M t s

ng d ng c a s ph c trong i s

2. M i i m trn ng trn c xem l i x ng v i chnh n qua . T nh ngha 3.2 suy ra r ng cc i m i x ng qua ng trn lin h v i nhau b i h th c w = z0 + Th t v y, t bi u th c v a vi t suy ra |w z0| |z z0| = R2 v arg(w z0) = arg(z z0). Trong hnh h c s c p ta bi t r ng hai i m z v z i x ng v i nhau qua ng trn khi v ch khi m i ng trn C i qua z v z u tr c giao v i . Ta c nh l sau. nh l 3.12. Tnh i x ng tng h gi a cc i m l m t b t bi n c a nhm cc ng c u phn tuy n tnh. Ch ng minh. K t lu n c a nh l c suy t nh l 3.7 v 3.9. T s b t bi n c a tnh i x ng gi a cc i m suy ra r ng trong tr ng h p khi ng trn bi n thnh ng th ng, tnh i x ng trng v i khi ni m i x ng thng th ng. Ta minh h a vi c p d ng tnh b t bi n c a cc i m i x ng qua ng c u phn tuy n tnh b ng cc nh l sau y. nh l 3.13. ng c u phn tuy n tnh b t k bi n n a m t ph ng trn ln hnh trn n v u c d ng w = ei trong R l s th c ty . z , z Im > 0, (3.51) R2 z z0

3.3. Phng trnh hm v i bi n i phn tuy n tnh

155

Ch ng minh. Gi s ng c u phn tuy n tnh w = w(z) nh x n a m t ph ng trn Im z > 0 ln hnh trn {|w| < 1} sao cho w() = 0 (Im > 0). Ta nh n xt r ng i m w = 0 v w = s tng ng v i cc gi tr lin h p c a z, do c = 0 (v n u c = 0 th i m s tng ng v i i m ). b d Cc i m w = 0, w = s tng ng v i cc i m v . Do c th a c b d az vi t = , = v w = a c cz V cc i m c a tr c th c c nh n m trn ng trn n v , t c l |w| = 1 khi z = x R, cho nn a x a = =1 c x c v a = cei. Nh v y w = ei z z Ta ch ng minh r ng l ng c u ph i tm. Th t v y, n u z = x R th hi n nhin |w| = 1. N u Im z > 0 th z g n hn so v i (t c l |z | < |z |) v do |w| < 1. Nh n xt 3.3. Trong nh x (3.51) gc quay c a cc ng cong t i i m l b ng v t (3.51) ta c 2 arg w () = 2

nh l 3.14. M i ng c u phn tuy n tnh bi n hnh trn {|z| < 1} ln hnh trn {|w| < 1} u c d ng w = ei z , 1 z (3.52)

trong || < 1, R l s th c ty . Ch ng minh. Gi s ng c u phn tuy n tnh w = w(z) bi n hnh trn {|z| < 1} ln hnh trn {|w| < 1} sao cho w() = 0 (|| < 1). Theo tnh ch t b o ton i m i x ng, cc i m w = 0, w = tng ng v i cc i m lin

156

Chng 3. M t s

ng d ng c a s ph c trong i s

h p z = v z =

1 , || < 1. Do b = , a d 1 = , c || < 1,

v w= a z a z a z = = 1 c c z 1 c 1 z z

V cc i m c a ng trn n v ph i bi n thnh cc i m c a ng trn n v nn |w| = 1 khi |z| = 1. V z z = |z|2 nn zz = 1 khi |z| = 1. V s 1 z v 1 z lin h p v i nhau v |1 z| = |1 z| nn n u |z| = 1 th |1 z| = |1 z| |z| = |z zz| = |z |. Do khi |z| = 1 th ta c: z = 1. 1 z Nhng khi |w| = 1 cho nn c (3.52). Ta c n ch ng minh r ng l ng c u mu n tm. Th t v y n u z = ei v = r1 ei th |w| = 1 r1 ei ei ei r1ei = = 1. 1 r1 ei ei 1 r1 ei ei a a = 1 v = ei, R. Nh v y ta thu c c

N u z = rei (r < 1) th
2 2 |z a|2 |1 z|2 = r2 2rr1 cos( ) + r1 (r1 r2 2r1 r cos( ) + 1) 2 = (r2 1)(1 r1 ) < 0

v do |z |2 |1 z|2 < 0 v |w| < 1.

3.3. Phng trnh hm v i bi n i phn tuy n tnh

157

Nh n xt 3.4. V dw dz = ei
z=

1 , 1 ||2

|| < 1,

cho nn v m t hnh h c b ng gc quay c a nh x (3.52) t i i m : = arg dw dz .


z=

T cng th c (3.52) ta cn rt ra h th c dw dz =
z=

1 1 ||2

v do gin d n n khi i m d n n bin c a hnh trn n v . Nh n xt 3.5. Php ng c u bi n hnh trn {|z| < R} ln hnh trn {|w| < R } c d ng w = RR ei z , z R2 || < R, R.

V d

3.52. Gi s U1 = {|z| < 1}, U2 = {|z 1| < 1} v D = U1 U2 . Tm

ng c u bi n mi n D ln n a m t ph ng trn. L i gi i. Giao i m c a cc cung trn gi i h n mi n D l cc i m sau: 1 1 3 3 , a = i a= +i 2 2 2 2 Gi s cung trn i qua i m z = 1 c k hi u l 1 v cung trn i qua i m z = 0 l 2 . Ta p d ng cc nh x trung gian sau 1. nh x z z1 = z 3 1 i 2 2 1 3 +i 2 2

158

Chng 3. M t s

ng d ng c a s ph c trong i s

bi n mi n cho D thnh m t gc trong m t ph ng z1 v i nh l z1 = 0. V 2 nn gc gi a hai cung trn 1 v 2 t i cc i m a cng nh a u b ng 3 2 . D dng th y r ng m c a gc v a thu c b ng 3 3 1 i 1 2 2 3 1 z1(1) = = +i 2 2 1 3 1 +i 2 2 1 3 z1(0) = i 2 2 v do gc - nh thu c c c nh i qua i m z1(1) v z1 (0). Ta k hi u gc l D(z1 ). z1 bi n gc D(z1 ) thnh gc c m c nh trng t 1 3 v i ph n dng c a tr c th c, cn c nh kia i qua i m + i 2 2 3 2 3 2 3. nh x c n tm c d ng w = z2 gc c m =! . 3 2 H p nh t 1) - 3) ta thu c 3 2 2z 1 + i 3 w= 2z 1 i 3 v hi n nhin ch l m t trong cc hm th c hi n nh x ph i tm. V d 3.53. nh x mi n D l gc {0 < arg z < , 0 < < 2} v i nht 2. nh x quay z2 = e
2i 3

c t theo m t cung c a ng trn n v t i m z = 1 n i m z = ei , 0 < < (hy v hnh). L i gi i. Ta s d ng cc nh x trung gian sau y 1. nh x z1 = z bi n gc cho thnh gc D(z1 ) c m b ng v i nht c t thu c ng trn n v i t i m z = 1 n i m z = ei . 2. nh x phn tuy n tnh z2 = z1 1 z1 + 1
1

3.3. Phng trnh hm v i bi n i phn tuy n tnh

159

bi n mi n D(z1 ) thnh n a m t ph ng trn v i nht c t theo tr c o t g c t a n i m i tan . Ta k hi u mi n nh l D(z2 ). 2


2 3. nh x z3 = z2 bi n mi n D(z2 ) thnh m t ph ng v i nht c t theo ; R. Ta k hi u mi n thu c l D(z3 ). tan2 2 Hi n nhin hm c n tm c d ng

w=

= z3 + tan 2
2

z 1 z +1
1

+ tan2

. 2

k t thc ph n ny, ta ch ng minh r ng nh x phn tuy n tnh (3.48) az + b , ad bc = 0 bi n n a m t ph ng trn ln chnh n khi v ch w = cz + d khi m i h s a, b, c, d u l nh ng s th c th a mn i u ki n ad bc > 0. Gi s nh x (3.48) bi n n a m t ph ng trn ln chnh n. Ta xt ba i m khc nhau z1, z2 v z3 c a tr c th c trong m t ph ng z. nh c a ba i m ny l nh ng i m bin c a n a m t ph ng Im w > 0, t c l cc s wk = w(zk ), k = 1, 2, 3 l nh ng s th c. T , ta thu c h phng trnh v i cc h s th c xc nh a, b, c, d. Do v i s chnh xc n m t th a s no t h phng trnh tuy n tnh v a thu c d dng suy ra r ng cc h s c a (3.48) u l th c. V w = u + iv, z = x + iy nn khi y > 0 ta c v > 0. Thay w = u + iv, z = x + iy vo (3.48) ta c y(ad bc) (cx + d)2 + (cy 2)

v=

T suy ra ad bc > 0. Ng c l i, n u cc h s a, b, c v d u th c th tr c th c c a m t ph ng (z) c nh x ln tr c th c c a m t ph ng (w) v v ad bc > 0 nn n a m t ph ng trn c nh x ln n a m t ph ng trn.

160

Chng 3. M t s

ng d ng c a s ph c trong i s

3.3.3

Phng trnh hm sinh b i hm phn tuy n tnh

Bi ton t ng qut 3.1. Xc nh cc hm s f (x) th a mn i u ki n sau f x + x+ = af (x) + b, x R \ {}, (3.53)

trong , , ; a, b l cc h ng s th c, a = 0, = 0. Ta kh o st bi ton t ng qut (3.53) trong ba tr ng h p c trng i n hnh sau y: (i) Phng trnh (x) = x c hai nghi m th c phn bi t. (ii) Phng trnh (x) = x c 1 nghi m kp (th c). (iii) Phng trnh (x) = x khng c nghi m th c. Nh n xt r ng, phng trnh trong tr ng h p (iii) tng ng v i phng trnh (x) = x c hai nghi m (ph c) l cc s lin h p ph c c a nhau. Ta chuy n bi ton t ng qut 3.1 v bi ton t ng qut sinh b i hm b c nh t quen bi t m cch gi i bi t Bi ton t ng qut 3.2. Xc nh cc hm s f (x) th a mn i u ki n sau f (x + )) = af (x) + b, x R, trong , , a, b l cc h ng s th c, a = 0, = 0. ho c v d ng bi ton t ng qut sinh b i php i h p b c n d ng sau y. Bi ton t ng qut 3.3. Xc nh cc hm s f (x) th a mn i u ki n sau f x + x+ = af (x) + b, x R \ {}, (3.55) (3.54)

trong , , , a, b l cc h ng s th c, a = 0, = 0, v n (x) x, k+1 := (k (x)), 0 (x) := x.

3.3. Phng trnh hm v i bi n i phn tuy n tnh

161

Ti p theo, ta minh h a cch gi i ng v i cc tr ng h p thng qua cc bi ton c th sau y. T k t qu kh o st c a ph n tr c, ta ch c n xt cc phng trnh hm sinh b i (x) c d ng (x) = V d m , m = 0 x+

3.54. Xc nh cc hm s f (x) th a mn i u ki n sau f 1 2x = 2f (x) 1, x R \ {2}. (3.56)

L i gi i. Nh n xt r ng phng trnh 1 =x 2x c hai nghi m th c x = 1 v x = 2. S d ng php i bi n x1 = t, x2 ta thu c x=2+ V y (3.56) c d ng f hay g trong g(t) = f V d 2+ 3 t1 . 1 t 2 = 2g(t) 1, t R \ {2 ; 1}, (3.57) 2+
1 t 2

3 1 3 , =2+ 1 t1 2x t1 2

3 1

= 2f

2+

3 t1

1, t R \ {2 ; 1},

3.55. Xc nh cc hm s f th a mn i u ki n sau f 2 3x = 3f (x) + 2, x R \ {3}. (3.58)

162

Chng 3. M t s

ng d ng c a s ph c trong i s

L i gi i. Phng trnh 2 =x 3x c m t nghi m (th c) kp x = 1. S d ng php i bi n 1 = t, x1 ta thu c 1 2 1 x=1+ , =1+ t 3x t1 V y (3.58) c d ng f hay g(t 1) = 3g(t) + 2, t R \ {2 ; 1}, trong g(t) = f 1+ 1 t . 1+ 1 t1 = 3f 1+ 1 t + 2, t R \ {0 ; 1},

V d 3.56. Xc nh cc hm s f th a mn i u ki n sau f 2 2x = 2f (x) + 5, x R \ {2}. (3.59)

L i gi i. y l tr ng h p phng trnh hm v i nghi m c trng c a 2 = phng trnh sinh (x) = x khng c nghi m th c. Phng trnh sinh 2x x, c nghi m z1,2 = 1 i. S d ng php i bi n x 1 = t, ta thu c x = 1 + t, 1+t 2 =1+ 2x 1t

v vi t phng trnh (3.59) d i d ng f hay g 1+t 1t = 2g(t) + 5, x R \ {1}, (3.60) 1+ 1+t 1t = 2f (1 + t) + 5, t R \ {1}.

3.4. Bi t p

163

trong g(t) = f (1 + t). Xt phng trnh hm (3.60) ng v i tr ng h p (t) = (3.61) 1+t v phng trnh 1t sinh tng ng (t) = t c hai nghi m thu n o i. Ta vi t (t) = 1 + t tan2 1+t 4 = , 1t 1 t tan2 4

do (t) c tnh tu n hon ( i h p) b c b n, ngha l ((((t)))) t. V v y, phng trnh hm (3.60)-(3.61) a v h phng trnh tuy n tnh v c nghi m duy nh t g(t) 5 f (x) = 5, x R \ {2}.

3.4

Bi t p

Bi 3.1. Xc nh c (c C) sao cho phng trnh 1 + ix 1 ix c cc nghi m u th c. Bi 3.2. Cho a th c P (x) const. Ch ng minh r ng h phng trnh sau ch c khng qu h u h n s nghi m th c x P (t) sin tdt =
0 x 0 2002

=c

0 0.

P (t) cos tdt =

Bi 3.3. Cho s nguyn dng n v cc s ak , bk R. Ch ng minh r ng phng trnh


n

x+
k=1

(ak sin kx + bk cos kx) = 0

c nghi m trong kho ng ( ; ).

164

Chng 3. M t s

ng d ng c a s ph c trong i s

Bi 3.4. Cho M > 0 v cho tam th c b c hai f (x) = x2 + bx + c c cc h s n m trong [M ; M]. G i x1 , x2 l hai nghi m th c ho c ph c c a f (x). Ch ng minh r ng (1 + |x1|)(1 + |x2 |) 4 3M. Bi 3.5. Cho tam th c b c hai f (x) = ax2 + bx + c c cc nghi m u th c v a th c P (x) = a0 + a1x + + an xn R[x] c 3 nghi m th c. Ch ng minh r ng khi a th c Q(x) = aP (x) + bP (x) + cP (x) cng c t nh t ba nghi m th c. Bi 3.6. Cho cc s th c a, b, c, d, e, r tho mn i u ki n abcder = 0, ar + be + cd = 0. Gi i h phng trnh ( n x, y, z, u, v): xz y 2 xu yz xv yu yu z 2 xu yv zv u2 = = = = = . a b c d e r Bi 3.7. Cho s t nhin p = a0a1 . . . an l m t s nguyn t . Ch ng minh r ng a th c tng ng P (x) = a0xn + a1 xn1 + + an s khng c nghi m h ut .

3.4. Bi t p

165

Bi 3.8. Ch ng minh r ng m i nghi m c a phng trnh 1 + ix 1 ix


n

1 + ia , 1 n N, a R. 1 ia

Bi 3.9. Gi i phng trnh ix i+x


n

cot + i , 1 n N, R. cot i

Bi 3.10. Gi i cc phng trnh sau :


2 1. xn naxn1 Cn a2xn2 an = 0.

2. x5 + x4 + x3 + x2 + x + 1 = 0. 3. x5 + x4 + 2 x3 + 3 x2 + 4 x + 5 = 0, 0 = C. Bi 3.11. Gi i cc h phng trnh sau trong C : 1. 3. z 3 + w7 z 5w11 z 3 + w5 z 2w4 =0 =1; =0 =1; z 5 w7 =1 z 2 w3 = 0 ; 13 19 =1 z w 5 7 =1 4. z w 2 z + w2 = 2. 2.

Bi 3.12. Gi i h phng trnh sau x + 3x y = 3 x2 + y 2 y x + 3y = 0. x2 + y 2 Bi 3.13. Gi i h phng trnh sau x 1 12 3x + y y 1 + 12 3x + y Bi 3.14. Gi i h phng trnh sau x3 3xy 2 = 1 3x2y y 3 = 3.

=2 = 6.

Chng 4

S ph c trong cc bi ton s h c v t h p
4.1 Gi i phng trnh Diophant

Vnh cc s ph c nguyn Z[i] v ni chung l cc vnh s nguyn i s c nh ng ng d ng kh hi u qu trong vi c gi i cc bi ton v phng trnh Diophant. y ta th ng dng n tnh ch t quen thu c sau y: n u a, b

l cc s nguyn (nguyn i s ) nguyn t cng nhau v tch a.b l lu th a ng b c n th a, b k t h p v i m t lu th a ng b c n. V d 4.1. Tm t t c cc nghi m nguyn dng c a phng trnh x2 +1 = y 3. L i gi i. Ta c (x + i)(x i) = y 3. Ta s ch ng minh hai s x + i v x i l nguyn t cng nhau. Gi s tri l i c s nguyn t Gauss sao cho | x + i v | x i. Suy ra | 2i do | 2. V y N ()|N (2) = 4, suy ra N () ch n. V N ()|N (x + i) = x2 + 1 = y 3 nn y ch n do x l v x2 + 1 = y 3 chia h t cho 8. Nhng x2 + 1 = 2(mod 4). Ta c mu thu n. V y (x + i, x i) = 1. Nh th x + i k t h p v i m t l p phng no . V 1 = (1)3 , i = (i)3, (i) = i3
166

4.2. Rt g n m t s t ng t h p

167

nn chnh x + i l m t l p phng. Ta c x + i = (a + ib)3 = (a3 3ab2) + i(3a2b b3), suy ra x = a(a2 3b); 1 = b(3a2 b2 ) hay |b| = 1; |3a2 b2 | = 1. Ta thu c |3a2 1| = 1 hay a = 0, b = 1. Do x = 0, y = 1 l nghi m duy nh t c a phng trnh cho. V d 4.2. Ch ng minh r ng v i m i s t nhin n, t n t i cc s nguyn

(a, b, c) v i (a ; b) = 1 sao cho a2 + b2 = cn. L i gi i. L y x, y l hai s nguyn dng nguyn t cng nhau v gi s (x + iy)n = a + ib. Khi a2 + b2 = N (a + ib) = N ((x + iy)n) = (N (x + iy))n = (x2 + y 2)n . t x2 + y 2 = c, ta c ngay h th c a2 + b2 = cn .

4.2

Rt g n m t s t ng t h p

Cn nguyn thu b c n c a n v v i tnh ch t c b n l 1 + k + + k(n1) = 0 v i (k, n) = 1 c ng d ng kh hi u qu trong vi c rt g n cc t ng t h p. Ngoi ra cng th c Euler ei = cos + i sin c th a cc t ng l ng gic thnh cc c p s nhn ho c cng th c khai tri n nh th c. D i y chng ta xem xt hai v d tiu bi u V d 4.3. Tnh t ng
[n/3] 3k Cn . k=0

168

Chng 4. S ph c trong cc bi ton s h c v t h p

L i gi i. Xt a th c
n

P (x) = (1 + x)n =
k=0

k Cn xk .

Xt l cn nguyn thu b c ba c a n v , t c l 2 + + 1 = 0 th ta c 2k + k + 1 = 0 khi k khng chia h t cho ba v b ng ba n u k chia h t cho ba. V th


n [n] 3 k Cn (1 + k + 2k ) = 3 k=0 k=0 3k Cn .

P (1) + P () + P (2 ) = Cu i cng, do P (1) = (1 + 1)n = 2n , P () = 1 + 1 3 +i 2 2 1 3 i 2 2


n

=
n

3 1 +i 2 2 3 1 i 2 2

= cos
n

n n + i sin , 3 3 n n i sin , 3 3

P (2 ) = 1 +

= cos

nn ta c t ng c n tm b ng V d 4.4. Tnh t ng

1 n n 2 + cos . 3 3
n k Cn cos kx. k=0

L i gi i. Xt cc t ng
n n k Cn k=0

C= Ta c
n

cos kx, S =
k=0

k Cn sin kx.

n k Cn (cos kx + i sin kx) = k Cn eikx = (1 + eix )n k=0

C + iS =
k=0

= (1 + cos x + i sin x)n = 2 cos = 2 cos x 2


n

cos

nx nx + i sin . 2 2

x 2

cos

x x + i sin 2 2

4.3. Cc bi ton m

169

T C = 2 cos hay
n k Cn cos kx = 2 cos k=0

x 2

cos

nx , 2
n

x 2

cos

nx 2

V d

4.5. Ch ng minh r ng
m

2m

cos

2m

x=
k=0

k C2m cos 2(m k)x.

eix + eix Do L i gi i. Ta c e = cos x + i sin x v cos x = 2


ix 2m 2m k C2m (eix)k (eix )2mk = k=0 m1 m1 k C2m e2(km)ix + k=0 m1 k=m+1 k m C2m cos 2(m k)x + C2m cos 2(m m)x k=0 m k m C2m e2(km)ix + C2m k=0 k C2m e2(km)ix

22m cos2m x = (eix + eix )2m =

= =
k=0

k C2m cos 2(m k)x.

4.3

Cc bi ton m

S ph c c nh ng ng d ng r t hi u qu trong cc bi ton m. V vai tr trung tm trong k thu t ng d ng s ph c vo cc bi ton m ti p t c l i l cn nguyn thu c a n v . Ch l n u l cn nguyn thu b c n c a n v th ta c i) 1 + + + n1 = 0, ii) 1 + k + + k(n1) = 0 v i (k ; n) = 1. y chnh l tnh ch t quan tr ng c a cn nguyn thu th ng c s d ng.

170

Chng 4. S ph c trong cc bi ton s h c v t h p

V d

4.6 (PTNK 2009). Tm s t t c cc s c n ch s l p t cc ch s

3, 4, 5, 6 v chia h t cho 3. L i gi i. G i cn l s cc s c n ch s th a mn yu c u bi. G i l m t nghi m c a phng trnh x2 + x +1 = 0. Khi 3 = 1 v 2k + k +1 = 0 n u k khng chia h t cho 3, 2k + k + 1 = 3 n u k chia h t cho 3. Xt a th c P (x) = (x3 + x4 + x5 + x6)n . D th y cn chnh l b ng t ng cc h s c a cc s m chia h t cho 3 trong khai tri n c a P (x). Ni cch khc, n u
6n

P (x) =
k=0 2n

ak xk

th cn =
k=0

a3k . M t khc ta c
6n 2n

P (1) + P () + P ( ) =
k=0

ak (1 + + ) =
k=0

2k

3a3k .

Cu i cng, do P (1) = 4n , P () = P (2 ) = 1 nn ta c
2n

cn =
k=0

a3k =

4n + 2 3

V d

4.7 (IMO 1995). Cho p l m t s nguyn t l . Tm s cc t p con A

c a t p h p {1, 2, . . . , 2p}, bi t r ng (i) A ch a ng p ph n t ; (ii) T ng cc ph n t c a A chia h t cho p. L i gi i. Xt a th c P (x) = xp1 + xp2 + + x + 1. a th c ny c p 1 nghi m ph c phn bi t. G i l m t nghi m b t k c a P (x). Ch r ng , 2 , . . . , p1 l p 1 nghi m phn bi t c a P (x) v p = 1. Do , theo nh l Vite, xp1 1 = (x )(x 2 ) (x p1 ).

4.3. Cc bi ton m

171

Xt a th c Q(x) = (x )(x 2 ) (x 2p ) v g i H = {A {1, 2, . . . , 2p} : |A| = p}. Gi s Q(x) =


k=0 2p

ak xk .

Khi ap =
AH

S(A) , S(A) =
xA

x.

V n u S(A) = j(mod p) th S(A) = j nn


p1

ap =
j=0

nj j ,

trong nj l s cc A H sao cho S(A) = j(mod p). M t khc Q(x) = (xp 1)2 , suy ra ap = 2. Thnh th
p1

nj j = 2.
j=0

(4.1)

Xt a th c
p1

R(x) =
j=0

nj xj + n0 2.

T ng th c (4.1), suy ra l m t nghi m c a R(x). V deg P = deg R v l m t nghi m b t k c a P (x) nn P (x) v R(x) ch sai khc nhau h ng s nhn. T np1 = np2 = = n1 = n0 2, suy ra
p C2p 2 np1 + np2 + + n1 + n0 2 = . n0 2 = p p

172

Chng 4. S ph c trong cc bi ton s h c v t h p

V y p s c a bi ton l
p C2p 2 n0 = 2 + p

V d

4.8 (Rookie Contest 1999). Cho n l s nguyn t v a1, a2, . . . , am

l cc s nguyn dng. G i f (k) l s cc b m s (c1 , c2 , . . . , cm ) tho mn i u ki n 0 ci ai v c1 + c2 + + cm = k(mod n). Ch ng minh r ng f (0) = f (1) = = f (n 1) khi v ch khi n | aj v i j no thu c {1, 2, . . . , m}. L i gi i. Xt = cos
m

2 2 + i sin . Ch r ng h th c sau ng n n X c1 ++cn


1ck ak

(X + X 2 + + X ak ) =
k=1

v
m

f (0) + f (1) + f (n 1)n1 =


1ck ak

X c1 ++cn =
k=1

( + 2 + + ak ).

T y suy ra f (0) = f (1) = = f (n 1) khi v ch khi f (0) + f (1) + + f (n 1)n1 = 0.


m

i u ny tng ng v i
k=1

(+2 + +ak ) = 0, t c l +2 + +ak = 0

v i j no thu c {1, 2, . . . , m}. T y suy ra aj 1 = 0, t c l n | aj .

4.4

S ph c nguyn v

ng d ng trong l thuy t s

Ta xt bi ton tm t t c cc c p s nguyn (x ; y) tho mn phng trnh x2 y 3 = 1. S d ng s ph c ta c th gi i n b ng cch tr c h t nh n xt phng trnh trn tng ng v i x2 + 1 = y 3 hay (x + i)(x i) = y 3 .

4.4. S ph c nguyn v ng d ng trong l thuy t s

173

Tch hai s l m t l p phng th b n thn m i s cng l m t l p phng. Thnh th x + i = (a + bi)3, trong a, b Z, hay x + i = (a3 3ab2 ) + i(3a2b b3). Tch ph n th c v ph n o hai v , ta thu c

x = a(a2 3b) ; 1 = b(3a2 b2). T phng trnh 1 = b(3a2 b2) suy ra |b| = 1, |3a2 b2| = 1 |3a2 1| = 1 hay a = 0. Suy ra x = 0 v y = 1. V y (0 ; 1) l nghi m nguyn duy nh t c a phng trnh cho. L i gi i trn cho p s ng v s ph c c dng tr l i cu h i v s t nhin! Tuy nhin, v phng di n logic ch t ch th cc l p lu n trn khng th ch p nh n c v ta m c nhin dng cc tnh ch t c a t p s nguyn Z cho m t t p h p khc, c th l t p cc s d ng a + bi v i a, b l s nguyn. Sau y chng ta ch ng t r ng t p cc s ph c d ng a + bi c r t nhi u tnh ch t nh t p Z do chng ta c th lm s h c trn cc s ph c ny. V i u quan tr ng hn l nh n chng ta c th gi i c cc bi ton v t p Z m n u ch ng trong Z ta s khng th tm c l i gi i. nh ngha 4.1. S ph c c d ng a + bi nguyn (hay s nguyn Gauss). T p t t c cc s ph c nguyn ( c Gauss kh o st u tin (1832) c k hi u l Z[i] v th ng c k hi u b ng cc ch ci Hyl p , , , . . . . R rng Z Z[i]. Nh v y, m t s nguyn thng th ng l s ph c nguyn c ph n o b ng 0. Trn m t ph ng ph c, t p Z[i] l t p cc i m (a ; b) c t a nguyn. D ki m tra r ng t ng, hi u, tch c a hai s ph c nguyn l i l m t s ph c nguyn. T suy ra t p Z[i] ng i v i php c ng, tr , nhn. a, b Z c g i l s ph c

174

Chng 4. S ph c trong cc bi ton s h c v t h p

Ngay t nm 300 tr c cng nguyn Euclide nh n th y r ng khi ni m chia h t v khi ni m s nguyn t l hai khi ni m quan tr ng nh t c a s nguyn. Nhi u tnh ch t c v r t hi n nhin c a cc s nguyn l i lin quan n nh ng suy lu n kh tinh vi. Ch ng h n, ch ng minh r ng n u tch c a hai s nguyn ab chia h t cho s nguyn t p th ho c a ho c b ph i chia h t cho p, ta ph i a ra khi ni m u c chung l n nh t c a hai s v dng thu t ton Euclide m t c chung l n nh t. Cng t ng i ta thi t l p nn nh l c b n c a s h c: N u n l m t s nguyn th n c th phn tch m t cch duy nh t thnh tch cc s nguyn t . Nh v y, mu n lm s h c trn cc s nguyn ph c Z[i], ta cng ph i xy d ng c khi ni m chia h t v khi ni m s nguyn t trong Z[i]. 4.4.1 Tnh ch t chia h t trong t p cc s ph c nguyn

nh ngha 4.2. Cho , Z[i] trong = 0. Ta ni chia h t hay chia h t cho n u t n t i Z[i] sao cho = . N u chia h t , ta . . ni l m t u c c a v vi t | hay l m t b i c a v vi t . . S ph c nguyn c g i l n v n u l u c c a m i s ph c nguyn . Chu n c a s ph c nguyn = a + bi, k hi u b i N (), c xc nh b i cng th c sau N () = ||2 = = a2 + b2 . Tnh ch t 4.1. N u , , Z[i] sao cho |, | th | Tnh ch t 4.2. N u , , z1, z2 Z[i] sao cho |, | th | (z1 + z2). Tnh ch t 4.3. Chu n N () l m t s t nhin. N () = 0 khi v ch khi = 0. Tnh ch t 4.4. N u = th N () = N ()N (). Ni ring, n u chia h t cho th N () chia h t cho N ().

4.4. S ph c nguyn v ng d ng trong l thuy t s

175

Tnh ch t 4.5. T p U t t c cc n v c a Z[i] l U = {1, i}. T p U l p thnh m t nhm nhn, ng i v i php l y lin h p v s ph c nguyn l m t n v khi v ch khi N () = 1. nh l 4.1 (Thu t chia Euclide). Cho , l hai s ph c nguyn b t k v i = 0. Khi t n t i cc s ph c nguyn , sao cho = + , Ch ng minh. Gi s 0 N () < N (). (4.2)

= u + iv v i u, v Q. Ta c th ch n x, y Z sao cho x g n u nh t v y g n v nh t t c l 1 |u x| , 2 1 |v y| . 2

t = x + iy Z[i], = . Khi ta c = + . Ta ch ng t r ng N () < N (). Th t v y N () = | |2 = |( M t khc | 1 1 )|2 = |(u x) + (v y)i|2 = |u x|2 + |v y|2 < + < 1. 4 4 )|2 = N ()| )|2.

Do N () < N (). Ta c th minh ho hnh h c thu t chia Euclidean nh sau: Cc b i c a s ph c nguyn c bi u di n b i cc nh c a m t l i cc vung v i vung c b n l vung v i b n nh l 0, , i, (1 + i). S nguyn s n m trong m t vung no . Ph n d chnh l hi u gia v i nh g n nh t v i c a vung. N () < N () v d ch ng minh c r ng trong m t hnh vung kho ng cch t m t i m b t k c a hnh vung t i nh g n nh t v i n th ph i b hn c nh c a hnh vung.

176

Chng 4. S ph c trong cc bi ton s h c v t h p

Ch . Bi u di n (4.2) l khng duy nh t. Ni cch khc, trong thu t chia ni trn th ph n d v thng s l khng duy nh t. Ch ng h n 5 + 4i = (3 + 2i) + (2 + 2i) = 2(3 + 2i) + (1), N (3 + 2i) = 13 > N (2 + 2i) = 8, N(3 + 2i) = 13 > 1 = N (1). nh ngha 4.3. Cho , Z[i] l hai s ph c nguyn khc khng. Chng c g i l nguyn t cng nhau n u t t c cc c chung c a v ch l {1 ; i}. Ni cch khc, n u | v | th {1 ; i}. nh l 4.2. Gi s v nguyn t cng nhau. Khi t n t i cc s ph c nguyn 0 , 0 sao cho 0 + 0 = 1. Ch ng minh. t A = { + }, trong , ch y trn t p Z[i] v l y A l ph n t m chu n N () c gi tr nh nh t trong cc chu n c a cc ph n t khc khng trong A. Theo thu t chia Euclide ta tm c , Z[i] sao cho = + , 0 N () < N ().

Ta ch ng t r ng A. Th t v y, v A nn = 1 + 1. Do = = (1 + 1) = (1 1 ) + (1) = 2 + 2. V r ng N () < N (), A v N () c gi tr nh nh t trong cc chu n c a cc ph n t khc khng trong A nn ta ph i c = 0. Do = . V y l m t c c a . Tng t , cng l m t u c c a t c l l m t c chung c a v . V r ng v l nguyn t cng nhau nn ph i l m t n v . Vy 1 = = (1 ) + (1 ) = 0 + 0. nh ngha 4.4. Cho , Z[i] l hai s ph c nguyn khc khng. Ta ni r ng Z[i] l u c chung l n nh t (UCLN) c a v v vi t ( ; ) =

4.4. S ph c nguyn v ng d ng trong l thuy t s

177

n u l c chung c a hai s , v chu n N () c gi tr l n nh t trong t p h p chu n c a t t c cc c chung c a v . UCLN lun t n t i v chu n c a c c a khng v t qu chu n c a . nh l 4.3. Gi s r ng ( ; ) = . Khi , t n t i cc s ph c nguyn , sao cho + = . N u l m t c chung b t k c a v th | . Ch ng minh. Theo gi thi t ta c = 1 , = 2 . Ta ch ng minh 1 v 2 nguyn t cng nhau. Th t v y gi s = (1 ; 2 ) 1 = 1 , 2 = 2 . V y = 1 , = 2 . Suy ra | , | t c l l c chung c a v . Theo nh ngha ta ph i c N ()N () = N () N () N () = 1. Suy ra l n v . V y 1 , 2 nguyn t cng nhau. Theo nh l 4.2 c t n t i cc s ph c nguyn , sao cho 1 + 2 = 1 Thnh th 1 + 2 = + = . Ti p theo, ta c |, | | ( + ) = . H qu 4.1. N u | v , nguyn t cng nhau, th | . Ch ng minh. Th t v y, t n t i cc s ph c nguyn , sao cho + = 1. Do = + . V | , | nn | ( + ) | . 4.4.2 S nguyn t Gauss

nh ngha 4.5. M t s ph c nguyn khc n v c g i l m t s nguyn t Gauss n u khng th bi u di n c d i d ng tch c a hai s ph c nguyn khc n v . Ni cch khc, c g i l m t s nguyn t Gauss n u t ng th c = ta ph i c ho c l n v . N u khng l s nguyn t Gauss

178

Chng 4. S ph c trong cc bi ton s h c v t h p

ta ni l m t h p s Gauss. Ni cch khc, c g i l h p s Gauss n u n c th vi t d i d ng = , v i , l hai s ph c nguyn khc n v . nh ngha 4.6. S g i l m t s k t h p v i n u = l m t

n v . Nhn hai v v i ta c = , do cng l k t h p v i . Nh v y ta c th ni v l hai s k t h p v i nhau. Quan h "k t h p" l m t quan h tng ng (c tnh ph n x , i x ng v b c c u). Tnh ch t 4.6. N u m t s nguyn thng th ng l s nguyn t Gauss th chnh b n thn n ph i l s nguyn t . Tuy nhin, i u ng c l i khng ng. M t s nguyn t thng th ng cha ch c l m t s nguyn t Gauss. Ch ng h n 5 l s nguyn t thng th ng nhng v 5 = (2 + i)(2 i) do 5 l h p s Gauss. Tnh ch t 4.7. S Gauss l s nguyn t Gauss khi v ch khi s k t h p v i n l s nguyn t Gauss. Tnh ch t 4.8. S Gauss l s nguyn t Gauss n u v ch n u n ch chia h t cho cc n v v cc s k t h p v i n. nh l 4.4. Gi s l m t s nguyn t Gauss. Khi , n u |() th | ho c |. M t cch t ng qut, n u |12 . . . n , (n 2) th chia h t m t th a s i no c a tch. Ch ng minh. Gi s khng ph i l c c a . Ta ch ng minh r ng

khi , l nguyn t cng nhau. Th t v y gi s khng ph i nh v y. G i l c chung khc n v c a v . Ta c = 1, = 1. V khng ph i l n v v l s nguyn t Gauss nn 1 l n v . Do = = | |. i u ny tri v i gi thi t. Thnh th , l nguyn t cng nhau. Do h qu 6.4, ta suy ra |.

4.4. S ph c nguyn v ng d ng trong l thuy t s

179

K t lu n t ng qut c ch ng minh b ng phng php quy n p. V i n = 2 th kh ng nh ng. Gi s kh ng nh ng v i n v |12 . . . n n+1 = n+1 trong ta t = 1 2 . . . n . Theo trn ta c | ho c |n+1 . N u | ta p d ng gi thi t quy n p k t lu n r ng t n t i i v i 1 i n | i . By gi ta c th ch ng minh nh l phn tch m t s ph c nguyn thnh cc th a s nguyn t Gauss. nh l 4.5 ( nh l c b n v cc s ph c nguyn). Cho l s ph c nguyn khc khng v n v . Khi c th bi u di n d i d ng = 1 2 . . . m , trong i l cc s nguyn t Gauss. N u c hai bi u di n = 12 . . . m = 1 2 . . . n th ta ph i c m = n v t n t i m t hon v (i1, . . . , in ) c a (1, 2, . . . , n) sao cho j v ij l hai s k t h p v i nhau (j = 1, 2, . . . , n). Ngha l, m i s ph c nguyn khc khng v n v c th bi u di n (phn tch) thnh tch c a cc s nguyn t Gauss. Thm vo , bi u di n ny l duy nh t, ch sai khc th t v cc th a s n v . Ch ng minh. N u l s nguyn t Gauss th chnh l thnh ph n duy nh t trong bi u di n. N u tri l i, c phn tch thnh tch c a hai s ph c nguyn khc khng v khc n v = 12. N u 1 l s nguyn t Gauss th ta gi nguyn n. N u tri l i n c phn tch thnh tch c a hai s ph c nguyn khc khng v khc n v 1 = 3 4. Ta cng lm i u tng t nh v y cho 2. Ta ti p t c qu trnh ny ch ng no cn c h p s Gauss xu t hi n (n u xu t hi n h p s ta l i phn tch n thnh tch c a hai s ph c nguyn khc khng v khc n v ). Sau m t s h u h n b c qu trnh ph i k t thc (t c l khng cn h p s n a). Th t v y gi s = 1 2 . . . n th

180

Chng 4. S ph c trong cc bi ton s h c v t h p

N () = N (1 ) . . . N (n ) 2n do n log2 N () do n khng th tng v h n c. Ti p theo ta ch ng t tnh duy nh t c a s phn tch (sai khc th t v cc th a s n v ). Gi s ta c = 1 2 . . . m = 1 2 . . . n (m n),

trong cc nhn t i v j l cc s nguyn t Gauss, khng nh t thi t phn bi t. V 1 |1 2 . . . m do theo nh l 4.4, 1 l c c a m t nhn t , k hi u nhn t l i1 , 1|i1 . V i1 l s nguyn t Gauss nn 1, i1 l hai s k t h p, i1 = 1 1. Gi n c hai v cho 1 ta thu c 2 . . . n = 12 . . . m . Ti p t c qu trnh ny ta thu c i2 = 22 , . . . , im = m m. N u n > m th ta c 1 = 1 . . . m j trong j {i1 ; . . . ; im }. V y j | 1 do j l n v . Ta c mu thu n. / Thnh th , m = n v = 1 2 . . . m = (11)(2 2) . . . (m m ) i l cc n v v i i = 1. nh l sau y c nhi u p d ng trong vi c gi i cc bi ton khc nhau. nh l 4.6. Cho v l hai s ph c nguyn nguyn t cng nhau. Gi s = k trong k 2 l m t s nguyn dng. Khi t n t i cc s ph c nguyn 1 , 1 v cc n v , sao cho = k 1 ;
k = 1 .

4.4. S ph c nguyn v ng d ng trong l thuy t s


s t sm t Ch ng minh. Gi s r ng = 11 . . . m , = 11 . . . nn

181

i , j l cc

s nguyn t Gauss i m t phn bi t, si , tj l cc s nguyn dng. T nh l 4.4, d th y {i , j } l t p t t c cc c nguyn t c a . Gi s =


r r l rm l l l k 11 . . . m 11 . . . nn . t 1 = 11 . . . rm , 1 = 11 . . . nn ta c = k 1 . 1

V k | v k , l hai s nguyn t cng nhau nn theo h qu 6.4 ta k t 1 1


k k lu n r ng k |. V |k 1 v , 1 l hai s nguyn t cng nhau nn theo h 1 1

qu 6.4 ta k t lu n r ng |k . Thnh th hai s v k l k t h p v i nhau. 1 1


k Tng t , hai s v 1 l k t h p v i nhau. V y nh l c ch ng minh.

V d 4.9. Cho p l s nguyn t . Khi phng trnh p = x2 + y 2 c nghi m nguyn khi v ch khi p khng c d ng 4k + 3. Ch ng minh. Gi s ng c l i, phng trnh p = x2 + y 2 c nghi m nguyn (a ; b) v p = 4k + 3. D th y a, b u khng chia h t cho p. Ta c a2 b2 (mod p) ap1 (1)
p1 2

bp1

(mod p) 1 1

(mod p).

Ta c mu thu n. o l i gi s p khng c d ng 4k + 3. Khi p = 2 ho c p = 4k + 1. N u p = 2 th phng trnh 2 = x2 + y 2 r rng c nghi m (1 ; 1). p1 l h th ng d thu g n Gi s p = 4k + 1. D th y t p 1; 2; . . . ; 2 (mod p) thnh th (1)(2) . . . V p1 l s ch n nn 2 p1 ! 2 t m =
2

p1 2

(p 1)! 1

(mod p).

+1 0

(mod p).

p1 ! v q = p . Xt t p {x + my, x, y = 0, 1, 2, . . . , q}. 2 V (q + 1)2 > p nn t n t i (x1 ; y1) = (x2 ; y2) sao cho x1 + my1 x2 + my2 (mod p) suy ra (x1 x2) m(y2 y1), hay l (x1 y1 )2 m2(y2 y1)2 .

182

Chng 4. S ph c trong cc bi ton s h c v t h p

t a = |x1 x2 |, b = |y1 y2|. V m2 1 (mod p) nn suy ra a2 + b2 0 (mod p). L i c a2 q 2 < p, b2 q 2 < p, a2 + b2 = 0 nn suy ra a2 + b2 = p. nh l sau y s xc nh t t c cc s nguyn t Gauss. nh l 4.7. Cho = a + bi l s ph c nguyn khc n v . Khi : 1) N u b = 0, a = 0 th l s nguyn t Gauss n u v ch n u a l s nguyn t thng th ng c d ng 4k + 3. 2) N u a = 0, b = 0 th l s nguyn t Gauss n u v ch n u b l s nguyn t thng th ng c d ng 4k + 3. 3) N u a = 0, b = 0 th l s nguyn t Gauss n u v ch n u N () = a2 + b2 l s nguyn t thng th ng. V y t p h p t t c cc s nguyn t Gauss g m T t c cc s nguyn t thng th ng p c d ng 4k + 3 v cc s ph c nguyn k t h p v i chng. T t c cc s ph c nguyn a + bi, trong (a ; b) l nghi m nguyn c a phng trnh p = a2 + b2 v i p = 2 ho c p l s nguyn t c d ng 4k + 1. (Nghi m (a ; b) lun t n t i theo bi ton 4.9). Ch ng minh. Ta c = a. N u l s nguyn t Gauss th a ph i l s nguyn t thng th ng. Ta ch ng minh a c d ng 4k + 3. Gi s a khng c d ng 4k + 3. Theo bi ton 4.9, t n t i s nguyn x, y sao cho a = x2 + y 2, x, y N a = (x + iy)(x iy). Do = a l h p s Gauss. Mu thu n. o l i gi s = a l s nguyn t thng th ng c d ng 4k + 3 v l h p s Gauss. Khi = N () = a2 = N ()N () N () = N () = a.

4.4. S ph c nguyn v ng d ng trong l thuy t s

183

Gi s = x + iy. Khi N () = x2 + y 2 = a. i u ny tri v i bi ton 4.9 v a l s nguyn t d ng 4k + 3. Ta c = ib. Do v b hai s ph c nguyn k t h p. Do l s nguyn t Gauss n u v ch n u b l s nguyn t Gauss. Do 1) ta c i u c n ch ng minh. Gi s r ng N () l s nguyn t thng th ng. N u l h p s Gauss

th ta c = trong , l cc s ph c nguyn khc n v . Do N () = N ()N (). Suy ra N () = 1 ho c N () = 1. Ta c mu thu n. Ng c l i gi s l s nguyn t Gauss. Ta ph i ch ng minh N () l s nguyn t thng th ng. Tr c h t ta ch ra r ng c t n t i s nguyn t thng th ng p sao cho |p. Th t v y v = N () do l u c c a s nguyn dng thng th ng N (). G i n l s nguyn dng thng th ng b nh t nh n l c. Khi n l s nguyn t thng th ng. Th t v y, n u tri l i gi s n = n1 n2 , 1 < n1 < n, 1 < n2 < n. V |n1n2 = n nn theo nh l 4.4 ta c |n1 ho c |n2. i u ny mu thu n v i cch ch n n. V y n ph i l m t s nguyn t thng th ng, ta k hi u n l p. Ta c p = (do | p). S khng ph i l n v v a = 0, b = 0. V y N () < N (p) = p2 . M N ()|N (p) = p2 nn suy ra N () = p. nh l c ch ng minh xong Ti p theo, ta xt m t s v n lin quan n ng d. nh ngha 4.7. Cho , l cc s ph c nguyn. Ta ni r ng ng d v i modulo n u |( ). Khi ta vi t (mod ). D th y quan h ng d modulo xc nh m t quan h tng ng trn Z[i]. nh l 4.8 (Tnh ch t c a quan h modulo ). 1. N u = m l m t s nguyn thng th ng, = a + bi, = x + iy th (mod ) n u v ch n u a x (mod m), b y (mod m).

184

Chng 4. S ph c trong cc bi ton s h c v t h p

2. N u 1 1 (mod ) v 2 2 (mod ) th 1 2 1 2 (mod ) v 1 2 1 2 (mod ).

nh l 4.9. Cho p > 2 l m t s nguyn t thng th ng v l m t s ph c nguyn. Khi (mod p) n u p = 4k + 3 mod p n u p = 4k + 1. 2 2) V i m i s nguyn t p thng th ng ta lun c p mod p. 1) p 3) p
2 1

1(mod p) n u p = 4k + 3, = 0mod p

p1 1(mod p) n u p = 4k + 1, N () = 0(mod p).


k Ch ng minh. Gi s = a + bi. V Cp 0 (mod p), (1 k p 1) v

theo nh l Fermat, ta c p = (a + bi)p ap + (ib)p a + ipb (mod p).

M t khc, n u p = 4k + 3 th ip = i v n u p = 4k + 1 th ip = i. Thnh th , p (mod p) v i p = 4k + 1 v p (mod p) for p = 4k + 3. N u p = 4k + 1 ta c p p (mod p). N u p = 4k + 3 ta c p p (mod p). Gi s = a + bi = 0 (mod p), p = 4k + 3. Khi a2 + b2 = 0 (mod p) (v n u tri l i th a b 0 (mod p) = a + bi 0 (mod p). Do t n t i c N sao cho c(a2 + b2 ) 1 (mod p). t = c(a bi) = c(a2 + b2) 1 (mod p). Do 2) ta c (p
2 1 2 2

1) 0 (mod p) (p

2 1

1) 0 (mod p) p

2 1

1 0 (mod p).

Gi s = a + bi = 0 (mod p), p = 4k + 3, N() = a2 + b2 = 0 (mod p). Tng t nh trn c t n t i sao cho 1 (mod p). B ng l p lu n nh 2) t p (mod p) ta d n n p1 p (mod p).

4.4. S ph c nguyn v ng d ng trong l thuy t s

185

4.4.3 V d

M t s p d ng s ph c nguyn 4.10. Tm t t c cc b ba Pitago (x ; y ; z) t c l tm t t c cc

nghi m nguyn dng (x ; y ; z) c a phng trnh x2 + y 2 = z 2 . L i gi i. V r ng b (x ; y ; z) l b ba Pitago khi v ch khi (kx ; ky ; kz) l b ba Pitago v i k N nn khng gi m t ng qut ta gi s r ng (x ; y ; z) = 1, t c l (x ; y ; z) l b ba Pitago nguyn thu . T suy ra (x ; y) = (y ; z) = (z ; x) = 1, do x, y khng cng ch n. Tuy nhin x, y khng th cng l v n u th th z 2 1+1 = 2 (mod 4) l v l. Gi s x ch n, y l . Khi (x+iy)(xiy) = z 2 . By gi ta ch ng minh hai s ph c nguyn x + iy v x iy l nguyn t cng nhau. Th t v y, gi s tri l i t n t i s nguyn t Gauss sao cho |x + iy, |x iy Suy ra |2x, |2iy. Ta ch ra r ng khng l c c a 2. Th t v y n u |2 th N ()|N (2) = 4. Do N () ch n. V N ()|N (x+iy) = x2 +y 2 do x2 + y 2 l s ch n. Mu thu n. T |x, |y. Suy ra N ()|N (x) = x2, N()|N (y) = y 2. Theo nh l 4.7 ta c N () = p l m t s nguyn t thng th ng, v y p|x, p|y. i u ny tri v i gi thi t (x ; y) = 1. Theo nh l 4.6 ta c x + iy = (m + in)2 = (m2 n2 ) + (2mn)i ho c x + iy = i(m + in)2 = 2mn (m2 n2 )i. V x ch n, y l x, y N , ta thu c x = 2mn, y = (m2 n2 ). T z = (m2 + n2 ). V (y ; z) = 1 nn (m ; n) = 1 v m, n khng c cng tnh ch n l . Ng c l i, ta x y z d ki m tra cng th c = 2mn = |m2 n2| = m 2 + n2 ho c x = |m2 n2 | y = 2mn z = m2 + n2 ,

186

Chng 4. S ph c trong cc bi ton s h c v t h p

trong m, n N , (m ; n) = 1, m = n (mod 2), xc nh cho ta m t b ba Pitago nguyn thu (x ; y ; z) V d 4.11. Tm t t c cc s nguyn dng a, b, c phn bi t sao cho a2 , b2, c2 l p thnh m t c p s c ng. L i gi i. Bi ton quy v vi c tm nghi m nguyn x = y c a phng trnh x2 + y 2 = 2z 2 (4.3)

V r ng b (x ; y ; z) l nghi m c a (4.3) khi v ch khi (kx ; ky ; kz) l nghi m c a (4.3) v i k N nn khng gi m t ng qut ta gi s r ng (x ; y ; z) = 1. N u x, y ch n th suy ra z ch n, v l. Gi s x l do y l . V y v tri chia 4 d 2 do z l . N u p|x, p|y th p|z (do p l ). V y (x ; y) = 1. Ta c (x + iy)(x iy) = 2z 2 = (1 + i)(1 i)z 2. Suy ra (1 + i)|x + iy ho c (1 + i)|x iy. V y x + iy = (1 + i)(u + iv) ho c x iy = (1 + i)(u + iv). Cn b ng ph n th c v o ta c x = u v, y = (u + v). Thay vo (4.3) ta c u2 + v 2 = z 2. V x = y nn u, v = 0. Do (x ; y) = 1 u v z nn (u ; v) = 1. Thnh th = 2mn = (m2 n2 ) = (m2 + n2) u = (m2 n2 ) v = 2mn z = (m2 + n2 ), (4.4)

ho c

trong m, n Z, (m ; n) = 1, m = n (mod 2). Ta c |x| = |u v|, |y| = u + v, |z| = m2 + n2 . T ta nh n c cng th c nghi m nguyn dng l a = |2mn + n2 m2 |, c = |2mn + m2 n2 |, b = m2 + n2

4.4. S ph c nguyn v ng d ng trong l thuy t s

187

hay d i d ng i x ng hn a = |(m + n)2 2n2 |, c = |(m + n)2 2m2 |, b = m2 + n2 trong m, n l cc s nguyn dng phn bi t nguyn t cng nhau, khc tnh ch n l . V d 4.12. Tm t t c cc c p s nguyn (x ; y) tho mn x2 + 1 = y 3.

L i gi i. Ta c (x + i)(x i) = y 3. Ta s ch ra hai s x + i, x i l nguyn t cng nhau. Gi s tri l i c s nguyn t Gauss sao cho |x + i, |x i. Suy ra |2i do |2. V y N ()|N (2) = 4, suy ra N () ch n. V N ()|N (x + i) = x2 + 1 = y 3 nn y ch n do x l v x2 + 1 = y 3 chia h t cho 8. Nhng x2 + 1 2 (mod 4). Ta c mu thu n. Ta th y x + i l k t h p v i m t l p phung no . M ta c 1 = (1)3 , i = (i)3, (i) = i3 nn chnh x + i l m t l p phng. V y x + i = (a + ib)3 = (a3 3ab2) + i(3a2b b3) x = a(a2 3b); 1 = b(3a2 b2 ) |b| = 1 ; |3a2 b2| = 1 |3a2 1| = 1 a = 0, b = 1. Do x = 0, y = 1 l nghi m duy nh t c a phng trnh cho. V d 4.13. Ch ng minh r ng m t s nguyn dng n > 1 c bi u di n

thnh t ng c a hai s chnh phng khi v ch khi trong phn tch tiu chu n c a n cc c nguyn t d ng 4k + 3 c lu th a ch n. L i gi i. Gi s r ng n = x2 + y 2 x, y N . Khi n = (x + iy)(x iy).

V n > 1 nn x + iy khng l n v . Phn tch x + iy thnh tch cc s nguyn t Gauss ta c x + iy = (i qi)si (j j )tj

188

Chng 4. S ph c trong cc bi ton s h c v t h p

trong i , j l cc n v , qi l cc s nguyn t thng th ng d ng 4k + 3 ( cng l cc s nguyn t Gauss) v j = aj + ibj l cc s nguyn t Gauss v i aj = 0, bj = 0. Ta c x iy = x + iy = (iqi )si (j j )tj . Vy
2s n = qi i (N (j ))tj .

t N (j ) = pj . Theo nh l 4.7, pj l cc s nguyn t thng th ng khng c d ng 4k + 3. V y phn tch tiu chu n c a n l


2s n = qi i pjj . t

Ng c l i, gi s
2s n = qi i pjj . t

Theo bi ton 4.9, pj c th vi t d i d ng t ng c a hai s chnh phng khc 0, pj = a2 + b2. t j = aj + ibj ta c N (j ) = j j = a2 + b2 = pj . Ta xt j j j j s ph c nguyn sau y
s = qi i jj . 2s Khi = qi i pjj = n. Gi s = x + iy. Khi n = x2 + y 2. t t

Ta c th xt s ph c nguyn khc sau y


s = qi i jj j rj l

trong lj , rj N sao cho lj + rj = tj . Khi


2s = qi i pjj l +rj 2s = qi i pjj = n. t

Gi s = u + iv th n = u2 + v 2 v ta c m t cch bi u di n khc c a n thnh t ng c a hai s chnh phng. C th ch ng minh r ng n u phn tch tiu chu n c a n l
2s n = qi i pjj t

4.5. Bi t p

189

th s nghi m nguyn khng m (x ; y) c a phng trnh n = x2 + y 2 l (1 + tj ). V d 4.14. Cho p l s nguyn t d ng 4k + 3 v x, y, z l cc s nguyn


p+1 2

dng tho mn x2 + y 2 = z

Gi s r ng x, y nguyn t cng nhau. Ch ng minh r ng xy chia h t cho p. L i gi i. Tr c h t ta ch ng minh hai s ph c nguyn x + iy, x iy l nguyn t cng nhau. N u gi s tri l i c s nguyn t Gauss sao cho |x + iy, |x iy. Khi |2x, |2iy. Ta ch ra r ng khng ph i l c c a 2. Th t v y, n u |2 th N ()|N (2) = 4. Do N () ch n. V N ()|N (x + iy) = x2 +y 2 = z
p+1 2 p+1 2 p+1 2

nn z

ch n. Do z ch n, i u ny ko theo 4|z

= x2 +y 2.

L i c (x ; y) = 1 nn x, y khng cng ch n. V y x2 + y 2 1, 2 (mod 4). Ta c mu thu n. V y |x, |y. Suy ra N ()|N (x) = x2, N()|N (y) = y 2 . Do nh l 4.7, N () = p l s nguyn t , nn p|x, p|y. i u ny tri v i gi thi t (x ; y) = 1. V x + iy = (a + bi)
p+1 2

nn

(x + iy)2 = (a + bi)p+1 = (a + bi)(a + bi)p. Do v y (a + bi)p (a bi) (mod p) nn (x + iy)2 = (x2 y 2) + i2xy (a2 + b2 ) (mod p). T 2xy 0 (mod p). V p l s nguyn t l nn ta k t lu n c p|xy.

4.5

Bi t p

Bi 4.1. Ch ng minh r ng m t s nguyn dng n > 1 bi u di n thnh t ng c a hai s chnh phng khi v ch khi trong phn tch tiu chu n c a n cc c nguyn t d ng 4k + 3 c lu th a ch n.

190

Chng 4. S ph c trong cc bi ton s h c v t h p

Bi 4.2 (IMO 1974). Ch ng minh r ng s


n 2k+1 C2n+1 23k k=1

khng chia h t cho 5 v i m i s nguyn n 0.


n

Bi 4.3. Tnh t ng
k=1

k Cn

cos kx v i x [0 ; ].

Bi 4.4 (Cu c thi Traian Lalescu - Romania, 2003). C bao nhiu s c n ch s ch n t t p h p {2 ; 3 ; 7 ; 9} v chia h t cho 3? Bi 4.5. Cho ba s nguyn dng m, n, p, trong m > 1 v n + 2

0 (mod m). Tm s b (x1 ; x2; . . . ; xp ) g m p s nguyn dng sao cho t ng (x1 + x2 + xp ) chia h t cho m, trong m i s x1, x2, . . . , xp u khng l n hn m. Bi 4.6 (IMO 2007 Shorlist). V i s nguyn dng n > 1, xt t p S = {1 ; 2 ; 3; . . . ; n}. T cc s c a S b ng 2 mu, u s c t mu v v s c t mu xanh. Hy tm s cc b (x ; y ; z) thu c S 3 sao cho a) x, y, z c t cng mu ; b) x + y + z chia h t cho n. Bi 4.7 (Vi t Nam TST 2008, Bi 6). K hi u M l t p h p g m 2008 s nguyn dng u tin. T t t c cc s thu c M b i ba mu xanh, vng, sao cho m i s c t b i m t mu v m i mu u c dng t t nh t m t s . Xt cc t p h p S1 = {(x ; y ; z) M 3 | x, y, z cng mu v x + y + z 0 (mod 2008)}; S2 = {(x ; y ; z) M 3 | x, y, z i m t khc mu v x + y + z 0 (mod 2008)}. Ch ng minh r ng 2|S1 | > |S2 |. (Ta k hi u M 3 := M M M v |X| l s ph n t c a t p h u h n X).

4.5. Bi t p

191

Bi 4.8. Tm cng th c t ng qut c a dy s {xn } xc nh b i x0 = 1, x1 = 0, x2 = 0, xn+3 = 2xn+2 3xn+2 + 2xn+1 xn . Bi 4.9. Xt khai tri n 1 = 1 + a1 x + a2 x2 + 1 + ax + bx2 Ch ng minh r ng n u aj > 0 v i m i j = 1, 2, 3, . . . th phng trnh 1 + ax + bx2 = 0 c cc nghi m u th c. Bi 4.10. Ch ng minh r ng n u x + 1 1 = 2 cos th xn + n = 2 cos(n). x x

Chng 5

M ts ng d ng c a s ph c trong hnh h c

Chng trnh Ton h c

b c Trung h c ph thng c a h u h t cc n c n c ta, sau nhi u l n c i cch, n i dung

u c ph n ki n th c s ph c.

s ph c cu i cng cng c a vo chng trnh Gi i tch 12, tuy nhin cn r t n gi n. V nhi u l do khc nhau, r t nhi u h c sinh, th m ch l h c sinh kh, gi i sau khi h c xong ph n s ph c cng ch hi u m t cch r t n s: s d ng s ph c, c th gi i c m i phng trnh b c hai, tnh m t vi t ng c bi t, ... Vi c s d ng s ph c trong nghin c u, kh o st hnh h c ph ng t ra c nhi u thu n l i, nh t l trong vi c xem xt cc v n lin quan n cc php bi n hnh c a m t ph ng cng v i hnh h c c a chng. Trong chng ny s m t m t s k t qu , khi ni m c b n c a Hnh h c Euclid ph ng d i d ng ngn ng s ph c nh gc, kho ng cch, s ng quy, th ng hng, ng th ng, ng trn cng v i m t s php d i hnh, ng d ng d ng c b n nh t.
192

5.1. M t m t s k t qu c a hnh h c ph ng b ng ngn ng s ph c

193

5.1

M t m t s k t qu c a hnh h c ph ng b ng ngn ng s ph c

Cho tr c hai i m M(m), N(n). Khi , di o n MN b ng MN = |n m| = d(m; n) Trong m t ph ng cho tr c o n th ng AB. Khi , i m M chia o n th ng AB theo t s k R \ {1} khi v ch khi MA = k MB, a m = k (b m) trong a, b v m l t a v c a cc i m A, B v M theo th t . T , n u k hi u [AB] l ch o n th ng AB, k hi u (AB) l ch ng th ng AB, k hi u [AB) l ch tia AB, ta c cc k t qu sau Cho tr c hai i m A(a), B(b) phn bi t v i m M(m). Khi M [AB] t 0 : z m = t (b m) t [0; 1] : m = (1 t)a + t b (1) M (AB) t R : m a = t(b a) t R : m = (1 t)a + tb (2) nh l 5.1. Cho tr c hai i m A(a), B(b) phn bi t v i m M(m). Khi , cc m nh sau tng ng M [AB) t > 0 : m = (1 t)a + tb arg (m a) = arg (b a) ma = t R+ ba

T , r ng t = t t R, ta thu c phng trnh c a ng th ng i qua hai i m W1(w1 ), W2(w2 ) l (z w1) (w2 w1) (z w1) (w2 w1) = 0 (3)

194

Chng 5. M t s

ng d ng c a s ph c trong hnh h c

5.1.1

Gc gi a hai ng th ng

Trong m t ph ng ph c, cho hai i m M1 (z1), M2 (z2) v k = arg zk , k = 1, 2. Khi , do (Ox, OM1 )+(OM1 ; OM2 ) (Ox; OM2 ) (mod 2) nn
M2 (z2 )

M1 (z1 )

(OM1 ; OM2 ) (Ox; OM2 ) (Ox; OM1 ) (mod 2) z2 T , n u cho z1 b n i m phn bi t Mk (zk ), k = 1, 2, 3, 4 th gc ( nh h ng) t o b i ng z4 z2 th ng M1 M3 v i ng th ng M2 M4 b ng arg z3 z1 hay gc nh h ng t o b i tia OM1 v i tia OM2 b ng arg nh l 5.2. Hai tam gic ABC, A B C ng d ng cng h ng khi v ch khi ca c a = ba b a V hai tam gic ABC, A B C ng d ng ng c h ng khi v ch khi c a ca = ba b a 5.1.2 Tch v h ng c a hai s ph c

Trong m t ph ng ph c cho hai i m M1 (z1), M2 (z2). Khi OM1 OM2 = OM1 OM2 cos M1 OM2

5.1. M t m t s k t qu c a hnh h c ph ng b ng ngn ng s ph c

195

N u zk c modul b ng rk , v c argument b ng k th OM1 OM2 = r1 r2 cos(2 1) = r1 r2(cos 1 cos 2 + sin 1 sin 2) Do < z1 ; z2 >= 1 (z1 z2 + z1 z2) 2

T suy ra < z1; z2 > =< z1; z2 > v do < z1; z2 > R Tch v h ng c a hai s ph c cng c cc tnh ch t nh tch v h ng c a hai vc-t. Ngoi ra < z1; zz2 >= z < z1 ; z2 > v < zz1; z2 >= z < z1; z2 >. Nh n xt 5.1. 1. Trong m t ph ng ph c cho hai i m M1 (z1), M2 (z2). Khi

< z1 ; z2 > b ng phng tch c a O v i ng trn ng knh M1 M2 2. N u A(a), B(b), C(c) v D(d) l b n i m phn bi t c a m t ph ng ph c, th AB CD < b a; d c >= 0 Re 5.1.3 ba dc =0

Tch ngoi c a hai s ph c. Di n tch tam gic

Trong m t ph ng ph c cho hai i m M1 (z1), M2 (z2). Khi OM1 OM2 = |OM1 | |OM2 | sin M1 OM2 N u zk c modul b ng rk , v c argument b ng k th OM1 OM2 = r1 r2 sin(2 1 ) = r1 r2 (sin 2 cos 1 cos 2 sin 1 ) Do z1 z2 = i (z1 z2 z1 z2 ) 2

T , do z1 z2 = z1 z2 nn suy ra Im z1 z2 = 0 Tch ngoi c a hai s ph c cng c cc tnh ch t nh tch ngoi c a hai vct trong m t ph ng, ngoi ra (zz1) z2 = z (z1 z2) v z1 (zz2) = z (z1 z2)

196

Chng 5. M t s

ng d ng c a s ph c trong hnh h c

Nh n xt 5.2.

1. Ba i m A(a), B(b), C(c) th ng hng khi v ch khi (b

a) (c a) = 0 2. N u A(a), B(b) l hai i m phn bi t, khng th ng hng v i O th a b = 2 [OAB], trong , k hi u [A1A2 . . . An ] l ch di n tch i s c a a gic nh h ng A1A2 . . . An 3. T nh n xt 2, v i ba i m A(a), B(b) v C(c) phn bi t, khng th ng hng, th [ABC] = 5.1.4 ng trn 1 (a b + b c + c a) 2

ng trn tm M0(z0 ) bn knh R l t p h p nh ng i m M(z) sao cho M0 M = R hay |z z0 | = R t c l zz z0z z0z + z0 z0 R2 = 0 T , m i ng trn u c phng trnh d ng zz + z + z + = 0 trong C v R. ng trn nay c tm v i t a v , bn knh R = 5.1.5 s ph c

M t cc php bi n hnh ph ng b ng ngn ng

Php d i hnh.

Php t nh ti n. Php t nh ti n theo vc-t = (v) l php bi n hnh v bi n i m M(z) thnh i m M (z ) sao cho MM = . Do , bi u th c c a v php t nh ti n l z = f (z) = z + v Php quay. Php quay tm M0 (z0 ) gc quay l php bi n hnh bi n M(z) thnh i m M (z )m M0 M = M0 M v (M0 M; M0 M ) (mod 2). T , bi u th c c a php quay l z z0 = ei(z z0 )

5.1. M t m t s k t qu c a hnh h c ph ng b ng ngn ng s ph c

197

Php i x ng-tr c. Php i x ng qua ng th ng l php bi n hnh bi n m i i m M(z) thnh i m M (z ) sao cho Php i x ng qua tr c th c: z = f (z) = z Php i x ng qua tr c o: z = f (z) = z Do 2(Ox; ) = (Ox; OM ) + (Ox; OM ( y = (z0)) nn php i x ng qua ng th ng z = f (z) = ei z v T , n u = T ( ) v i = (z0) th php i x ng qua c bi u th c v z = eiz + 2z0 i qua g c O v i m z0 = ei 2 c bi u th c

l trung tr c c a MM . T

M (z) M (z) v M (z )
2
M (z )

2 v

M (z )

Php v t

Php v t tm C(z0), t s r R lf php bi n hnh bi n i m M(z) thnh i m M (z ) m CM = r CM. Do , c bi u th c z = r (z z0) + z0

198

Chng 5. M t s

ng d ng c a s ph c trong hnh h c

5.1.6

i u ki n ng quy, th ng hng, vung gc v cng n m trn m t ng trn ( ng vin)

nh l 5.3. Cho ba ng th ng v i phng trnh 1 : (z z1) u1 = 0 2; (z z2) u2 = 0 3 : (z z3) u3 = 0. Khi , ba ng th ng 1, 2 , 3 ng quy khi v ch khi (u1 u2 )2 + (u2 u3)2 + (u3 u1 )2 = 0 (u1 u2 )(z3 u3) + (u2 u3)(z1 u1 ) + (u3 u1 )(z2 u2) = 0 nh l 5.4. Ba i m M1 (z1), M2 (z2), M3 (z3) th ng hng khi v ch khi z3 z1 R z2 z1 hay Im z3 z1 z2 z1 =0

nh l 5.5. B n i m Mk (zk ), k = 1, 2, 3, 4 cng n m trn m t ng th ng hay ng trn khi v ch khi z3 z2 z3 z4 : R z1 z2 z1 z4 5.1. B n i m Mk (zk ), k = 1, 2, 3, 4 cng n m trn m t ng th ng z3 z4 z3 z2 R v R khi v ch khi z1 z2 z1 z4 B n i m Mk (zk ), k = 1, 2, 3, 4 cng n m trn m t ng trn khi v ch z3 z2 z3 z4 z3 z2 z3 z4 khi : R nhng R v R z1 z2 z1 z4 z1 z2 z1 z4 nh l 5.6. N u A(a), B(b), C(c) v D(d) l b n i m phn bi t c a m t ph ng ph c, th AB CD < b a; d c >= 0 Re ba dc =0 H qu

5.2

M t s v d

p d ng

Cho A(a), B(b), C(c) l ba nh c a m t tam gic. Khi tam gic ABC l tam gic u khi v ch khi hai tam gic ABC, BCA l ng d ng cng

5.2. M t s v d p d ng

199

h ng. i u ny tng ng v i ab ca = (c a)(c b) = (a b)(b a) ba cb c2 ca bc + ab = a2 b2 + 2ab a2 + b2 + c2 = ab + bc + ca M t khc, tam gic ABC u, nh h ng dng (tng ng m) khi v ch khi php quay tm A, gc quay + (tng ng ) bi n B thnh C, do 3 3 Tam gic ABC u, nh h ng dng khi v ch khi a + b + c 2 = 0, v tam gic ABC u, nh h ng m khi v ch khi a 2 + b + c = 0, trong 3 = 1. V d 5.1 (Napolon). L y cc c nh BC, CA, AB c a tam gic ABC lm ng A0, B0 , C0. Ch ng

y, d ng ra ngoi cc tam gic u v i tm tng minh r ng A0 , B0, C0 l nh c a m t tam gic u.

L i gi i. Gi s tam gic ABC nh h ng dng. G i x l t a v c a i m X no trong m t ph ng. Ta c a + c1 + b 2 = 0, b + a1 + c 2 = 0, c + b1 + a 2 = 0 Do A0 , B0, C0 theo th t l tr ng tm cc tam gic BCA1, CAB1, ABC1 nn 3a0 = b + c + a1, 3b0 = c + a + b1 , 3c0 = a + b + c1 T 3(c0 + a0 + b0 2 = a + b + c1 + (b + c + a1) + 2 (c + a + b1) = (b + a1 + c 2 ) + (c + b1 + a 2) + (a + c1 + b 2 ) 2 = 0 Suy ra i u ph i ch ng minh. L i gi i 2. Gi s tam gic ABC nh h ng m, v x l t a v c a i m X no trong m t ph ng. Khi , ta c c = ei 3 (b a0) + a0 , a = ei 3 (c b0 ) + b0, b = ei 3 (a c0 ) + c0
2 2 2

200

Chng 5. M t s

ng d ng c a s ph c trong hnh h c

Suy ra b = ei 3 (ei 3 (c b0 ) + b0 c0 ) + c0 = ei 3 (ei 3 (b a0) + a0 b0 ) + ei 3 (b0 c0 ) + c0 = b a0 + ei 3 (a0 b0) + ei 3 (b0 c0 ) + c0 T c0 a0 = ei 3 (b0 a0) i u c ngha l tam gic A0B0 C0 u
B1
4 2 4 2 2 2 2

A C1 C0

B0

B A0

A1

V d

5.2 (BMO 1990 - Shortlist). Cho tam gic ABC. L y cc c nh lm

y, d ng ra ngoi ba ngic u. Tm t t c cc gi tr c a n sao cho tm c a ba a gic u l nh c a m t tam gic u. Gi i. Gi s tam gic ABC nh h ng m. G i A0, B0 , C0 l tm c a cc a gic u d ng trn c nh BC, CA, AB (hnh v ). Khi BA0C = CB0A = AB0C =
2 n
2

t = ei n v g i a, b, c, a0, b0, c0 l n l t l t a v c a cc i m A, B, C, A0, B0, C0 theo th t . T gi thi t, ta c a = b0 + (c b0), b = c0 + (a c0), c = a0 + (b a0)

5.2. M t s v d p d ng

201
C0
2 n

A B0
2 n

2 n

A0

T b0 = a c b a c b ; c0 = ; a0 = 1 1 1

Tam gic A0B0 C0 u khi v ch khi a2 + b2 + c2 = a0 b0 + b0 c0 + c0 a0 0 0 0 Thay a0, b0, c0 tm c trn vo, khai tri n, rt g n, ta c

(1 + + 2 ) (b a)2 + (a c)2 + (c b)2 = 0 i u ny tng ng v i 1 + + 2 = 0 do n = 3. V d 5.3. Trn cc c nh c a l c gic l i c tm i x ng A1A2 A3A4A5A6 ,

d ng v pha ngoi cc tam gic u Ak Ak+1 Bk (v i k = 1, 2, . . . , 6 v quy c A7 A1). Ch ng minh r ng trung i m cc o n th ng Bk Bk+1 l nh c a m t l c gic u (v i k = 1, 2, . . . , 6 v B7 B1 ) L i gi i. Gi s l c gic A1A2A3 A4A5A6 nh h ng m. Ch n tm i x ng O c a l c gic lm g c, g i x l t a v c a i m X trong m t ph ng ph c, t ei 3 = . Khi ak+3 = ak . Do tam gic Ak Ak+1 Bk u, c h ng dng, nn bk + 2 ak + 4 ak+1 bk = ak+1 + ak . Suy ra bk = bk+3

202

Chng 5. M t s

ng d ng c a s ph c trong hnh h c

B1 P6 B6 A1 A2
3

P1

B2

P2 A3

A6 P5 A5

O A4

B3 B5 P4 B4 P3

Do Pk l trung i m c a Bk Bk+1 nn pk = pk + pk+3 =

bk +bk+1 2

k = 1, 2, . . . , 6 T

(bk + bk+3 ) + (bk+1 + bk+4 ) bk + bk+1 bk+3 + bk+4 + = =0 2 2 2

do l c gic P1 P2 P3 P4 P5 P6 nh n O lm tm i x ng. K hi u f l php quay tm O gc quay . Ta c 3 f (p1 ) = ( 1 b1 + b2 ) = (a2 + a1 + a3 + a2) 2 2

1 = (a2 + a1 + a3) 2 1 = a3 + a2 + 2 a1 2 1 = (a3 + a2 + a4 ) = p2 2 Do , f (p1 ) = p2 . Tng t , cng c f (p2 ) = p3 , f(p3 ) = p4 , pcm. V d 5.4 (IMO 1977). Cho hnh vung ABCD. D ng v pha trong hnh

vung cc tam gic u ABK, BCL, CDM v DAN . Ch ng minh r ng trung

5.2. M t s v d p d ng

203

i m cc o n th ng KL, LM, MN, NK, BK, BL, CL, DM, DN v NA l nh c a m t th p nh gic u. L i gi i. Gi s hnh vung ABCD nh h ng dng. Ch n tm O c a hnh vung lm g c, g i x l t a v c a i m X trong m t ph ng ph c. Khi b = ia, c = a, d = ia. t ei 3 = ta c k = (i + )a, = ( + i)a, m = (i )a, n = ( i)a r ng a gic P1 Q1S1 P2 Q2S2 P3 Q3S3 P4 Q4 S4 nh n O lm tm i x ng, do v i f l php quay tm O, gc quay + th ch c n ch ng minh f (pk ) = 6 qk , f(qk ) = sk v f (sk ) = pk+1 (k = 1, 2) l
D K C

P1 Q1 L S1 P2

S4

Q4

P4

S3 N Q3 Q2 S2 P3

M A B

T cch d ng, ta c a 1 a p1 = (k + ) = [(i 1) + (i + 1)] , p2 = [(i + 1) + (i 1)] , 2 2 2

204

Chng 5. M t s

ng d ng c a s ph c trong hnh h c

a a q1 = [i(1 + ) + ] , s1 = [1 + i + ] 2 2 Khi , v i = ei 6 th f (p1 ) = p1 = a [(i 1) + (i + 1)] = q1 2 a f (q1 ) = q1 = [i + i + ] = s1 2 a f (s1 ) = s1 = [ + i + ] = p2 2

M t cch tng t , cng c f (p2 ) = q2 , f(q2) = s2 , f(s2) = p3 (PCM) Nh n xt. Bi ton ny hon ton c th gi i b ng phng php t a nh trong [5], hay phng php t ng h p nh trong [6], tuy nhin l i gi i qu di. L i gi i c trnh by trn c xu t pht t t ng s d ng php quay

vc-t, tuy nhin b ng cng c s ph c, lm gi m i ng k cc ng tc bi n i ph c t p trn cc vc-t V d 5.5 (SEA-MO 1998). Cho tam gic ABC. L y i m P khc pha v i

C i v i ng th ng AB, i m Q khc pha v i B i v i ng th ng CA v i m R cng pha v i A i v i ng th ng BC sao cho cc tam gic BCR, ACQ v BAP ng d ng. Ch ng minh r ng t gic AP RQ l m t hnh bnh hnh. L i gi i 1. Gi s tam gic ABC nh h ng dng v g i x l t a v c a i m X. t
BP BA

AQ AC

BR BC

= t, ABP = CAQ = CBR = , = ei .

Khi , t gi thi t suy ra p = (t + 1)b ta, q = (t + 1)a tc v r = (t + 1)b tc

5.2. M t s v d p d ng Q A

205

R P B C

Khi p+q = (t+1)(a+b)t(a+c) = (t+1)btc+(t+1)ata = a+r (PCM) L i gi i 2. T gi thi t, suy ra cc tam gic BCR, ACQ v BAP ng d ng cng h ng. V y rb qa pb = = =zC rc qc pa T p= Suy ra p+q = V d b zc + (1 z)a = a+r 1z b za a zc b zc ; q= ; r= 1z 1z 1z

5.6. Trong m t ph ng cho b n tam gic ABC, AB1C1, A2BC2 v

A3B3 C ng d ng, cng h ng. G i A0, B0 v C0 theo th t l trung i m c a A2A3 , B1B3 v C1 C2. Ch ng minh r ng A0B0 C0 ABC

L i gi i. G i x l t a v c a i m X. Gi s php ng d ng f1 (z) = 1 z + 1 bi n tam gic ABC thnh tam gic AB1C1 , php ng d ng f2 (z) = 2 z + 2 bi n tam gic ABC thnh tam gic A2BC2, php ng d ng f3 (z) = 3 z + 3 bi n tam gic ABC thnh tam gic A3B3 C.

206

Chng 5. M t s

ng d ng c a s ph c trong hnh h c

Khi 1 1 b0 a0 = (b1 + b3 a2 a3) = [(b1 a) + (a b) + (b a2) + (b3 a3)] 2 2 1 = (1 + 2 + 3 1) (b a) 2 1 Tng t , cng c c0 a0 = (1 + 2 + 3 1) (c a) 2 c0 a0 c a Vy (PCM) = b0 a0 b a Nh n xt 5.3. N u t AB = t v = (AC; AB), th b ng cch lm tng t AC nh l i gi i 1 bi ton 5, ta cng ch ng minh c b0 a0 = tei(c0 a0) v cng c i u ph i ch ng minh. B ng nh ng cch lm nh trn, khng nh ng ta ch ng minh c cc tam gic ng d ng, m cn ch ra c chng ng d ng cng h ng, v cng tm c t s ng d ng theo cc t s cho. V d 5.7 (Italy MO 1996). Cho ng trn (O) v i m A ngoi (O).

V i m i i m P trn ng trn, d ng hnh vung AP QR, v i cc nh theo ng c chi u kim ng h . Tm qu tch i m Q khi P ch y kh p trn (O). L i gi i. Khng m t t ng qut, coi ng trn (O) c tm t i g c, bn knh b ng 1, g i x l t a v c a i m X trn m t ph ng. Khi , ta c q = ei 2 (a p) + p q = ia + (1 i)p Do , qu tch c a i m Q l ng trn c tm t i i m A (ia) (t c l A = QO 2 (A)), bn knh R = |(1 i)p| = 2 V d 5.8 (Bulgaria MO 1997). Cho hai hnh vung n v K1 , K2 v i tm

M, N trong m t ph ng sao cho MN = 4. Bi t r ng hnh vung K1 c hai c nh song song v i MN , hnh vung K2 c m t ng cho n m trn ng th ng MN , tm qu tch trung i m XY , trong X l m t i m trong c a K1 , Y l m t i m trong c a K2

5.2. M t s v d p d ng

207

L i gi i. Khng m t t ng qut, coi M(2), N(2) v g i w l t a v c a i m


i i i W trong m t ph ng ph c. Khi a1 = 5 2 , b1 = 3 2 , c1 = 3 + 2 , d1 = 2 2 2

5 + 2

i 2

v a2 = 2

1 , b2 2

=2

i , c2 2

=2+

1 , d2 2

=2+

i 2

D2 D1 X M A1 B1 B2 C1 Z Y A2 N C2

Ta c X n m trong hnh vung A1B1 C1D1 khi v ch khi x = x1 + x2i, xk R v i |x2| < 1 , |x1 + 2| < 2 |y1 + y2 2| <
1 2 1 2

V Y n m trong hnh vung A2 B2C2 D2 khi v ch khi y = y1 + y2 i, yk R v i v |y1 y2 2| <


1 2 x1 +y1 2 x2 +y2 2

V y, v i Z l trung i m XY th z = T 2
1 2

+i

= u + iv.

< x1 < 2 + 1 , 2 2

Tng t , cng c |v| <

1+ 2 , |u 2

1 2

< y1 y2 < 2 +

+ v|, |u v| <

1 2 2+1 2 2

suy ra |u| <

2+1 2

bt V y, qu tch i m Z l mi n gic gi i h n b i cc ng th ng |x| = 2+1 2+1 2+1 2+1 , |y| = , |x + y| = , |x y| = 2 2 2 2 2 2 Nh n xt. V m t hnh h c, qu tch i m Z l mi n trong a gic u c nh l trung i m cc o n n i cc nh c a hai hnh vung l nh c a A1B1 C1D1 , A2B2 C2D2 tng ng qua cc php t nh ti n theo cc vc-t 1 MN 2 v 1 NM (hnh v ) 2 V d 5.9 (Poland MO 1999). Cho l c gic l i ABCDEF c A+C +E = 360 v AB CD EF = BC DE F A. Ch ng minh r ng AB F D EC = BF DE CA

208

Chng 5. M t s

ng d ng c a s ph c trong hnh h c

L i gi i. G i w l t a v c a i m W trong m t ph ng ph c. t b a = x, c b = y, d c = z, e d = t, f e = u, a f = v. Do AB CD EF = BC DE F A nn |xzu| = |ytv| Do A + C + E = 360 nn arg


x v x v

(1)

z y

u t

= 0 i u ny c ngha l (2)

z y

u t

l m t s th c dng.

T (1),(2) suy ra xzu = vyt hay xzu + vyt = 0 Do x + y + z + t + u + v = 0 nn xt(x + y + z + t + u + v) + (xzu + vyt) = 0 x2 t + xty + xtz + xt2 + xtu + xtv + xzu + vyt = 0 (xt2 + xtz + xtu + xzu) + (x2 t + xty + xtv + vyt) = 0 x(t + z)(t + u) + t(x + y)(x + v) = 0 Do |x(t + z)(t + u)| = |t(x + y)(x + v)| (PCM) V d 5.10. Cho t gic l i ABCD. D ng cc hnh vung AMBN v CP DQ cng h ng. Ch ng minh r ng |MQ2 NP 2 | = 4SABCD Gi i. Coi t gic ABCD nh h ng m (hnh v ). G i X, Y, Z, T theo th t l trung i m cc c nh AB, BC, CD, DA c a t gic ABCD. G i w l t a v c a i m W trong m t ph ng ph c. r ng MP 2 NQ2 = (MP + NQ)(MP NQ) Ta c (p m)2 (q n)2 = 2 < z x; n m + p q > = 2 < z x; i(a b + d c) > = 4i < z x; t y > V y |(p m)2 (q n)2 | = 4SABCD (Do SABCD = 2SXY ZT )

5.2. M t s v d p d ng M B X A N T Q Y

209

D Z C

V d

5.11. Xt t

gic ABCD khng c hai c nh no song song. G i

Ga , Gb , Gc , Gd theo th t l tr ng tm cc tam gic BCD, CDA, DAB, ABC. Ch ng minh r ng n u AGa = BGb v CGc = DGd th ABCD l m t hnh thang cn. L i gi i. G i x l t a v c a i m X trong m t ph ng ph c v t s = a + b + c + d. Ta c ga = sa b+c+d sb sc sd = , gb = , gc = , gd = 3 3 3 3 3

Do AGa = BGb nn |a ga | = |b gb | |4a s| = |4b s| < 4a s; 4a s >=< 4b s; 4b s > T 2(|a|2 |b|2) =< (a b); s > Tng t , t CGc = DGd cng c 2(|c|2 |d|2) =< (c d); s > (1) (2)

210

Chng 5. M t s

ng d ng c a s ph c trong hnh h c

Tr (1) cho (2) v i v , ta c 2(|a|2 |b|2 |c|2 + |d|2 ) =< (a b c + d); (a + b + c + d) > 2(|a|2 |b|2 |c|2 + |d|2 ) = |a + d|2 |b + c|2 aa ad ad + dd = bb bc bc + cc |a d|2 = |b c|2 T c l AD = BC C ng (1) v i (2) v i v , ta c 2(|a|2 |b|2 + |c|2 |d|2 ) =< a b d + c; a + b + c + d > v tng t nh trn, thu c AC = BD T (3) v (4) suy ra i u ph i ch ng minh. V d 5.12. G i G l tr ng tm t gic ABCD. Ch ng minh r ng GA (4) (3)

GD AD = MN , trong M, N theo th t l trung i m AD, BC. L i gi i. G i w l t a v c a i m W trong m t ph ng ph c. Do G(g) l tr ng tm t gic ABCD nn g =


a+b+c+d 4

Ta c GA GD < a g; d g >= 0 a+b+c+d a+b+c+d < a ;d >= 0 4 4 < 3a b c d; 3d b c a >= 0 < (a b c + d) + 2(a d); (a b c + d) 2a d) >= 0 < a + d b c; a + d b c >= 4 < a d; a d > a+d b+c 2 2 V d
2

= |a d|2 MN = AD

5.13 (St. Petersburg 2000). Cho tam gic ABC nh n n i ti p trong l ti p tuy n c a t i B, K l hnh chi u c a

ng trn . ng th ng

tr c tm H c a tam gic ABC trn , v g i M l trung i m AC. Ch ng minh r ng tam gic BKM cn.

5.2. M t s v d p d ng

211

L i gi i. Khng m t t ng qut, coi l ng trn n v , a = x + yi, b = i, c = z + ti. Khi =


x+z 2

+i

y+t 2

Do H l tr c tm c a tam gic, nn h = x + z + (y + t + 1)i. Khi k = x + z + i. Ta c |b | = V |k | =


x+z 2 2 x+z 2 2 y+t2 2 2

=
1 2

1 2

(x + z)2 + (y + t 2)2 (x + z)2 + (y + t 2)2

(1) (2)

2yt 2 2

T (1),(2) suy ra i u ph i ch ng minh. V d 5.14. Cho tam gic ABC n i ti p ng trn . G i A1 l trung i m c nh BC v A2 l hnh chi u c a A1 trn ti p tuy n c a t i A. Cc i m B1 , B2, C1 , C2 c xc nh m t cch tng t . Ch ng minh r ng cc ng th ng A1A2 , B1B2 , C1C2 ng quy. Hy xc nh v tr hnh h c i m ng quy. L i gi i. Khng m t t ng qut, coi l ng trn n v . G i w l t a v c a i m W trong m t ph ng ph c.

A2 A

HN

O C

A1

Ta c a1 =

b+c 2

v ng th ng A1A2 l ng th ng i qua A1 (a1), song

212

Chng 5. M t s

ng d ng c a s ph c trong hnh h c

song v i OA, do A1 A2 c phng trnh az az = a b+c a 2 b+c 2

Do aa = 1 nn phng trnh c vi t l i d i d ng z a2z = hay z a2z = a+b+c a+b+c a2 2 2


a+b+c 2

b+c a2 2

b+c 2

G i N l tm ng trn Euler c a tam gic, th n =

do A1A2 i

qua N . Tng t cng c B1 B2 , C1C2 i qua N (PCM)

5.3

Ch ng minh b t ng th c hnh h c

Vi c bi u di n cc i m trong m t ph ng b ng cc s ph c (to v ) cho php chng ta a cc b t ng th c hnh h c v cc b t ng th c v m-un s ph c. Khi , cc h ng ng th c i s v b t ng th c tam gic n gi n: |z1 + z2| ? |z1| + |z2|, trong r t nhi u tr ng h p, l cha kho cho l i gi i bi ton. V d 5.15 (B t ng th c Ptolemy). Cho t gic ABCD. Ch ng minh r ng ta lun c AB.CD + AD.BC AC.BD. D u ng th c x y ra khi v ch khi A, B, C, D theo th t l nh c a m t t gic l i n i ti p m t ng trn. L i gi i. Xt m t ph ng ph c, g i a, b, c, d l t a v c a cc nh A, B, C, D trong m t ph ng ph c. Ta c AB.CD + AD.BC = |a d| |d c| + |d a| |c b| |(a d) (d c) + (d a) (c b)| = |(c a)(d b)| = AC.BD

5.3. Ch ng minh b t ng th c hnh h c

213

D u ng th c x y ra khi (b a)(d c) = t(d a)(c b), t > 0. Khi 1 dc da da = arg ba t cb ba = arg dc cb

hay DAB = DCB hay t gic ABCD n i ti p ng trn. V d 5.16. Cho tam gic ABC v m t i m M ty n m trong m t ph ng tam gic. Ch ng minh r ng MB.MC MC.MA MA.MB + + 1 AB.AC BC.BA CA.CB D u ng th c x y ra khi no? L i gi i. Ta c (m a)(m b) (m b)(m c) (m c)(m a) + + = 1. (c a)(c b) (a b)(a c) (b c)(b a) (5.1)

Ch n h t a nh n ng trn ngo i ti p tam gic lm ng trn n v . G i m, a, b, c tng ng l to v c a M, A, B, C tng ng. Khi MA = |m a|, MB = |m b|, M C = |m c|, AB = |a b|, AC = |a c|, BC = |b c|. p d ng b t ng th c v gi tr tuy t i, t (5.1) suy ra |m a| |m b| |m b| |m c| |m c| |m a| + + 1. |c a| |c b| |a b| |a c| |b c| |b a| v chnh l i u ph i ch ng minh. V d 5.17. Cho tam gic ABC v m t i m M b t k n m trong m t ph ng tam gic. Ch ng minh r ng MB.MC MC.MA MA.MB + + 1. AB.AC BC.BA CA.CB D u ng th c x y ra khi no?

214

Chng 5. M t s

ng d ng c a s ph c trong hnh h c

L i gi i. Ta c (m a)(m b) (m b)(m c) (m c)(m a) + + = 1. (c a)(c b) (a b)(a c) (b c)(b a) (5.2)

Ch n h t a nh n ng trn ngo i ti p tam gic lm ng trn n v . G i m, a, b, c tng ng l to v c a M, A, B, C, tng ng. Khi MA = |m a|, MB = |m b|, M C = |m c| AB = |a b|, BC = |b c|, CA = |c a|. p d ng b t ng th c tr tuy t i, t (5.3), suy ra |m a||m b| |m b||m c| |m c||m a| + + 1, |c a||c b| |a b||a c| |b c||b a| v chnh l i u ph i ch ng minh.

5.4

Cc bi ton hnh h c ch ng minh v tnh ton

S ph c c ng d ng to l n v hi u qu trong cc bi ton hnh h c. B ng cch bi u di n to v cc i m c a m t hnh hnh h c b ng cc s ph c, ta c th bi u di n cc i u ki n bi c b n ch t hnh h c b ng cc ng th c i s v chuy n k t lu n hnh h c v cc ng th c i s . Nh v y, bi ton ch ng minh hnh h c c th a v vi c ki m tra m t h ng ng th c, ho c m t h ng ng th c c i u ki n. V d 5.18. Cho tam gic ABC. Trong n a m t ph ng b AB ch a i m C, d ng hnh vung ABDE. Trong n a m t ph ng b BC ch a i m A, d ng hnh vung BCF G. Ch ng minh r ng GA vung gc v i CD v GA = CD. L i gi i. L y h t a vung gc c g c t i B, vct BC l chi u dng c a tr c th c. K hi u nhn c a cc nh c a tam gic ABC tng ng l a, b = 0, c.

5.4. Cc bi ton hnh h c ch ng minh v tnh ton

215

Khi t a c a G l ic. T a c a i m D l ia. G i gc gi a GA v CD k hi u l th = arg Xt


iac aic

ia c . a ic

= i. Do arg (i) = nn GA vung gc v i CD. Ngoi ra th 2 |GA| = |a ic| = | ia c| = |CD|.

(iac)(a+ic) (aic)(a+ic)

i(a2 +c2 ) a2 +c2

V y ta c i u ph i ch ng minh. Nh n xt 5.4. n bi u th c to c a cc php bi n hnh, ta th y php t nh ti n tng ng v i php c ng s ph c, php quay l php nhn v i s ph c c m-un b ng 1, php v t l php nhn v i s th c, php v t quay l php nhn v i s ph c b t k. V d 5.19 (IMO 1986). Trong m t ph ng cho tam gic A1A2A3 v i m P0 . V i m i s 4 ta t As = As3 . D ng dy i m P0 , P1 , . . . sao cho i m Pk+1 2 theo chi u l nh c a Pk v i php quay tm Ak+1 (k = 0, 1, . . . ) m t gc 3 kim ng h . Ch ng minh r ng n u P1986 = P0 th tam gic A1A2A3 l tam gic u. L i gi i. V i m i k k 0, tam gic Ak+1 Pk Pk+1 cn v i Pk Ak+1 Pk+1 = G i pk , ak l t a c a cc i m Pk , Ak tng ng v i k = 0, 1, . . . . 2 2 + i sin . Th pk+1 ak+1 = (pk ak+1 v i = cos 3 3 C th , p1 = a1 + (P0 a1 ), p2 = a2 + (p1 a2 ), p3 = a3 + (p2 a3 ). Do p2 = a2 a2 + [a1 + (p0 a1)] = (1 )a2 + (1 )a1 + 2 p0 , 2 . 3

216

Chng 5. M t s

ng d ng c a s ph c trong hnh h c

p3 = a3 a3 +[(1)a2 +(1)a1 +2p0 ] = (1)(a3 +a2 +2 a1 ]+3p0 , L i do 3 = 1 nn p3 = (1 )(a3 + a2 + 2 a1 + p0 , Nh n xt r ng p0 .p3 , p6 , . . . l p thnh c p s c ng v i s h ng u tin p0 v v i cng sai (1 )(a3 + a2 + 2 a1). V y nn n u P1986 = P0 th 3 662(1 )(a3 + a2 + 2 a1 ) + p0 = p0 . Suy ra a3 + a2 + 2 a1 M = 2 = 1 v 2 = 4 == 3 = v i = cos Thay vo (5.3) ta c a3 + ( 1)a2 + a1 = 0 suy ra a3 = a2 + (a1 a2) nn tam gic A1A2A3 u. V d 5.20. Cho tam gic ABC tr c tm H, v ng trn ng knh CH, c t cc c nh AB v AC t i P v Q. Ch ng minh r ng nh ng ti p tuy n t i i m P v Q i v i ng trn c t nhau t i i m gi a c a AB. L i gi i. Ch n h t a v i ng trn ngo i ti p tam gic ABC l ng trn n v . Do P, Q l chn ng cao c a tam gic h t A, B nn a p = 1 (a + b + c bc) 2 1 b) q = 2 (a + b + c ac (5.3) + i sin . 3 3

5.4. Cc bi ton hnh h c ch ng minh v tnh ton

217

Tm O c a ng trn ng knh CH l trung i m CH nn 1 1 1 a = (c + h) = (c + a + b + c) = a + (a + b). 2 2 2 G i M l trung i m c a AB, th m = Ta c


1 (a + b) 1 (a + b + c bc) a mp 2 2 = 1 1 0p c + 2 (a + b) 2 (a + b) bc a

a+b . 2

= Tng t ta c:

b 1 a ba bc c a = = c + bc a 1 + b a b+a ab mq = 0q a+b

Cc t s trn l s

o. Th t v y n u b = x0 + iy0, a = x1 + iy1 th

ab a+b

c ph n

th c l phn s v i t s b ng 0:
2 2 (x0 x1 )(x0 + x1 ) + (y0 y1 )(y0 + y1 ) = (x2 + y0 ) (x2 + y1 ) = 0. 0 1

Ch ng t MP OP , MQ OQ. ngha l MP, MQ l ti p tuy n c a ng trn (pcm). Nh n xt r ng, s ph c t ra c bi t hi u qu v i cc bi ton lin quan n vung gc. V d 5.21. V pha ngoi c a t gic l i ABCD, l n l t d ng cc hnh vung nh n AB, BC, CD, DA lm c nh. Cc hnh vung ny c tm l O1 , O2 , O3 , O4 . Ch ng minh r ng O1 O3 vung gc v i O2 O4 v O1 O3 = O2 O4 . L i gi i. Gi s cc hnh vung l ABMM , BCNN , CDP P , DAQQ c tm l O1 , O2 , O3 , O4 . Ta quy u c ch ci th ng l to v c a cc nh, ch ng h n a l to v c a i m A. Ta nh n th y r ng, i m M nh n c t php quay tm B, gc quay /2. T suy ra m = b + (a b)i.

218

Chng 5. M t s

ng d ng c a s ph c trong hnh h c

Tng t n = c + (b c)i, p = d + (c d)i, q = a + (d a)i. Do o1 = a + b + (a b)i a+m b + c + (b c)i = , o1 = , 2 2 2 c + d + (c d)i d + a + (d a)i , o4 = o3 = 2 2

Suy ra o3 o1 (c + d a b) + i(c d a b) = i. = o4 o2 a + d b c + i(d a b + c) Do O1 O3 vung gc O2 O4 . Hn n a, o3 o1 = | i| = 1 o4 o2 nn O1 O3 = O2 O4 . . V d 5.22 (IMO 17, 1975). V pha ngoi c a tam gic ABC, l n l t d ng cc tam gic ABR, BCP , CAQ sao cho P BC = CAQ = 450 , BCP = QCA = 300 , ABR = RAB = 150 . Ch ng minh r ng QRP = 900 , RQ = RP. L i gi i. Ta xt bi ton trong m t ph ng ph c. G i M l chn ng vung gc h t i m P xu ng ng th ng BC. Ta qui c ch ci th ng l t a v c a nh tng ng, ch ng h n, a l t a v c a i m A. V MP = MB v MC = 3 nn MP pm cm = i v = i 3. bm pm Do c + 3b bc +i p= 1+ 3 1+ 3 c + 3a ac +i q= 1+ 3 1+ 3

Tng t ta cng tnh c

5.4. Cc bi ton hnh h c ch ng minh v tnh ton

219

i m B nh n ng t i m A b ng php quay tm R, gc quay q = 1500 . Do 3 1 b=a + i . 2 2

T , b ng cc php bi n i i s , ta c c + 3b p bc c + 3a ac +i : +i = i. = q 1+ 3 1+ 3 1+ 3 1+ 3 Suy ra QR vung gc v i P R hay QRP = 900 . Hn n a, |p| = |iq| = |q| nn RQ = RP . Bn c nh cc bi ton ch ng minh vung gc, s ph c cng t ra hi u qu trong cc bi ton v th ng hng, ng quy. V d 5.23. Cho ABCD v BNMK l hai hnh vung khng giao nhau, E

l trung i m c a AN . G i F l chn ng vung gc h t B xu ng ng th ng CK. Ch ng minh r ng cc i m E, F, B th ng hng. L i gi i. Ta xt bi ton trong m t ph ng ph c. Ch n F lm g c to v CK, F B l n l t l tr c honh v tr c tung. G i c, k, bi l n l t l to v c a cc i m C, K, B v i c, k, b R. Php quay tm B, gc quay q = 900 bi n i m C thnh i m A, do A c to v l a = b(1 i) + ci. Tng t , i m N l nh c a i m K qua php quay tm B, gc quay q = 900 nn i m N c to v l n = b(1 + i) ki. T suy ra to v i m E, trung i m c a o n th ng AN l e= a+n ck =b+ . 2 2

T suy ra i m E n m trn ng th ng F B hay cc i m E, F, B th ng hng. V d 5.24. Trn cc c nh AB, BC, CA c a tam gic ABC ta l n l t d ng cc tam gic ng d ng c cng h ng l ADB, BEC, CF A. Ch ng minh r ng cc tam gic ABC v DEF c cng tr ng tm.

220

Chng 5. M t s

ng d ng c a s ph c trong hnh h c

L i gi i. Ta qui c ch ci th ng l t a v c a nh tng ng, ch ng h n, a l t a v c a i m A. V ADB, BEC, CF A l cc tam gic ng d ng c cng h ng nn eb f c da = = = z. ba cb ac Do d = a + (b a)z, e = b + (c b)z, f = c + (a c)z. Suy ra c+d+f a+b+c = . 3 3 Hay cc tam gic ABC v DEF c cng tr ng tm. V d 5.25 (IMO Shortlist). Cho ABC l m t tam gic u c tm l S v

A B O l m t tam gic u khc c cng h ng. G i M, N l n l t l trung i m c a cc o n th ng A B v AB . Ch ng minh r ng cc tam gic SB M v SA N ng d ng. L i gi i. G i R l bn knh c a ng trn ngo i ti p c a tam gic ABO, t e = cos 2p 2p + i sin 3 3

Ta xt bi ton trong m t ph ng ph c. Ch n S l g c t a v SO l tr c th c (tr c honh). Khi , to c a cc i m O, A, B l R, Re, Re2 . G i R + z l t a c a i m B , th R ze l t a c a i m A . Suy ra to c a M, N l zM Re2 + R ze R(e2 + 1) ze Re ze e(R + z) zB + zA = = = = , = 2 2 2 2 2 z Re + R z R(e + 1) + z Re2 + z zA + zB = = = = 2 2 2 2 2
R e

zN = Ta c

R ze 2e

zB zS R+z zA zS R ze = e(R+z) = Rze e = 1 |e|2 = 1. e zM zS zN zS 2e


2

T suy ra cc tam gic SB M v SA N ng d ng.

5.4. Cc bi ton hnh h c ch ng minh v tnh ton

221

5.4.1

S ph c v a gic u

Cn b c n c a n v l cc s ph c c bi u di n trn m t ph ng to l nh c a m t n-gic u. Tnh ch t n gi n ny c th s d ng gi i nhi u bi ton lin quan n n- gic u. V d 5.26 (Romania 1997). Cho n > 2 l m t s nguyn v f : R2 R l

m t hm s sao cho v i m i n-gic u A1A2 . . . An , ta c f (A1 ) + f (A2 ) + + f (An ) = 0. Ch ng minh r ng f (A) = 0 v i m i A thu c R2 . L i gi i. Ta ng nh t R2 v i m t ph ng ph c v t = e2i/n . Khi i u ki n bi chnh l ng v i m i s ph c z v s th c t ta u c
n

(z + t)j = 0.
j=1

T , nh nh ng tr ng h p ring, ta c v i m i k = 1, 2, . . . , n
n

(z k + t)j = 0.
j=1

C ng cc ng th c ny l i, ta c
n n

(z (1 m ))k = 0.
m=1 k=1

V i m = n t ng trong b ng nf (z); v i cc gi tr m khc, t ng trong l i ch y qua nh c a n-gic u, do b ng 0. V y f (z) = 0 v i m i z C. V d 5.27 (Balkan MO 2001). M t ng gic l i c cc gc b ng nhau v c cc c nh l cc s h u t . Ch ng minh r ng ng gic u. L i gi i. Ta dng s ph c gi i. Gi s nh c a a gic l i l cc s ph c v1, v2, . . . , v5. Xt z1 = v2 v1 , z2 = v3 v2, z3 = v4 v3, z4 = v5 v4, z5 = v1 v5.

222

Chng 5. M t s

ng d ng c a s ph c trong hnh h c

Khi ta c z1 + z2 + z3 + z4 + z5 = 0. t wj = zj /z1 . Ta c w1 = 1 v w1 + w2 + w3 + w4 + w5 = 0. V ng gic l i c cc gc b ng nhau, ta c (nh s l i n u c n), w2 = a2, w3 = a3 2 , w4 = a4 3 , w5 = a5 4 , trong = e2pi/5 v m i aj l cc s h u t . Nh v y l nghi m c a a th c v i h s h u t 1 + a2 + a3 2 + a4 3 + a5 4 = 0. Nhng a th c t i ti u c a trn Q[x] l 1 + t + t2 + t3 + t4, t c 1 + + 2 + 3 + 4 = 0. T suy ra a2 = a3 = a4 = a5 = 1, t c l |wj | = 1, suy ra |zj | = |z1|, ngha l ng gic l u. 5.4.2 V d ng th c l ng gic trong tam gic 5.28. Cho tam gic ABC c A = /7, B = 2/7, C = 4/7. Ch ng OH = OIa = R 2. L i gi i. Ta c 2 4 a = 2R sin , b = 2R sin , c = 2R sin . 7 7 7 V y nn OH 2 = 9R2 (a2 + b2 + c2) = 9R2 4R2 sin2 3 1 2 2 3 1 9R2 4R2 + 2 2 t z = cos + i sin , ta c 7 7 9R2 4R2 z + z3 + z5 = 2 4 + sin2 + sin2 7 7 7 2 4 cos + cos + cos 7 7 7 3 5 cos + cos + cos . 7 7 7

minh r ng

1 z 1 z7 z = 2 = z2 1 z 1 1z

5.5. B ng cc cng th c c b n ng d ng s ph c vo gi i ton hnh h c

223

L y ph n th c hai v ta c 3 5 1 + cos + cos = 7 7 7 2 V y OH 2 = 9R2 7R2 v OH = R 2. cos V d 5.29. Cho tam gic ABC c A = /7, B = 2/7, C = 4/7. Ch ng

minh r ng R = 2ra . L i gi i. Ta c
2 OIa = R2 +

sin sin 2 sin 4 abc 7 7 7 = R2 + 4R2 b+ca sin + sin 2 + sin 4 7 7 7

Do 4 2 3 5 7 + sin sin = sin + sin sin + sin 7 7 7 7 7 7 7 4 3 4 3 4 cos 2 sin sin = 2 sin cos cos = 2 sin 7 7 7 7 7 7 7 2 4 sin = 4 sin sin 7 7 7 2 V y nn OIa = R2 + R2 = 2R2 suy ra OIa = R 2. sin V d 5.30. Cho tam gic ABC c A = /7, B = 2/7, C = 4/7. Ch ng a2 + b2 + c2 = 7R2 . L i gi i. Ta c a2 + b2 + c2 = 4R2 sin2 2 4 + sin2 + sin2 7 7 7 = 4R2 7 = 7R2 4

minh r ng

5.5

B ng cc cng th c c b n gi i ton hnh h c

ng d ng s ph c vo

Sau y l m t b ng (li t k) cc cng th c c b n c n dng n trong vi c gi i ton Hnh h c ph ng b ng s ph c m chng ta s g p sau ny.

224

Chng 5. M t s

ng d ng c a s ph c trong hnh h c

l nh ng cng th c tnh di, tnh l n c a gc, tnh di n tch, tnh t s n (c a ba i m), tnh t s kp (c a b n i m); thi t l p i u ki n song song, vung gc c a cc ng th ng, i u ki n (d u hi u) ng d ng c a hai tam gic, i u ki n lin thu c, phng trnh ng trn... 1. OA = aa, AB = (a b)(a b) 2. C (AB) AB AC = (b a)(c a) (abc) = ca ca = = (abc) cb cb (t s n c a ba i m A(a), B(b), C(c) l s th c)
2 2

3. B n i m A, B, C, D th ng hng hay ng vin c n v l t s kp (a, b, c, d) l th c (a, b, c, d) = C th A, B, C, D th ng hng khi v ch khi ca da ca da : ; v R cb db cb db ca da ca da : = : = (a, b, c, d) cb db cb db

A, B, C, D ng vin c n v l

ca da ca da : R nhng v R cb db cb db

4. B n i m A(a), B(b), C(c), D(d) khng th ng hng. Khi AB v AB CD ba iR t c l Re dc ba dc =0 CD ba R dc

5. Tam gic ABC u, c h ng thu n ( nh h ng dng) khi v ch khi a + b + c 2 = 0 trong = cos 2 + i sin 2 = ei 3 3 3
2

5.5. B ng cc cng th c c b n ng d ng s ph c vo gi i ton hnh h c

225

(Hay c bi t, c th x y ra A B C) Tam gic ABC u, c h ng ngh ch ( nh h ng m) khi v ch khi a 2 + b + c = 0 trong = cos 2 + i sin 2 = ei 3 3 3
2

6. Hai tam gic A1B1 C1 v A2B2 C2 ng d ng cng h ng khi v ch khi b2 a2 b1 a1 = = ( C) c1 a1 c2 a2 Hai tam gic A1B1 C1 v A2B2 C2 ng d ng ng c h ng khi v ch khi b2 a2 b1 a1 = = ( C) c1 a1 c2 a2 7. Di n tch c a tam gic ABC nh h ng, v i cc nh A(a), B(b), C(c), c tnh theo cng th c i 4 a a 1 b b 1 c c 1 a a 1 b b 1 c c 1 =0

Do A(a), B(b), C(c) th ng hng khi v ch khi

8. Phng trnh ng th ng z + z + = 0 trong C , R 9. Kho ng cch t i m M(z0 ) n ng th ng : z + z + = 0 b ng | z0 + z0 + | 2 10. Phng trnh ng trn z z + z + z + trong C, R 11. Phng trnh ng trn i qua ba i m A(a), B(b), C(c) phn bi t, khng th ng hng c d ng za zc za zc : = : ba bc ba bc

226

Chng 5. M t s

ng d ng c a s ph c trong hnh h c

(b a)(d c) + (b a)(d c) 12. cos(AB; CD) = 2|b a| |d c| T AB CD (b a)(d c) + (b a)(d c) = 0 (b a)(d c) + (b a)(d c) 13. sin(AB; CD) = 2i|b a| |d c| 14. AB v CD l hai dy cung c a ng trn z z = R2 . Khi AB CD ab = cd v AB CD ab+cd = 0 (aa = bb = cc = dd = R2 )

15. AB, CD l hai dy cung c a ng trn n v zz = 1, (AB)(CD) = S. Khi s= a + b (c + d) (a a = b b = c c = d d = 1) ab cd

16. N u C l chn ng cao, h t nh C c a tam gic ABC n i ti p trong ng trn n v z z = 1, th c = ab 1 a+b+c 2 c

v i aa=bb=cc=dd=1 17. AB l dy cung c a ng trn n v z z = 1. Khi Z [AB] hay Z (AB) z + abz = a + b 18. C l giao i m cc ti p tuy n t i A v B c a ng trn n v z z = 1. Khi 2 1 1 = + c a b 19. AB l dy cung c a ng trn n v z z = 1. Khi v i m i i m M khng n m trn ng trn, hnh chi u (vung gc) c a M trn ng th ng AB c xc nh b i h= 1 (m abm + a + b) 2

5.6. Bi t p

227

20. Tam gic ABC n i ti p trong ng trn n v z z = 1, th th di n tch S c a n c xc nh b i S= i (a b)(b c)(c a) 4 abc

5.6

Bi t p

Bi 5.1. Cho ABCD l m t hnh vung c nh. Xt t t c cc hnh vung P QRS sao cho P, R n m trn hai c nh khc nhau v Q n m trn m t ng cho c a hnh vung ABCD. Tm t t c cc v tr c th c c a i m S. Bi 5.2 (Colombia MO 1997). Xt i m P n m bn trong ho c trn bin c a hnh vung ABCD. Tm gi tr l n nh t, gi tr nh nh t c th c c a f (P ) = ABP + BCP + CDP + DAP Bi 5.3 (Greece MO 1997). Cho t gic l i ABCD. G i E, F, G, H theo th t l tm cc hnh vung v i cc c nh AB, BC, CD, DA d ng ra pha ngoi t gic. Ch ng minh r ng - Trung i m cc ng cho c a hai t gic ABCD, EF GH l nh c a m t hnh vung. - EF v GH vung gc v i nhau v b ng nhau Bi 5.4. Trn cc c nh AB, BC, CA c a tam gic ABC, d ng ra pha ngoi ba tam gic ng d ng ABC1, A1BC, AB1C. Ch ng minh r ng tr ng tm hai tam gic ABC, A1B1 C1 trng nhau. H i k t lu n c a bi ton cn ng khng n u cc tam gic ABC1, A1 BC, AB1C d ng vo pha trong c a tam gic ABC? Bi 5.5. Trn ng trn cho tr c hai i m A, B c nh v m t i m M di ng trn . Trn tia MA l y i m P sao cho MP = MB. Tm qu tch i m P .

228

Chng 5. M t s

ng d ng c a s ph c trong hnh h c

Bi 5.6. Cho t gic l i ABCD. L y cc c nh AB, CD lm y, d ng ra ngoi cc tam gic vung cn ABX, CDY . Ch ng minh r ng 2XY AC + BD Bi 5.7. Cho tam gic ABC n i ti p trong ng trn n v . Bi t r ng t n t i (0; ) sao cho a + b sin + c cos = 0 ( y a, b, c l t a v c a cc 2 1+ 2 nh A, B, C), ch ng minh r ng 1 < SABC 2 Bi 5.8 (Romanian 2003). G i M, N, P, Q, R, S theo th t l trung i m cc c nh AB, BC, CD, DE, EF, F A c a l c gic ABCDEF . Ch ng minh r ng RN 2 = MQ2 + P S 2 MQ P S Bi 5.9 (Romanian 1994). Cho t gic l i ABCD. G i E, F, G, H theo th t l trung i m cc c nh AB, BC, CD, DA. Ch ng minh r ng AB CD BC 2 + DA2 = 2(EG2 + F H 2 ) Bi 5.10. Xt i m M trn ng trong ngo i ti p c a tam gic ABC, khng trng v i cc nh c a tam gic. Ch ng minh r ng tm ng trn Euler c a cc tam gic MBC.MCA, MAB l nh c a m t tam gic ng d ng v i tam gic ABC. Bi 5.11. Ch ng minh r ng tam gic ABC vung khi v ch khi ng trn Euler ti p xc v i ng trn ngo i ti p. Bi 5.12. Cho tam gic ABC. G i a , b v c theo th t l cc ng trn nh n cc ng trung tuy n k t A, B v C lm ng knh. Ch ng minh r ng, n u hai trong ba ng trn ny ti p xc v i ng trn n i ti p c a tam gic ABC, th ng trn th ba cng ti p xc v i ng trn n i ti p c a tam gic. Bi 5.13 (Romanian 2008).

5.6. Bi t p

229

Bi 5.14. Cho t gic ABCD n i ti p. G i Ea , Eb , Ec , Ed theo th t l tm ng trn Euler c a cc tam gic BCD, CDA, DAB, ABC. Ch ng minh r ng cc ng th ng AEa, BEb , CEc , DEd ng quy. Hy xc nh v tr hnh h c i m ng quy. Bi 5.15. Cc c nh AB, BC v CA c a tam gic ABC c chia thnh ba o n b ng nhau b i cc i m M, N ; P, Q v R, S. V pha ngoi c a tam gic ABC, d ng cc tam gic u MND, P QE, RSF . Ch ng minh r ng DEF l tam gic u. Bi 5.16. V pha ngoi c a tam gic ABC ta d ng cc tam gic hnh vung ABEF v ADGH l n l t c tm l O v Q. M l trung i m c a o n BD. Ch ng minh r ng OMQ l tam gic vung cn t i M. Bi 5.17 (IMO 1982 shortlist). V pha ngoi c a t gic l i ABCD, ta d ng cc tam gic u ABM, CDP ; v pha trong c a t gic, ta d ng cc tam gic u BCN, ADQ. Ch ng minh r ng t gic MNP Q l hnh bnh hnh. Bi 5.18. Cho ABC l tam gic nh n. Trn cng m t pha c a ng th ng AC ch a i m B, ta d ng cc tam gic vung cn DAB, BCE, AF C t i cc nh A, C, F . Ch ng minh r ng D, E, F th ng hng. Bi 5.19. G i A, B, C l cc i m i x ng c a A, B, C tng ng qua cc c nh BC, CA, AB. Tm i u ki n c n v gi a cc gc A, B, C c a tam gic ABC sao cho i) AB, BC, CA ng quy t i m t i m. ii) A, B, C th ng hng Bi 5.20. Cho A, B, C l ba nh lin ti p c a m t n-gic u, M l m t i m n m trn ng trn ngo i ti p n-gic u sao cho B v M n m khc pha i v i AC. Ch ng minh r ng MA + MC = 2MB cos n

230

Chng 5. M t s

ng d ng c a s ph c trong hnh h c

Bi 5.21. Cho P l m t i m b t k n m trn ng trn ngo i ti p hnh vung ABCD. Tm t t c cc s nguyn dng n sao cho i l ng P An + P B n + P C n + P Dn khng ph thu c vo v tr c a P . Bi 5.22. Cho tam gic ABC c cc c nh BC = a, CA = b, AB = c. M l m t i m b t k n m trong m t ph ng tam gic. Ch ng minh r ng a) a.MA2 + b.MB 2 + cMC 2 abc b) a.MB.MC + b.MC.MA + c.MA.MB abc. Bi 5.23 (China TST 1998). Cho tam gic nh n ABC c cc c nh BC = a, CA = b, AB = c. P l m t i m b t k n m trong m t ph ng tam gic. Ch ng minh r ng a.P B.P C + b.P C.P A + c.P A.P B = abc khi v ch khi P trng v i tr c tm c a tam gic ABC. Bi 5.24. Cho tam gic ABC c tr ng tm G. G i R, R1 , R2 , R3 l n l t l bn knh ng trn ngo i ti p cc tam gic ABC, GBC, GCA, GAB. Ch ng minh r ng R1 + R2 + R3 3R.

Chng 6

Kh o st dy s v phng trnh sai phn


6.1 M t s khi ni m c b n v tnh ch t c a sai phn
f : R R l m t hm s cho tr c v h =

nh ngha 6.1. Gi s

constant = 0. Ta g i sai phn c p 1 c a f l i l ng f (x) = f (x + h) + f (x). Gi s nh ngha c sai phn c p n 1 c a f . Khi , sai phn c p n c a f c nh ngha nh sau: n f (x) = [n1f (x)](n 1), 0 f (x) := f (x).

Ch 6.1. M c d h c th l m t h ng s b t k nhng gi tr th ng dng l h = 1. M t trong nh ng l do d n n ch xt h = 1 l: vi c tnh ton sai phn h u h n r t c n thi t cho vi c tnh t ng c a chu i. D n hay hi n, m i s h ng c a chu i l m t hm s bi n s nguyn xc nh v tr c a s h ng trong chu i v v y bi n c l p c gi s l gi tr nguyn sai khc m t n v . V i h = 1 ta c f (x) = f (x + 1) f (x).
231

232

Chng 6. Kh o st dy s v phng trnh sai phn

nh ngha 6.2. Bi u th c giai th a: Cho hm s fx : R R. Ta g i bi u th c fx fx1 fx2 fxn+1 l bi u th c giai th a. Xt d ng giai th a quan tr ng sau:

(a + bx)(n) = (a + bx)(a + bx 1) (a + bx n + 1). c bi t, khi a = 0, b = 1 ta c x(n) = x(x 1)(x 2) (x n + 1). nh ngha 6.3. Gi s hm y = f (x) cho d i d ng b ng yi = f (xi ) t i cc m c xi cch u: xi+1 xi = h = constant(i c xc nh nh sau: yi = yi+1 yi 2yi = (yi) = yi+1 yi, n yi = (n1 yi) = n1 yi+1 n1 yi . V d 6.1. x2 = (x + 1)2 x2 = 2x + 1; lg x = lg(x + 1) lg x = lg
x+1 x

0). Khi sai phn c a dy yi

= lg 1 +

1 x

sin x = sin(x + 1) sin x = 2 sin 1 cos(x + 1 ). 2 2 ax = (a 1)ax


b b sin(a + bx) = 2 sin 2 cos a + 2 + bx b cos(a + bx) = 2 sin 2 sin a + b 2

+ bx

x! = xx! (a + bx)(n) = bn(a + bx)(n1) , c bi t x(n) = nx(n1) .


b n sin(a + bx) = 2 sin 2 b n cos(a + bx) = 2 sin 2 n n(b+) 2 n(b+) 2

sin a + bx +
n

cos a + bx +

6.1. M t s khi ni m c b n v tnh ch t c a sai phn

233

nh ngha 6.4. Ta g i Efx = fx+1 l ton t d ch chuy n. D th y E m fx = E(E m1 )fx = fx+m . G i I l ton t ng nh t, t c l Ifx = fx , ta c = E I v v i s nguyn dng m ta c
m

m fx = (E I)m fx =
i=0

i (1)i Cm E mi fx , E 0 = I.

nh l 6.1. a. Sai phn c a h ng s b ng 0. b. Sai phn m i c p l ton t tuy n tnh. c. n (xn ) = n!hn ; m (xn ) 0(m > n) n, m nguyn dng. H qu 6.1. a. n axn = a(n!) b. n+1 xn = 0. c. N u fx l m t a th c b c n c a x c d ng fx = A0xn + A1xn1 + + An th n fx = A0(n!). nh l 6.2. a. N u P (x) l a th c b c n th theo cng th c Taylor
n

P := P (x + h) P (x) =
i=1

hi (i) P (x). i!

b. f (x + nh) = c. n f (x) =

n i=0

i Cn i f (x).

n i i i=0 (1) Cn f (x

+ (n i)h).

d. Gi s f C n [a, b] v (x, x + nh) [a, b]. Khi n f (x) = f (n) (x + nh), hn (0, 1).

234

Chng 6. Kh o st dy s v phng trnh sai phn

H qu 6.2. N u f C n [a, b] th khi h nh f (n) (x) n f (x) . hn

nh l 6.3. a. fx gx = fx gx + gx+1 fx (Cng th c sai phn t ng ph n).


n

b.
k=m

fx = fn+1 fm ,

(m < n).

nh l 6.4. (Newton) N u fx l m t a th c b c n c a x th n c th bi u di n d i d ng fx = f0 + x(1.8)f0 + x(1.10) 2 x(3) 3 x(n) n f0 + f0 + + f0 . 2! 3! n!

Ch ng minh: Theo nh ngha c a bi u th c giai th a, ta c th gi s fx = a0 + a1x(1.8) + a2x(1.10) + + an x(n) . L n l t l y sai phn cc c p c a fx , ta c fx = a1 + 2a2 x(1.8) + 3a3 x(1.10) + + nan x(n1) 2fx = 2 1 a2 + 3 2 a3x(1.8) + + n (n 1) an x(n2) 3fx = 3 2 1 a3 + + n (n 1) (n 2) an x(n3) n fx = an (n!). Cho x = 0 trong cc ng nh t th c trn ta c a0 = f0 , a1 = f0, a2 = V d 2f0 n f0 , , an = . 2! n!

6.2. Gi s fx l m t a th c. Tm s h ng th chn f8 c a dy sau:

f0 = 1, f1 = 4, f2 = 10, f3 = 20, f4 = 35, f5 = 56.

f8 = f0 + 8f0 +

87 2 876 3 f0 + f0 = 1 + 8 3 + 28 3 + 56 1 = 165. 2! 3!

6.1. M t s khi ni m c b n v tnh ch t c a sai phn

235

V d

6.3. Tm s h ng t ng qut c a fx bi t: f0 = 1, f1 = 4, f2 = 10, f3 =

20, f4 = 35, f5 = 56. x(1.10) 2 x(3) 3 1 f0 + f0 = [x3 + 6x2 + 11x + 6]. 2! 3! 6

fx = f0 + x(1.8)f0 + V d

6.4. Bi u di n fx = 2x3 3x2 + 3x 10 d i d ng chu i giai th a. fx = 2x3 3x2 + 3x 10 = 2x(3) + 3x(1.10) + 2x(1.8) 10.

Ch 6.2. Ta c th dng nh l php chia c d tnh cc h s trong bi u di n a th c thnh chu i giai th a nhanh hn cch dng b ng sai phn nh trn. Th t v y, gi s Pn (x) l a th c b c n d ng Pn (x) = b0 xn + b1 xn1 + b2xn2 + + bn1 x + bn . Theo nh l Newton ta c th bi u di n Pn (x) d i d ng Pn (x) = A0 + A1x + A2 x(x 1) + A3 x(x 1)(x 2) + + An x(x 1)(x 2) (x n + 1). (1.2) (1.1)

Ta c n xc nh cc Ai , i = 0, 1, 2, n. Chia Pn (x) trong (1.2) cho x. Ph n d l A0 = Pn (0) = bn v thng l Pn1 (x) = A1 +A2(x1)+A3(x1)(x2)+ +An(x1)(x2) (xn+1). Chia Pn1 (x) cho (x 1). Ph n d l Pn1 (1.8) = A1, v thng l Pn2 (x) = A2 + A3 (x 2) + + An (x 2)(x 3) (x n + 1). Ti p t c qu trnh cu i cng ta nh n c P1 (x) = An1 + An (x n + 1). Chia P1 (x) cho (x n + 1) ta nh n c ph n d An1 = P1 (n 1) v thng An . R rng An = b0 h s c a xn trong Pn (x). Cn b ng h s trong hai d ng c a Pn (x) ta tm c cc An .

236

Chng 6. Kh o st dy s v phng trnh sai phn

V d 6.5. Bi u di n fx = 2x3 3x2 + 3x 10 d i d ng chu i giai th a. fx = 2x3 3x2 + 3x 10 = Ax(x 1)(x 2) + Bx(x 1) + Cx + D. R rng D = 10 v A = 2. Chia fx cho x, thng l 2x2 3x + 3, d l 10 = D. Chia 2x2 3x + 3 cho (x 1), thng l 2x 1, d l C = 2. Chia 2x 1 cho (x 2) thng l 2, d l B = 3. V y fx = 2x3 3x2 + 3x 10 = 2x(3) + 3x(1.10) + 2x(1.8) 10. Nh n xt 6.1. Trong nhi u bi ton c s d ng sai phn, ta th ng ph i khai tri n bi u th c giai th a x(n) thnh a th c c a x hay khai tri n n th c xn thnh chu i cc bi u th c giai th a d ng x(i) . Cc s Stirling lo i 1 v lo i 2 cho ta cc h s trong cc khai tri n trn. Cc s Stirling T nh ngha bi u th c giai th a, ta c cc cng th c sau a. x(n) = x(x 1) (x n + 1) b. x(n) = x(m)(x m)(nm) , m<n
n i=1 n

n n n c. x(n) = S1 x + S2 x2 + + Sn xn =

Sin xi

d. x(n+1) = x(n)(x n) =
i=1 n+1

Sin xi (x n)

e.
i=1

Sin+1 xi =

Sin xi(x n)
i=1

Do ta nh n c
n Sin+1 = Si1 nSin . n R rng, Sn = 1, n S0 = 0,

Sin = 0, i > n. Ta g i cc s Sin xc nh theo

cng th c trn l cc s Stirling lo i 1. Cc s ny c dng bi u di n h s c a xn trong khai tri n x(n) thnh a th c.

6.1. M t s khi ni m c b n v tnh ch t c a sai phn

237

n 1 2 3 4 5 6 7 Ta c

n S1 1 1 2 6 24 120 720

n S2

n S3

n S4

n S5

n S6

n S7

1 3 11 50 274 1764

1 6 35 225 1624

1 10 85 735

. 1 15 175
n

1 21

n n n x(n) = S1 x + S2 x2 + + Sn xn = i=1

Sin xi .

Cho x = 1 ta c
n

Sin = 0.
i=1

Do , t ng cc ph n t trn m t hng b ng 0. V d 6.6. Dng b ng trn vi t bi u th c giai th a x(6) d i d ng a th c. Ta c x(6) = x6 15x5 + 85x4 225x3 + 274x2 120x. S Stirling lo i 2:

Cc s Stirling lo i 2 c dng bi u di n h s c a x(n) trong khai tri n xn thnh chu i giai th a.

1 1 x = x(1.8) = S1 x(1.8) do S1 = 1 2 2 2 2 x2 = x(1.8) + x(1.10) = S1 x(1.8) + S2 x(1.10) do S1 = 1, S2 = 1 3 3 3 3 3 3 x3 = x(1.8) + 3x(1.10) + x(3) = S1 x(1.8) + S2 x(1.10) + S3 x(3) do S1 = 1, S2 = 3, S3 = 1.

T ng qut,
n n n n n xn = S1 x(1.8) + S2 x(1.10) + S3 x(3) + + Si1 x(i1) + Sin x(i) + + Sn x(n) (1.3)

238

Chng 6. Kh o st dy s v phng trnh sai phn

hay
n

x =
i=1

Sin x(i).

Theo cng th c Newton,


n

fx =
i=0

x(i)

i f0 = i!

i=0

ifx i!

.
x=0

Thay fx = xn ta c xn = x(1.8)[xn ]x=0 + x(1.10)[ hay


n

n x n 2 x n ]x=0 + + x(n) 2! n! i xn i!

x=0

x =
i=1

x(i)

.
x=0

Ta l i c Sin = v x(i+1) + ix(i) = x x(i). Theo nh ngha


n n+1 n+1 n+1 xn+1 = S1 x(1.8) + S2 x(1.10) + + Sin+1 x(i) + + Sn+1 x(n+1) = i=1

i xn i!

(1.4)
x=0

(1.5)

Sin+1 x(i) (1.6).

T (1.3) ta c
n

n+1

n n n n S1 xx(1.8)+S2 xx(1.10)+ +Si1 xx(i1) +Sin xx(i)+ +Sn xx(n)

=
i=1

Sin xx(i). (1.7)

n H s c a x(i) trong (1.6) l Sin+1 , h s c a x(i) trong (1.7) l Si1 + iSin . V y

ta c
n Sin+1 = Si1 + iSin .

6.2. Tnh t ng b ng phng php sai phn

239

V d

6.7.
5 4 4 5 4 4 S3 = 3S3 + S2 = 3(6) + 7 = 25; S4 = 4S4 + S3 = 4(1.8) + 6 = 10.

n 1 2 3 4 5 6 7

n S1 1 1 1 1 1 1 1

n S2

n S3

n S4

n S5

n S6

n S7

1 3 7 15 31 63

1 6 25 90 301

1 10 65 350

. 1 15 140 1 21

V d 6.8. Dng b ng trn khai tri n x6 thnh chu i cc bi u th c giai th a. Ta c x6 = x(6) + 31x(5) + 90x(4) + 65x(3) + 15x(1.10) + x(1.8).

6.2

Tnh t ng b ng phng php sai phn

Tr c tin ta xt bi ton sau: Xc nh gx sao cho gx = fx , v i fx l hm bi t. Nh n xt r ng, n u gx l m t l i gi i c a bi ton trn th gx + C v i C l h ng s b t k cng l l i gi i c a n. Trong ti li u ny ta s k hi u gx + C = 1fx , C R.

Ta d dng ki m tra c cc tnh ch t sau y c a 1 . nh l 6.5. a. 1 0 = C, C R, C R,

b. 1 (fx gx ) = 1fx 1 gx + C, c. 1 kfx = k1fx + C, k, C R,

d. 1[fx gx ] = fx gx 1 [gx+1 fx ] + C, V d 6.9. Ta c


x(3) 3

C R.

+ C = x(1.10). Do 1 x(1.10) = x(3) +C 3

240

Chng 6. Kh o st dy s v phng trnh sai phn


ax a1

V d 6.10. a. 1ax = b. 1 x(n) =


x(n+1) n+1

+ C, C R,

C R,

+ C,

c. 1 (a + bx)(n) =

(a+bx)(n+1) b(n+1) 1 b 2 sin 2 1 b 2 sin 2

+ C,
b 2

C R, + bx +C, C R, C R,

d. 1 sin(a + bx) = e. 1 cos(a + bx) =

cos a

b sin a 2 + bx +C,

f. 1 (xx!) = x! + C,

C R.

V d 6.11. Tnh a. 13x , b. 1 (x3 2x2 +7x12), c. 1 [x(x+1)(x+2)]. a. 1 3x =


3x 31

+C =

3x 2

+ C,

C R.

b. x3 2x2 + 7x 12 x(3) + x(1.10) + 6x(1.8) 12. 1 (x3 2x2 + 7x 12) = 1 (x(3) + x(1.10) + 6x(1.8) 12) = 1 x(3) + 1 x(1.10) + 61 x(1.8) 121 x(0) x(4) x(3) + + 3x(1.10) 12x(1.8) + C, C R. = 4 3 c. 1 [x(x + 1)(x + 2)] = 1 (x + 2)(3) (x + 2)(4) +C = 4 (x + 2)(x + 1)(x)(x 1) + C, = 4 V d 6.12. Tnh 1 x3x . S d ng tnh ch t d c a nh l 5, ta t fx = x, fx = 1, Do ta nh n c 1 x3x = x 3x 3 1 [ 3x 1] + C 2 2 3x 3 3x x 3 = x + C = 3x [ ] + C, 2 2 2 2 4 gx = 1 3x = 3x , 2 gx+1 = gx = 3x . Khi

C R.

3 3x+1 = 3x . 2 2

C R.

6.2. Tnh t ng b ng phng php sai phn

241

V d

6.13. Tnh 1 2x1 . 2x1

Ta ph i tm gx sao cho gx = fx = 2x 1 . 2x1

S d ng nh ngha sai phn ta c th gi s gx = f (x) , 2x1

trong f (x) l m t a th c c a x. V gx = fx = nn f (x) f (x + 1) f (x) x1 = x1 x 2 2 2 hay f (x + 1) 2f (x) = 4x 2. V ph i c a phng trnh ny l tuy n tnh. Do v tri ph i tuy n tnh, nn f (x) ph i c d ng f (x) = ax + b, ko theo f (x + 1) = ax + a + b. Do (ax + a + b) 2(ax + b) = 4x 2. Cn b ng h s ta nh n c ax 2ax = 4x, a + b 2b = 2. 2x 1 2x1

Suy ra a = 4, b = 2 v f (x) = 4x 2. T gx = V y 1 2x + 1 2x 1 = x2 + C, 2x1 2 C R. 4x 2 2x + 1 = x2 . 2x1 2

242

Chng 6. Kh o st dy s v phng trnh sai phn

Ti p theo ta c p n vi c tnh t ng b ng phng php sai phn. Gi s ta ph i tnh t ng


n1 k=1

ak khi ta tm dy {xk } sao cho xk+1 xk = ak . T c

l xk = ak . Khi ta c
n1 n1

ak =
k=1 k=1

xk = xn x1 = xk |n = 1 ak |n . 1 1

V d 6.14. Tnh t ng 1 2 + 2 3 + + n (n + 1). Ta c fx = x(x + 1) = (x + 1)(1.10).


n

fx = 1(x + 1)(1.10)|n+1 1
1

(n + 2)(n + 1)n (1 + x)(3) n+1 (n + 2)(3) 2(3) |1 = = . = 3 3 3 3 V d 6.15. Tnh t ng 12 + 22 + + n2 . Ta c fx = x2 = x(x 1) + x = x(1.10) + x(1.8).
n

fx = 1 [x(1.10) + x(1.8)]|n+1 1
1

(n + 1)n(n 1) (n + 1)n x(3) x(1.10) n+1 (n + 1)(3) (n + 1)(1.10) + ]|1 = + = + 3 2 3 2 3 2 n(n + 1)(2n + 1) . = 6 = [ V d qut l x3 + 7x. Ta c fx = x3 + 7x = x(3) + 3x(1.10) + 8x(1.8).
n

6.16. Tnh t ng c a n s h ng u tin c a chu i v i s h ng t ng

fx = 1 [x(3) + 3x(1.10) + 8x(1.8)]|n+1 1


1

6.2. Tnh t ng b ng phng php sai phn

243

x(4) (n + 1)(4) + x(3) + 4x(1.10)]|n+1 = + (n + 1)(3) + 4(n + 1)(1.10) 1 4 4 1 2 n(n + 1)(n + n + 14). = 4 = [ V d 6.17. Tnh t ng
n

S=
k=1

ak ,

ak =

1 . (k + 1) k + k k + 1

Ta c ak = 1 1 = (k + 1) k + k k + 1 k k + 1( k + k + 1) 1 k+1 k 1 1 = . = = k+1 k k+1 k k


n n

Do S=
k=1

ak =

1 1 1 1 =1 . = n+1 n+1 k k=1

V d 6.18. Ch ng minh r ng t n t i cc s A, B tho mn v i m i n nguyn dng, ng th c sau ng a1 + a2 + + an = Atgn + Bn, Ta c tg1 = tg[k (k 1)] = suy ra tg1 + tg1[tgk tg(k 1)] = tgk tg(k 1) = tg(k 1). Suy ra tgk tg(k 1) = Do
n n

ak = tgk tg(k 1).

tgk tg(k 1) , 1 + tgk tg(k 1)

tg(k 1) 1. tg1
n

ak =
k=1 k=1

tgk tg(k 1) = B = 1.

1 tg1

tg(k 1)
k=1 k=1

1=

1 (tgn tg0) n. tg1

1 T c A = tg1 ,

244

Chng 6. Kh o st dy s v phng trnh sai phn

V d 6.19. Cho c p s c ng a1, a2, , an , cng sai d. Tnh t ng


n n

Sn =
k=1

sin ak ,

Tn =
k=1

1 . sin ak sin ak+1

Ta c 2 sin an sin d d 3 1 = 2 sin[a1 +(n1)d]sin = cos[a1 +(n )d]cos[a1 +(n )d]. 2 2 2 2

t bn = cos[a1 + (n 3 )d], ta c 2 2 sin an sin Do d d 1 2Sn sin = bn = b1 bn+1 = cos(a1 ) cos(a1 + (n )d). 2 2 2 k=1 T ta nh n c Sn = Ta c cotgan cotgan+1 = Suy ra 1 1 (cotgan cotgan+1 ) = cotgan . = sin an sin an+1 sin d V y Tn = V d qut l x2x . (x + 2)! sin nd 1 1 sin(an+1 a1) 1 (cotgan+1 cotga1) = . = sin d sin d sin a1 sin an+1 sin d sin a1 sin(a1 + nd) 6.20. Tnh t ng c a n s h ng u tin c a chu i v i s h ng t ng sin(an+1 an ) sin d = . sin an sin an+1 sin an sin an+1 sin(a1 +
n1 d) sin( n d) 2 2 . sin d 2 n

d = bn bn+1 = bn . 2

6.2. Tnh t ng b ng phng php sai phn

245

Ta ph i tm gx sao cho gx = fx = x2x . (x + 2)!

S d ng nh ngha sai phn ta c th gi s gx = f (x) x 2 , (x + 1)!

trong f (x) l m t a th c c a x. T gx = gx+1 gx = fx , ta c f (x + 1)2x+1 f (x)2x x2x = (x + 2)! (x + 1)! (x + 2)! hay 2f (x + 1) (x + 2)f (x) = x. V ph i c a phng trnh ny l tuy n tnh. Do v tri ph i tuy n tnh, nn f (x) ph i l hm h ng. Gi s f (x) k. Khi f (x) = k f (x + 1) = k

v 2k (x + 2)k = x. T ta c k = 1 = f (x). V y gx = 1 2x . (x + 1)!

fx =
1 1

x2x (x + 2)!

246

Chng 6. Kh o st dy s v phng trnh sai phn

2x n+1 x2x n+1 |1 = | (x + 2)! (x + 1)! 1 2n+1 . = 1 (x + 2)! = 1 V d 6.21. Tnh tch phn h u h n: 1 2x1 . 2x1 Ta ph i tm gx sao cho gx = fx = Gi s gx = V gx = fx = nn f (x + 1) f (x) f (x) x1 = x1 2x 2 2 hay f (x + 1) 2f (x) = 4x 2. V ph i c a phng trnh ny l tuy n tnh. Do v tri ph i tuy n tnh, nn f (x) ph i c d ng f (x) = ax + b, ko theo f (x + 1) = ax + a + b. Do (ax + a + b) 2(ax + b) = 4x 2. Cn b ng h s ta nh n c ax 2ax = 4x, a + b 2b = 2. 2x 1 2x1 f (x) . 2x1 2x 1 . 2x1

Suy ra a = 4, b = 2 v f (x) = 4x 2. T gx = V y 1 2x + 1 2x 1 = x2 + C. 2x1 2 4x 2 2x + 1 = x2 . x1 2 2

6.2. Tnh t ng b ng phng php sai phn

247

a th c Bernoulli Trong vi c nghin c u a th c ta g p bi ton: Tm a th c b c n c a x, Pn (x) sao cho Pn (x) = hay Pn (x) = 1 Gi thi t Pn (x) c d ng
n

xn1 (n 1)!

(1.8)

xn1 + C. (n 1)!

(1.9)

Pn (x) =
i=0

Ai

xni xn xn1 = A0 + A1 + + An1 x + An . (n i)! n! (n 1)!

(1.10)

T (1.8) ta c Pn (x + 1) Pn (x) = Pn (x + 1) Pn (x) = Pn (x) =


xn2 (n2)! xn1 (n1)! xn2 (n2)!

M t khc, cng t (1.8) ta c Pn1 (x) = Suy ra Pn (x) = Pn1 (x) hay Pn (x) Pn1 (x) = k. T (1.10) ta c Pn (0) = An1 , Do k = 0. V y Pn (x) = DPn (x) = Pn1 (x). Pn (x) c g i l hm Bernoulli. Pn1 (0) = An1 .
xn2 (n2)!

248

Chng 6. Kh o st dy s v phng trnh sai phn

Ta c P2 (x) = P1 (x) = A0 x + A1 . T (1.8) v (1.9) ta nh n c P1 (x) = 1, suy ra P1 (x) = 1 x(0) = x + C1 , suy ra A0 = 1. M t khc, P3 (x) = P2 (x) = A0 2 x + A1x + A2. 2!
x(2) 2

T (1.8) v (1.9) ta nh n c P2 (x) = x, suy ra P2 (x) = 1 x(1) = C2 =


x(x1) 2

+ C2 , suy ra A1 = 1 . Ti p t c nh v y ta xc nh c t t c 2

cc h s A2, A3, . Tuy nhin ta c nh n xt sau y: xn xn1 + A1 + + An1 x + An . n! (n 1)! xn xn1 + A1 + + An1 x. n! (n 1)!
n1

Pn (x) = A0 Suy ra

Pn (x) = A0 Suy ra

Pn (x)|x=0 =
i=0

Ai . (n i)! = 0, do

T (1.8), n u n > 1 th Pn (x)|x=0 =


n1

| (n1)! x=0

xn1

i=0

Ai = 0. (n i)!

V v y, + V i n = 3: + V i n = 4: Tng t , A4 = 1/120, A5 = 0, A6 = 1/30240, A7 = 0, Ta thu c cc hm Bernoulli sau y: P1 (x) = x


1 2 A0 3! A0 4!

+ +

A1 2! A1 3!

+ +

A2 1! A2 2!

= 0, suy ra A2 = 1/12. +
A3 1!

= 0, suy ra A3 = 0.

6.2. Tnh t ng b ng phng php sai phn

249

P2 (x) = P3 (x) = P4 (x) = P5 (x) =

x2 2 x3 6 x4 24

x 2 x2 4

1 12 x 12 x2 24 x3 72

+ +

x3 12 x4 48

1 720 x . 720

x5 120

Tnh ch t c a a th c Bernoulli: 1. DPn (x) = Pn1 (x) suy ra DPn+1 (x) = Pn (x), suy ra
b

Pn (x)dx = Pn+1 (b) Pn+1 (a).


a

NX:
n1

Pn (0) = An , Pn (1.8) =
i=0

Ai + An = 0 + An = An . (n i)!

Do

Pn (x)dx = Pn+1 (1.8) Pn+1 (0) = An+1 An+1 = 0.


0

2. A2k1 = 0 v i k > 1. Ch ng minh: Ta c P2k+1 (x) = , P2k+1 (x + 1) P2k+1 (x) = Thay x b i x ta c P2k+1 (1 x) P2k+1 (x) = t F (x) = P2k+1 (1 x), suy ra P2k+1 (x) = P2k+1 (1 (x + 1)) = F (x + 1). Do F (x + 1) F (x) = F (x) = x2k . (2k)! x2k . (2k)! x2k . (2k)! x2k (2k)!

250

Chng 6. Kh o st dy s v phng trnh sai phn

Suy ra F (x) = P2k+1 (x) suy ra F (x) P2k+1 (x) = c hay F (x) P2k+1 (x) = 0 hay P2k+1 (1 x) P2k+1 (x) hay P2k (1 x) = P2k (x). o hm hai v ng th c ny theo x ta c P2k (1 x) = P2k (x) hay P2k1 (1 x) = P2k1 (x), hay P2k1 (x) + P2k1 (1 x) = 0. Do P2k1 (0) + P2k1 (1.8) = A2k1 + A2k1 = 2A2k1 = 0, Ta g i Bn (x) = n!Pn (x) l a th c Bernoulli. V d 6.22. B0 (x) = 1 B1(x) = x
1 2 1 6 x 2 1 30 x 6 x2 2

k>1

k > 1.

B2(x) = x2 x +

B3(x) = x3 3 x2 + 2

B4(x) = x4 2x3 + +x2 B5(x) = x5 5 x4 + 5 x3 2 3 B6(x) = x6 3x5 + 5 x4 2

1 42

B7(x) = x7 x5 7 x6 + 7 x5 7 x3 + x . 2 2 6 6 Ta c Pn (x) = xn1 , (n 1)!

6.2. Tnh t ng b ng phng php sai phn

251

do n!Pn (x) = nxn1 hay Bn (x) = nxn1 hay Bn+1 (x) = (n + 1)xn . Suy ra 1 xn = Suy ra
x1

1 Bn+1 (x) + C. n+1 Bn+1 (x) Bn+1 (1) . n+1

xn = 1 xn |x = 1
x=1

V d
x1

6.23. x(x 1)(2x 1) B3 (x) B3 (1.8) = . 3 6

x2 = 12 +22 + +(x1)2 = 1 x2 |x = 1
x=1

x1

x3 = 13 + 23 + + (x 1)3 = 1 x3 |x = 1
x=1

x(x 1) B4 (x) B4 (1.8) = 4 2

Hm Gamma Hm Gamma c xc nh b i tch phn

(n) =
0

xn1 ex dx

(1.11).

(n) xc nh v i b t k gi tr th c c a n lo i tr 0 v cc s nguyn m. V i n nguyn dng ta s ch ng t r ng (n) = (n 1)!.

252

Chng 6. Kh o st dy s v phng trnh sai phn

T nh ngha ta c

(1) =
0

ex dx = ex | = 0 + 1 = 1 0

(1.12).

Tch phn t ng ph n ta c
t t n1 x n1 t

x
0

e dx = t

+ (n 1)
0

xn2 ex dx.

Dng nh l LHospital ta c tn1 et ti n n 0 khi t ra . V v y,


(n) =
0

xn1 ex dx = (n 1)
0

x(n1)1 ex dx

(1.13)

hay (n) = (n 1)(n 1) v thay n b i n + 1 ta c (n + 1) = n(n), T (1.14) suy ra (n) = (n 1)(n 1) = (n 1)(n 2)(n 2) = (n 1)(n 2)(n 3) 3 2 1 (1) = (n 1)! (n) = (n + 1) . n (1.15) (1.14)

T (1.12) ta c (1) = 1, do (n) = (n 1)!. Ng i ta tnh c cc gi tr c a (n) v i 1 < n < 2 v nh cc cng th c (1.14) v (1.15) ta c th tnh (n) v i m i gi tr dng c a n.

6.2. Tnh t ng b ng phng php sai phn

253

V d

6.24. a. (3.2) = (2.2)(1.2)(1.2) = (2.2)(1.2)(0.9182) = 2.424.


(1.6) 0.6

b. (0.6) = c. (0.5) =

0.8935 0.6

= 1.489.

V i n l s th c m ta s dng cng th c (1.15) tnh (n). V d 6.25. (0.4) =


(0.6) 0.4

(1.6) (0.4)(0.6)

= 3.723

Ch 6.3. Ng i ta ch ng minh c r ng v i n = 0 v n nguyn m th (n) khng xc nh. Hm Beta Hm Beta c nh ngha b i


1

(m, n) =
0

xm1 (1 x)n1 dx

(1.16).

Hm Beta xc nh v i m i m, n > 0. t y = 1 x ta c
1 1

(m, n) =
0

m1

(1 x)

n1

dx =
0

y n1 (1 y)m1 dy = (n, m).

(1.17)

Ti p theo ta s tm m i lin h gi a hm Gamma v hm Beta. Trong (1.11) t x = z 2 , dx = 2zdz; ta c

(n) = 2
0

z 2n1 ez dz.

T ta c

(m) = 2
0

ex x2m1 dx

(n) = 2
0

ey y 2n1 dy

254

Chng 6. Kh o st dy s v phng trnh sai phn


(m)(n) = 4
0 0

ex

2 y 2

x2m1 y 2n1dydx.

Chuy n sang t a c c ta c

(m)(n) = 4
0 0

er r2m1 (cos )2m1 r2n1 (sin )2n1 rdrd


2

= 2
0

er r2(m+n)1 dr 2
0
2

(cos )2m1 (sin )2n1 d

= (m + n) 2
0

(cos )2m1 (sin )2n1 d.

Ta s ch ng minh r ng
2

(m, n) = 2
0

(cos )2m1 (sin )2n1 d.

t x = cos2 , (1 x) = sin2 , dx = 2 cos sin d. Ta c


2

(cos )
0

2m1

(sin )

2n1

0 (

cos2 )m1 (sin2 )n1 (2 cos sin d)

d =

2
2

= 2
0

(cos )2m1 (sin )2n1 d.

V y ta c (m)(n) = (m + n)(m, n) hay (m, n) = (m)(n) . (m + n)

6.2. Tnh t ng b ng phng php sai phn


2

255

V d 6.26. Tnh tch phn


0

sinn xdx, dy. Khi


1

n > 1. t y = sin x, dy = cos xdx.

Suy ra dx =

dy cos x

= (1 y 2 )
2

1 2

sinn xdx =
0 0 dz . 2 z

y n (1 y 2 )

1 2

dy.

t z = y 2, dz = 2ydy, dy =

Ta c

y n (1 y 2)
0

1 2

dy =
0

z 2 2 (1 z)
1

1 2

dz

1 = 2
0

n+1 1 2

(1 z) 2 1 dz

= =

1 n+1 1 , 2 2 2 n+1 1 2 2 2
n+1 2

1 2

n+1 2 n+2 2

Bi t p 1. Tnh cc t ng sau:
n

1. S = 1 1! + 2 2! + + n n! = 2. S = 13 + 23 + + n3 =
n k=1 k=1

k k!.

k 3.

3. S = sin x + sin 2x + + sin nx 4. S = cos x + cos 2x + + cos nx 5.S = a + aq + + aq n1 6. S = sin(a + x) + sin(a + 2x) + + sin(a + nx)

256

Chng 6. Kh o st dy s v phng trnh sai phn

7. S = cos(a + x) + cos(a + 2x) + + cos(a + nx) 8. S = 1 q + 2 q 2 + + n q n 9. S =


12 1 12 +22 2 12 +22 +32 3 12 +22 +32 ++n2 . n

+ +

10. 1 3 + 2 4 + 3 5 + + n(n + 2). 11. 1 22 + 2 32 + 3 42 + + n(n + 1)2 . 12. S = 13. S =


1 12

1 23

+ +

1 . n(n+1) 1 (n2)(n1)n

1 123

1 234

+ +
2

1 . (n1)n(n+1)

14. S = sin x + sin x + + sin n1 x. 2

15. S = 21 sin2

21

+ 22 sin2

22

+ + 2n sin2

2n

16. 6 9 + 12 21 + 20 37 + 30 57 + 42 81 + (n s h ng). 17. S =


1 14

1 47

1 710

+ (n s h ng).

2. Tnh cc t ng sau: 1. 12 2 + 22 22 + 32 23 + + n2 2n . 2. 2 2 + 6 22 + 12 23 + 20 24 + 30 25 + (n s h ng).


n

3.
1

x sin x.

4. Gi s fx l m t hm kh tch h u t b c n. Ch ng minh r ng, tch phn t ng ph n lin ti p cho ta cng th c 1 axfx = ax a a fx fx + a1 a1 a1


2

2fx + +(1)n

a a1

n fx .

5. S d ng k t qu cu 4 tnh
n n

3x x(2),
1 1

2x (x3 3x + 2).
1 . n(n+1)(n+3)

6. S =
n

1 124

1 233

1 346

+ +

7.
1 n

1 (5x2)(5x+3) 1 . (2x1)(2x+1)(2x+5) 12 3

8.
1

9. S =

23 32

34 33

45 34

+ (n s h ng).

3. Tnh cc t ng sau:

6.3. Phng trnh sai phn tuy n tnh v i h s h ng


n x 2x . (x+1)(x+2) 2x1 . 2x1 x2 +x1 . (x+2)! x! !. (2x+1)

257

1.
1 n

fx , fx = fx , fx =
1 n

2. 3.
1 n

fx , fx =

4.
1 n

fx , fx = 2x x fx , fx =
(a+x)2 . 3a+x

5.
0

4 Ch ng minh cc ng th c sau:
1

1.
0
2

x2n dx 1x2

2n+1 2

n+1 2

n+1
m+1 2

2.
0

sin x cos xdx = xn eax dx =


0 1 dx n 1x

1 2

(n+1) . an+1
1 n

n+m +1 2

3. 4.
0

=
n . 2

1 +1 n 2

5.
0

ex dx =

6.3

Phng trnh sai phn tuy n tnh v i h s h ng

i v i phng trnh sai phn tuy n tnh th b ng php i bi n ta a v h phng trnh tuy n tnh c p 1. Trong m c ny, h th ng l i m t s k t qu v cng th c nghi m phng trnh c p cao c suy ra m t cch tng t t phng trnh c p 1. nh l 6.6. Nghi m t ng qut xn c a (2.2) b ng t ng xn v x , v i x l n n m t nghi m ring b t k c a (2.2). nh ngha 6.5. xn1 , , xnk c g i l k nghi m c l p tuy n tnh c a

258

Chng 6. Kh o st dy s v phng trnh sai phn

(2.3) n u t h th c C1xn1 + + Ck xnk = 0 suy ra C1 = = Ck = 0. nh l 6.7. N u xn1 , xnk l k nghi m c l p tuy n tnh c a (2.3), th nghi m t ng qut xn c a (2.3) c d ng xn = C1 xn1 + + Ck xnk , trong C1, C2 , , Ck l cc h ng s tu . nh l 6.8. N u 1 , 2 , , k l k nghi m th c khc nhau c a (2.4) v c1 , c2 , , ck l k h ng s tu th xn = c1 n + c2 n + + ck n 1 2 k l nghi m t ng qut c a phng trnh sai phn tuy n tnh thu n nh t (2.3). Ch 6.4. N u phng trnh c trng (2.4) c nghi m th c j b i s, th ngoi nghi m n , ta c nn , n2 n , , ns n cng l cc nghi m c l p tuy n j j j j tnh c a (2.3) v do
s1 k i Cj ni n j i=0

xn =

+
j=i=1

Ci n . i

V d 6.27. Tm cc hm f : Z R th a mn cc i u ki n 5 f (x + y) + f (x y) = f (x)f (y), x, y Z, f(0) = 0, f(1) = . 2 Cho x = n Z, y = 1 ta c f (n + 1) + f (n 1) = f (n)f (1). t f (n) = un ta thu c phng trnh sai phn 5 5 un+1 = un un1 , u0 = f (0) = 0, u1 = . 2 2

6.3. Phng trnh sai phn tuy n tnh v i h s h ng

259

Cho x = 1, y = 0 ta c f (1)f (0) = 2f (1), suy ra f (0) = 2 = u0 . Ta d dng tm c nghi m f (x) = 2x + 1 , 2x x Z.

nh l 6.9. N u phng trnh c trng c nghi m ph c j = a + ib = r(cos + i sin ) th


k

xn =
j=i=1

1 2 Ci n + rn (Cj cos n + Cj sin n). i

V d

6.28. Cho f : N R th a mn cc i u ki n f (n + 2) = f (n + 1) f (n), f(1) = 1, f(2) = 0.

Ch ng minh r ng |f (n)|

2 3 , 3

n N .

t f (n) = un ta c bi ton gi tr ban u un+2 = un+1 un , u1 = f (1) = 1, u2 = f (2) = 0. Phng trnh c trng c nghi m ph c 1+i 3 1i 3 , 2 = . 1 = 2 2 Ta c = cos + i sin . Ta d dng tm c nghi m c a bi ton gi tr ban 3 3 u l n n 3 un = cos + sin . 3 3 3 |f (n)| 12 2 3 3 , + = 9 3 n N .

Do

nh l 6.10. N u phng trnh c trng c nghi m ph c j b i s th


k

xn =
j=i=1

Ci n +rn [(A1+A2n+ +As ns1 ) cos n+(B1 +B2n+ +Bs ns1 ) sin n]. i

260

Chng 6. Kh o st dy s v phng trnh sai phn

M t s tr ng h p c th tm nghi m ring m t cch n gi n. Tr ng h p fn = Pm (n),


yn = Qm (n),

mN

1. N u 1 , , k l cc nghi m th c khc 1 c a phng trnh (2.4) th m N, v i Qm (n) l a th c cng b c m v i fn . m N, v i Qm(n)


2. N u (2.4) c nghi m = 1 b i s th yn = ns Qm (n),

l a th c cng b c m v i fn . V d 6.29. Cho f : N R th a mn cc i u ki n f (n + 1) 2f (n) + f (n 1) = n + 1, f(1) = 1, f(2) = 0. Ch ng minh r ng (6f (n) 24) l b i c a n v i n bi ton gi tr ban u un+1 2un + un1 = n + 1, u1 = f (1) = 1, u2 = f (2) = 0. Phng trnh c trng c nghi m kp = 1. Nghi m t ng qut c a phng trnh thu n nh t l A + nB. Ta tm nghi m ring d i d ng n2(an + b). D dng tm c a = 1 , b = 1 . Do 6 2 un = A + Bn + n2 v nghi m c a bi ton gi tr ban u l un = f (n) = 4 Do (6f (n) 24) = (n3 + 3n2 22n) chia h t cho n. V d 6.30. ( d tuy n IMO - 1992) Gi s a, b l 2 s th c dng. Tm n3 n2 11 n+ + . 3 6 2 1 1 n+ 6 2 6. t f (n) = un ta c

t t c cc hm f : [0, ) [0, ) th a mn i u ki n f (f (x)) + af (x) = b(a + b)x.

6.3. Phng trnh sai phn tuy n tnh v i h s h ng

261

V phng trnh hm trn ng v i m i x [0, ) nn f (f (f (x))) + af (f (x)) = b(a + b)f (x), x = f (x). Tng t nh v y ta thu c f n+2 (x) + af n+1 (x) = b(a + b)f n (x). C nh x ta thu c phng trnh sai phn un+2 + aun+1 = b(a + b)un . Phng trnh c trng c 2 nghi m = b, = a b. Khi f n (x) = un = K bn + L (a b)n . Ta c u0 = x = K + l, u1 = f (x) = Kb L(a + b). V f n : [0, ) [0, ) nn 0 M t khc, do
b a+b n

f n (x) b =K n (a + b) a+b

+(1)n L.

0 khi n nn ta ph i c L = 0. V y f (x) = Kb = bx.

Tr ng h p fn = Pm (n) n
1. N u cc nghi m c a (2.4) u l cc nghi m th c khc th yn =

Qm (n) n, v i Qm (n) l a th c b c m.
2. N u (2.4) c nghi m = b i s th yn = ns Qm (n) n , v i Qm (n) l a

th c b c m. V d 6.31. Xt phng trnh sai phn xn+4 10xn+3 + 35xn+2 50xn+1 + 24xn = 48 5n .

262

Chng 6. Kh o st dy s v phng trnh sai phn

Phng trnh c trng c cc nghi m 1 = 1, 2 = 2, 3 = 3, 4 = 4 u khc 5. T ta nh n c x = 2 5n . n Tr ng h p fn = cos nx + sin nx, Tm nghi m ring d i d ng
yn = a cos nx + b sin nx.

, R

V d 6.32. Tm nghi m ring x phng trnh sai phn n xn+3 2xn+2 xn+1 + 2xn = (2 n n + 2 sin . 2) cos 4 4

S d ng phng php v a trnh by ta d dng tm c x = cos n Tr ng h p fn = gn1 + + gns


Tm nghi m ring yni ng v i hm gni , i = 1, , s. Nghi m ring yn ng

n . 4

v i fn s l
yn

=
i=1

yni .

V d 6.33. Tm nghi m ring x phng trnh sai phn n n n 3 3 cos + 10 2n + 2. xn+4 3xn+3 + 3xn+2 3xn+1 + 2xn = sin 2 3 2 3 Dng nguyn l ch ng nghi m v p d ng phng php trong 3 tr ng h p nu ta c x = sin n Bi t p 1. Xc nh s h ng t ng qut un c a dy s n u bi t n + n 2n n. 3

6.3. Phng trnh sai phn tuy n tnh v i h s h ng

263

un+1 = un + 2n, p s : un = n2 n + 2. u1 = 2. un+1 = 15un 14n + 1, b. p s : un = 99 n2 . u0 = 7. un+1 = 2un + 3n , c. p s : un = 7 2n + 3n . u0 = 8. un+1 = 7un + 7n+1 , p s : un = (101 + n)7n . d. u0 = 101. 1 1 un+1 = 2 un 2 sin n , 4 p s : un = cos n . e. 4 u0 = 1. 2. Dng phng php bi n thin h ng s tm nghi m ring u c a cc phng n a. trnh sai phn sau a. un+1 = un + n n!. p s : u = n!. n b. un+1 = 2un + 6 2n . p s : u = 3n 2n . n c. un+1 = un + cos nx. p s : d. un+1 = un +
1n . 2n+1 sin n 1 2 2 sin
x 2

u n

sin x = 0. 2
nn! . 5

p s : u = n

n . 2n

e. un+1 = 5un + 1 (n2 3n + 1)n!. p s : u = n 5 phng trnh sai phn sau

3. Dng phng php phng php hm Green tm nghi m ring x c a cc n 1. xn+1 = 2xn + n2 n + 1. S: x = n2 n 3 n 2. xn+1 = 5xn + n2 + 3n + 2. S: x = 1 k 2 7 k n 4 8 3. xn+1 = 3xn + (2 n)2n . S: x = n2n n 4. xn+1 = 2xn + cos n 2 sin n . S: x = sin n . n 2 2 2 4. Xc nh s h ng t ng qut un c a dy s n u bi t un+1 = (n + 1)un + 2n (n 1), a. u1 = 0. n un+1 = n+1 (un + 1), b. u1 = 0. c. d. un+1 = u1 = 0. un+1 = u1 = 0.
n(n+1) (un (n+2)(n+3) 25 32

+ 1), + 1),

n(n+1)(n+k) (un (n+k+1)(n+2k+1)

264

Chng 6. Kh o st dy s v phng trnh sai phn

5. Gi i cc phng trnh sai phn sau a. xn+3 7xn+2 + 16xn+1 12xn = 0. H ng d n: Phng trnh c trng c cc nghi m 1 = 2 (kp), 2 = 3. b. xn+3 5xn+2 + 8xn+1 6xn = 0. H ng d n: Phng trnh c trng c cc nghi m 1 = 3, 2 = 1 + i, 2 = 1 i. c. xn+6 3xn+5 + 4xn+4 6xn+3 + 5xn+2 3xn+1 + 2xn = 0. H ng d n: Phng trnh c trng c cc nghi m 1 = 1, 2 =, 3 = i (kp), 3 = i (kp). d. xn+3 7xn+2 + 16xn+1 12xn = n + 1. H ng d n: Phng trnh c trng c cc nghi m 1 = 2 (kp), 2 = 3 u khc 1. e. xn+4 xn+3 3xn+2 + 5xn+1 2xn = 1. H ng d n: Phng trnh c trng c cc nghi m 1 = 1 (b i 3), 2 = 2. f. xn+3 7xn+2 + 16xn+1 12xn = 2n (24 24n). H ng d n: Phng trnh c trng c cc nghi m 1 = 2 (kp), 2 = 3. 6. Tm t t c cc hm s f tho mn i u ki n a. f : R R, f (f (x)) = 3f (x) 2x, x R.

H ng d n: V phng trnh hm trn ng v i m i x R nn f (f (x)) = 3f (x) 2x, x R. Tng t nh v y ta thu c f n+2 (x) = 3f n+1 (x) 2f n (x). C nh x ta thu c phng trnh sai phn un+2 3un+1 + 2un , u0 = x, u1 = f (x). b. f : N N, f(1) = 1, 2f (n)f (k + n) 2f (k n) = 3f (n)f (k), k n.

6.3. Phng trnh sai phn tuy n tnh v i h s h ng

265

H ng d n: Cho k = n = 0 ta c f 2 (0) = 2f (0) suy ra f (0) {0, 2}. Gi s f (0) = 0. Thay n = 0 vo phng trnh hm trn ta c f (k) = 0k N nn f (1) = 0 (v l). V y f (0) = 2. Thay n = 1 vo phng trnh hm ta thu c bi ton gi tr ban u thu n nh t b c 2. c. f : N Z, f(1) = 1, f (k + n) 2f (n)f (k) + f (k n) = 3n 2k . H ng d n: Cho k = n = 0 ta c 2f 2 (0) + 2f (0) = 0 suy ra f (0) {0, 1}. Gi s f (0) = 0. Thay n = 0 vo phng trnh hm trn ta c

2f (k) = 0k N nn f (1) = 0 (v l). V y f (0) = 2. Thay n = 1 vo phng trnh hm ta thu c bi ton gi tr ban u khng thu n nh t b c 2. 7. Xc nh s h ng t ng qut c a dy {xn } n u x1 = a, xm+n = xm + xn + mn, p s : xn = n
1 (n 2

m, n.

1) + a . Th l i th y k t qu ny th a mn bi.

8. T n t i hay khng m t dy s {xn } m m, n N ta c xm+n = xm + xn + mn. H ng d n: Gi s x1 = a. Gi i tng t v d trn ta c 1 xn = n[ (n + 1) + a] 1. 2 Th l i th y k t qu ny khng th a mn bi v i m i m, n. 9. Xc nh s h ng t ng qut c a dy {xn } n u bi t x1 = , x2 = , x m+n = xm +xn m, n N , m+n N. 2 2
2

H ng d n: D th y xn = x (n+1)+(n1) .
2

266

Chng 6. Kh o st dy s v phng trnh sai phn

Gi i phng trnh sai phn tuy n tnh thu n nh t v i i u ki n ban u x1 = , x2 = ta c xn = 2 + ( )n. 10. Xc nh dy s {xn } n u bi t xmn = xm xn . H ng d n: Ta c xm = xm1 = xm x1 suy ra x1 = 1. xn = xpk1 pk2 pk = k1 k2 k . 1 2
1 2

11. Xc nh s h ng t ng qut c a dy {xn } n u bi t x1 = , xn+1 = axn + bx2 + c, n a2 b = 1, > 0, a > 1.

H ng d n: Gi i phng trnh sai phn tuy n tnh thu n nh t v i x1 = , x2 = a + b2 + c = ta c xn = 2 n + 1 n , 1 = a + a2 1, 2 = 12 1 12 2 1 2 a a2 1. 12. Xc nh s h ng t ng qut c a dy {xn } n u bi t x1 = , x2 = 2 xn+1 = xn +a . xn1 H ng d n: a v phng trnh xn = t(xn+1 + xn1 ), t = 13. Hy tm t t c cc gi tr c a a R x1 = a xn+1 =
xn , 2+xn

x2 . x3 + x2

nN

xc nh m t dy, hy tm s h ng t ng qut c a dy s .

6.3. Phng trnh sai phn tuy n tnh v i h s h ng

267

H ng d n: t dy s ph yn =

1 , xn

khi ta c

yn+1 2yn = 1. Gi i phng trnh ny ta nh n c yn = suy ra xn = a . (a + 1)2n1 a (a + 1)2n1 a , a

Ta ph i tm gi tr c a a sao cho xn = 2, n. 14. Xc nh s h ng t ng qut c a dy {xn } n u bi t x1 = , x2 = xn =


x2 +2bxn1 bxn2 +c n1 . xn2 +b

H ng d n: t yn = xn + b. Khi ta c yn =
2 yn1 + c . yn2

Phng trnh d ng ny bi t cch gi i. 15. Xc nh s h ng t ng qut c a dy {xn } n u bi t x1 = , xn+1 = t yn =


1 , xn x n a+ b+x2 n

> 0, a > 1, a2 b = 1.

ta c yn+1 = ayn +
2 byn + c.

y l phng trnh sai phn bi t cch gi i. 16. Xc nh s h ng t ng qut c a dy {xn } n u bi t x1 = , xn+1 = an xn + fn , t dy s ph xn = yn


n1 k=0

an = 0.

ak .
n1

xn = [ + a0

fk
k i=0

k=1

ai

]
k=0

ak .

268

Chng 6. Kh o st dy s v phng trnh sai phn

Tr ng h p an = c = constan, ta c xn = [ + c
n1

k=1

fk n ]c , ck1

c > 1.

17. Xc nh s h ng t ng qut c a dy {xn } n u bi t x1 = > 0, an > 0, n N, k R xn+1 = an xk . n Logarit ho hai v c a phng trnh theo c s e ta c ln xn+1 = ln an + k ln xn . t dy s ph ln xn = yn a v phng trnh d ng yn+1 kyn = ln an . t dy s ph yn = k n1 un . xn = ek
n1 u n

= xn = ek

n1 [ln +

n1 ln ai ] i=1 ki

18. Xc nh s h ng t ng qut c a dy {xn } n u bi t x1 = > 0, xn+1 = fn+1 xk , n fk


n

fn > 0, n N, k R.

Chuy n v d ng xk xn+1 = n, k fn+1 fn t dy s ph vn =


xn . fn

Ta c
k vn+1 = vn .

Logarit c s e hai v , ta c ln vn+1 = k ln vn .

6.3. Phng trnh sai phn tuy n tnh v i h s h ng

269

t dy s ph un = ln vn . xn = fn [ kn1 ] . f1

19. Xc nh s h ng t ng qut c a dy {xn } n u bi t x0 = , xn+1 = ax2 b, n t dy s ph xn = byn suy ra y0 =


b

ab = 2, a, b R. = . Ta c

2 byn+1 = ab2yn b

hay
2 yn+1 = 2yn 1.

Xt tr ng h p || < 1 v ||

1, xn = b cos 2n hayxn = bch2n .

20. Xc nh s h ng t ng qut c a dy {xn } n u bi t x0 = , xn+1 = ax3 3xn , n t xn =


2 yn . a

a > 0.

Ta c

a x0 a y0 = = = 2 2
3 yn+1 = 4yn 3yn .

Xt tr ng h p || < 1 v || < 1. 2 1 n n xn = sin 3n = [( a + a2 4)3 + ( a a2 4)3 ], a 2 a v 1 xn = [( + a 2 1)3 + (


n

2 1)3 ].

21. Xc nh s h ng t ng qut c a dy {xn } n u bi t x0 = , xn+1 = ax3 + 3xn , n a > 0.

270

Chng 6. Kh o st dy s v phng trnh sai phn

t xn =

2 yn . a

Ta c

a = y0 = 2
3 yn+1 = 4yn + 3yn .

Do y0 = nn t n t i sao cho ch = . Ch ng minh b ng quy n p ta c yn = sh3n . Do 2 xn = sh3n . a 22. Xt phng trnh xn+1 =
axn +b , cxn +d

x0 cho tr c,

a, b, c, d R. Ch ng

minh r ng: N u (yn , zn ) l nghi m c a h phng trnh yn+1 = ayn + bzn zn+1 = cyn + dzn , v i n = 0, 1, 2 v y0 = , z0 = 1 th xn = sai phn h u t xn+1 = v i n = 0, 1, 2 v x0 = . H ng d n: Khi n = 0 th m nh trn ng do x0 =
y0 z0 yn zn

l nghi m c a phng trnh

axn + b , cxn + d

= . Gi s m nh

trn ng v i n, ta ch ng minh n ng v i n + 1. Ta c xn+1 r ng h yn+1 = ayn + bzn zn+1 = cyn + dzn , v i n = 0, 1, 2 v y0 = , z0 = 1 l h phng trnh sai phn tuy n tnh thu n nh t c p 2 bi t cch gi i. a yn + b axn + b yn+1 ayn + bzn zn = . = = = yn zn+1 cyn + dzn c zn + d cxn + d

6.4. H phng trnh sai phn tuy n tnh thu n nh t v i h s h ng

271

6.4

H phng trnh sai phn tuy n tnh thu n nh t v i h s h ng

Xt h phng trnh sai phn tuy n tnh thu n nh t k n d ng U n+1 = AU n , trong A l ma tr n vung c p k v U0 l vc t cho tr c. Gi s v1 , v2 , , vk l cc vc t ring c l p tuy n tnh tng ng v i cc gi tr ring 1 , 2 , , k c a A. Khi t n t i cc s 1 , 2, , k sao cho U 0 = 1 v 1 + 2 v 2 + + k v k . Ta c U n+1 = AU n = A2U n1 = = An+1 U 0 v U n = An U 0 = An (1 v1 + 2 v2 + + k vk ) = 1An v 1 + 2 An v 2 + + k An vk = 1 n v 1 + 2 n v 2 + + k n v k . 1 2 k Ta s ch ng t U n = 1 n v 1 + 2 n v 2 + + k n v k 1 2 k th a mn (3.1). Th t v y, AU n = 1 n Av1 + 2n Av2 + + k n Avk 1 2 k = 1 n+1 v1 + 2 n+1 v2 + + k n+1 vk 1 2 k = U n+1 . Vy U n = 1 n v 1 + 2 n v 2 + + k n v k 1 2 k (3.1)

272

Chng 6. Kh o st dy s v phng trnh sai phn

l nghi m t ng qut c a h phng trnh sai phn tuy n tnh (3.1). T ng h p l i v n v a nu ta c nh l sau: nh l 6.11. N u v 1, v 2, , vk l cc vc t ring c l p tuy n tnh tng ng v i cc gi tr ring 1 , 2 , , k c a A th nghi m t ng qut c a h (3.1) c d ng U n = 1 n v 1 + 2 n v 2 + + k n v k , 1 2 k trong 1, 2 , , k l cc s thu c tr ng K. minh h a nh ngha, ta tm nghi m t ng qut c a h phng trnh sai phn tuy n tnh U n+1 = AU n trong ta xt hai tr ng h p sau: Tr ng h p 1: v i A= Tr ng h p 2: v i A= i v i tr ng h p 1, ta c Gi tr ring c a A l 1,2 = eit . x G i v = y l vc t ring c a A tng ng gi tr ring . Ta c v1 = sin t i sin t , v2 = sin t i sin t (t = k, k Z). cosh t sinh t sinh t cosh t cos t sin t sin t cos t

Nghi m t ng qut c a h c d ng U n = 1 n v 1 + 2 n v 2 1 2 sin t sin t = 1 eitn i sin t + 2 eitn i sin t = sin t(1eitn + 2 eitn ) . i sin t(1eitn 2eitn )

6.4. H phng trnh sai phn tuy n tnh thu n nh t v i h s h ng

273

i v i tr ng h p 2, b ng phng php tng t ta tnh c nghi m t ng qut c a h l U n = sinh t 1 ent + 2ent ) . 1ent 2 ent )

i v i m t s tch phn suy r ng, n u ch s d ng phng php tch phn quen thu c th bi ton c th r t ph c t p. B ng vi c p d ng h phng trnh sai phn tuy n tnh, ta c th d dng tnh c cc tch phn . V d 6.34. Tnh cc tch phn sau

In =
0

xn ex sin xdx,

Kn =
0

xn ex cos xdx.

Tch phn t ng ph n : t u = xn ex , ta c In = nKn1 Kn , Kn = nIn1 + In . T ta c In + Kn = nKn1 , In Kn = nIn1 hay In = Kn i bi n In = n!xn , 2 Kn = n!yn , 2 n n (In1 + Kn1 ), I0 = 2 2 n n (In1 + Kn1 ), K0 = = 2 2 dv = sin xdx,

274

Chng 6. Kh o st dy s v phng trnh sai phn

ta c h xn = xn1 + yn1 , yn = xn1 + yn1 , H ny tng ng v i h xn+1 = xn + yn , yn+1 = xn + yn , Ta vi t h trn d i d ng U n+1 = AU n , v i A= 1 1 1 1 , Un = xn yn , U0 =


n 2 n 2

x0 =

n 2 n y0 = 2

x0 =

n 2 n y0 = 2

Cc gi tr ring c a A l 1,2 = 1 i. Cc vc t ring c l p tuy n tnh c a A tng ng v i cc gi tr ring 1 , 2 l v1 = Nghi m t ng qut c a h l Un = = = 1(1 + i)n + 2 (1 i)n 1i(1 + i)n 2 i(1 i)n (2)n ((1 + 2 ) cos n + i(1 2 ) sin n ) 4 4 ( 2)n ((1 2)i cos n (1 + 2 ) sin n ) 4 4 n n n (2) ( cos 4 + i sin 4 ) . ( 2)n (i cos n sin n ) 4 4 1 1 , v2 = 1 i .

T ta nh n c n n xn = ( 2)n ( cos + i sin ) 4 4 n n sin ) yn = ( 2)n (i cos 4 4

6.4. H phng trnh sai phn tuy n tnh thu n nh t v i h s h ng

275

Cho n = 0 ta c = 1 , i = 1 , do 2 2 xn yn V y, In Kn n n!( 2)n n + sin ) = (cos n+1 2 4 4 n n!( 2)n n sin ). = (cos 2n+1 4 4 n n ( 2)n (cos + sin ) = 4 4 2 n n n ( 2) (cos sin ). = 2 4 4

i v i m t s bi ton kh c a s h c, ta cng c th gi i quy t b ng cch p d ng h phng trnh sai phn tuy n tnh. minh h c i u ny, ta xt v d sau: V d 6.35. Cho n, k Z + . Ch ng minh r ng v i (x0, y0 ), x0, y0 Z + , t n

t i duy nh t (xn , yn ), xn , yn Z + th a mn (x0 + y0 k)n = xn + yn k (x0 y0 k)n = xn yn k. (3.2) (3.3)

T suy ra r ng: N u phng trnh x2 ky 2 = 1, k Z + c nghi m nguyn dng v i k no th n c v s nghi m nguyn dng. Ch ng minh b ng phng php quy n p. Ta ch c n ch ng minh (3.2) cn (3.3) ch ng minh tng t . Th t v y, v i n = 1 th (3.2) ng. Gi s (3.2) ng v i n t c l (x0 + y0 k)n = xn + yn k, ta ch ng minh (3.2) ng v i n + 1.

276

Chng 6. Kh o st dy s v phng trnh sai phn

Ta c (x0 + y0 k)n+1 = (x0 + y0 k)n (x0 + y0 k) = xn x0 + kyn y0 + (yn x0 + xn y0) k = xn+1 + yn+1 k, t ta c xn+1 = x0 xn + ky0 yn Z + yn+1 = y0 xn + x0 yn Z + . Ch ng minh tnh duy nh t: Gi s t n t i (xn , yn ) , xn , yn Z + th a (3.1). Khi ta c xn + yn k = xn + yn k xn xn = (yn yn ) k. Gi s yn = yn suy ra
xn xn yn yn

= k, nhng do k khng chnh phng nn

ta c i u mu thu n. V y, yn = yn v do xn = xn . Ta gi thi t k khng chnh phng v n u k chnh phng th k = x2 ky 2 = 1 x2


2 2 2

suy ra

y =1

x y = 1 x + y = 1 t y ta c x = 1 y = 0 do k khng th a gi thi t c a bi ton. By gi gi s phng trnh x2 ky 2 = 1, k Z +

6.4. H phng trnh sai phn tuy n tnh thu n nh t v i h s h ng

277

c nghi m nguyn dng (x0 , y0), x0, y0 Z + . Khi


2 x2 ky0 = 1 (x0 + 0

ky0)(x0

ky0 ) = 1.

V i m i n Z + ta c ky0)n (x0 ky0 )n = 1 (xn + yn k)(xn yn k) = 1 (x0 +


2 x2 kyn = 1. n

R rng, (xn , yn ) l nghi m c a phng trnh x2 ky 2 = 1, k Z + . V d 6.36. a th c Chebyshev c nh ngha nh sau:


sin(n+1) sin

V i x = cos ta t Tn (x) = cos n v Un (x) =

l cc a th c

Chebyshev lo i I v lo i II. Chng l nghi m c a cc phng trnh sai phn sau: Tn+1 = 2xTn Tn1 , T0 = 1, T1 = x v Un+1 = 2xUn Un1 , U0 = 1, U1 = 2x. Phng trnh (3.4) tng ng v i h phng trnh sau Tn+1 = 2xTn Sn Sn+1 = Tn . Ta vi t l i h ny d i d ng U n+1 = AU n , trong Un = Tn Sn Gi tr ring c a A l 1,2 = x A= x2 1. 2x 1 1 0 . (3.5) (3.4)

278

Chng 6. Kh o st dy s v phng trnh sai phn

G i v l vc t ring c a A tng ng v i cc gi tr ring 1 , 2 , ta c v1 = 1 , x x2 1 v2 = 1 . x + x2 1

Nghi m t ng qut c a h phng trnh sai phn tuy n tnh c d ng U n = 1 n v 1 + 2 n v 2 1 2 x 1 (x + 2 1)n + 2 (x x2 1)n . 1 (x + x2 1)n (x x2 1) + 2 (x x2 1)n (x + x2 1)

T ta c Tn = 1 (x + x2 1)n + 2 (x x2 1)n .

Cho n b ng 0 v b ng 1 ta thu c 1 + 2 = 1 1(x + x2 1) + 2 (x x2 1) = x, t ta tm c 1 = 1 , 2 V y, 1 1 Tn = (x + x2 1)n + (x x2 1)n . 2 2 Phng trnh (3.5) tng ng v i h Un+1 = 2xUn Rn Rn+1 = Un , t ta c ngay Un = 1(x + x2 1)n + 2(x x2 1)n . 2 = 1 . 2

Cho n b ng 0 v b ng 1 ta thu c 1 + 2 = 1

6.5. H phng trnh sai phn tuy n tnh v i h s h ng

279

1(x + t ta tm c

x2 1) + 2(x x2 1) = 2x, x x2 1 2 = . 2 x2 1

x + x2 1 1 = , 2 x2 1

Do

(x + x2 1)n+1 (x x2 1)n+1 . Un (x) = 2 x2 1 2 x2 1

6.5

H phng trnh sai phn tuy n tnh v i h s h ng

Xt h phng trnh sai phn tuy n tnh khng thu n nh t v i h s h ng un+1 = Aun + bn , (3.6)

trong A l ma tr n vung c k k, u0 l vc t cho tr c, bn l v ph i c a h . D th y nghi m c a (3.6) c d ng

un = A u0 +
j=0

Anj1 bj .

V n l ta c n tnh An . gi i quy t v n ny, chng ti xin gi i thi u m t s phng php nh sau: S d ng nh l Caley-Hamilton G i c() = det(I A) = k + ck1 k1 + + c1 + c0 = ( 1 )( 2 ) ( 2 ) l a th c c trng c a A. Theo nh l Caley-Hamilton th c(A) = 0, hay Ak + ck1 Ak1 + + c1A + c0 I = (A 1 I)(A 2 I) (A k I) = 0. Ta c cc nh l sau dng tnh An .

280

Chng 6. Kh o st dy s v phng trnh sai phn

nh l 6.12. Cho (A)kk l ma tr n khng suy bi n; 1 , 2 , , k l cc gi


j

tr ring c a A. t M(0) = I, M (j) =


i=1

(A i I), j 1. N u xj (n), j =

1, 2, , k th a mn x1(n + 1) = 1 x1 (n), x1 (0) = 1; xj+1 (n + 1) = j+1 uj+1 (n) + xj (n), xj+1 (0) = 0, j = 1, 2, , k 1,
k1

th An =
j=0

xj+1 (n)M(j).

Ch ng minh: Ta c x1(n) = n ;
n1

xj+1 (n) =
i=0

ni1 xj (i), j = 1, 2, , k 1; M(k) = 0. j+1

t
k1

(n) =
j=0

xj+1 (n)M(j).

Ta c n ch ng minh (I) = 0, (n + 1) = A(n). R rng


k1

(0) =
j=0

xj+1 (0)M(j) = x1 (0)M(0) = I.

Ta c
k1 k1

(n + 1) A(n) =
j=0

xj+1 (n + 1)M(j)
j=0

xj+1 (n)AM(j).

V AM(j) = (A j+1 I + j+1 I) Mj = M(j + 1) + j+1 Mj , nn


k1 k1

(n + 1) A(n) =
j=0

(xj+1 (n) j+1 xj+1 (n))M(j)


j=0

xj+1 (n)M(j + 1).

6.5. H phng trnh sai phn tuy n tnh v i h s h ng

281

M t khc, ta c
k1 k k1

xj+1 (n)M(j + 1) =
j=0 i=1

xi (n)M(i) =
j=1

xj (n)M(j),

do (n + 1) A(n) = (x1(n + 1) 1 u1(n))


k1

+
j=1

(xj+1 (n) j+1 (n) xj (n))M(j) = 0.

nh l c ch ng minh. V d 6.37. Xt A= Ta c
n1

0 1 1 2 3 1 3 1 4

c() = ( 2)2 ( 3), u1 (n) = n , uj+1 (n) = 1


i=0

nj1 uj (i). j+1

Do u1(n) = 2n ,
n1 n1

u2(n) =
i=0 n1

ni1 i

2 =
i=0

2n1 = n2n1 ,

u3(n) =
i=0

3ni1 i2i1 3 2
n1 n1

= V y A =
n

i
i=0

2 3

= 2n + 3n n2n1 .

n21 2n + 3n 2n1 3n n2n1 n n n1 n1 2 3 n2 (n + 2)2 2n + 3n n+1 n n1 n1 2 23 n2 n2 2n + 23n

282

Chng 6. Kh o st dy s v phng trnh sai phn

nh l 6.13. Cho (A)kk l ma tr n khng suy bi n, c() = k + ck1 k1 + + c1 + c0 l a th c c trng c a A, z(n) l nghi m c a phng trnh sai phn z(n + k) + ck1 z(n + k 1) + + c1 z(n + 1) + c0z(n) = 0, v i z(0) = z(1) = = z(k 2) = 0, z(k 1) = 1; q1(n), q2(n), , qk (n) th a c c2 1 . c2 q1(n) . . . . = . . . ck1 1 qk (n) 1 0 Khi An =
j=0

. . . ck1 . . . 1 . . . . . . . . .
k1

1 0 . . . . 0

z(n) . . . . z(n + k 1)

qj+1 (n)Aj .

Ch ng minh: t
k1

(n) =
i=0

qj+1 (n)Aj .

Ta c n ch ng minh (0) = I, (n + 1) = A(n). Ta c


k1

(0) =
j=0

qj+1 (0)Aj = q1 (0)A0 = I.

T gi thi t ta c q1(n+1) = c1 z(n+1)+c2z(n+2)++c(k1)z(n+k1)+z(n+k) = c0z(n), suy ra q1 (n + 1) + c0 z(n) = 0.

6.5. H phng trnh sai phn tuy n tnh v i h s h ng

283

Tng t q2 (n + 2) + c1 z(n + 1) = q1(n + 1) . . .

qk (n + k) + ck1 z(n + k 1) = qk (n + k 1). V qk (n) = z(n) nn q1(n + 1) + c0qk (n) = 0 q2(n + 1) + c1qk (n) = q1(n) . . .

qk (n + 1) + ck1 qk (n) = qk1 (n). Khi (n + 1) A(n) = = q1(n + 1)I + c1qk (n)A1 ck1 qk (n)Ak1 qk (n)Ak = q1(n + 1)I qk (n) c1 A1 + + ck1 Ak1 qk (n)An = q1(n + 1)I + c0 q k I = 0. nh l c ch ng minh. V d 6.38. Xt A= 0 1 1 2 3 1 3 1 4 .

284

Chng 6. Kh o st dy s v phng trnh sai phn

Ta c c() = 3 72 + 16 12. Phng trnh z(n + 3) 7z(n + 2) + 16z(n + 1) 12z(n) = 0, z(0) = z(1) = 0, z(2) = 1 c nghi m l z(n) = (2 + n)2n1 + 3n . Khi q1 (n) = 3(1 + n)2n + 43n q2(n) = (8 + 5n)2n1 4.3n q3 (n) = (2 + n)2n1 + 3n . Ta tnh c A0 = I, A2 = V y
2

5 4 5 9 8 5 14 4 14

A =
j=0

qj+1 (n)A =

2n1 3n n2n1 n21 2n + 3n 2n 3n n2n1 (n + 2)2n1 2n + 3n n+1 n n1 2 23 n2 n2n1 2n + 23n

nh l 6.14. Cho (A)kk l ma tr n khng suy bi n, c() = k + ck1 k1 + + c1 + c0 l a th c c trng c a A. Gi s y1(n), y2(n), , yk (n) l cc nghi m c a phng trnh c(E)y(n) = 0. N u {(y1(n), y2 (n), , yk (n))} l t p c l p tuy n tnh th t n t i cc ma tr n E1 , E2, Ek sao cho An = y1(n)E1 + + yk (n)Ek . Ch ng minh: Xt h + . . . + yk (0)Ek = y1 (0)E1 I A = y1 (1)E1 + . . . + yk (1)Ek . . . k1 = y1 (k 1)E1 + . . . + yk1 Ek . A

6.5. H phng trnh sai phn tuy n tnh v i h s h ng

285

V y1(0) y2 (0) . . . yk (0) y1(1) y2 (1) . . . yk (1) . . . y1(k 1) y2 (k 1) . . . yk (k 1) nn h trn c nghi m duy nh t (E1 , E2 , , Ek ) th a An = y1(n)E1 + + yk (n)Ek . nh l c ch ng minh. V d 6.39. Xt A= Ta c
3

=0

0 1 1 2 3 1 3 1 4

A =
j=1

yj (n)Ej ,

trong y1 (n) = 2n , y2(n) = n2n1 , y3 (n) = 3n , E1 , E2 , E3 l nghi m c a h I = y1(0)E1 + y2 (0)E2 + y3 (0)E3 A = y1(1)E1 + y2 (1)E2 + y3 (1)E3 A2 = y1(2)E1 + y2 (2)E2 + y3(2)E3 , suy ra E1 = V y A =
n

2 0 1 1 1 1 3 1 4

, E2 =

1 1 0 1 1 1 3 1 4

, E3 =

1 0 1 1 0 1 2 0 2

n21 2n + 3n 2n1 3n n2n1 n n n1 n1 2 3 n2 (n + 2)2 2n + 3n n+1 n n1 n1 2 23 n2 n2 2n + 23n

286

Chng 6. Kh o st dy s v phng trnh sai phn

V d 6.40. Xt A= Ta c A =
j=1 n

8 19 12 1 0 0 0 1 0
3

yj (n)Ej ,

trong y1(n) = 1n , y2(n) = 3n , y3 (n) = 4n , E1 , E2 , E3 l nghi m c a h I = E1 + E2 + E3 A = E1 + 3E2 + 4E3 A2 = E1 + 9E2 + 16E3 , D ki m tra c D = 6; DE1 = A2 7A + 12I; DE2 = 3A2 + 15A 12I; DE3 = 2A2 8A + 6I; trong A2 = Do 45 140 96 8 19 12 1 0 0.
16 3 4 3 1 3

E1 = V y

1 6 1 6 1 6

9 45 7 2 18 2 6 2 7 2 , E2 = 3 15 6 , E3 = 6 2 2 5 7 2 2 1 6 2 2
n n 1 93 + 164 6 2n 3n 1 33 + 44 6 2n 3 n 1 32 + 43 6

64 16 3 16 4 . 3 4 1 3

An =

7 + 453 6 2n 7 + 153 6 2n 7 + 53 6 2

644n 3 164n 3 44n 3

2 18 3n + 16 4n 2 6 3n + 4 4n . 2 2 3n + 4n

nh l 6.15. Cho (A)kk l ma tr n khng suy bi n, khi ma tr n A c phn tch duy nh t d i d ng A = S + N , (trong SN = NS, S n a n, N ly linh) v An = (S + N )n = S n + + n 2 n1 n2 2 1 S N + 2 S N ++ n nk+1 k1 N . k1 S

6.5. H phng trnh sai phn tuy n tnh v i h s h ng

287

Ch ng minh: Ta vi t c()d i d ng
s

c() =
i=1

( i ) i , ni 1, n1 + n2 + + ns = k.

Khi ta c th phn tch c()1 nh sau 1 c1() c2 () cs () = + ++ . n1 n2 c() ( 1 ) ( 2 ) ( s )ns Xt a th c fi () = ci ()


j=i

( j )nj ,

khi 1 = f1 () + f2 () + + fs (). V v i Fi = fi (A) ta c cc h th c sau F1 + F2 + F s = I, Fi Fj = Fj Fi = 0, Fi2 = Fi. t S=


i=1 s

i Fi , N = A S.

Ta c A = S + N, SN = NS, N k = 0, S n a n. nh l c ch ng minh. V d 6.41. Xt A= Ta c 1 1 1 = . + c() ( 2)2 3 1 = (1 )( 3) + ( 2)2 = f1 () + f2 (), Fi = f1(A). F1 = 2 0 1 1 1 1 2 0 1 , F2 = 1 0 1 1 0 1 2 0 2 . 0 1 1 2 3 1 3 1 4 .

288

Chng 6. Kh o st dy s v phng trnh sai phn

S = 2F1 + 3F2 =

1 0 1 1 2 1 2 0 4

, N = AS =

1 1 0 1 1 0 1 1 0

N 2 = 0, SN = NS = 2N, An = S n + n2n1 N. Tnh lu th a c a ma tr n Jordan Ta bi t r ng, v i ma tr n A cho lun t n t i ma tr n Q khng suy bi n sao cho = QAQ1 l ma tr n Jordan. Vi t phng trnh (3.6) d i d ng Qun+1 = QAQ1Qun + Qbn , t vn = Qun ta c

(3.7)

vn+1 = v 0 + Q
j=0

Anj1 bj .

Tr c h t ta nh c l i cch a m t ma tr n v ma tr n Jordan. Tm a th c c trng A I v s c trng c a ma tr n A c p n cho. Gi s i l nghi m c trng b i mi . i v i m i nghi m c trng i b i mi ta tm cc Jordan tng ng v i n (s l ng v c c a chng). S cc Jordan tng ng v i gi tr ring i b ng n ri , v i ri l h ng c a ma tr n A i I. Nh v y, n u n ri = mi th i tng ng v i mi d ng Ji (i ). N u n ri < mi th i tng ng v i gh d ng Jh (i ), v i gh = r(Ai I)h1 2r(Ai I)h + r(Ai I)h+1 , h = 1, 2, , ;
h=1

= mi .

c bi t, n u i l nghi m n c a phng trnh c trng th c m t Jordan Ji (i ) tng ng v i nghi m . Ti p theo ta nh c l i cch tm ma tr n Q a m t ma tr n cho v d ng Jordan. N u ma tr n A c p n c d n n d ng cho = Q1 AQ th cc to c a n vc t ring c l p tuy n tnh c a ma tr n A l cc c t c a ma tr n Q.

6.5. H phng trnh sai phn tuy n tnh v i h s h ng

289

N u d ng Jordan c a ma tr n A khng l d ng cho th ta cn c vo nguyn t c sau: N u f l nh x tuy n tnh m ma tr n c a n trong c s {e1, , en} l A, cn {e1, , en } l c s m trong nh x f c ma tr n Jordan , th ma tr n Q l ma tr n chuy n t c s {e1, , en } n c s {e1, , en}. T nh n xt trn, ta rt ra cc b c nh tm Q nh sau: - Tm d ng Jordan c a ma tr n cho; - Tm to c a cc vc t c s e1 , , en trong c s {e1, , en }; - Tm ma tr n chuy n t c s {e1 , , en } n c s {e1, , en }. V d 6.42. Tm ma tr n Q a ma tr n A= v d ng Jordan. - D th y a th c c trng c a A c 2 nghi m l = 2 v = 3. Do d ng Jordan c a A l = 2 0 0 3 . 5 6 1 0

- Hai vc t ring u, v c a A tng ng v i = 2, = 3 l u = (2b, b), v = (3a, a), a, b R. Do ta c Q= V d 3a 2b a b .

6.43. Tm ma tr n Q a ma tr n A= 1 0 1 0 2 0 1 0 3 .

v d ng Jordan. - D ng Jordan c a A l = 2 0 0 0 2 1 0 0 2 .

290

Chng 6. Kh o st dy s v phng trnh sai phn

T ma tr n ta c f (e1) = 2e1 f (e2) = 2e1 f (e3) = e2 2e3 Do e1 l vc t ring c a A tng ng v i gi tr ring 2. To x1, x2, x3 3x1 + 0x2 + x3 = 0 0x1 + 0x2 + 0x3 = 0 Ta c a e1 trong c s {e1, e2, e3 } l nghi m c a h x1 + 0x2 5x3 = 0. nh n c e1 = 0e1 + ke2 + 0e3 . D th y e2 l vc t ring c a A tng ng v i gi tr ring 2. Tng t , ta tm c e2 = te1 + 0e2 + te3. Gi s z = (z1 , z2, z3) l to c a e3. Khi Az = (t, 0, t) 2(z1 , z2, z3 ), t suy ra e3 = (t s)e1 + 0e2 + se3 ; s, t R, |s| + |t| = 0. V y Q= 0 t s t k 0 0 0 t s .

Bi t p 1. Tm nghi m t ng qut c a h phng trnh sai phn thu n nh t un+1 = Aun v i ma tr n A c cho b i 0 1 4 2 , 1 0 , 5 2 5 3 2 8 5 4 4 3 3 ,

7 6 2 6 ,

2 1 0 1 3 1 0 1 2

0 1 1 1 1 0 1 , 3 1 1 0 2 2 1 1 3 1 1 , 9 3 4

1 4 2 1 , 1 1 0 1 0 0 0 1 , 1 3 3

6.6. M t s l p phng trnh sai phn phi tuy n c ch m

291

1 0 0 1 0 0 2. Tm nghi m

4 2 2 , 0 1 0 t ng qut

0 1 1 1 2 0 , 0 1 0 3 0 0 c a h phng trnh

0 0 . 3 sai phn khng thu n nh t

un+1 = Aun + bn , v i ma tr n A v vc t bn cho b i 2k 3 1 . a. 3 5 , 4k 2 2 5k+1 . b. 3 4 , 1/2 2 1 1 0 3 1 1 k . c. , 9 3 4 0 1 1 0 0 k 2 1 2 , . d. 3 2 1 2k

6.6

M t s l p phng trnh sai phn phi tuy n c ch m

Xt phng trnh sai phn phi tuy n d ng xn+1 = F (xn , xn1 , , xnk ) , n = 0, 1, , (4.5)

trong xk , xk+1 , , x0 l cc s cho tr c, F C I k+1 , I , v i I l kho ng s th c v k l s nguyn dng cho tr c. nh ngha 6.6. M t dy s th c (xn ) n=k c g i l nghi m c a phng trnh (4.5) n u n th a mn (4.5) v i m i n = 0, 1, . Cho tr c (k + 1) s th c ai , i = k, k + 1, , 0 th phng trnh (4.5) c nghi m duy nh t (xn ) n=k th a mn i u ki n ban u xi = ai , i = k, k + 1, , 0. nh ngha 6.7. M t nghi m (xn ) n=k c a phng trnh (4.5) c g i l nghi m dng n u xn > 0, n.

292

Chng 6. Kh o st dy s v phng trnh sai phn

nh ngha 6.8. M t nghi m dng (xn )n=k c a phng trnh (4.5) c g i l gi i n i ng t n u 0 < lim inf xn
n n

lim sup xn < .

nh ngha 6.9. M t dy (xn ) c g i l dao ng xung quanh i m 0 hay n gi n l dao ng n u cc s h ng khng ng th i dng ho c khng ng th i m. Ng c l i, ta ni dy (xn ) khng dao ng. M t dy (xn ) c g i l dao ng ng t n u n0 0, n1, n2 n0 sao cho xn1 .xn2 < 0. M t dy (xn ) c g i l dao ng xung quanh x n u dy (xn x) dao ng. M t dy (xn ) c g i l dao ng ng t xung quanh x n u dy (xn x) dao ng ng t. nh ngha 6.10. Cho I l kho ng s th c, x I c g i l i m cn b ng c a phng trnh (4.5) n u x = F (, x, , x ). x i) i m cn b ng x c a phng trnh (4.5) c g i l n nh a phng n u > 0, > 0 sao cho m i nghi m v i i u ki n ban u xk , xk+1 , , x0 ( ; x + ) th xn ( ; x + ), n N. x x ii) i m cn b ng x c a phng trnh (4.5) c g i l n nh ti m c n a phng n u n n nh a phng v n u > 0 sao cho v i xk , xk+1 , , x0 I m xk , xk+1 , , x0 ( ; x + ) th lim xn = x. x
n

iii) i m cn b ng x c a phng trnh (4.5) c g i l ht ton c c n u m i nghi m (xn )n=k c a phng trnh (4.5) u h i t n x khi n . iv) i m cn b ng x c a phng trnh (4.5) c g i l n nh ti m c n ton c c n u n n nh a phng v ht ton c c. v) Phng trnh
k

yn+1 =
i=0

F (, x, , x) yni , x xni

n = 0, 1, ,

6.6. M t s l p phng trnh sai phn phi tuy n c ch m

293

c g i l phng trnh tuy n tnh lin k t v i phng trnh (4.5) xung quanh i m cn b ng x. vi) Phng trnh
k

n+1

=
i=0

F (, x, , , x) ni , x xni

n = 0, 1, ,

c g i l phng trnh c trng lin k t v i phng trnh (4.5). nh ngha 6.11. Gi s (xn ) n=k l m t nghi m dng c a phng trnh (4.5). Khi ta g i: i) M t n a chu trnh dng c a m t nghi m (xn )n=k c a phng trnh (4.5) xung quanh i m cn b ng x l m t dy (xl , xl+1 , , xm ) sao cho t t c xi x, i = l, l + 1, , m v sao cho ho c l = k hay v ho c l m = hay m < v xm+1 < x. ii) M t n a chu trnh m c a m t nghi m (xn ) n=k c a phng trnh (4.5) xung quanh i m cn b ng x l m t dy (xl , xl+1 , , xm ) sao cho t t c xi < x, i = l, l + 1, , m v i = k hay v ho c l m = hay m < v xm+1 x. > k v x 1 x, > k v x 1 < x,

nh ngha 6.12. N a chu trnh u tin c a m t nghi m l n a chu trnh b t u v i s h ng xk v n dng n u xk x, n m n u xk < x.

294

Chng 6. Kh o st dy s v phng trnh sai phn

M nh

6.1. Cho F C I k+1 , I (I l kho ng s th c ty ), k N . r ng J I, lim sup xn = S I. Khi t n t i hai dy (Jn ) n= v


n

G i (xn ) n=k l m t nghi m b ch n c a phng trnh (4.5). Gi s lim inf xn = n (Sn ) n= th

a mn phng trnh sai phn (4.5) v i m i n Z m J0 = J ,

S0 = S; Jn , Sn [J ; S] , n Z v v i m i N Z th JN , SN l hai i m gi i h n c a dy (xn )n=k . Hn n a, v i m i m


n

k, t n t i hai dy con (xrn ) v (xln ) c a nghi m


n

(xn )n=k th a lim xrn +N = JN , lim xln+N = SN , v i m i N m. V d 6.44. Xt phng trnh sai phn xn+1 = xn , n = 0, 1, , (4.6)

v i i u ki n ban u x0 . Khi i m cn b ng x = 0 l n nh a phng, nhng khng n nh ti m c n a phng. Th t v y, nghi m c a phng trnh (4.6) c d ng xn = c, c R. Do > 0, = sao cho b t k nghi m xn = c th a |x0 0| = |c| < , ta c |xn 0| = |c| < = . i m cn b ng x = 0 khng n nh ti m c n a phng v c s > 0 (khng ph thu c ) v nghi m xn = c sao cho |x0 | < nhng xn khng h i t v 0. V d 6.45. Xt phng trnh sai phn 1 xn+1 = xn , 3 n = 0, 1, , (4.7)

v i i u ki n ban u x0. Khi i m cn b ng x = 0 l n nh ti m c n ton c c. Th t v y, nghi m c a phng trnh (4.7) c d ng xn =


n 1 n 3

x0. Do

lim xn = lim

1 n 3

x0 = 0.

6.6. M t s l p phng trnh sai phn phi tuy n c ch m

295

V i m cn b ng x = 0 l n nh a phng. Th t v y, > 0, = () = > 0, sao cho v i nghi m xn = th |xn 0| =


1 n 3 1 n 3

x0 th a mn |x0 0| = |x0| <

x0

|x0 | < = .

M nh sau th ng c s d ng kh o st tnh n nh a phng c a nghi m phng trnh (4.5) trong tr ng h p k = 1. M nh 6.2. Xt phng trnh xn+1 = F (xn , xn1 ) , n = 0, 1, . (4.8)

Gi s x l i m cn b ng c a phng trnh (4.8) v phng trnh c trng lin k t v i phng trnh (4.8) c d ng 2 r s = 0. Khi i) N u hai nghi m c a phng trnh (4.9) n m trong hnh trn n v || < 1 th i m cn b ng x l n nh ti m c n a phng. ii) N u c t nh t m t nghi m c a phng trnh (4.9) c gi tr tuy t i l n hn 1 th x khng n nh. iii) i u ki n c n v hai nghi m c a phng trnh (4.9) n m trong hnh trn n v || < 1 l |r| < 1 s < 2. iv) i u ki n c n v m t nghi m c a phng trnh (4.9) c gi tr tuy t i b hn 1 v nghi m cn l i c gi tr tuy t i l n hn 1 l r2 > 4s v |r| > |1 s|. v) i u ki n hai nghi m c a phng trnh (4.9) n m trong hnh trn n v l || < 1 l |r| + |s| < 1. V d 6.46. Xt phng trnh sai phn xn+1 = + xn1 , xn n = 0, 1, , (4.10) (4.9)

296

Chng 6. Kh o st dy s v phng trnh sai phn

v i i u ki n ban u x1 , x0 . Khi , i m cn b ng x = + 1 c a phng trnh l n nh ti m c n a phng n u > 1, khng n nh ti m c n n u 0 < < 1. Th t v y, ta c phng trnh c trng lin k t v i phng trnh (4.10) xung quanh i m cn b ng x = + 1 l 2 + Ta c |r| + |s| = 2 . +1 1 1 = 0. +1 +1 (4.11)

N u > 1 th |r| + |s| < 1. Do , hai nghi m c a phng trnh (4.11) n m trong hnh trn n v || < 1. Theo M nh 4.5 th i m cn b ng x = + 1 l n nh ti m c n a phng. N u < 1 th |r| + |s| > 1. Do , phng trnh (4.11) c t nh t m t nghi m c gi tr tuy t i l n hn 1. Theo M nh 4.5 th i m cn b ng x = + 1 l khng n nh ti m c n. V d 6.47. Xt phng trnh sai phn xn+1 = xn , n = 0, 1, , xn1 (4.12)

trong i u ki n ban u x1 , x0; , , l cc s th c dng sao cho =


(+)2 4

v > . Khi , i m cn b ng c a phng trnh ny l khng n

nh. Th t v y, xt phng trnh xc nh i m cn b ng x= hay ()2 ( + ) x + = 0, x x , x

6.6. M t s l p phng trnh sai phn phi tuy n c ch m

297

nn x Do x= + 2

= 0.

+ . 2

Phng trnh c trng lin k t v i phng trnh (4.12) xung quanh i m cn b ng x =


+ 2

l 2 + + 2 = 0.
2

2 t r = , s =

+ ,

khi |r| + |s| =


+ 2

>1+

> 1. Theo

M nh 4.5 th i m cn b ng x =

khng n nh.

V m t l p phng trnh sai phn h u t b c 1 Trong m c ny ta nghin c u s h i t c a nghi m phng trnh sai phn h ut b cm t xn+1 = + xn , A + Bxn (4.13)

trong n N0 v x0 l s th c khng m cho tr c. Tr c h t ta xt v d sau: Xt phng trnh xn+1 = 1 + Ta s ch ng t limn xn = 1 , 1 + xn x0 = 1.

2. T ta s pht bi u nh l t ng qut

v s h i t c a nh ng dy {xn } th a mn phng trnh xn+1 = f (xn ) v i x0 > 0 cho tr c, f l hm dng, lin t c, ngh ch bi n trn [0, ). Xt hm f (x) = 1 + 1 , 1+x

298

Chng 6. Kh o st dy s v phng trnh sai phn

ta c 1 < f (x) T

2, f (x) l hm dng, lin t c, ngh ch bi n trn [0, ). 1 = f (xn ) 1 + xn

xn+1 = 1 + ta suy ra 1 < xn+1 Ta xt hai tr ng h p:

2, v i m i n = 0, 1, 2, .

Tr ng h p 1: x0 < x2 suy ra f (x0 ) > f (x2 ) hay x1 > x3 suy ra f (x1) < f (x3 ) hay x2 < x4 suy ra f (x2 ) > f (x4 ) hay x3 > x5 , Quy n p : x2n < x2n+2 . Th t v y, gi s x2k < x2k+2 suy ra f (x2k ) > f (x2k+2 ) hay x2k+1 > x2k+3 suy ra f (x2k+1 ) < f (x2k+3 ) hay x2k+2 < x2k+4 suy ra {x2n} l dy tng. Ch ng minh tng t ta c {x2n+1 } l dy gi m. Ta c: - Dy {x2n } tng v b ch n trn nn {x2n } h i t . Gi s x2n khi n . - Dy {x2n+1 } gi m v b ch n d i nn {x2n+1 } h i t . Gi s x2n+1 khi n . Do hm f lin t c nn ta c h = f (), = f (), hay 1 , 1+ 1 = 1+ . 1+ = 1+ T ta nh n c == 2.

6.6. M t s l p phng trnh sai phn phi tuy n c ch m

299

Tr ng h p 2: x0 x2 . L y x0 = 2 suy ra 4 10 24 17 x1 = , x2 = , x3 = , x4 = 1 + . 3 7 17 41 Ta c: - Dy {x2n} gi m v b ch n d i nn {x2n } h i t . Gi s x2n khi n . - Dy {x2n+1} tng v b ch n trn nn {x2n+1 } h i t . Gi s x2n+1 khi n . Tng t tr ng h p 1 ta cng thu c = = lim xn = 2. 2. V y,

nh l sau cho ta m t i u ki n m i nghi m c a m t l p phng trnh sai phn phi tuy n b c m t h i t . nh l 6.16. Gi s f : [0, +) (0, +) l hm s lin t c, ngh ch bi n trn [0, ) v h phng trnh x = f (y), y = f (x) c nghi m duy nh t x = y = . Khi m i nghi m c a phng trnh sai phn phi tuy n b c m t xn+1 = f (xn ), h it t i . Ch ng minh: Ta xt hai tr ng h p: Tr ng h p 1: x0 < x2 . Ta c f (x0 ) > f (x2 ) hay x1 > x3 suy ra f (x1 ) < f (x3 ) hay x2 < x4 suy ra f (x2 ) > f (x4 ) hay x3 > x5, . n N0, (x0 > 0 cho tr c)

300

Chng 6. Kh o st dy s v phng trnh sai phn

Ta s ch ng minh quy n p r ng x2n < x2n+2 ,

n N0 . Th t v y, gi s

x2k < x2k+2 suy ra f (x2k ) > f (x2k+2 ) hay x2k+1 > x2k+3 suy ra f (x2k+1 ) < f (x2k+3 ) hay x2k+2 < x2k+4 suy ra {x2n} l dy tng. Ch ng minh tng t ta c {x2n+1 } l dy gi m. Dy {x2n} tng v b ch n trn, dy {x2n+1} gi m v b ch n d i nn chng h i t . Gi s x2n u, x2n+1 v khi n . Do f l hm lin t c nn ta c f (u) = v. L lu n tng t ta cng c f (v) = u. V y ta nh n c h phng trnh v = f (u), u = f (v). Theo gi thi t h ny c nghi m duy nh t u = v = . Tr ng h p 2: x0 x2 . Trong tr ng h p ny, dy {x2n } gi m v b ch n d i, dy {x2n+1} tng v b ch n trn. Gi s x2n u, x2n+1 v khi n . Tng t tr ng h p 1 ta thu c u = v = . V y ta c
n

lim xn = .

nh l c ch ng minh. By gi ta p d ng nh l 6.16 kh o st s h i t c a nghi m phng trnh (4.13). Xt hm s f (x) =


x+ , Bx+A

0. Ta c f l hm lin t c v

ngh ch bi n trn [0, +) n u A < B. M t khc, d ki m tra c h phng trnh x = f (y), y = f (x) c nghi m dng duy nh t x=y= A+ (A )2 + 4B . 2B

6.6. M t s l p phng trnh sai phn phi tuy n c ch m

301

Do , theo nh l 6.16, m i nghi m c a (4.13) h i t t i s dng = V d A+ (A )2 + 4B . 2B

6.48. Tnh cn b c hai dng c a m t s dng a.

Xt phng trnh sai phn h u t xn+1 = xn + a , xn + 1 n N0 , (x0 > 0 cho tr c). a.

D th y, m i nghi m c a phng trnh ny h i t t i

Nh n xt 6.2. Ta c th s d ng nh l 6.16 kh o st s h i t c a nghi m nhi u phng trnh sai phi tuy n khc phng trnh (4.13). Ch ng h n, ta c v d sau: V d 6.49. Kh o st s h i t c a nghi m phng trnh xn+1 = Xt hm s f (x) = x 3+ , x2 1 x [ 3, +). 3+ xn , x2 1 n n N0, (x0 3 cho tr c).

R rng f lin t c trn [ 3, +). Ta c 1 < 0, (x2 1)3 Do f ngh ch bi n trn [ 3, +). f (x) = Xt h phng trnh u , 3+ u2 1 v , 3+ u = v2 1 v = x [ 3, +).

u, v [ 3, +).

Ta s ch ng minh h ny c nghi m duy nh t u = v. Th t v y, 3 u2 1 + u, v u2 1 =

302

Chng 6. Kh o st dy s v phng trnh sai phn

u v2 1 = 3 v 2 1 + v,

u, v [ 3, +).

L y phng trnh th hai tr v theo v cho phng trnh th nh t c a h ta c u v 2 1 v u2 1 = 3 v 2 1 3 u2 1 + v u. Vi t l i phng trnh ny d i d ng v 2 1 u 3 u2 1 v 3 = v u. Ta ch ng minh u = v. Gi s tri l i, u > v, khi v ph i m. Ta ch ng t 3 minh v tri dng. Th t v y, xt hm s f (t) = t2 1 , t 3. D th y, f l hm n i u tng. Do ta c f (u) > f (v) v 3 u 3 > u2 1 v2 1 v 2 1 u 3 > u2 1 v 3 v 2 1 u 3 u2 1 v 3 > 0. i u ny ch ng t v tri dng. Ta c i u v l. Do u = v. Xt phng trnh = V [ 3, +) nn 0 < 3+
1 2

[ 3, +).
1

() = sin .

< 1. Do t n t i (0, /2) sao cho

Khi phng trnh c d ng 1 1 = 3+ (sin cos ) + 3 sin cos = 0. sin cos Gi i phng trnh () v i i u ki n [ 3, +) ta c nghi m duy nh t 1 3( 5 + 1) = = . sin 2 =

6.6. M t s l p phng trnh sai phn phi tuy n c ch m

303

V y, m i nghi m c a phng trnh sai phn trn v i i u ki n ban u thu c [ 3, +) h i t n s dng 3( 5 + 1) . = 2 Nh n xt 6.3. D th y r ng, n u (yn , zn ) l nghi m c a h phng trnh sai phn yn+1 = yn + zn , zn+1 = Byn + Azn, th xn =
yn zn

y0 = x0 , z0 = 1

l nghi m c a phng trnh (4.13). Do , vi c kh o st s h i t


yn zn

c a nghi m phng trnh (4.13) c th chuy n v vi c tm nghi m (yn , zn ) c a h phng trnh sai phn trn, sau tnh gi i h n khi n ti n ra v cng.

Tuy nhin, cch ny khng g n b ng cch s d ng nh l 6.16. V m t s phng trnh sai phn h u t b c hai Trong m c ny ta kh o st s h i t c a nghi m phng trnh sai phn sau xn+1 = xn + , A + xn + Cxn1 (4.14)

trong n N v x0 , x1 l 2 s th c khng m cho tr c. Ta gi thi t cc tham s trong phng trnh (4.14) l cc s th c dng. Trong m c ny ta lun gi thi t f : [0, +) [0, +) [0, +) l hm lin t c. Cc b sau r t c n thi t kh o st s h i t c a nghi m phng trnh (4.14). B 6.1. N u m i nghi m c a phng trnh xn+1 = f (xn , xn1 ), n N, (x0 , x1 > 0 cho tr c) (4.15)

h i t n m t s dng , th h phng trnh x = f (y, x),

304

Chng 6. Kh o st dy s v phng trnh sai phn

y = f (x, y) c nghi m dng duy nh t x = y = . Ch ng minh: G i (x, y) l m t nghi m dng c a h phng trnh trn. Xt phng trnh (4.15) v i x1 = x v x0 = y. Th th x2 = f (x1 , x0) = f (x, y) = y v x3 = f (x2, x1 ) = f (y, x) = x. Ta ch ng minh b ng quy n p r ng x2k = y v x2k+1 = x v i m i k. Gi s x2k = y, x2k+1 = x v i m t s t nhin k no , ta s ch ng minh x2(k+1) = y, x2(k+1)+1 = x. Th t v y, ta c x2(k+1) = f (x2k+1 , x2k ) = f (x, y) = y, x2(k+1)+1 = f (x2k+2 , x2k+1) = f (y, x) = x. Nh v y x2k = y, x2k+1 = x v i k N0. Theo gi thi t {xn }n h i t n s dng , nn ta c cc dy con {x2k } , {x2k+1} h i t n , t c l k=0 k=0 x = y = . B c ch ng minh. B sau s ch ra r ng i u ki n c a b 6.1 l n u hm f b ch n v n i u gi m theo bi n x, n i u tng theo bi n y. Tr c h t ta xt v d sau: V d 6.50. Xt phng trnh sai phn xn+1 = xn + 1 , xn + xn1 + 1 x0 , x1 cho tr c .

Ch n x0, x1 (0, 1). Xt hm s f (x, y) = x+1 . x+y+1

D th y f ng bi n theo x, ngh ch bi n theo y. M t khc, infx,y 0 f (x, y) = 0 := 0 , v xn (0, 0), n 2. supx,y 0 f (x, y) = 1 := 0

6.6. M t s l p phng trnh sai phn phi tuy n c ch m

305

t 1 = infx,y(0 ,0 )f (x, y), Ta c 1 1 = f (0 , 0) = , 2 Tng t n+1 = infx,y(n ,n )f (x, y) = f (n , n ), n+1 = supx,y(n ,n )f (x, y) = f (n , n ), 1 = f (0 , 0) = 1. 1 = supx,y(0 ,0 )f (x, y).

v i n = 0, 1, 2, . Ta c {n }n l dy n i u tng v b ch n trn b i 1, {n }n l dy n i u gi m v b ch n d i b i 0, do chng h i t . Gi s


n

lim n = , lim n = .

Ta thu c h sau = f (, ) = f (, ). Suy ra 1 == . 2 M t khc, ta ch ng minh b ng quy n p c xn+2k (k , k ), Do 1 lim xn = . 2 B 6.2. Gi s hm f n i u gi m theo bi n x v i m i y > 0 v n i u tng theo bi n y v i m i x > 0. Gi thi t thm r ng, M := supx,y v h phng trnh u = f (v, u),
0

k = 0, 1, 2, .

f (x, y) <

306

Chng 6. Kh o st dy s v phng trnh sai phn

v = f (u, v) c nghi m duy nh t u = v = . Khi m i nghi m c a (4.15) h i t n . Ch ng minh: Theo gi thi t ta c xn+1 = f (xn , xn1 ) < M v i m i n N2 . Khng m t tnh t ng qut ta gi s xn < M v i m i n N0. Ta xt h phng trnh sai phn sau un+1 = f (vn , un ), vn+1 v i n N0. = f (un , vn )

y ta t u0 = 0, v0 = M. R rng, u0 < xn < v0 v i m i n N0 .

Do hm f n i u gi m theo bi n x v n i u tng theo bi n y, nn xn+2 = f (xn+1 , xn ) < f(u0 , v0) = v1 v tng t , xn+2 = f (xn+1 , xn ) > f (v0, u0) = u1 v i m i n N0 .

B ng phng php ch ng minh quy n p, ta c th th y r ng uk < xn+2k < vk v i m i k, n N0 .

M t khc u0 < u1 v v0 > v1. Cng do hm f n i u gi m theo bi n x v n i u tng theo bi n y nn ta c u2 = f (v1 , u1) > f (v0, u0 ) = u1 v tng t v2 < v1. B ng ch ng minh quy n p ta c dy {uk }k n i u tng v dy {vk }k n i u gi m. G i u, v l n l t l gi i h n c a cc dy {uk }k v {vk }k . Ta c u v v th a mn h phng trnh u = f (v, u),

6.6. M t s l p phng trnh sai phn phi tuy n c ch m

307

v = f (u, v). Theo gi thi t u = v = . Nh v y cc dy {uk }k v {vk }k l h i t v gi i h n c a hai dy ny l . Theo trn ta c uk < xn+2k < vk , k, n N0 nn ta c dy {xn } h i t v limn xn = . B c ch ng minh. n=0 Ch 6.5. N u hm f b ch n th v i m i nghi m {xn }n c a (4.15), t n t i hai dy gi i h n y {un }nZ v {vn }nZ tho mn (4.15) v i m i n Z sao cho u0 = lim sup xn ,
n

v0 = lim inf xn ,
n

un , vn [v0, u0] v i m i n Z.

Hai dy gi i h n y ny c ch n t t p gi i h n c a nghi m {xn }n . B 6.3. Gi s hm f n i u gi m theo bi n y v i m i x > 0 v M := sup f (x, y) < .


x,y 0

Th th v i m i nghi m {xn }n c a (4.15) ta c


0 x M

max f (x, 0)

lim sup xn
n

lim inf xn
n

0 x M

min f (x, M).

Ch ng minh: Ch n hai dy gi i h n y {un }nZ v {vn }nZ t t p gi i h n c a {xn }n sao cho u0 = lim sup xn ,
n

v0 = lim inf xn ,
n

un , vn [v0, u0 ] v i m i n Z.

Ta c u0 Tng t , ta nh n c v0 f (v0, v0 ). u1 = f (u0 , u1 ) f (u0 , u0).

T y suy ra nghi m dng duy nh t c a phng trnh x = f (x, x) ph i n m trong [v0, u0]. M t khc, u0 = f (u1 , u2) f (u1 , 0)
0 x M

max f (x, 0).

308

Chng 6. Kh o st dy s v phng trnh sai phn

Tng t , ta c v0 B c ch ng minh. By gi ta kh o st tnh ch t c a nghi m phng trnh (4.14). ti n theo di ta nh c l i k t q a sau c a .V. Giang. nh l 6.17. Gi s /A. N u m t trong cc i u ki n sau tho mn
0 x M

min f (x, M).

th d on c a G. Ladas l ng. (i) A; 1; 4/(C 1).

(ii) > A v C

(iii) > A, C > 1 v ( A)2

Nh n xt 6.4. K t h p nh l 6.17 v b 6.3 ta th y r ng, n u cc i u ki n (i)-(iii) c a nh l 6.17 khng x y ra th v i m i nghi m {xn }n c a (4.14) ta c a trong a = b = 1 A 2 1 A+ 2 ( A)2 ( A)2 4 , C1 4 . C 1 lim inf xn
n

lim sup xn
n

b,

Ti p theo ta nghin c u d on c a Ladas trong tr ng h p = A. ng ti c trong tr ng h p ny ta v n ph i h n ch trn cc tham s , v C. nh l 6.18. Gi s = A v C < 1. N u < 4 2/(C + 1) th d on c a G. Ladas l ng.

6.6. M t s l p phng trnh sai phn phi tuy n c ch m

309

Ch ng minh: Tr c h t r ng n u

2, ta c th p d ng tr ng h p

(i) c a nh l 6.17. V v y, khng m t tnh t ng qut ta gi s r ng > 2. M t khc ta c = v |xn+1 | = |( )(xn ) C (xn1 )| . xn + Cxn1 + (4.17) C +1 (4.16)

t n = |xn |. Ta nh n c n+1 | |n + C n1 .

Xt phng trnh sai phn tuy n tnh yn+1 = D th y n | |yn + C yn1 v i n N, (y0 = 0 , y1 = 1 ).

yn v i m i n N0 v yn c d ng yn = 1 n + 2 n , 1 2

trong 1,2 l cc nghi m c a phng trnh 2 =| | + C . (4.18)

Ta ch ng minh r ng cc nghi m c a phng trnh (4.14) c gi tr tuy t i nh hn 1 (v h qu l yn 0 khi n ). i u ny tng ng v i vi c ch ng minh > | | + C . (4.19)

Cu i cng ta hy xt hai tr ng h p c th x y ra: N u < 2(C + 1), ta c > v h qu l | | + C = + (C 1) < (v C < 1). Tr ng v | | + C = (C + 1) =

h p th hai l 2(C + 1). Ta c

(C + 1) < 2 = (v < 4 2 /(C + 1)). nh l c ch ng minh.

310

Chng 6. Kh o st dy s v phng trnh sai phn

nh l 6.19. Gi s = A v 1 c a Ladas l ng.

2. N u < 9 2 /(C + 1) th d on

Ch ng minh: Tr c h t ch r ng n u

2, ta c th p d ng tr ng h p

(i) nh l 6.17. V v y khng m t tnh t ng qut, ta gi s > 2. Xt hm f (x, y) = Ta s ch ng t supx,y


0

x + . x + Cy +

f (x, y) = . Th t v y, ta c

f (x, y) =

x + Cy 2x x + = x + Cy + x + Cy + Cy + x( ) . = x + Cy +

v i m i x, y 0. L y > 0 nh ty , ta ch ng minh t n t i (x, y) [0, ) [0, ) sao cho f (x, y) >


, t c l x + Cy 2x < x + Cy +

hay (x + Cy)( ) + + 2x > 0. x + Cy + N u th b t ng th c trn hi n nhin ng v i m i (x, y) [0, ) < th ta ch n x =


1 (x 2

[0, ). Cn n u

+ Cy)( ) v y ty thu c

[0, ), khi b t ng th c trn th a mn. V y supx,y0 f (x, y) = . M t khc, ta c


f (x, y) y

C (x+Cy+)2

< 0,

x, y

0 nn hm f (x, y)

n i u gi m theo bi n y trn [0, ), do f (x, y) Hn n a, ta c


f (x, y) x

f (x, /),
Cy+( 2 ) (x+Cy+)2

y [0, /].

v do C 1 nn

f (x, /) > 0. x

6.6. M t s l p phng trnh sai phn phi tuy n c ch m

311

Suy ra f (x, y) T ta thu c inf


x,y(0,/)

f (x, /)

f (0, /),

x, y [0, /].

f (x, y) = f (0, /) =

> = . + C (C + 1) C+1

r ng xn+1 = f (xn , xn1 ) nn xn > C +1 v i n N4 .

M t khc, t n = |xn |, t (4.17) ta c n+1 | |n + C n1 . 2

Xt phng trnh sai phn tuy n tnh yn+1 = D th y n | |yn + C yn1 2 v i n N5 , (y4 = 4 , y5 = 5 ).

yn v i m i n N4 v yn c d ng yn = 1 n + 2 n , 1 2

trong 1,2 l 2 nghi m phn bi t c a phng trnh 22 = | | + C . (4.20)

Ta ch ng minh r ng cc nghi m c a phng trnh (4.20) c gi tr tuy t i nh hn 1 (v h qu l yn 0 khi n ). i u ny tng ng v i vi c ch ng minh 2 > | | + C . (4.21) =

Xt hai tr ng h p sau c th x y ra: N u < (C + 1) 2 , ta c > /(C + 1) v h qu l | | + a = + (C 1) a 2). Tr ng h p th +

< 2 (v v

hai l (C + 1) 2. Ta nh n c

312

Chng 6. Kh o st dy s v phng trnh sai phn

| |+C = (C+1) = nh l c ch ng minh

(C + 1) < 3 = 2 (v < 9 2/(C+1)).

nh l sau cho m t i u ki n m i nghi m c a (4.14) h i t . nh l 6.20. N u < A th m i nghi m c a (4.14) h i t t i . Ch ng minh: Xt hm s H(x, y, u, v) = y + . v + Bu + A

r ng H(x, y, x, y) = f (x, y). Hn n a, H(x, y, u, v) l hm n i u tng theo cc bi n x, y v n i u gi m theo cc bi n u, v. Xt h phng trnh sai phn sau un+1 = H(un , un1 , n , n1 ), n+1 = H(n , n1 , un , un1 ) Trong , 0 = 1 = 0, u0 = u1 = M + R rng, xn+1 = f (xn , xn1 ) M = supx,y . A
0

v i n N.

f (x, y) v i m i n N. V v y

khng m t tnh t ng qut ta gi s x0 , x1 u0 0 0 1 u1 1 x0 x1

M. Ta c u2 , 2 , u0 , u1 .

B ng quy n p, ta c th ch ng minh r ng {n }n l dy n i u khng gi m, {un }n l dy n i u khng tng v n xn un v i n N. G i l gi i

6.6. M t s l p phng trnh sai phn phi tuy n c ch m

313

h n c a dy {n }n v u l gi i h n c a {un }n . Th th u = =
u+ , (B+1)+A + . (B+1)u+A

Theo gi thi t < A, do t h phng trnh ny ta thu c u = = . nh l c ch ng minh. V l p phng trnh sai phn h u t b c k trn b c (k 1) Xt phng trnh sai phn h u t b c k trn b c (k 1) xn+1 = + xnk , f (xn , xn1 , , xnk+1 ) n = 0, 1, , (4.22)

trong 0, k N, i u ki n ban u xk , xk+1 , , x0 l cc s th c dng cho tr c. Gi s f l hm th a gi thi t (H) sau y: f : [0; )k (0; ) l hm lin t c. V i u ki n ban u l cc s th c dng v do gi thi t (H) nn nghi m c a phng trnh (4.22) l dng. t g(u) := f (u, u, , u) , u 0. Do gi thi t (H) nn g l hm tng v lun nh n gi tr trong kho ng (0; ). Karakostas v Stevic nghin c u tnh b ch n, tnh ht ton c c, tnh dao ng v tnh tu n hon c a nghi m c a phng trnh (4.22) v i cc i u ki n trn. nh l sau cho php ta xc nh c di t i a c a m i n a chu trnh c a nghi m. nh l 6.21. Gi s hm H i t t p [0; )
k+1

vo t p [0; ) c cc tnh

ch t: i0 {1, 2, , k} sao cho H (z1, z2, , zk , y) l hm khng tng theo

314

Chng 6. Kh o st dy s v phng trnh sai phn

m i bi n zi , i {1, 2, , k} \ {i0 }, gi m theo zi0 v tng theo y. G i x l i m cn b ng c a phng trnh sai phn xn+1 = H (xn , , xnk+1 , xnk ) , n = 0, 1, . (4.23)

Khi , tr n a chu trnh u tin, m i nghi m dao ng c a phng trnh sai phn (4.23) v i i u ki n ban u dng u c cc n a chu trnh c di t i a l k. Ch ng minh: G i (xn ) n=k l nghi m dao ng c a phng trnh (4.23) m nghi m ny c t nh t hai n a chu trnh. Gi s t n t i m t n a chu trnh c di l n hn k, khi t n t i s t nhin N sao cho

xN k < x < xN k+1 , , xN , xN +1, ho c xN k x > xN k+1 , , xN , xN +1. Tr ng h p xN k < x < xN k+1 , , xN , xN +1. Ta c xN +1 = H (xN , , xN k+1 , xN k ) < H (, x, , x) = x, x suy ra xN +1 < x (v l).

Tr ng h p xN k x > xN k+1 , , xN , xN +1 . Ta c xN +1 = H (xN , , xN k+1 , xN k ) > H (, x, , x) = x, x suy ra xN +1 > x (v l).

Nh v y, c hai tr ng h p ta u a ra i u v l. Do m i nghi m

6.6. M t s l p phng trnh sai phn phi tuy n c ch m

315

dao ng v i i u ki n ban u dng u c cc n a chu trnh v i di khng v t qu k (tr n a chu trnh u tin). H qu 6.3. Gi s hm f th a mn gi thi t (H) v x l m t i m cn

b ng dng c a phng trnh (4.22). Khi , tr n a chu trnh u tin, m i nghi m dao ng v i gi tr ban u dng c cc n a chu trnh v i di khng v t qu k. Ch ng minh: t H (z1, z2, , zk , y) = + y . f (z1 , z2, , zk )

Do f : [0; )k (0; ) l hm lin t c v lun nh n gi tr dng nn H (z1 , z2, , zk , y) l hm lin t c. M t khc, f l hm khng gi m v i m i bi n v tng v i t nh t m t bi n nn v i cch t nh trn th hm H khng tng v i m i bi n zi , i {1, 2, , k} \ {i0}; gi m theo bi n zi0 v tng theo y. Khi hm H th a mn gi thi t c a nh l 6.21. V v y, tr n a chu trnh u tin, m i nghi m dao ng c a phng trnh sai phn xn+1 = H (xn , , xnk+1 , xnk ) , n = 0, 1, , v i i u ki n ban u dng u c cc n a chu trnh v i di khng v t qu k. Hay, tr n a chu trnh u tin, m i nghi m dao ng c a phng trnh xn+1 = + xnk , n = 0, 1, , f (xn , xn1 , , xnk+1 )

v i i u ki n ban u dng c cc n a chu trnh v i di khng v t qu k. Ti p theo ta nghin c u tnh b ch n, h i t v tu n hon c a nghi m phng trnh (4.22). Tr ng h p g () > 1. Ta c g l hm tng trn [0; ), nn v i m i u > th g (u) > g () > 1.

316

Chng 6. Kh o st dy s v phng trnh sai phn

N u = 0 th 0 l i m cn b ng duy nh t c a phng trnh xn+1 = xnk . f (xn , , xnk+1 ) (4.24)

Th t v y, gi s x l i m cn b ng c a phng trnh (4.24), t c l x= hay x 1 Do x = 0. Ta c xn+1 = xnk f (xn , xn1 , , xnk+1 ) xnk xnk = , < f (0, 0, , 0) g (0) 1 g () x = 0. x , f (, x, , x) x

suy ra i m cn b ng x = 0 l ht ton c c. Gi s > 0. Hai nh l sau y cho ta d u hi u nh n bi t phng trnh (4.22) c duy nh t i m cn b ng dng v m i nghi m dng c a phng trnh ny b ch n. nh l 6.22. Gi s g () > 1 v hm f th a mn gi thi t (H). Khi phng trnh sai phn (4.22) c duy nh t i m cn b ng dng x. Ch ng minh: Gi s x l i m cn b ng c a phng trnh (4.22). Ta c x=+ Xt hm s F : [, ) R x x x . g(x) x . g () x

6.6. M t s l p phng trnh sai phn phi tuy n c ch m

317

R rng F lin t c trn [0; ), th a F (0) = < 0 v lim F (x) = +.


x+

Do t n t i x (0; ) sao cho F () = 0, hay x x=+ x . g () x

Ta ch ng minh x l duy nh t. Th t v y, x, y [0; ) : x > y, ta c F (x) F (y) = x x g (x) y y g (y)

= (x y) = =

xg (y) yg (x) g (x) g (y)

(x y) g (x) g (y) (x y) g (x) + x [g (x) g (y)] g (x) g (y) (x y) g (x) [g (y) 1] + y [g (x) g (y)] . g (x) g (y)

Do g (x) > g (y) > g () > 1 nn F (x) F (y) > 0, suy ra F l hm tng trn [0; ). V y x l i m cn b ng dng duy nh t c a phng trnh (4.22). nh l 6.23. Gi s g () > 1 v hm f th a mn gi thi t (H). Khi m i nghi m v i i u ki n ban u xk , xk+1 , , x0 l cc s th c dng s b ch n b i s M0 := max {xk , xk+1 , , x0} + g () . g () 1

Ch ng minh: Th t v y, gi s (xn ) n=k l m t nghi m tu c a phng trnh (4.22). V xn > , n 1 nn xn+1 = + xnk xnk <+ , f (xn , , xnk+1 ) g () n = 0, 1, .

V i b t k m N v r {0, 1, , k}, ta c x(k+1)m+r+1 < + x(k+1)m+rk . g ()

318

Chng 6. Kh o st dy s v phng trnh sai phn

M t khc x(k+1)m+rk = x(k+1)m+rk1+1 = x(k+1)(m1)+r+1 <+ M x(k+1)(m1)+rk < + nn x(k+1)m+r+1 < + 1 g () + 1 x(k+1)(m2)+rk g () x(k+1)(m3)+rk g () x(k+1)(m2)+rk , g () x(k+1)(m1)+rk . g ()

<+

1 + 2 g () g ()

=+ . . . <+

1 + 2 + 3 x(k+1)(m3)+rk g () g () g ()

1 + 2 ++ m + m+1 xrk g () g () g () g () 1 1 1 + 2 ++ m g () g () g () 1 1 1 + 2 ++ m g () g () g () 1 g m+1 1 g m+1 () ()

< 1+

xrk

= 1+ tq=
1 g()

xrk .

< 1 (do g () > 1). Ta c


m

x(k+1)m+r+1 <
j=1

q j + xrk q m+1 <

+ xrk , 1q

6.6. M t s l p phng trnh sai phn phi tuy n c ch m

319

suy ra 1 . 1 g()

xn Hay

max {xk , , x0} +

xn

max {xk , , x0} +

g () := M0 . g () 1

Cc nh l sau c p n tnh ht ton c c c a nghi m dng phng trnh (4.22). nh l 6.24. Gi s > 0, g () > 1 v hm f th a mn gi thi t (H). Khi , n u hm x
g(x)g() x

gi m trn (; ) th m i nghi m dng c a

phng trnh (4.22) h i t . Ch ng minh: Gi s (xn ) n=k l nghi m dng c a phng trnh (4.22), theo nh l 6.23 th (xn ) n=k b ch n. Do t n t i
n

lim inf xn := l (

l < ) v lim sup xn := L (l


n

L < ) . Hi n

nhin g(L) l m t s th c xc nh. L y gi i h n trn v gi i h n d i hai v c a (4.22), ta c l+ Ta c l+ suy ra (l ) g (L) l. Hay (l ) [g (L) g ()] g () + [1 g ()] l. Tng t L + L g (l) (4.25) l , g (L)
l g(L)

v L

L . g(l)

320

Chng 6. Kh o st dy s v phng trnh sai phn

(L ) [g (l) g ()] g () + [1 g ()] L. N u l = th + g (l) > g () . Gi s r ng l < L. T (4.25) v (4.26), ta c (l ) [g (L) g ()] g () + [1 g ()] l g () + [1 g ()] L (L ) [g (l) g ()] , suy ra (l ) [g (L) g ()] (L ) [g (l) g ()] . Nn g (L) g () g (l) g () . L l M t khc, hm
g(x)g() x , g(L)

(4.26)

suy ra

0 (v l). Do l > , nn

gi m trn (; ), nn v i < l < L ta c g (L) g () g (l) g () > . l L

Khi g (l) g () g (l) g () > . l l (Ta c i u v l). V y L = l, t c l m i nghi m dng c a phng trnh (4.22) h i t . H qu 6.4. Gi s > 0, g () > 1 v hm f th a mn gi thi t (H). Khi , n u hm g lm ch t trn (; ) th m i nghi m dng c a phng trnh (4.22) h i t . Ch ng minh: t G : (; ) R x G (x) =
g(x)g() . x

6.6. M t s l p phng trnh sai phn phi tuy n c ch m

321

Theo nh l 6.24, ta ch c n ch ng minh hm G(x) gi m trn (; ) . T c l ch ng minh G (x) < 0, x (; ). Ta c G (x) = (x ) g (x) [g (x) g ()] . (x )2

V g kh vi trn (; ), nn theo nh l Langrange t n t i c (; x) sao cho g (x) g () = g (c) , x hay g (x) g () = (x ) g (c) . M t khc, g lm ch t trn (; ) nn g (x) < 0, x (; ), t c l g (x) gi m trn (; ), suy ra g (c) > g (x) . Vy G (x) = (x ) g (x) (x ) g (c) (x )2 (x ) [g (x) g (c)] < 0, x (; ) . (x )2

= Ta k hi u M1 :=

g() . g()1

nh l 6.25. Gi s > 0, g () > 1 v hm f th a mn gi thi t (H). N u hm g th a mn b t ng th c |ug (u) vg (v)| g 2 () |u v| , u, v [; M1 ] ,

th m i nghi m dng c a phng trnh (4.22) h i t v i m cn b ng x. Ch ng minh: Gi s (xn )n=k l nghi m dng c a phng trnh (4.22). Theo nh l 6.23 th xn (; M0] , n 1, v i M0 := M1 + max {xk , , x0 }. S

322

Chng 6. Kh o st dy s v phng trnh sai phn

d ng M nh 1.2, ta c hai dy gi i h n y (ym ) v (zm ) th a mn phng trnh sai phn (4.22) v i m i m Z v


n

lim inf xn = z0

zm , ym

y0 = lim sup xn , m Z.
n

(4.27)

D nh n th y t t c s h ng c a hai dy (ym ) , (zm ) u thu c [; M1 ]. Gi s r ng z0 < y0. T cng th c (4.22), ta c y0 v z0 + T (4.28) ta c y0 suy ra y0 1 Nn y0 g () := M1 1 = g () 1 1 g() M0 . 1 g () . + y0 g (z0) + y0 , g () z0 . g (y0 ) (4.29) + y0 , g (z0 ) (4.28)

Do , t t c cc s h ng c a hai dy gi i h n y thu c [; M1 ]. T b t ng th c (4.29) v tnh n i u c a hm g, ta c y0 z0 (Ta c i u v l). Vy y0 = z0 . nh l 6.26. Gi s > 0, g () > 1 v f l hm th a mn gi thi t (H). N u hm g kh vi v th a mn m t trong cc i u ki n sau: g (v) < 1 [g (v) 1]2 , v [; M1 ] , (4.30) z0 y0g (y0) z0g (z0) y0 < g (z0 ) g (y0) g 2 () y0 z0.

6.6. M t s l p phng trnh sai phn phi tuy n c ch m

323

1 g (v) < 2 (v) g

1 1 g ()

v [; M1 ] , v [; M1 ] ,

(4.31) (4.32)

g (v) g () 1 < , g (v) [g (v) 1] g ()

th m i nghi m dng c a phng trnh (4.22) h i t v i m cn b ng x. Ch ng minh: G i (xn ) n=k l nghi m dng c a phng trnh (4.22). Khi , nh nh l 6.25 c hai dy gi i h n y (ym ) , (zm ) th a (4.27), (4.28), (4.29) v th a mn phng trnh sai phn (4.22) v i m i m Z. Hm g th a i u ki n (4.30), ta c n ch ng minh y0 = z0 . Gi s y0 > z0 . nh ngha hm (u, v) := v t (r) := [(1 r) z0 + ry0 , (1 r) y0 + rz0] . Ta c (0) = (z0 , y0) = v (1) = (y0 , z0) = Nh v y (1) M t khc r0 (0; 1) th a mn (1) (0) = (r0 ) , suy ra (r0 ) Nh n xt r ng (r) = g [(1 r) y0 + rz0 ] (1 r) z0 + ry0 , g [(1 r) y0 + rz0] 1 0. 0 (0) . g (z0 ) y0 g (z0) 1 0. g (y0 ) z0 0, g (y0 ) 1 u g (v) , g (v) 1

324

Chng 6. Kh o st dy s v phng trnh sai phn

suy ra (r) = Do (r0) 0 nn 0, (v0 = (1 r) y0 + rz0 [; M1 ]) , 1 (y0 z0 ) g [(1 r) y0 + rz0 ] (y0 z0) . (g [(1 r) y0 + rz0 ] 1)2

(y0 z0 ) g (v0) 1 (y0 z0) [g (v0) 1]2 suy ra

1 g (v0) [g (v0 ) 1]2 Hay g (v0 ) Vy y0 = z0 = x.


1

0.

[g (v0) 1]2 (tri gi thi t).

Hm g th a i u ki n (4.31), ta c n ch ng minh y0 = z0. Gi s y0 > z0. nh ngha hm (u, v) := 1 v t (r) := [(1 r) z0 + ry0 , (1 r) y0 + rz0] . Ta c (0) = (z0, y0) = 1 v (1) = (y0 , z0) = 1 Nh v y (1) M t khc, r0 (0; 1) th a mn (1) (0) = (r0 ) , 0 (0) . 1 y0 g (z0) 0. 1 0, z0 g (y0) 1 , u g (v)

6.6. M t s l p phng trnh sai phn phi tuy n c ch m

325

suy ra (r0 ) Khi u2 0 g (v0 ) , g 2 (v0 ) u0 = (1 r) z0 + ry0 , v0 = (1 r) y0 + rz0 . 0.

Nh n xt r ng u0 [; M1 ] , nn 2 = 2 u0 M1 Do g (v0) 1 [g () 1]2 2 = 2 (v ) 2 () g 0 u0 g (Mu thu n v i gi thi t). Vy y0 = z0 = x. Hm g th a i u ki n (4.32), ta c n ch ng minh y0 = z0 . Gi s y0 > z0 . nh ngha hm hai bi n Z (u, v) := u v t (r) := Z [(1 r) z0 + ry0, (1 r) y0 + rz0 ] . Ta c (0) = Z (z0, y0) = z0 v (1) = Z (y0 , z0) = y0 y0 g (z0) 0. z0 0, g (y0) u , g (v) 1 1 g ()
g() g()1 2 2

[g () 1] . g 2 ()

326

Chng 6. Kh o st dy s v phng trnh sai phn

Nh v y (1) Theo nh l Langrange r0 (0; 1) th a mn (1) (0) = (r0 ) , suy ra (r0 ) Khi 1 g (v0) u0g (v0 ) , u0 = (1 r) z0 + ry0 , v0 = (1 r) y0 + rz0, g 2 (v0) g (vo ) [g (vo ) 1] Hay g (v0) 1 . g (vo ) [g (vo ) 1] u0 Do u0 [; M1 ] , nn 1 1 g () 1 . = u0 M1 g () Khi g (v0) 1 g () 1 . g (vo) [g (vo) 1] u0 g () (Mu thu n v i gi thi t). Vy y0 = z0 = x. nh l 6.27. Gi s > 0, g () > 1, hm f th a mn gi thi t (H) v h l hm cho b i cng th c h (u) :=
1 1 g(u)

(0) .

0.

1 suy ra

u0g (v0 ) .

. N u hm h p h h lm trn [; M1 ]

th m i nghi m dng c a phng trnh (4.22) h i t .

6.6. M t s l p phng trnh sai phn phi tuy n c ch m

327

Ch ng minh: Gi s (xn )n=k l m t nghi m c a phng trnh (4.22). Khi t n t i hai dy gi i h n y (ym ), (zm ) th a mn phng trnh (4.22) v i m i m Z, m t t c s h ng c a hai dy ny u thu c [; M1 ] sao cho lim inf xn = z0 zm , ym y0 lim sup xn

v z0 , y0 th a z0 + Ta c z0 + suy ra z0 1 Hay z0 Khi y0 suy ra y0 1 hay y0 M t khc, ta c z0 + suy ra z0 1


1 g(M2 )

z0 , y0 g (y0)

y0 . g (z0 )

z0 z0 + , g (y0) g (M1 ) 1 g (M1 )

g (M1 ) := N1 . g (M1 ) 1 y0 g (z0) y0 , g (N1)

1 g (N1 )

g (N1) := M2. g (N1 ) 1 z0 z0 + , g (y0) g (M2 ) g (M2 ) := N2 . g (M2 ) 1

328

Chng 6. Kh o st dy s v phng trnh sai phn

C nh v y, ta nh n c hai dy (Mn ) v (Nn ) m cc s h ng c a hai dy ny thu c [, M1 ], c nh ngha nh sau: Nn := g (Mn ) g (Nn1 ) , Mn := . g (Mn ) 1 g (Nn1 ) 1

D dng ch ng minh c dy (Nn ) khng gi m v dy (Mn ) khng tng (do hm s y =


x x1

gi m trn (1; )). V v y gi i h n c a hai dy ny t n t i v

th a i u ki n N0
n n

z0

y0

M0 ,

v i N := lim Nn , M := lim Mn , M = h (N ) , N = h (M). Khi h h (M) = h (N ) = M, h h (N ) = h (M) = N. Gi s r ng N < M , ta c h [h ()] = h g () g () 1 = h (M1 ) = 1


1 g(M1 )

(4.33)

= N1 > .

V h h lm trn [; M1 ] v h [h ()] > nn th hm s h h (x) c t ng phn gic th nh t t i nhi u nh t l m t i m (mu thu n v i (4.33)). V y M = N, d n n

N0 = z0 = y0 = M0 . Tr ng h p g () < 1. Trong ph n ny, ta xt tr ng h p g () < 1, k = 2m + 1 v cc s h ng l c a nghi m khng nh h ng n m u c a phng trnh (4.22), t c l nghin c u tr ng h p f (u1, u2 , u3, , u2m1 , u2m , u2m+1 ) := F (u1, u3 , , u2m1 , u2m+1)

6.6. M t s l p phng trnh sai phn phi tuy n c ch m

329

v phng trnh (4.22) c d ng xn+1 = + xn2m1 . F (xn , xn2 , , xn2m ) (4.34)

Khi gi thi t (H) i v i hm F c pht bi u l i nh sau: (H ) : F : [0; )


m+1

(0; ) l m t hm lin t c, khng gi m v i m i

bi n v tng v i t nh t m t bi n. V hm g (u) := F (u, u, , u) l hm tng nn c hm ng c g 1 . Ta c nh l sau: nh l 6.28. Gi (H ). Hn n a s 0, g () < 1 v hm F th a mn gi


b b (xn ) n=k

thi t

x > : g (x) >

= , trong b c nh thu c kho ng , n = k, k + 1, c a phng trnh

m (; g 1 ()). Xt nghi m

(4.34) sao cho v i m i j = m, m 1, , 1, 0 ta c

x2j1 < b,

x2j > g 1 Khi


n

b b

:= P.

lim x2n = ,

v
n

lim x2n+1 =

1, 1L
1 L

trong L =: lim g (u) .


u

Trong tr ng h p g khng b ch n, ta t

= 0.

Ch ng minh: T tnh n i u tng c a hm g, ta c L g (P ) = g g 1 b b = b > 1. b

330

Chng 6. Kh o st dy s v phng trnh sai phn

Ta c < x1 = + suy ra < x1 < + L i c x2 = + x2m . F (x1, x1 , , x2m+1 ) b


b b

b x2m1 <+ , F (x0 , x2 , , x2m ) g (P ) = b < g 1 (1) .

V x2i+1 < b, i = 0, m v F l hm lin t c, khng gi m v i m i bi n, tng v i t nh t m t bi n, nn ta c F (x1, x1 , , x2m+1 ) < g (b) . Do x2 > + Tng t < x3 = + v x4 = + . . . b x2r2m1 <+ = b < g 1 (1) , F (x2r , x2r2, , x2r2m ) g (P ) >+
1 x g(b) 2r2m

1 x2m > x2m > P. g (b)

b x2m+1 <+ = b < g 1 (1) , F (x2 , x0, , x2m+2 ) g (P ) 1 x2m+2 >+ x2m+2 > P. F (x3 , x2, , x2m+3 ) g (b)

x2r+1 = + x2r+2 = + Nh v y

x2r2m F (x2r+1 , x2r1 , , x2r2m+1 )

> x2r2m > P.

x2r+1 (; b) , x2(r+1) > +

1 1 x2r2m = + x2([r(m+1)+1]) . g (b) g (b)

6.6. M t s l p phng trnh sai phn phi tuy n c ch m

331

Quy n p, ta c x2[(m+1)r+j] > + =+ >+ =+ . . . x2j + 2 + + r1 + r g (b) g (b) g (b) g (b) 1 x2j , j {m, m + 1, , 1, 0} . g r (b) 1 x2[(m+1)r(m+1)+j] g (b) 1 x2[(m+1)(r1)+j] g (b) 1 + x2[(m+1)(r2)+j] g (b) 1 + 2 x2[(m+1)(r2)+j] g (b) g (b)

>+ > r + Khi

lim x2n = .

Ti p theo ta ch ng minh lim x2n+1 = 1. 1L

V lim x2n = v L = lim g (u) nn


n u

lim F (x2n , x2n2 , , x2n2m ) = L.

i u ny v t

x2r+1 (; b), suy ra t n t i gi i h n h u h n li :=


n

lim inf x2n+1 , ls := lim sup x2n+1 th a mn li + li , L

332

Chng 6. Kh o st dy s v phng trnh sai phn

v ls Khi li 1 suy ra 1 1 , ls L li nn li ls . Vy
n 1 L

ls . L
1 L

v ls 1

lim x2n+1 = li = ls =

1. 1 L

Tr ng h p g () = 1 Xt phng trnh (4.22) trong tr ng h p g () = 1. N u > 0 th ch ng minh tng t nh nh l 6.22, ta c phng trnh (4.22) c i m cn b ng dng duy nh t x. nh l sau l k t qu chnh c a m c ny: nh l 6.29. Gi s > 0 v hm lin t c f (z1, , zk ) khng gi m v i m i bi n th a g () = 1. Khi m i nghi m khng dao ng (xn )n=k c a phng trnh (4.22) h i t n nghi m (n ) c chu k (k + 1). Hn n a, n u c m t ch s i0 {1, 2, , k} m hm f (z1 , , zk ) tng t i zi0 trong m t ln c n ph i c a th m i nghi m khng dao ng c a phng trnh (4.22) h i t n i m cn b ng x. Ch ng minh: Gi s (xn ) n=k l nghi m khng dao ng c a phng trnh (4.22). Khi ta c cc tr ng h p sau: Tr ng h p 1: xn x, n k. Ta c xn+1 x = + x xnk + f (xn , , xnk+1 ) g () x

6.6. M t s l p phng trnh sai phn phi tuy n c ch m

333

x xnk f (xn , , xnk+1 ) g () x

= V

x [f (xn , , xnk+1 ) g ()] xnk x x . f (xn , , xnk+1 ) g () f (xn , , xnk+1 ) x

xn x > , n k, nn f (xn , , xnk+1 ) > f (, , ) = g () = 1. Do xn+1 x suy ra xn+1 xnk . xnk x,

T i u ny v b ng quy n p, ta nh n c xr(k+1)+n xr(k+1)+nk1 = x(r1)(k+1)+n , r 0, n 0.

V v y t n t i gi i h n

n := lim xr(k+1)+n , n.
r

M t khc n = lim xr(k+1)+n = lim x(r1)(k+1)+n+k+1 = n+(k+1) .


r r

V y, m i nghi m khng dao ng (xn ) n=k c a phng trnh (4.22) h i t n nghi m (n ) c chu k (k + 1). Tr ng h p 2: xn < x, n k Ch ng minh tng t nh tr ng h p 1, ta c i u c n ch ng minh.

334

Chng 6. Kh o st dy s v phng trnh sai phn

Ti p theo, gi s c i0 (1, 2, , k) m f (z1, z2 , , zk ) tng t i zi0 trong m t ln c n ph i c a , ta ch ng minh m i nghi m khng dao ng c a phng trnh (4.22) h i t n i m cn b ng x. V m i nghi m khng dao ng c a phng trnh (4.22) h i t n nghi m n c chu k (k + 1) nn ta ch c n ch ng minh n = x, n k. Th t v y, khng m t tnh t ng qut, ta gi s xn x, n k. N u n0 k : n0 > x th n0 x suy ra f (n0 1 , , n0 k ) L i c 1 = g () nn f (n0 1 , , n0 k ) = 1 = g () = f (, , ) . Nh v y n0 i0 = < x Vy n k : n = x. nh l 6.30. Gi s = 0, f (u1, , uk ) l hm lin t c v tng v i m i bi n trong ln c n ph i c a 0. N u g(0) = 1 th m i nghi m dng c a phng trnh xn+1 = xnk , f (xn , xn1 , , xnk+1 ) n = 0, 1, , (4.35) (v l). f (n0 1 , , n0 k ) , 1. n0 (k+1) x , f (n0 1 , , n0 k )

h i t n nghi m c chu k (k + 1) d ng p, 0, 0, , 0, p, 0, 0, , 0, p, , v i p [0, ) .

6.6. M t s l p phng trnh sai phn phi tuy n c ch m

335

Ch ng minh: Tr c tin ta ch ng minh phng trnh (4.35) c duy nh t i m cn b ng x = 0. Th t v y, x l nghi m c a phng trnh x= x g () x 1 g () x =0

x 1

x = 0 ho c 1

1 =0 g () x

x = 0 ho c g(x) = 1 = g(0) x = 0. G i (xn ) n=k l nghi m dng c a phng trnh (4.35) v i i u ki n ban u dng. Khi (xn ) n=k khng dao ng v h i t n nghi m c chu k (k + 1), k hi u l (n ). Gi s n0 : n0 > 0. T phng trnh (4.35) ta c n0 = suy ra 1 = g (0) Nn n0 j = 0, j = 1, k. T c l, (xn ) n=k h i t n nghi m (n ) c chu k (k + 1) d ng n0 , 0, 0, , 0, n0 , 0, 0, , 0, n0 , . V l p phng trnh sai phn h u t b c (k 1) trn b c k Xt phng trnh sai phn h u t b c (k 1) trn b c k xn+1 = A + f (xn , ,xnk+1 ) , n = 0, 1, , xnk (4.36) f (n0 1 , n0 2 , , n0 k ) = 1. n0 (k+1) n0 = , f (n0 1 , n0 2 , , n0 k ) f (n0 1 , n0 2 , , n0 k )

336

Chng 6. Kh o st dy s v phng trnh sai phn

trong k l s t nhin dng, A (0; ) v f : Rk R+ l hm lin t c, + khng gi m v i m i bi n. Ta s nghin c u tnh b ch n, h i t c a nghi m phng trnh (4.36) v i cc gi thi t trn. Trong m c ny, ta nghin c u s t n t i v tnh duy nh t c a i m cn b ng dng x c a phng trnh (4.36). nh l sau cho ta d u hi u nh n bi t s t n t i v tnh duy nh t c a i m cn b ng dng x. nh l 6.31. Xt phng trnh sai phn (4.36), v i A > 0 v f : Rk R+ + l hm lin t c. Gi s r ng hm g(u) = f (u, u, , u) th a mn t nh t m t trong cc i u ki n sau:
u+

lim inf

g(u) < 1, u2 A(u v), g(v) . v

(4.37) (4.38) (4.39)

u > v A g(u) g(v) u>vA g(u) u

Khi t p S g m t t c cc i m cn b ng c a phng trnh (4.36) l t p con khc r ng c a (A; ). Hn n a, n u i u ki n (4.38) ho c i u ki n (4.39) c th a mn th t p S ch c m t ph n t , t c l phng trnh (4.36) c duy nh t i m cn b ng. Ch ng minh: Ta c n ch ng minh phng trnh g(u) = u(u A) c nghi m l n hn A. Th t v y, gi s phng trnh g(u) = u(u A) khng c nghi m l n hn A, t c l th hai hm s g(u) v u(u A) khng c giao i m c honh thu c (A; ). Nh n xt r ng hm s g(u) tng trn (0; ) v nh n gi tr trong (0; ) nn t i u gi s trn ta c g(u) > u(u A), u > A.

6.6. M t s l p phng trnh sai phn phi tuy n c ch m

337

Gi s i u ki n (4.37) c th a mn, ta c
u+

lim inf g(u) lim inf 1 A = 1 (v l). u2 u


u+

Nh v y phng trnh g(u) = u(u A) lun c nghi m l n hn A, gi s l x. Khi g() = x( A), x x suy ra xA= Do S = . Gi s i u ki n (4.38) c th a mn. Khi v i b t k u > A, ta c g (u) g (A) suy ra g (A) + A (u A) g (u) > u (u A) . Nn g (A) > u (u A) A (u A) = (u A) , u > A.
2 u+ 2

g() x . x

A (u A) ,

(4.40)

V lim (u A) = + v g(A) l m t s th c xc nh nn t (4.40), ta c i u v l. V y phng trnh g(u) = u(u A) c nghi m x > A. Ta ch ng minh x l duy nh t. Th t v y, gi s x, y l hai nghi m c a phng trnh g(u) = u(u A) th a A < x < y . Khi 0 < g (y ) g () = y ( A) x ( A) x y x = ( x) ( + x A) y y suy ra y+x 2A (v l). A ( x) , y

338

Chng 6. Kh o st dy s v phng trnh sai phn

V y x = y. Gi s i u ki n (4.39) c th a mn. t h (u) = Khi h (A) = V iu>


g(A) A

g (u) (u A) . u g (A) > 0. A


g(A) A

+ A th h (u) =

g(u) u

u+A

u + A < 0, ngha l

g (A) g (u) (u A) < 0, u > + A. u A Hay g (u) < u (u A) , u >


g(A) A

+A

(v l).

V y phng trnh g(u) = u(u A) lun c nghi m x > A. Ta ch ng minh x l duy nh t. Th t v y, gi s x, y l hai nghi m c a phng trnh g (u) = u (u A) th a A < x < y. Khi g () y y suy ra yA Hay y Vy x = y. nh l 6.32. Xt phng trnh sai phn (4.36), v i A >0 v f : Rk R+ + l hm lin t c. Gi s r ng hm g(u) = f (u, u, , u) th a mn i u ki n lim g(u) < 1. u2 x (v l). x A. g () x , x

u+

Khi S l t p compact.

6.6. M t s l p phng trnh sai phn phi tuy n c ch m

339

Ch ng minh: Gi s

lim g(u) 2 u+ u

< 1, ta ch ng minh S compact. Th t v y, n u


g(uk ) u2 k

t n t i m t dy (uk ) trong S d n t i + th

= 1

A uk

1, ta c i u

v l. Do (uk ) S th (uk ) b ch n, ngha l S b ch n. Ta cn ph i ch ng minh S ng. Th t v y, (uk ) S : uk u0 , ta ch ng minh u0 S. V g lin t c nn g (uk ) g (u0), m t khc uk S nn uk = A + suy ra
k+

f (uk , uk , , uk ) g (uk ) =A+ , uk uk lim g (uk )


k+

lim uk = A +

k+

lim uk

nn u0 = A + Hay u0 S. V y S ng v b ch n, nn S compact. g (u0) . u0

nh l sau cho php ta xc nh c di t i a c a m i n a chu trnh c a nghi m. nh l 6.33. Gi s A > 0, f : Rk R+ l m t hm lin t c, tng v i + m i bi n v th a i u ki n (4.39). G i x l i m cn b ng dng duy nh t v (xn ) n=k l nghi m dng, dao ng b t k c a phng trnh (4.36). Khi , m i n a chu trnh dng v m i n a chu trnh m c a (xn ) n=k xung quanh x c khng qu (2k + 1) s h ng. Ch ng minh: Gi s xs , xs+1 , , xs+2k+1 l (2k + 2) s h ng lin ti p c a (xn ) n=k th a xs+i x, i {0, 1, , 2k + 1}. G i i0 l ph n t l n nh t

340

Chng 6. Kh o st dy s v phng trnh sai phn

trong t p {0, 1, , 2k + 1} th a xs+i0 = max {xs , xs+1 , , xs+2k+1 } . N u i0 {0, 1, , k} th max {xs+i0 +k , , xs+i0 +1 } < xs+i0 . (V n u max {xs+i0 +k , , xs+i0 +1 } = xs+i0 th i = i0 + n0 , 1 n0 k: xs+i0 =

xs+i = xs+i0 +n0 , mu thu n v i i0 = max {0, 1, , 2k + 1}). V f l hm lin t c, tng v i m i bi n nn xs+i0 +k+1 = A + suy ra xs+i0 +k+1 < A + Do xs+i0 x nn p d ng (4.39) ta c g (xs+i0 ) xs+i0 Khi xs+i0 +k+1 < A +
g( ) x x

f (xs+i0 +k , , xs+i0 +1 ) f (xs+i0 , , xs+i0 ) < A+ , xs+i0 xs+i0 g (xs+i0 ) . xs+i0

g () x . x

=x

(v l).

N u i0 {k + 1, , 2k + 1} v min {xs+i0 1 , , xs+i0 k1 } < xs+i0 th ta c x(s+i0 1)+1 = A + suy ra xs+i0 .xs+i0 k1 = Axs+i0 k1 + f (xs+i0 1 , , xs+i0 k ) . Hay xs+i0 k1 = < Axs+i0 k1 + f (xs+i0 1 , , xs+i0 k ) xs+i0 Axs+i0 + f (xs+i0 , , xs+i0 ) xs+i0 A+ g () x = x. x f (xs+i0 1 , , xs+i0 k ) , xs+i0 k1

6.6. M t s l p phng trnh sai phn phi tuy n c ch m

341

Nh v y xs+i0 k1 < x (v l).

N u k < xs+i0 = xs+i0 1 = = xs+i0 k1 th ta c xs+i0 k1 = Axs+i0 k1 + f (xs+i0 1 , , xs+i0 k ) xs+i0 Axs+i0 + f (xs+i0 , , xs+i0 ) xs+i0 A+ Suy ra xs+i0 k1 < x (v l).

g () x = x. x

V y m i n a chu trnh dng c a (xn )n=k xung quanh x c khng qu (2k + 1) s h ng. Ch ng minh tng t cho tr ng h p n a chu trnh m. Ti p theo ta nghin c u tnh b ch n c a nghi m dng phng trnh (4.36). nh l 6.34. Xt phng trnh sai phn (4.36), v i A > 1 v f : Rk R+ + th a tnh ch t y1, , yk R+ : f (y1, , yk ) c max {y1 , , yk } + b,

trong b, c c nh th a c (0; A) , b 0. Khi m i nghi m dng c a phng trnh (4.36) b ch n. Ch ng minh: Gi s t n t i nghi m (xn ) n=k ch a m t dy con (xsn ) d n n +, ta c th gi s xm < xsn , m < sn . V i m i sn k + 2, ta c xsn = A + f (xsn 1 , , xsn k ) xsn k1

342

Chng 6. Kh o st dy s v phng trnh sai phn

A+ A+ suy ra

c max {xsn 1 , , xsn k } + b xsn k1 cxsn b + , A A A2 + b + cxsn .

Axsn Hay xsn

A2 +b Ac

(v l).

V y m i nghi m dng c a phng trnh (4.36) b ch n. nh l 6.35. Gi s f : Rk R+ l hm tng v i m i bi n v th a i u + ki n


k

f (u1, u2 , , uk )
i=1

ai ui ; ui 0, i = 1, , k.

N u A th a mn b t ng th c ai1 ai2 ait+1 < At+1,


i1 ++ij+1 =k+1, j=1,t

(4.41)

(v i t l s t nhin dng cho tr c) th m i nghi m dng c a phng trnh (4.36) b ch n. Ch ng minh: Gi s r ng t n t i nghi m (xn ) ch a m t dy con (xsn ) d n n +, ta c th gi s xm < xsn , m < sn . V i n l n, ta c f (xn1 , , xnk ) xn = A + xn(k+1) Suy ra xn A+ 1 xn(k+1)
k k

A+

1 xn(k+1)

aixni .
i=1

ai1 A +
i1 =1 k

1 xni1 (k+1)

ai2 xni1 i2
i2 =1

A+
i1 =1

ai1 +
i1 =1, i2 =1

ai1 ai2

xni1 i2 xn(k+1)xni1 (k+1)

(do xn > A, n)

6.6. M t s l p phng trnh sai phn phi tuy n c ch m


k

343

= A+
i1 =1 k

ai1 +
i1 + i2 =k+1

ai1 ai2 1 Ai

xni1 i2 + xn(k+1)xni1 (k+1) ai1 ai2


i1 + i2 =k+1

ai1 ai2
i1 + i2 =k+1

xni1 i2 xn(k+1) xni1 (k+1)

A+
i1 =1

ai1 +

ai1 ai2 +
1 + i2 =k+1

1 x . xn(k+1) xni1 (k+1) ni1 i2

Suy ra
k

xn

A+
i1 =1

ai1 +

1 Ai

ai1 ai2
1 + i2 =k+1

+
i1 + i2 =k+1 k

1 1 A+ ai1 ai2 xn(k+1) xni1 (k+1) xni1 i2 (k+1) 1 A ai1 ai2 + ai1 ai2 ai3 1 A2

ai3 xni1 i2 i3
i3 =1

A+
i1 =1

ai1 +

ai1 ai2 ai3


i1 +i2 =k+1i1 +i2 +i3 =k+1

+
i1 +i2 =k+1i1 +i2 +i3 =k+1 k

xni1 i2 i3 xn(k+1) xni1 (k+1) xn(i1 +i2 )(k+1) ai1 ai2


i1 + i2 =k+1

A+
i1 =1

ai1 +

1 A

ai1 ai2 +
i1 + i2 =k+1

1 x xn(k+1) xni1 (k+1) ni1 i2

xn v i B = A+

B+
i1 , i2 , ,it+1 =1i1 +i2 ++ij+1 =k+1, j=1, t

ai1 ai2 ait+1

xn(i1 +i2 ++it+1 ) , xn(k+1) xni1 (k+1) xn(i1 +i2 ++it )(k+1)

ai1 +

1 A

ai1 ai2 +

1 A2

ai1 ai2 ai3 + +


i1 +i2 =k+1

1 At

ai1 ai2 ait+1 .


i1 +i2 ++ij+1 =k+1, j=1,t1i1 +i2 ++ij+1 =k+1

Suy ra
k

xsn

B+
i1 , i2 , ,it+1 =1 i1 +i2 ++ij+1 =k+1, j=1, t

ai1 ai2 ait+1

344

Chng 6. Kh o st dy s v phng trnh sai phn

xsn (i1 +i2 ++it+1 ) xsn (k+1) xsn i1 (k+1) xsn (i1 +i2 ++it )(k+1)

(4.42).

V xm < xsn , m < sn v xn > A, n nn t (4.42) ta c

xn suy ra

B+
i1 , i2 , ,it+1 =1 i1 +i2 ++ij+1 =k+1, j=1, t

ai1 ai2 ait+1

xsn , At+1

xsn
i1 , i2 , ,it+1 =1 i1 +i2 ++ij+1 =k+1, j=1, t

ai1 ai2 ait+1

xsn At+1

hay xsn 1 hay xsn 1


i1 , i2 , ,it+1 =1 i1 +i2 ++ij+1 =k+1, j=1, t

ai1 ai2 ait+1


i1 , i2 , ,it+1 =1 i1 +i2 ++ij+1 =k+1, j=1, t

1 At+1

B
k

. ai1 ai2 ait+1 At+1


1

Do (xsn ) b ch n (V l). nh l c ch ng minh. Tr ng h p c bi t, v i k = 1, i u ki n (4.41) tr thnh A > 0. Do trong tr ng h p ny ta c k t qu : V i m i s th c dng A > 0, m i nghi m dng c a phng trnh xn+1 = A +
xn ,n xn1

= 0, 1, b ch n.

Trong m c ny, ta nghin c u tnh ht ton c c c a nghi m dng phng trnh (4.36). nh l 6.36. Gi s f : Rk R+ l hm lin t c, khng gi m v i m i + bi n v tng v i bi n th nh t, hm g(u) = f (u, u, , u) th a mn i u ki n (4.38). Khi m i nghi m b ch n c a phng trnh (4.36) h i t n i m cn b ng x.

6.6. M t s l p phng trnh sai phn phi tuy n c ch m

345

Ch ng minh: Gi s (xn )n=k l nghi m b ch n c a phng trnh (4.36). Khi , c hai dy gi i h n y (ym ) v (zm ) c a (xn ) n=k th a phng trnh (4.36) v i m i m Z v
n

lim inf xn = z0

zm , ym

y0

lim sup xn .

Gi s dy (xn ) n=k khng h i t , khi z0 < y0 . T (4.36) v v f l hm khng gi m v i m i bi n, nn ta c y0 < A + v z0 > A + Do g (y0) + Az0 > z0y0 > Ay0 + g (z0) , suy ra g (y0) g (z0 ) > A (y0 z0) nh l c ch ng minh. nh l 6.37. Xt phng trnh sai phn (4.36), v i A > 1 v f : Rk R+ + l hm lin t c th a mn cc i u ki n a) f khng gi m v i m i bi n v tng v i bi n th nh t. b) g (u) g (v) c (u v) , u > v 0; c c nh thu c [0; A). (v l). g (z0) . y0 g (y0 ) z0

Khi m i nghi m dng c a phng trnh (4.36) h i t n i m cn b ng dng c a phng trnh ny. Ch ng minh: Do f khng gi m v i m i bi n v tng v i bi n th nh t nn f (y1, y2 , , yk ) Do (b) ta c g (max {y1 , y2, , yk }) g (0) c max {y1 , y2 , , yk } , g (max {y1, y2 , , yk }) , y1, y2, , yk R.

346

Chng 6. Kh o st dy s v phng trnh sai phn

suy ra g (max {y1, y2 , , yk }) Do f (y1, , yk ) Vy f (y1 , y2, , yk ) c max {y1 , y2, , yk } + g (0) . g (max {y1, , yk }) c max {y1, , yk } + g (0) . c max {y1 , y2 , , yk } + g (0) .

T i u ny v nh l 6.34, ta c m i nghi m dng c a phng trnh (4.36) th b ch n. B i hm g th a i u ki n (4.38) c a nh l 6.31 nn phng trnh (4.36) c duy nh t i m cn b ng x > A. Do nh l 6.36, nn m i nghi m dng c a phng trnh (4.36) h i t n i m cn b ng x. nh l c ch ng minh. nh l 6.38. Xt phng trnh sai phn (4.36), v i A > 1 v f : Rk R+ + l hm lin t c, khng gi m v i m i bi n v th a mn i u ki n sau g (y) = f (y, y, , y) = y, y R+ .

Khi , m i nghi m dng c a phng trnh (4.36) h i t t i i m cn b ng A + 1. Ch ng minh: Ta c phng trnh (4.36) c duy nh t i m cn b ng x = A + 1 v th a i u ki n g (u) g (v) Ta c f (y1 , , yk ) f (max {y1, y2 , , yk } , , max {y1 , y2, , yk }) A (u v) , u > v A.

= g (max {y1, , yk }) = max {y1 , , yk } , y1 , , yk R+ .

6.6. M t s l p phng trnh sai phn phi tuy n c ch m

347

Theo nh l 6.34 th dy (xn )n=k b ch n (v i c = 1 < A v b = 0). M t khc, do hm g th a i u ki n g (u) g (v) = 1. (u v) ,

u > v 0,

nn theo nh l 6.36, dy (xn )n=k h i t n i m cn b ng x = A + 1. nh l c ch ng minh.

V m t l p phng trnh sai phn phi tuy n c ch m Xt phng trnh sai phn phi tuy n v i m t ch m d ng xn+1 = xn + F (xnm ), trong m l m t s nguyn dng c nh, n N0 ; xi , (4.43) (i = m, 0) l cc

s dng cho tr c; (0, 1) v F C([0, )). Phng trnh (4.43) xu t hi n trong nhi u ng d ng v c th c xem nh l k t qu c a s r i r c ho phng trnh vi phn phi tuy n c ch m x(t) = x(t) + f (x(t )), trong t > 0, f C([0, )), v l cc tham s dng. Trong m c ny chng ta nghin c u m t s tnh ch t c a nghi m phng trnh sai phn phi tuy n v i m t ch m (4.43). C th l, ta s xc nh m t s i u ki n m i nghi m c a phng trnh trn l h i t v 0, gi i n i ng t hay h i t t i tr ng thi cn b ng dng duy nh t. c bi t l i u ki n t n t i nghi m tu n hon khng t m th ng c a (4.43). Xt phng trnh sai phn phi tuy n c ch m (4.43). Ta c cng th c bi n thin h ng s nh sau
n

(4.44)

xn+1 =

n+1

x0 +
i=0

ni F (xim )

v i n N0 .

(4.45)

348

Chng 6. Kh o st dy s v phng trnh sai phn

Cng th c (4.45) c ch ng minh d dng b ng cch s d ng phng php quy n p theo n. nh l sau s cho ta m t i u ki n c n v m i nghi m c a (4.43) h i t t i 0. nh l 6.39. i u ki n c n v m i nghi m {xn }n c a (4.43) h i t t i 0 khi n ti n ra v cng l F (u) < (1 )u v i m i u > 0. Ch ng minh: Tr c h t gi s r ng F (u) < (1 )u v i m i u > 0. G i {xn }n l m t nghi m c a (4.43) v M := maxm
i 0

xi . Ta ch ng minh r ng xn

v i m i n. Th t v y, dng phng php quy n p gi s r ng xk k n. Ta c xn+1 = xn + F (xnm ) M + (1 )M = M. V v y xn M v i m i n. t


1 2

M v im i

= lim sup xn < +,


n

= lim sup F (xnm ) < +.


n 2

L y

> 0 l m t s nh ty . t N = N ( ) sao cho F (xnm ) <

v i m i n > N . V i n > N ta c xn+1 = xn + F (xnm ) < xn + L y gi i h n trn hai v ta nh n c


1 2

+ .

+ . 1
2

l s nh bao nhiu ty nn i u ny cho ta


2 1

(4.46)

6.6. M t s l p phng trnh sai phn phi tuy n c ch m

349

M t khc, {xn }n v {F (xnm )}n l cc dy b ch n nn ta c th ch n m t dy con {nk } c a t p s t nhin sao cho


2

= lim F (xnk m ).
k 3.

Ta cng c th gi s r ng dy con {xnk m } h i t t i gi i h n hm lin t c nn ta c


2

V F l

= F ( 3 ). N u
2

> 0, th

= F ( 3) < (1 ) 3 .

R rng,

1.

V v y
2

< (1 ) 1 . T (4.46) ta c
1

< (1 )
3

1 1

2.

T c l

<

(v l). V y
2

= 0 do
1

= F ( 3 ) = F (0) = 0 suy ra

= 0.

Nh v y,
n

lim xn = 0.

Ng c l i, gi s r ng F (u) < (1 )u khng th a mn v i m i u > 0. Hai tr ng h p sau c th x y ra: (i) T n t i a > 0 sao cho F (a) = (1 )a. (ii) F (u) > (1 )u v i m i u > 0. Trong tr ng h p th nh t dy {xn }n v i xn = a, n l m t nghi m i=

dng nn khng h i t n 0. Ta xt tr ng h p th hai. t xi = 2,

m, 0. Ta ch ng minh r ng xn > 1 v i m i n. B ng quy n p, gi s r ng xk > 1 v i k n. Th th xn+1 = xn + F (xnm ) > + (1 ) = 1.

350

Chng 6. Kh o st dy s v phng trnh sai phn

V v y xn > 1 v i m i n, do xn khng h i t t i 0. nh l c ch ng minh. Nh n xt 6.5. D th y r ng n u F (x) c (h ng s ) th limn xn =


c . 1 c . 1

c Th t v y, phng trnh xn+1 = xn + c c nghi m t ng qut l xn = n + 1 .

Do (0, 1) nn ta c ngay limn xn =

nh l sau l m t i u ki n m i nghi m c a (4.43) l gi i n i ng t. nh l 6.40. Gi s r ng F (x) = H(x, x), trong H : [0, ) [0, ) [0, ) l hm lin t c, ng bi n theo bi n th nh t nhng ngh ch bi n theo bi n th hai v H(x, y) > 0 n u x, y > 0. Gi thi t thm r ng H(x, y) < 1 , x (x,y)(,) lim sup H(x, y) > 1 . (x,y)(0,0) x lim inf Khi m i nghi m {xn }n c a (4.43) l gi i n i ng t. Ch ng minh: Tr c h t ta ch ng minh r ng {xn }n l b ch n trn. B ng phng php ch ng minh ph n ch ng, gi s lim supn xn = . V i m i s nguyn n m, ta nh ngha kn := max{ : m Nh n xt r ng km km+1 n, x = max xi }.
m i n

(4.47)

(4.48)

kn v do

lim xkn = .

Ch n n0 > 0 sao cho kn0 > 0. V i n > n0 ta c xkn = xkn 1 + H(xkn 1m , xkn 1m ) xkn + H(xkn 1m , 0)

6.6. M t s l p phng trnh sai phn phi tuy n c ch m

351

v v v y
n

lim H(xkn 1m , 0) = .

i u ny ko theo
n

lim xkn 1m = .

M t khc xkn = xkn 1 + H(xkn 1m , xkn 1m ) xkn + H(xkn , xkn 1m ) (v xkn xkn 1m v H(x, y) l hm ng bi n theo bi n x) nn ta c H(x, y) x (x,y)(,) lim sup lim sup
n

H(xkn , xkn 1m ) x kn

1 .

i u ny mu thu n v i (4.47). Do , {xn }n b ch n trn. Ti p theo, ta ch ng minh r ng lim inf n xn > 0. B ng phng php ch ng minh ph n ch ng, gi s r ng lim inf n xn = 0. V i m i s nguyn n ta nh ngha sn := max{ : m R rng, sm sm+1 n, x = min xi }.
m i n

m,

sn v do
n

lim xsn = 0.

G i C l m t c n trn c a {xn }n v n0 > 0 sao cho sn0 > 0. V i n > n0 ta c xsn = xsn 1 + H(xsn 1m , xsn 1m ) xsn + H(xsn 1m , C) v do
n

lim H(xsn 1m , C) = 0.

352

Chng 6. Kh o st dy s v phng trnh sai phn

i u ny d n n
n

lim xsn 1m = 0.

M t khc, xsn = xsn 1 + H(xsn 1m , xsn 1m ) xsn + H(xsn , xsn 1m ) (1 )xsn (v xsn H(xsn , xsn 1m )

xsn 1m v H(x, y) l hm ng bi n theo bi n x) nn ta nh n c H(x, y) (x,y)(0,0) x lim inf lim inf


n

H(xsn , xsn 1m ) xsn

i u ny tri v i (4.48). nh l c ch ng minh. nh ngha 6.13. V i m t nghi m gi i n i ng t {xn }n c a (4.43) ta g i t p t t c cc i m t c a dy cc vc t {vn = (xnm , xnm+1 , , xn )}n l t p gi i h n m ga c a {xn }n v k hi u l (x). Nh n xt 6.6. T p gi i h n (x) compact v b t bi n i v i nh x T : Rm+1 Rm+1 + + xc nh b i T vn = vn+1 . N u m t nghi m {xn }n l tu n hon th t p h p gi i h n (x) g m h u h n i m. Ng c l i, n u t p h p gi i h n (x) g m h u h n i m, th b n thn n l m t nghi m tu n hon (xem [?]). Hn n a, nh x T : (x) (x) l ton nh. V v y, t n t i hai nghi m c ngu n g c {Pn }nZ v {Qn }nZ (gi tr ban u c ch n trong t p gi i h n (x)) c a phng trnh (4.43) v i m i n sao cho lim sup xn = P0 ,
n

lim inf xn = Q0
n

v Q0 Pn P0 , Q0 Qn P0 , n Z.

6.6. M t s l p phng trnh sai phn phi tuy n c ch m

353

Ta c P0 = P1 + F (Pm1 ), v h qu l, P0 F (Pm1 ) , 1 Q0 F (Qm1 ) . 1 Q0 = Q1 + F (Qm1 ),

T cng th c ny ta c 1 inf F (x) 1 x>0 lim inf xn


n

lim sup xn
n

1 sup F (x). 1 x>0

T y ta lun gi s r ng phng trnh x = x + F (x) c nghi m duy nh t x = x (0, ). Ta s xc nh i u ki n m i nghi m c a (4.43) h i t t i tr ng thi cn b ng duy nh t x v i t t c cc ch m. nh l 6.41. Gi s F l hm n i u tng v lim sup
x

F (x) < 1 , x F (x) > 1 . x

(4.49) (4.50)

lim inf
x0

Khi m i nghi m {xn }n c a (4.43) h i t n x. Ch ng minh: V i m i x [0, ) t H(x, y) = F (x), y [0, ), th th i u ki n (4.47) v (4.48) l th a mn v nh l 6.40 c p d ng. i u ny c ngha r ng m i nghi m c a (4.43) l gi i n i ng t. V v y, v i m i nghi m {xn }n c a (4.43), t n t i hai nghi m c ngu n g c {Pn }nZ v {Qn }nZ c a (4.43) sao cho lim sup xn = P0 ,
n

lim inf xn = Q0
n

(4.51)

v Q0 Hn n a, P0 F (Pm1 ) 1 F (P0) 1 (4.52) Pn P0 , Q0 Qn P0 , n Z.

354

Chng 6. Kh o st dy s v phng trnh sai phn

v tng t Q0 t (x) = F (x) (1 ). x 0 v (Q0 ) 0. M t khc, t (4.49) F (Qm1) 1 F (Q0) . 1 (4.53)

T (4.52) v (4.53) ta thu c (P0 )

suy ra lim supx (x) < 0, v t (4.50) ta nh n c lim inf x0 (x) > 0. Do , hai tr ng h p sau c th x y ra: Ho c l trong (0, Q0 ] v [P0 , ) c hai i m K , K khc nhau sao cho (K ) = (K ) = 0, ho c P0 = Q0 = x. Theo gi thi t th tr ng h p th hai x y ra. nh l c ch ng minh. nh l 6.42. Gi s F l hm n i u gi m. t f (x) = F (x) . 1

Gi thi t thm r ng h hai phng trnh sau = f (), = f ()

c nghi m duy nh t = . Khi m i nghi m {xn }n c a (4.43) h i t n x. Ch ng minh: V i m i y [0, ) t H(x, y) = F (y), x [0, ), th th i u ki n (4.47) v (4.48) l th a mn v nh l 6.40 c p d ng. Do v y, v i m i nghi m {xn }n c a (4.43), t n t i hai nghi m c ngu n g c {Pn }nZ v {Qn }nZ c a (4.43) sao cho lim sup xn = P0 ,
n

lim inf xn = Q0
n

v Q0 V v y, P0 F (Pm1 ) 1 F (0) = f (0) =: b1 1 Pn P0 , Q0 Qn P0 , n Z.

6.6. M t s l p phng trnh sai phn phi tuy n c ch m

355

v tng t Q0 F (Qm1 ) 1 f () =: a1.

Xt h cc phng trnh sai phn sau an+1 = f (bn ), bn+1 = f (an ) v i n N.

Th th c P0 v Q0 cng thu c vo o n [an, bn ] v i m i n N. Dy {an }n l n i u tng v dy {bn }n l n i u gi m. V v y t n t i hai gi i h n tng ng l v . Hn n a, cc gi i h n ny th a mn h = f (), = f ().

Theo gi thi t c a ta th = = x. V v y, limn an = limn bn = x v do , P0 = Q0 = x. nh l c ch ng minh. Ti p theo, ta gi s r ng v i y0 > 0, ta c F (y0) = max F (x)
x 0

v F l hm n i u tng trong [0, y0], n i u gi m trong (y0, ). Trong tr ng h p ny F c g i l hm hnh chung. t f (x) = F (x) . 1

Gi thi t thm r ng {xn }n l m t nghi m gi i n i ng t c a (4.43). G i {Pn }nZ v {Qn }nZ l hai nghi m c ngu n g c c a phng trnh (4.43) sao cho lim sup xn = P0 ,
n

Q0

Pn

P0 ,

n Z.

(4.54)

V v y, P0 F (Pm1 ) 1 F (y0) = f (y0 ). 1 (4.55)

356

Chng 6. Kh o st dy s v phng trnh sai phn

nh l 6.43. Gi s r ng f (y0 )

y0 v (4.50) cng c gi thi t l ng.

Gi s {xn }n l m t nghi m gi i n i ng t c a (4.43). Th th {xn }n h i t n x. Ch ng minh: T (4.54) v (4.55) ta c Pn hm tng trong [0, y0 ] nn ta thu c P0 v tng t Q0 t (x) = T (4.56) v (4.57) suy ra (P0 ) 0, (Q0 ) 0. F (x) (1 ). x F (Qm1) 1 F (Q0) . 1 (4.57) F (Pm1 ) 1 F (P0) 1 (4.56) P0 y0 , n Z. Nhng F l

M t khc, r rng lim supx (x) < 0 v t (4.50) ta c lim inf x0 (x) > 0. Do , hai tr ng h p sau c th x y ra: Ho c l trong (0, Q0] v [P0, ) c hai i m K , K khc nhau sao cho (K ) = (K ) = 0, ho c P0 = Q0 = x. Do gi thi t c a ta tr ng h p th hai x y ra. nh l c ch ng minh. Xt tr ng h p f (y0 ) > y0. Tr c tin, ta nh c l i nh l sau c a Ivanov c trnh by trong [?]: nh l 6.44. [?] Gi s t n t i m t o n I trong R l b t bi n i v i nh x f C(R), t c l f (I) I. Gi thi t thm r ng, c duy nh t m t i m x intI l i m ht ton c c c a f , t c l f (x) = x v limn f n (x) = x v i m i x intI. Th th, m i nghi m {xn }nNm , xi intI, c a phng trnh xn+1 = h i t t i x. 1 xn + f (xnm ), +1 +1 >0 i = m, 0

6.6. M t s l p phng trnh sai phn phi tuy n c ch m

357

t I l o n [0, f(y0 )]. R rng hm f a I vo chnh n. T (4.55) ta c xn I v i t t c n tr m t s h u h n ch s n. M t khc, v x l nghi m dng duy nh t c a phng trnh x = x + F (x) nn n cng l nghi m dng duy nh t c a phng trnh f (x) = x. i u ny c ngha x intI l i m c nh duy nh t c a f . Ta c b sau: B 6.4. Gi s r ng limn f n (x) = x v i t t c x I. Th th m i

nghi m gi i n i ng t c a (4.43) h i t t i x. Ch ng minh: Nh c p trn v i m t nghi m gi i n i ng t {xn }n ta ph i

c xn I v i t t c n tr m t s h u h n ch s n. V v y khng m t tnh t ng qut ta gi s r ng xn I v i m i n. Theo nh l 6.44 ta c i u ph i ch ng minh. B 6.5. Gi s hm f c o hm n c p 3 trn I, |f (x)| hm Schwarzian Sf (x) = f (x) 3 f (x) 2 f (x) 2 f (x) 1 v o

c a f m trong I \ {x}. Th th limn f n (x) = x v i t t c x I.

Php ch ng minh c a b 6.5 c th tm th y cho ta nh l sau:

[?], [?]. B 6.4 v 6.5

nh l 6.45. Gi s hm f c o hm n c p 3 trn I, |f (x)| hm Schwarzian Sf (x) = f (x) 3 f (x) 2 f (x) 2 f (x)

1 v o

c a f m trong I \ {x}. Th th m i nghi m gi i n i ng t c a (4.43) h i t t i x.

358

Chng 6. Kh o st dy s v phng trnh sai phn

By gi chng ta nghin c u hi u su t c a ch m m i v i s h i t c a nghi m phng trnh (4.43) t i tr ng thi cn b ng dng x. Ta gi thi t f (y0 ) > y0 . i u ny ko theo x > y0 . M nh 6.3. V i m i nghi m gi i n i ng t {xn }n c a (4.43) ta c m+1 x < lim inf xn
n

lim sup xn
n

f (y0 ).

Ch ng minh: G i {Pn }nZ v {Qn }nZ l cc nghi m c ngu n g c c a phng trnh (4.43) v i P0 = lim supn xn v Q0 = lim inf n xn . Ta c Q0 = Q1 + F (Q1m ) do Q0 f (Q1m ). Nhng Q0 Q0 + F (Q1m ), f (Q1m ). M t x. T y suy ra

Q1m , v v y Q1m

khc, ta c y < f(y) v i m i y (0, x). V v y, Q1m P0 x. Hn n a, t cng th c bi n thin h ng s ta c Q0 = Q1 + F (Q1m) = (Q2 + F (Q2m )) + F (Q1m ) = 2 Q2 + F (Q2m ) + F (Q1m)
m

= m+1 Q1m +
j=0

j F (Q1mj ) > m+1 x.

M t khc, P0 = P1 + F (P1m ) nn P0 f (P1m ) < f (y0 ). Nhng P0 P0 + F (P1m ), P1m , do P1m f (P1m ).

M t khc, ta c y > f (y) v i m i y (x, ). V v y, P1m ra Q0 x. M nh c ch ng minh.

x. T y suy

nh l 6.46. Gi s t n t i cc h ng s dng L1 , L2 sao cho hm f tho mn i u ki n 0 f (x) x L1 (x x) v i m i x [m+1 x, x],

6.6. M t s l p phng trnh sai phn phi tuy n c ch m

359

x f (x)

L2 (x x)

v i m i x [x, f(y0 )].

(4.58)

Khi m i nghi m gi i n i ng t {xn }n c a (4.43) h i t n x n u 1 . m+1 > 1 L1 L2 Ch ng minh: G i {Pn }nZ v {Qn }nZ l cc nghi m c ngu n g c c a phng trnh (4.43) v i P0 = lim supn xn v Q0 = lim inf n xn . T m nh 6.3 ta c m+1 x < Q0 Pm1 x Qm1 P0 f (y0 ).

T cng th c bi n thin h ng s ta c
m

x Q0 = x

m+1

Q1m
j=0

j F (Q1mj )
m

x 1 = (1 )

m+1

(1 )
j=0

j f (Q1mj )

j (x f (Q1mj ))
j=0

(1 )
{0 j m: x f (Q1mj )}

j (x f (Q1mj )) )(P0 x)L2 .

(1 Tng t ,

m+1

P0 x =

m+1

P1m x +
j=0

j F (P1mj )
m

m+1

1)x + (1 )
j=0 m

j f (P1mj )

= (1 )
j=0

j (f (P1mj ) x) j (f (P1mj ) x)
{0 j m: f (P1mj ) x}

(1 )

360

Chng 6. Kh o st dy s v phng trnh sai phn


m

= (1 )
j=0

j L1(x Q0) L1 (x Q0)


2

m+1

1 m+1 L1 L2 P0 x . Nhng t gi thi t c a ta, 1 m+1 L1 L2 < 1, nn P0 = Q0 = x. nh l c ch ng minh. M nh m0 6.4. Gi s cc gi thi t c a nh l 6.46 c tho mn. Cho
2

0 l m t s nguyn sao cho m0 < m v 1 . m0 +1 > 1 L1 L2

Th th m i nghi m (khc h ng) {xn }n c a (4.43) khng tu n hon v i chu k m m0 . Ch ng minh: Gi s tri l i, t c t n t i {xn }n l m t nghi m tu n hon (khc h ng) v i chu k m m0 . Th th {xn }n l nghi m c a phng trnh xn+1 = xn + F (xnm0 ). Ch m trong phng trnh ny l m0 , nn p d ng nh l 6.46, ta c lim xn = x.

Nhng {xn }n l dy tu n hon nn xn = x v i m i n. i u ny mu thu n v i gi thi t {xn }n l nghi m khc h ng. M nh c ch ng minh. Trn y ta nghin c u hi u su t c a ch m m i v i s h i t c a nghi m phng trnh (4.43). Ta ch ng minh r ng v i ch m nh v F l hm phi tuy n hnh chung, th m i nghi m gi i n i ng t h i t n tr ng thi

6.6. M t s l p phng trnh sai phn phi tuy n c ch m

361

cn b ng dng x. By gi ta s nghin c u tnh tu n hon c a nghi m trong tr ng h p ch m m l n. V i gi thi t f (x) > x khi x < x v f (x) < x khi x > x, ta ch ng minh r ng t t c cc nghi m gi i n i ng t {xn }n c a (4.43) th a mn m+1 x < lim inf xn
n

lim sup xn
n

m+1 x x x

max

f (x).

(4.59)

H qu l, n u m t nghi m gi i n i ng t khng dao ng xung quanh tr ng thi cn b ng dng x, th n ph i h i t n x. Cng v y, r rng r ng m i nghi m tu n hon khc h ng s ph i dao ng xung quanh x. Cho nn, trong m c ny ta ch quan tm nghi m dao ng xung quanh tr ng thi cn b ng dng x. Ta gi s t n t i m t o n compact I = [a, b] x sao cho f (I) I,

f (x) > x v i x (a, x) v f (x) < x v i x (x, b]. K hi u K l kh i [x, b]m+1 . R rng, K l t p l i compact c a Rm+1 . Ta nghin c u nghi m dao ng c a (4.43) xu t pht t K. M nh 6.5. Gi s {xn }n l m t nghi m c a (4.43) xu t pht t K. Th th xn I v i t t c n N. Ch ng minh: Ta ch ng minh quy n p theo n. Gi s xk I = [a, b] v i t t c k n. Th th xn+1 = xn + (1 )f (xnm ) a + (1 )a = a, b, v do , xn+1 I.

b i v f nh x o n I vo chnh n. Tng t , xn+1 M nh c ch ng minh.

M nh 6.6. T n t i m t nghi m dao ng c a (4.43) xu t pht t K. Ch ng minh: Gi s tri l i r ng m i nghi m xu t pht t K l khng dao ng. Th th t (4.59) ta suy ra t t c cc nghi m u h i t n tr ng thi

362

Chng 6. Kh o st dy s v phng trnh sai phn

cn b ng x. M t khc, xt nh x K : K K (xm , xm+1 , , x0) (xm , xm+1 , , x2m).

R rng K l m t nh x lin t c. nh (, x, , x) l m t i m b t ng x c c bin c a nh x K. Theo nh l i m b t ng (khng c c bin) Browder (xem [?]), K c m t i m b t ng khc bn trong K. G i {yn }n l m t

nghi m c a (4.43) xu t pht t i m b t ng ny. Th th {yn }n l m t nghi m tu n hon khc h ng c a (4.43). i u ny mu thu n v i gi thi t r ng m i nghi m xu t pht t K h i t t i tr ng thi cn b ng dng. M nh c ch ng minh. nh ngha 6.14. M t nghi m {xn }n c a (4.43) xu t pht t K c g i l dao ng ch m xung quanh tr ng thi cn b ng dng x n u t n t i dy cc s nguyn dng n 1 < n2 < < nk < sao cho nk+1 nk > m v xn2k , xn2k +1 , , xn2k +m x,

xn2k1 , xn2k1 +1 , , xn2k1 +m < x v i t t c cc s nguyn dng k. M nh 6.7. M i nghi m dao ng c a (4.43) xu t pht t K l dao ng ch m. Ch ng minh: Xt m t nghi m dao ng {xn }n xu t pht t K. T nh ngha c a K ta c xm , xm+1 , , x0 x. Gi s n1 l ch s nh nh t sao cho

xn1 < x. Th th xn1 , xn1 +1 , , xn1 +m < x. Th t v y, gi s tri l i, t c l c k [0, m) sao cho xn1 +k+1 x v xn1 +k < x. Khi ,

(1 )f (xn1 +km ) = xn1 +k+1 xn1 +k > x x,

6.6. M t s l p phng trnh sai phn phi tuy n c ch m

363

suy ra f (xn1 +km ) > x. Nh gi thi t trn hm f , ta nh n c xn1 +km < x. i u ny mu thu n v i tnh nh nh t c a n1 . V v y, xn1 , xn1 +1 , , xn1 +m < x. By gi gi s n2 > n1 l ch s nh nh t sao cho xn2 Ta s ch ng minh r ng xn2 , xn2 +1 , , xn2 +m t n t i k [0, m) tho mn xn2 +k+1 < x v xn2 +k x. R rng, n2 > n1 +m.

x. Th t v y, gi s tri l i, x. Khi ,

(1 )f (xn2 +km ) = xn2 +k+1 xn2 +k < x x, ko theo f (xn2 +km ) < x. Nh gi thi t c a hm f , ta nh n c xn2 +km > x. i u ny mu thu n v i tnh nh nh t c a n2 . V v y, (xn2 , xn2 +1 , , xn2 +m ) K. B ng quy n p, ta c th xc nh dy n1 < n2 < < nk < cc s nguyn dng sao cho nk+1 nk > m v xn2k , xn2k +1 , , xn2k +m x,

xn2k+1 , xn2k+1 +1 , , xn2k+1 +m < x v i t t c cc s nguyn dng k. M nh c ch ng minh. By gi ta nghin c u s t n t i nghi m tu n hon khng t m th ng c a (4.43) khi ch m m l n. Tuy n tnh ho (4.43) t i tr ng thi cn b ng ( t xn = x + yn , v i > 0 l m t s nh tu ) ta c yn+1 = yn + F (x)ynm . Tm nghi m d i d ng yn = z n , ta nh n c phng trnh c trng z m+1 = z m + F (x).

364

Chng 6. Kh o st dy s v phng trnh sai phn

n nh tuy n tnh

y c xc nh nh l n c a z. i u ki n n

nh l | z |< 1 v khng n nh khi | z |> 1. Tr ng h p | z |= 1 th hi n t ng r nhnh Hopf x y ra. H s r nhnh c xc nh nh sau: Ch n z = cos + i sin v t D = F (x), ta c (cos + i sin )m+1 = (cos + i sin )m + D, cos(m + 1) + i sin(m + 1) = (cos m + i sin m) + D. T y ta nh n c cos(m + 1) = cos m + D, sin(m + 1) = sin m. Suy ra 1 = 2 + D2 + 2D cos m, hay cos m = hay m = arccos M t khc, ta cng c cos m cos sin m sin = cos m + D, sin m cos + cos m sin = sin m. Gi i h ny ta c cos = hay = arccos 1 + 2 D 2 . 2 1 + 2 D 2 , 2 1 2 D 2 . 2D 1 2 D 2 , 2D

6.6. M t s l p phng trnh sai phn phi tuy n c ch m

365

Do , h s r nhnh l m =

arccos 1 D 2D
D arccos 1+2
2

2 2

trong F (x) [1 , 1 + ] [1 , 1 + ]. Theo nguyn l r nhnh Hopf ta c k t qu sau cho s t n t i nghi m tu n hon khng t m th ng c a (4.43). nh l 6.47. N u hm F kh vi t i x v ch m m tho mn i u ki n m> arccos 1 D 2D
D arccos 1+2
2 2 2 2

D = F (x) [1, 1+][1, 1+]

(4.60)

th (4.43) nh n m t nghi m tu n hon khng t m th ng, xu t pht t K v dao ng ch m xung quanh tr ng thi cn b ng dng x. nh ngha 6.15. M t nghi m {xn }n c a m hnh qu n th (4.43) c g i l di t vong n u limn xn = 0; c g i l tr ng t n n u 0 < lim inf xn
n

lim sup xn <


n

v c g i l pht tri n b n v ng n u t n t i gi i h n limn xn (0, ). V d 6.51. (M hnh qu n th chim ct bang Wisconsin)

Kh o st s di t vong, tr ng t n, pht tri n b n v ng v tu n hon c a m hnh qu n th chim ct xn+1 = xn + bang Wisconsin h p ch ng qu c Hoa K xnm 1 + xk nm (0 < < 1, , k > 0).

Phng trnh ny thu c d ng (4.43) v i F (x) = S di t vong x , 1 + xk f (x) = F (x) . 1

366

Chng 6. Kh o st dy s v phng trnh sai phn

N u +

1 hay F (x) =

1 th (1 )x < (1 )x, 1 + xk x > 0.

x 1 + xk

Theo nh l 6.39 ta c limn xn = 0. S tr ng t n Ti p theo ta xt + > 1. t H(x, y) = x 1 . 1 + yk

R rng, H l hm ng bi n trn [0, +) i v i x v ngh ch bi n trn [0, +) i v i y; hn n a H lin t c v F (x) = H(x, x). Ta c lim sup
(x,y)(,) 1 x 1+yk

= lim sup
y

= 0 < 1 , 1 + yk

lim inf
(x,y)(0,0)

1 x 1+yk

= lim inf
y0

= > 1 . 1 + yk

V y theo nh l 6.40 ta c 0 < lim inf xn


n

lim sup xn < .


n

b S pht tri n b n v ng Ta c F (x) = Do n u k 1 + (1 k)xk . (1 + xk )2

1 th F (x) > 0 v F l hm ng bi n. Hn n a cc i u ki n

(4.49) v (4.50) c a nh l 6.41 c th a mn. T c l lim sup


x

F (x) = lim sup = 0 < 1 , k x x 1 + x F (x) = lim inf = > 1 . x0 1 + xk x

lim inf
x0

6.6. M t s l p phng trnh sai phn phi tuy n c ch m

367
x 1+xk

Xt phng trnh F (x) = (1 )x, x > 0 ta c thu c x =


k

= (1 )x t ta

+1 1

duy nh t (v x > 0). Theo nh l 6.41 ta c lim xn =


k

+1 . 1

By gi ta xt tr ng h p k > 1. Trong tr ng h p ny dng nh l 6.43 ta tnh c F (x) = 0 = 1 + (1 k)xk = 0 = x = Xt y0 =


k k

1 > 0. k1

1 k1

> 0 ta c F (y0) = max F (x)


x 0

F (y0) =

y0 (k 1) y0 = k 1 + y0 k
+1

v ta nh n c r ng n u 0 < k < 1 F (y0) = (1 )y0 k t suy ra


n

th

(1

+1 )y0 = (1 )y0 , +1 . 1

lim xn =

Ti p theo ta xt tr ng h p k> . +1

p d ng nh l 6.45, tr c h t ta tnh f (x). Ta c f (x) = 1 { k( + 1)}. 1. i u ny cho ta 2 . +1

Ta c n tm i u ki n | f (x) | k

368

Chng 6. Kh o st dy s v phng trnh sai phn

Ta s ch ng minh r ng v i k [0, f(y0 )]. Ta c Sf (x) =

2 o hm Schwarzian Sf l m trn o n

k(k 1)xk {(k 1)(k 2)xk + 2(k + 1)} . 2x2 {(k 1)xk 1}2 2.

V v y Sf (x) < 0 v i m i x > 0 n u k

Trong tr ng h p 1 < k < 2 ta ph i gi thi t thm r ng 1 nh n c s


k

2(k + 1) k 2k k1

n nh ti m c n ton c c c a x. Th t v y, i u ki n o

hm Schwarzian m trong [0, f(y0)] l (k 1)(k 2)xk + 2(k + 1) > 0 x < Ngoi ra ta cn c n
k k

2(k + 1) 2k

1 = k1

2(k + 1) y0 . 2k

2(k + 1) y0 2k
k

f (y0 ) =

(k 1) y0 k(1 )

2(k + 1) 2k 1

(k 1) k(1 )
k

k 2(k + 1) . 2k k1

p d ng nh l 6.46 ta tm s L sao cho |f (x) x| Ta c f (x) = t (y) = [1 + (1 k)y] , v i y = xk . 1 (1 + y)2 [(1 + xk ) kxk1 x] F (x) [1 + (1 k)xk ] = = . 1 1 (1 + xk )2 1 (1 + xk )2 L|x x| v i m i x [y0, f(y0 )].

6.6. M t s l p phng trnh sai phn phi tuy n c ch m

369

V (y) lun m nn ta xt (y) =| (y) |= D dng tnh c (y) = [(1 k)y 2 + 2y + k + 1] (1 )(1 + y)4
k+1 . k1

(k 1)y 1 , 1 (1 + y)2

k y0 .

v phng trnh (y) = 0 c 2 nghi m y1 = 1, y2 =


k y[y0 ,f (y0 )k ]

V y

max

(y) = (

(k 1)2 k+1 )= = L = L1 = L2 . k1 1 4k 1 L

Do , n u m+1 > 1 v n u f (m+1 x) x, t c l, n u +1 1 th ta c


n

1 m+1 , m+1 (m+1)k

lim xn = x

v i m i nghi m {xn }n c a (4.43). T ng h p l i cc k t qu N u + trn ta c:

1 th m i nghi m di t vong.

N u + > 1 th m i nghi m tr ng t n. V i i u ki n ny th tr ng thi cn b ng dng duy nh t c a m hnh l x=


k

+1 . 1

Khi m i nghi m pht tri n b n v ng (limn xn = x) n u m t trong hai i u ki n sau y tho mn:
2 (i) k 0, +1 2, +1 ,

(ii) 1 m+1 <

4k (k1)2

1m+1 m+1 (m+1)k

+1 . 1

370

Chng 6. Kh o st dy s v phng trnh sai phn

Nh n xt 6.7. K t qu ny l m i v c ngha, b i v tr c y (xem [?], [?], [?]), cc tc gi ch ng minh s tuy nhin s d ng thm gi thi t khc. Trong [?] cc tc gi ch ng minh r ng n u k< 2 1 +1 n nh ton c c v i t t c cc ch m,

th tr ng thi cn b ng dng x l n nh ti m c n a phng. K t qu c a ta l n nh ti m c n ton c c nn i h i ph i thm i u ki n v cc tham s . By gi ta nghin c u tnh ch t tu n hon c a nghi m. Gi s k> Th th F (y0) = v F (x) = 1 [ k( + 1)] < 0. (k 1) y0 = max F (x), x 0 k 2 +1 v y0 =
k

1 . k1

R rng, f (y0) > y0 v f n i u tng trong o n [y0, f(y0 )]. T n t i m t o n ng I = [a, b] [y0, f(y0)] sao cho f nh x o n ny vo chnh n. Khi , v i ch m m l n t n t i m t nghi m tu n hon khc h ng s xu t pht t kh i [x, b]m+1. Ch r ng, nh n c (4.60) i h i ph i c k< 1 2 . +1 1

V d 6.52. (M hnh qu n th ru i xanh Nicholson). Kh o st s di t vong, tr ng t n, pht tri n b n v ng v tu n hon c a m hnh qu n th ru i xanh Nicholson xn+1 = xn + pxnm eqxnm , (0, 1), p, q (0, ).

6.6. M t s l p phng trnh sai phn phi tuy n c ch m

371

Phng trnh ny thu c d ng (4.43) v i F (x) = pxeqx , f (x) = F (x) . 1

R rng, hm pht tri n trong m hnh ny l hm hnh chung v i b t k p, q (0, ); trong khi , v i 0 < k 1 th hm pht tri n trong m hnh

qu n th chim ct l hm n i u tng trn [0, ). Ta d dng nh n c cc i u ki n sau cho s di t vong, tr ng t n, pht tri n b n v ng v tu n hon c a qu n th ru i xanh Nicholson: N up 1 th m i nghi m di t vong.

N u p > 1 th m i nghi m tr ng t n. N u p > 1 th tr ng thi cn b ng dng duy nh t l x= p 1 ln . q 1

Khi n u m t trong 2 i u ki n sau tho mn th m i nghi m l pht tri n b n v ng: (i) p e2 (1 ),


ln
p(1)e2 p

(ii) e2 (1 ) < p < e2 v m < N up e2 v arccos m> arccos

ln

p 12 [(1)(1ln 1 )]2 p 2[(1)(1ln 1 )]

1+2 [(1)(1ln 2

p )]2 1

th t n t i nghi m tu n hon khc h ng s . Bi t p

372

Chng 6. Kh o st dy s v phng trnh sai phn

1. Ch ng minh nh l sau: "Gi s hm f n i u gi m theo bi n x v i m i y > 0 v n i u tng theo bi n y v i m i x > 0. Gi thi t thm r ng, M := supx,y
0

f (x, y) < v h phng trnh u = f (v, u), v = f (u, v)

c nghi m duy nh t u = v = . Khi m i nghi m c a xn+1 = h i t n ". 2. Gi s /A. Ch ng minh r ng, n u m t trong cc i u ki n sau tho + xn1 A + Bxn + xn1

mn th m i nghi m c a phng trnh sai phn xn+1 = h it n (i) = A; 1; 4/(B 1).


(A)2 +4(B+1)+A : 2(B+1)

+ xn1 A + Bxn + xn1

(ii) > A v B

(iii) > A, B > 1 v ( A)2

3. Cho dy (xn ) n=1 xc nh theo cng th c xn+1 = + xn1 , n = 0, 1, , xn

trong x1 , x0 l cc s th c dng cho tr c. Ch ng minh r ng a) N u > 1 th dy (xn )n=1 h i t . b) N u 0 < 1 v x1 , x0 th a 0 x1 < 1, x0


1 1

th

lim x2n = ,

6.6. M t s l p phng trnh sai phn phi tuy n c ch m

373

v
n

lim x2n+1 = .

4. Cho dy (xn ) n=k xc nh theo cng th c xn+1 = + xnk , 1 + x n n = 0, 1, ,

trong xk , , x0 l cc s th c dng cho tr c, 0 < < 1 v (0; 1 + ). Ch ng minh r ng dy (xn ) n=k b ch n. 5. Cho dy (xn ) n=k xc nh theo cng th c xn+1 = + + xnk 1 2 xn xn1 x k nk+1 , n = 0, 1, ,

trong xk , , x0 l cc s th c dng cho tr c, j (0; 1) , j = 1, k sao cho 1 + 2 + + k =: (0; 1) , 1. Ch ng minh r ng dy (xn ) n=k h it . 6. Cho dy (xn ) n=k xc nh theo cng th c xn+1 = + 1+ 1 x1 n + xnk 2 2 xn1 + + k xk nk+1 , n = 0, 1, ,

trong xk , , x0 l cc s th c dng cho tr c, > 0; j 0, j = 1, k


k

sao cho
j=1

j > 0, i (0; 1) , j = 1, 2, , k. Ch ng minh r ng dy (xn ) n=k

h it . 7. Cho dy (xn ) n=k xc nh theo cng th c xn+1 = + tx1 x2 n n1 xnk , x k nk+1 n = 0, 1, .

trong xk , , x0 l cc s th c dng cho tr c, 1 , 2 , , k l cc s th c dng, sao cho 1 + 2 + + k = 1. Ch ng minh r ng, dy (xn ) n=k h i t n u m t trong cc i u ki n sau c th a mn a) > 0, t > 0 v t 3. b) > 0 v 2t 3 + 5.

374

Chng 6. Kh o st dy s v phng trnh sai phn

8. Cho dy (xn )n=k xc nh theo cng th c xn+1 = + (1 + 2 + + k ) xnk , 1 x + 2 x + + k x n1 n nk+1 n = 0, 1, ,

trong xk , , x0 l cc s th c dng cho tr c, > 1, > 0, i > 0, i = 1, 2, k. Ch ng minh r ng, dy (xn ) n=k h i t n u m t trong cc i u ki n sau c th a mn: a) + 2 < + . b) 2 > + 2 + 2 + 4. 9. Cho dy (xn ) n=1 xc nh theo cng th c xn+1 = A + xn , n = 0, 1, , xn1

trong A > 0, x1 , x0 l cc s th c dng cho tr c. Ch ng minh r ng dy (xn ) n=1 b ch n. 10. Cho dy (xn ) n=2 xc nh theo cng th c xn+1 = A + a1 xn + a2xn1 , xn2 n = 0, 1, ,

trong x2 , x1 , x0 , a1 , a2, A l cc s th c dng cho tr c. Ch ng minh r ng a) N u a2 < A th m i nghi m c a phng trnh b ch n. b) N u a1 + a2 < A th m i nghi m c a phng trnh h i t n i m cn b ng x = A + a1 + a2. 11. Cho dy (xn ) n=4 xc nh theo cng th c xn+1 = 2 + 4xn + 2xn1 + xn2 + xn3 , n = 0, 1, , 8xn4

trong x4, x3, x2, x1 , x0 l cc s th c dng cho tr c. Ch ng minh dy (xn ) n=4 h i t v i m 3. 12. Cho dy (xn )n=k xc nh theo cng th c xn+1 = A + a1 xn + a2xn1 + + ak xnk+1 , n = 0, 1, , xnk

6.6. M t s l p phng trnh sai phn phi tuy n c ch m

375

trong xk , xk+1 , , x0, a1, a2 , , ak l cc s th c dng cho tr c, A > 1. Ch ng minh r ng, n u a1 + a2 + + ak = 1 th dy (xn ) n=k h i t v i m cn b ng x = A + 1.

Chng 7

Kh o st cc phng trnh i s

376

7.1. Nh c l i cc ki n th c c b n v s ph c v hm ph c

375

7.1

Nh c l i cc ki n th c c b n v s ph c

ph c v hm

7.2. S nghi m c a phng trnh a th c trn m t kho ng

409

7.2

S nghi m c a phng trnh a th c trn m t kho ng

442

Chng 7. Kh o st cc phng trnh i s

7.3

nh gi kho ng nghi m

7.4. Gi i g n ng phng trnh a th c

481

7.4

Gi i g n ng phng trnh a th c

Ph l c A

Hm sinh v p d ng
P-1 V d minh h a

Tr c h t ta xt hai v d sau V d P.1. V i n N ta k hi u ln l s cch phn tch n thnh t ng c a cc s t nhin l , cn kn l s cch bi u di n n thnh t ng c a cc s t nhin i m t khc nhau. Ch ng minh r ng ln = kn , n N . L i gi i. V i |x| < 1, xt cc hm s p(x) =
iN, i l

1 + xi + x2i + x3i + ,

q(x) =

(1 + xj ).
jN

(0)

Ta c cc nh n xt sau:

H s c a xn trong khai tri n p(x) trong (0) chnh l ln v n b ng s cch ch n cc th a s xt sao cho t ng cc ly th a c a x b ng n. H s c a xn trong khai tri n v ph i c a q(x) chnh l kn . Ch r g khi x (1; 1), ta c 1 + x + x2 + x3 + + xn + = do 1 1 1 . 3 1 x5 1x1x 1 1 1 1 x 2 1 x4 1 x6 = . Cn q(x) = 2 1 x3 3 1 x5 1x 1x 1 x1 x V y nn p(x) = q(x), suy ra ln = kn , n N . p(x) =
517

1 , 1x

518

Ph l c A

V d

P.2. Cho (an ) N, (n N) l dy s khng gi m sao cho m i s t

nhin u c th bi u di n m t cch duy nh t d i d ng ai + 2aj + 4ak trong i, j, k N, khng nh t thi t ph i khc nhau. Hy tnh a2004. L i gi i. t F (x) = xa0 + xa1 + xa2 + v i |x| < 1. Khi F x2 = x2a0 + x2a1 + x2a2 + ; T ta c F (x)F x2 F x4 =
i;j;kN

F x4 = x4a0 + x4a1 + x4a2 + . xai +2aj +4ak := M(x).

Theo gi thi t, m i s t nhin u c th phn tch c d i d ng ai + 2aj + + 1 4ak , nn ta c M(x) = Ta c dy bi n i sau xn M(x) = 1x n=0 1 1 F x2 F x4 F x8 = 1x 1 x2 1 = F (x)F x2 F x4 (1 + x)F x2 F x4 F x8 = 1x F (x)F x2 F x4 = F (x) = (1 + x)F x8 = (1 + x) 1 + x8 F x8 1 + x8
2 2

= (1 + x) 1 + x8

1 + x8

V y ai l cc s t nhin m khi vi t theo c s 8 ch c m t cc ch s 0 v 1. xc nh a2004 tr c h t ta bi u di n s 2004 theo c s 2 r i thay c s 2 b i c s 8. M 2004 = 22 + 24 + 26 + 27 + 28 + 29 + 210 nn ta c a2004 = 82 + 84 + 86 + 87 + 88 + 89 + 810 .

P-2

Khi ni m v hm sinh

Trong l i gi i c a hai v d trn ta s d ng khi ni m hm sinh (Generating function). Khi ni m ny cn c s d ng trong cc d ng bi t p khc v dy s nn ta s nh c qua y. B n c quan tm c th c bi vi t

c a Srini Devadas v Eric Lehman trong t p ch "Mathematic for Computer

P-2. Khi ni m v hm sinh

519

Science", April, 2005.

C hai khi ni m trong Ton ph thng m

y ta mu n m r ng, l:

1. Khi ni m t ng c a c p s nhn li v h n (Sch gio khoa i s v Gi i tch 11 nng cao, trang 133).
n

2. Khi ni m a th c P (x) = a0 + a1x + a2x + + an x :=


k=0

ak xk .

nh ngha P.1. Cho dy hm (un (x)) v i un (x) F (X), X R. Cho X. G i Sn (x) =


k=0 n

uk (x) l t ng ring th n c a dy. N u (Sn (x))


+

S(x) trn

, th S(x) cn c g i l t ng v h n cc s h ng c a dy hm (un (x)). Ta vi t S(x) =


n=0 + +

un (x). un (x) cn c g i l chu i (hm) v i s h ng


n=0 +

K hi u
n=0 +

un (x) hay l

t ng qut un (x), cn S(x) c g i l t ng c a chu i . Ta cn ni r ng chu i un (x) h i t v S(x) trn v k hi u l


n=0 + n=0

un (x)

S(x) trn hay l

un (x)
n=0

S(x) n u r mi n .
+

nh ngha P.2. Chu i


n=0 +

an xn v i an R c g i l chu i ly th a.
+ n=0

V d (1; 1),

P.3. Chu i
n=0 +

u1xn l m t chu i ly th a.

u1 xn

u1 trn 1x

hay l
n=0

u1 xn = u1 + u1 x + u1 x2 + =

u1 , x (1; 1). 1x

Ch P.1.
+

1. Chu i ly th a
n=0 +

an xn = a0 + a1 x + a2x2 + + an xn + l m r ng

c a t ng v h n
n=0

u1 xn khi cc h s c a t ng khc nhau, cng l m r ng

520

Ph l c A

c a a th c P (x) khi s s h ng l v h n. C th coi chu i ly th a l a th c b c v cng.


+

2. Trong chu i
n=0 +

an xn l y x = 1, c chu i s a0 + a1 + a2 + + an + :=

an . i v i chu i s ta cng c cc khi ni m t ng ring, h i t , phn k,


n=0

v.v. nh v i chu i hm. C th coi chu i s l tr ng h p ring c a chu i hm v i un (x) = an .


+

nh l P.1 (i u ki n c n chu i s h i t ). N u
n=0 +

an h i t th an 0.

H qu P.1. Chu i
n=0

an v i (an ) khng ph i l m t VCB th phn k.


+

nh l P.2. N u n N : un (x) C(X) v


n=0

un (x)

f (x) trn X th

f (x) C(X).
+

nh l P.3. N u n N : un (x) c o hm trn X v


n=0 +

un (x)

f (x)

trn X th f (x) cng c o hm trn X v


n=0

un (x)

f (x) trn X.
+

nh l P.4. N u n N : un (x) C([a; b]), (a < b) v


+ x n=0 x

un (x)

f (x)

trn [a; b] th f (x) C([a; b]). Ngoi ra,


n=0 a

un (t) dt
a

f (t) dt v i m i

x [a; b]. Hi n nhin chu i ly th a th a mn t t c cc i u ki n c a ba nh l trn. xc nh s h i t c a m t chu i hm, th ng s d ng tiu chu n Weierstrass nh l P.5 (Tiu chu n h i t Weierstrass cho chu i hm). Cho chu i hm
+ n=0 + +

un (x) v i un : X R. N u t n t i dy s (cn ) R+ sao cho |un (x)| cn h i t th chu i hm


n=0 n=0

cn , x X, n N v chu i s u trn X.

un (x) h i t

P-2. Khi ni m v hm sinh

521

nh ngha P.3 (Khai tri n Taylor). 1. Bi u di n (hnh th c) hm s f (x)1 v khai tri n Maclaurin2 d i d ng
f(x) = f(a)+(xa) f (a) f (n) (a) f (a) +(xa)2 + +(xa)n + := 1! 2! n! f (n) (a) (xa)n n!

c g i l khai tri n hm s thnh chu i Taylor. 2. c bi t, khi a = 0, chu i Taylor cn c g i l chu i Maclaurin f (x) = f (0) + f (a) f (a) 2 f (n) (a) n x+ x + + x + := 1! 2! n! f (n) (a) n x . n!

Ta s khng i su hn m ch lu r ng trong hai bi u di n trn, cha ch c v ph i h i t v v tri. Tuy nhin, n u x y ra s h i t th khai tri n d ng l duy nh t. Trong ph n ny ta quy c n u vi t c d i m t trong hai d ng trn th v tri h i t v v ph i. nh l P.6 (Khai tri n Maclaurin c a m t s hm s s c p). xn x x2 x3 1. V i m i x R, c ex = 1 + + + + + + . (1) 1! 2! 3! n! x2 x3 x xn 2. V i m i x R, c ex = 1 + + + (1)n + . (2) 1! 2! 3! n! x2 x4 x2n ex + ex = 1+ + + + + (3) 3. V i m i x R, c cosh x := 2 2! 4! (2n)! ex ex x3 x5 x2n+1 4. V i m i x R, c sinh x := = x+ + + + + 2 3! 5! (2n + 1)! (4) 2 4 6 2n x x x x + + + (1)n + .(5) 5. V i m i x R, c cos x = 1 2! 4! 6! (2n)! x3 x5 x2n+1 + + (1)n + (6) 6. V i m i x R, c sin x = x 3! 5! (2n + 1)! Ch P.2. Ta khng s d ng n i u ki n x R nn cc cng th c trn cng ng v i x C. Trong cng th c (1) thay x b i iz ta c eiz = 1 +
1

(iz)n iz (iz)2 (iz)3 + + + + + 1! 2! 3! n!

Brook Taylor (1685-1731), nh ton h c ng i Anh, cng th i v i Newton v Leibniz. Khi ni m chu i Taylor c ng a ra khi m i 29 tu i, trong cu n " Methodus incrementorum directa et inversa", London, 1715. 2 Colin Maclaurin (1698-1746), nh ton h c ng i Scotland. Khai tri n ny c ng a ra nm 1742.

522

Ph l c A

z3 z5 z2 z4 z6 + + + i z + . Suy ra 2! 4! 6! 3! 5! eiz = cos z + i sin z. = 1 Tng t , c eiz = cos z i sin z T (7) v (8) ta c hai cng th c sau cos z = sin z = eiz + eiz = cosh(iz) 2

(7)

(8)

(9) (10)

1 eiz eiz = sinh(iz) hay l sinh(iz) = i sin z. 2i i

Cc cng th c (9), (10) g i l cc cng th c Euler. V i z = x + iy, x; y R, ta c ez = ex+iy = ex eiy = ex (cos y + i sin y) (11)

Ta bi t r ng m i s ph c u c th vi t d i d ng l ng gic z = r(cos + i sin ). Theo (7) cn c th vi t s ph c z d i d ng z = rei . l d ng m c a s ph c. nh l P.7 (M t s khai tri n Maclaurin khc). 1. V i m i x (1; 1), c 1 = 1 + x + x2 + x3 + + xn + . 1x 2. V i m i x (1; 1), t R, c t(t 1) 2 t t(t 1) (t n + 1) n x+ x + + x + . (13) 1! 2! n! x2 x3 xn+1 + + (1)n + . 2 3 n+1 (12)

(1 + x)t = 1 +

3. V i m i x (1; 1), c ln(1 + x) = x (14)

B ng php th ; l y o hm hai v ; l y tch phn hai v ta c th tm c khai tri n c a cc hm s khc n a.

P-2. Khi ni m v hm sinh

523

V d

P.4. Ch ng minh chu i i u ha 1 +


n

1 1 1 + + + + phn k. 2 3 n

L i gi i. Xt dy cc t ng ring Sn =
j=1

1 . G i k N, sao cho j

2k Ta c 1 2k

n < 2k+1 k

log2 n < k + 1 k = [log2 n].

1 1 > k+1 . Th c hi n dy bi n i v nh gi sau n 2 1 1 1 1 1 1 1 + + + + k + + k+1 + k+1 + + 2 3 4 2 +1 2 2 +1 n 2i 1 2 1 1 + + i+1 +1 2 v (do 1 > 1 v b i cc s h ng dng 2


2i

Sn = 1 +
k

>
i=0 k

cu i)

>
i=0

1 1 + + i+1 2i + 1 2

>
l=1

1 2i+1

1 2

= V d

1 k+1 = [log2 n] + 1 + Sn +. 2 2 P.5. Ch ng minh r ng t p cc s nguyn t l v h n.


+

1 1 1 < 1). h i t (v | | n p 2 n=0 p Gi s t p l h u h n v = {p1 ; p2 ; ; pl }. l + + 1 1 Khi , do m i chu i , (j 1..l) h i t nn chu i n pn n=0 pj n=0 j j=1 L i gi i.3 V i m i p , ta c chu i
+

:=

cn cng h i t . Nhng cc s cn c d ng
n=0 +

1 pk1 pk2 pkl 1 2 l + 1


n=0

nn theo khai tri n

l phn k theo v d n P.4. i u mu thu n ch ng t gi s c a ta l sai, t c l t p v h n. cn =


n=0

tiu chu n c a s nguyn dng, ta c

V d
3 4

P.6. Ch ng minh r ng

1 1 1 1 = 1 + + 4 3 5 7 9

(16)4

L i gi i c a Leonard Euler (1707-1783). K t qu ny c G.W.Leibnitz a ra nm 1666.

524

Ph l c A

L i gi i. C ta c

1 = 1 + x2
x

+ n=0

(1)n x2n , x [1; 1]. T , v i m i x [0; 1],

arctan x =
0

dt = 1 + t2

(1) t dt =
n=0 0 n=0

n 2n

(1)n

x2n+1 , x [1; 1]. 2n + 1

Cho x = 1 ta c i u ph i ch ng minh. nh ngha P.4 (Hm sinh). Hm sinh c a dy s (gn ) v i d ng khai tri n < g0 ; g1 ; g2 ; ; gn ; > l m t chu i ly th a hnh th c G(x) := g0 + g1 x + g2 x2 + + gn xn + :=
+ n=0 +

gn xn . gn xn h i t . N u chu i

Ta vi t "chu i hnh th c" v cha ch c chu i


n=0

h i t v hm s G(x) th ta vi t < g0 ; g1 ; g2 ; ; gn ; > g0 +g1 x+g2 x2 + +gn xn + = [ Hay l ng n g n hn (gn ) G(x).


+

]G(x).

V d P.7. 1. < 0; 0; 0; >


n=0

0xn = 0.
n

2. < a; 0; 0; > a + 0x + 0x2 + + 0xn + = a. 3. < a0 ; a1; a2; an ; 0; 0; 0; >


k=0 2

ak xk . 1 1x

4. < 1; 1; 1; 1; > 1 + x + x + + xn + = nh l P.8 (Cc php ton i v i hm sinh).

1. N u (gn ) g(x) ; (fn ) f (x) v a, b l cc h ng s th (a gn + b fn ) a g(x) + b f (x). 2. N u < f0 ; f1; f2 ; > f (x) th xk f (x) 3. N u < f0 ; f1; f2 ; f3 ; > f (x) th < f1 ; 2f2 ; 3f3 ; > f (x). < 0; 0; ; 0; f0; f1 ; f2 ; >
k ch s 0

P-3. M t s v d p d ng

525

V d

P.8. Tm hm sinh c a dy cc s chnh phng (n2 ).

L i gi i. Xt dy tng ng sau 1 1 1 < 1; 2; 3; 4; > = 1x 1x (1 x)2 x x 1+x < 0; 1; 2; 3; > < 1; 4; 9; 16; > = 2 2 (1 x) (1 x) (1 x)3 x(1 + x) (n2 ) < 0; 1; 4; 9; 16; > (1 x)3 < 1; 1; 1; 1; > V d P.9. 1. Tm hm sinh c a dy s (an ) cho b i a0 = 0 ; a1 = 1 ; an+2 = 3an+1 2an . 2. T , hy xc nh s h ng t ng qut c a dy s ny. L i gi i. 1. Gi s (an ) f (x). Ta c dy < 0 1 0 0 0 3a2 3a3 + < 0 3a0 3a1 < 0 0 2a0 2a1 2a2 < a0 a1 a2 a3 a4 bi > > > > n i x 3xf (x) 2x2f (x) (15)

f (x) x V y f (x) = x + 3xf (x) 2x2f (x) f (x) = l hm sinh c n 2x2 3x + 1 tm. + 1 1 2. Vi t Suy ra an xn f (x) = 1 2x 1 x n=0 f (x) = 1 + (2x) + (2x2 ) + + (2x)n 1 + x + x2 + + xn +
+

=
n=0

(2n 1)xn an = 2n 1 l s h ng t ng qut c n tm.

P-3
V d

M t s v d

p d ng

P.10. Cho p1 ; p2 ; ; pn ; l dy cc s nguyn t lin ti p.

Cho x0 (0; 1). Dy s (xk ) c xc nh nh sau n u xk1 = 0 0 pk xk = n u xk1 = 0, xk1

526

Ph l c A

trong ta k hi u {a} := a [a] l ph n phn c a s th c a. Tm x0 trong dy s (xk ) ch c h u h n s khc 0. L i gi i. Nh n xt r ng n u xk Q (Q := t p cc s h u t ) th xk1 = 0 Khi xk = 0, ta c pk N xk1 Q. xk1 pk pk pk Khi xk = 0, ta c xk = xk1 = xk1 Q. pk xk1 xk1 xk + xk1 V y n u xk Q th xk1 Q. Ta c n ph i ch ng minh t n t i k xk = 0. Khi cc s h ng ti p theo u b ng 0 v trong dy s c khng qu k s h ng khc 0 l x0 ; x1; ; xk1 . Th nhng, n u xk = 0 Q th theo trn xk1 Q xk2 Q x0 Q. m r Ng c l i, n u x0 Q x0 = v i 0 < m < n th x1 = trong r l s n m d trong php chia np1 cho m. T suy ra m u s c a x1 nh hn m u s c a x0 . Tng t , ch ng minh c t n t i xl sao cho m u s c a xl b ng 1 v khi pk pk N, suy ra xl+1 = = 0 nn t n t i k, (k = l + 1), sao cho ; xk = 0. xl xl V y x0 Q l cc gi tr c n tm. V d P.11. Tm s cc tam gic c s o cc c nh l cc s nguyn dng

khng v t qu 2n. L i gi i. G i f (n) l s c n tm ; f1 (n) l s cc tam gic c s o m t c nh l 2n + 1, s o cc c nh kia khng qu 2n + 1 ; f2 (n) l s cc tam gic c s o m t c nh l 2n + 2, s o cc c nh kia khng qu 2n + 2. Khi d th y f (1) = 3 ; f (n + 1) = f (n) + f1 (n) + f2 (n)
(c 3 tam gic c s o cc c nh khng qu 2 l cc tam gic (1; 1; 1) ; (1; 2; 2) ; (2; 2; 2)).

G ia

c l s o ba c nh c a m t tam gic th f1(n) l s cc tam gic

P-3. M t s v d p d ng

527

c a = 2n + 1 ; b = 2n + 1 k, (k 0..2n) ; c = x (v i x N, x Nhng b+ c > a 2n + 1 k + x > 2n +1 x > k k +1 V y v i m i gi tr c a k


n

b).

2n +1 k ; k

n.

n, x nh n 2n + 1 2k gi tr v do (2n + 1 2k) = = (n + 1)2 .


k=0 n

f1 (n) =

Hon ton tng t ta cng c f2(n) =


k=0

(2n + 2 2k) = = (n +

1)(n + 2). T suy ra f (n + 1) = f (n) + (n + 1)2 + (n + 1)(n + 2) = f (n) + 2n2 + 5n + 3. Gi i phng trnh sai phn tuy n tnh c p 1 trn v i f (1) = 3 ta c f (n) = n(n + 1)(4n + 5) l s c n tm. 6 V d P.12. Cho Sn = {1; 2; ; n}. Ph n t j Sn c g i l i m b t ng c a song nh5 , p : Sn Sn n u p(j) = j. G i f (n) l s song nh t Sn ln Sn khng c i m b t ng, cn g(n) l s song nh c ng m t i m b t ng. Ch ng minh r ng |f (n) g(n)| = 1. L i gi i. G i p l m t song nh t Sn ln Sn m c ng m t i m b t ng j. C n cch ch n j Sn v f (n 1) cch l p cc song nh t Sn \ {j} ln Sn \ {j} m khng c i m b t ng. V y g(n) = nf (n 1), n 2. (1)

By gi , g i r l song nh t Sn ln Sn m khng c i m b t ng. Khi , r(1) = j v i j = 1 v c n 1 cch ch n j nh v y.


5

V khi ni m song nh, xem trong chuyn Phng trnh hm

528

Ph l c A

N u r(j) = 1 th c f (n 2) cc song nh t Sn \ {1; j} ln Sn \ {1; j} m khng c i m b t ng. N u b sung thm r(1) = j, r(j) = 1 th song nh r t Sn ln Sn cng khng c i m b t ng. V y c (n 1)f (n 2) cc song nh t Sn ln Sn lo i ny. N u r(j) = 1 th c f (n 1) cc song nh t Sn \ {1} ln Sn \ {1} m khng c i m b t ng. G i q l m t song nh nh v y. B ng cch t p(i) = q(i) p(i) = 1, p(1) = j n u q(i) = j n u q(i) = j

ta cng c song nh p khng c i m b t ng t Sn ln Sn . V y c (n 1)f (n 1) cc song nh t Sn ln Sn lo i ny. Tm l i, ta c f (n) = (n 1)f (n 2) + (n 1)f (n 1), n T (1) v (2) suy ra v i m i n 2 ta lun c 3. (2)

f (n+1)g(n+1) = n[f (n)+f (n1)](n+1)f (n) = nf (n1)f (n) = g(n)f (n). M f (1) g(1) = 0 1 = 1 f (2) g(2) = 1, . B ng phng php quy n p ta ch ng minh c f (n) g(n) = (1)n |f (n) g(n)| = 1. V d P.13. Trn m t ph ng cho 2n + 1 ng th ng (n N ). Chng c t n(n + 1)(2n + 1) . 6

nhau t o thnh cc tam gic. Ch ng minh r ng s cc tam gic nh n t o thnh khng v t qu

L i gi i. G i s cc tam gic nh n t o thnh l f (n). Ta ph i ch ng minh f (n) n(n + 1)(2n + 1) , n N . 6

C th gi s trong 2n + 1 ng th ng cho khng c hai ng th ng no song song, khng c hai ng th ng no vung gc v i nhau v khng c ba ng th ng no ng quy. Th t v y, n u c hai ng th ng song song ho c vung gc th ta ch vi c

P-3. M t s v d p d ng

529

quay chng m t gc nh sao cho cc tam gic nh n v n l cc tam gic nh n, khi s cc tam gic nh n khng gi m. N u c ba ng th ng no ng quy th ta t nh ti n song song m t ng m t kho ng nh , s tam gic nh n cng khng gi m. Nh v y, ba ng th ng b t k trong s cc ng th ng cho lun c t nhau v t o thnh m t tam gic ho c nh n ho c t. G i g(n) l s cc tam gic t. Ta g i m t tam gic t o b i ba ng th ng a, b, c no l gi nh n c nh a n u cc gc chung c nh a c a tam gic l cc gc nh n. Ch n m t ng th ng l no v coi n l tr c honh. Cc ng th ng cn l i c chia thnh hai t p: T p T + g m cc ng th ng v i h s gc dng v t p T g m cc ng th ng v i h s gc m. Hai ng th ng t o v i l m t tam gic gi nh n n u m t ng thu c T + , ng kia thu c T . G i p l s ng th ng thu c T + , q l s ng th ng thu c T . Khi p + q = 2n v s tam gic gi nh n c nh l s l pq. Nh r ng p+q 2 pq = n2 . 2 Nhng l c th l ng th ng b t k trong s 2n + 1 ng th ng cho nn ta c s c p ( ng th ng l ; tam gic gi nh n c nh l) s khng qu n2 (2n + 1). Trong cch tnh trn m i tam gic nh n c tnh 3 l n (theo 3 c nh) cn m i tam gic t ch c tnh m t l n nn 3f (n) + g(n)
3 Th nhng t ng s cc tam gic l C2n+1 , t c l 3 f (n) + g(n) = C2n+1 =

n2 (2n + 1).

(2n + 1)(2n)(2n 1) 6

V y ta c f (n)

n(n + 1)(2n + 1) n2(2n + 1) (f (n) + g(n)) = 2 6

V d P.14. Cho (f (n)), (n N) l dy s t nhin khng gi m sao cho m i s t nhin u c th bi u di n m t cch duy nh t d i d ng f (i)+2f (j)+4f (k) trong i, j, k N, khng nh t thi t ph i khc nhau. Hy tnh f (2004).

530

Ph l c A

L i gi i. (y l l i gi i khc c a v d P.2). Ta c cc nh n xt sau

Nh n xt P.1. Dy (f (n)) tng ng t. Th t v y, n u f (j) = f (j + 1) th n = f (i) + 2f (j) + 4f (k) = f (i) + 2f (j + 1) + 4f (k) l i u v l v s n c hai cch bi u di n khc nhau.

Nh n xt P.2. C duy nh t m t dy (f (n)) tho mn yu c u bi. Th t v y, gi s c hai dy (f (n)), (g(n)) cng tho mn yu c u bi ra. G i n l ch s nh nh t m f (n) = g(n), t c l f (i) = g(i), i < n. Gi s f (n 1) < g(n) < f (n). Hi n nhin, f (0) = g(0) = 0. Do g(n) N nn g(n) = f (i) + 2f (j) + 4f (k), v i i, j, k < n. M g(n) < f (n) nn ta c f (i) = g(i), f(j) = g(j), f(k) = g(k) g(n) = g(i) + 2g(j) + 4g(k). Nhng g(n) = g(n) + 2g(0) + 4g(0) suy ra i u v l. V y dy (f (n)) tho mn yu c u bi ra l duy nh t. Ta xc nh m t s s h ng ban u. C f (0) = 0, f(1) = 1. V i m m = x0 + 2x1 + 4x2 , V im (xi {0; 1}) f (2) = 8, f(3) = 9. 7 c

63 c m = x0 + 2x1 + 22 x2 + 23 x3 + 24 x4 + 25 x5, (xi {0; 1}), suy ra m = (x0 + 8x3 ) + 2(x1 + 8x4 ) + 4(x2 + 8x5 ).

Nhng xi + 8xi+3 {0; 1; 8; 9} v 9 + 2.9 + 4.9 = 63 nn f (4) = 64, f(5) = 65. Ni chung, n u m = x0 + 2x1 + 22 x2 + 23 x3 + (xi {0; 1}) th

m = (x0 + 8x3 + 82 x6 + ) + 2(x1 + 8x4 + 42 x7 + ) + 4(x2 + 8x5 + 82 x8 + )

P-3. M t s v d p d ng

531

nn ta ch n f (n) d i d ng f (n) = y0 + 8y1 + 82 y2 + theo h c s 2 th f (n) = n0 + 8n1 + 82 n2 + .

(yi {0; 1}).

Do (f (n)) tng ng t nn n u n = n0 + 2n1 + 22 n2 + l cch bi u di n s n c bi t, do 2004 = 22 + 24 + 26 + 27 + 28 + 29 + 210 nn ta c f (2004) = 82 + 84 + 86 + 87 + 88 + 89 + 810. V d P.15. Cho f (x) Q[x], degf 2. Dy cc s (ai ) Q th a mn i u

ki n f (an+1 ) = an v i m i n N . Ch ng minh r ng t n t i k = l ak = al . L i gi i. Tr c h t ta ch ng minh t p {an } gi i n i. Th t v y do deg f nn t n t i x0 > 0 sao cho |f (x0)| > 2|x| v i m i x th a mn |x| > x0. Gi s t n t i n sao cho v i |an | > x0 ta c f (an ) > 2|an |. Th th an1 = f (an ) > 2|an | > 2x0 an2 = f (an1 ) > 2|an1 | > 22 |an | > 22 x0 2

a1 > 2n1 x0 n h u h n {an } b ch n. Ta ch ng minh s m u s c a cc phn s a1, a2, a3, . . . l h u h n. Th t v y, k hi u L l m u s chung nh nh t c a cc s h s trong a th c f (x). Ta c f (x) = 1 (b0xn + b1xn1 + + bn1 x + bn ), L (bi Z).

Khi v i m tu thu c N am = f (am+1 ) = 1 L


n n

bj anj m+1
j=0 j=0

bj anj = Lam . m+1

(1)

t t = b0b, b Z, sao cho t chia h t cho m u s c a am . Nhn hai v c a (1) v i bn1 bn ta c bn bn an + b1bn1 bn an1 + + bn bn1 bn = bn1 bn Lam Z. 0 m+1 0 0 m+1 0 0 Suy ra b0 bam+1 = tam+1 l nghi m c a m t phng trnh a th c v i cc h s nguyn v h s chnh b ng 1 nn tam+1 Z suy ra t chia h t cho m u s

532

Ph l c A

c a am+1 . V y t chia h t cho m u s c a t t c cc s an (do

trn ta ch n m

b t k, thu c N ). V y cc m u s c a cc s an ch c th l cc c s c a m t s t c nh. Do , s m u s c a cc phn s a1 , a2 , a3 , l h u h n. Tr l i bi ton. Do t p cc phn s {an } l b ch n v s cc m u s c a cc phn s an l h u h n nn cc s an ch nh n h u h n gi tr . Nhng c v s cc s an nn theo nguyn l Dirichlet, t n t i k = l ak = al. V d P.16. Cho f (x) = x3 6x2 + 9x. Hy xc nh s nghi m c a phng trnh f (f ( f (f (x)) )) = 0.
7 l n l y hm h p f

L i gi i. Nh n xt r ng phng trnh f (x) = 0 x = 0 x = 3. K hi u fk (x) = f (f ( f (f (x)) )) . Gi s


k l n l y hm f

phng trnh fk (x) = 0 c ak

nghi m ; phng trnh fk (x) = 3 c bk nghi m. V fk (x) = 0 fk1 (x) = 0 fk1 (x) = 3 nn ak = ak1 + bk1 . (1)

T b ng bi n thin c a hm s y = f (x) ta c f : [0; 4] [0; 4] v v i m i a [0; 4], phng trnh f (x) = a c ng ba nghi m phn bi t nn bk = 3bk1 m b1 = 3 bk = 3k . T (1) v do a1 = 2 (d th y), ta c ak = ak2 + bk2 + bk1 = ak3 + bk3 + bk2 + bk1 = = a1 + b1 + b2 + + bk1 = 2 + 3 + 32 + + 3k1 3k + 1 3k 3 37 + 1 = a7 = = 1094. . =2+ 31 2 2 V d 1 2an ; an+1 = v dy 2 1 + a2 n s (bn ) xc nh b i b0 = 4 ; bn+1 = b2 2bn + 2. n 2b0 b1b2 bn1 . Ch ng minh r ng v i m i n N ta lun c an = bn P.17. Cho dy s (an ) xc nh b i a0 =

P-3. M t s v d p d ng

533

L i gi i. Xt hm s f (x) =

1x . Ta c 1+x 1
2

f (an+1 ) =

1 an+1 1 + an+1

2an 1 an 1 + a2 n = = 2an 1 + an 1+ 2 1 + an 1

= f 2 (an ).

(1)

1 2 = 1 , ta c p d ng lin ti p (1) v i f (a0 ) = 1 3 1+ 2 f (an+1 ) = f 2 (an ) = f 2 (an1 ) = = f 2


n+1 2 n+1

(a0) =

1 3
n

2n+1

(2)

1 2n+1 32 1 32 1 1 an+1 an = 2n = an+1 = 2n+1 Suy ra 1 + an+1 3 3 +1 3 +1 L i xt hm s g(x) = x 1. C f (bn+1 ) = bn+1 1 = b2 2bn + 1 = (bn 1)2 = f 2 (bn ). V y n bn 1 = f (bn ) = f 2 (bn1 ) = f 2 (bn2 ) = = f 2 (b0 ) = 32 bn = 32 + 1. T c an bn = 32 1 = (32
n n1 2 n n n

+ 1) (32 + 1)(32 1) = bn1 bn2 b1.2.b0

Ch P.3. L p lu n m ta s d ng trong (2) c g i l quy. Phng php tm s h ng t ng qut c a dy s s d ng trong v d trn c g i l phng php hm l p. Ta th ng xc nh cc hm s f (x), h(x) sao cho f (un ) = h(f (un1 )). p d ng quy suy ra f (un ) = h(f (un1 )) = h(h(f (un2 ))) := h2 (f (un2 )) = = hn (f (u0)). T thu c bi u th c c a un . Hm s f (x) c g i l hm s ph , cn hm s h(x) c g i l hm l p. Trong v d trn, c hm l p l h(x) = x2 . V khi ni m hm l p, xem trong chuyn "Phng trnh hm". c hai tr ng h p ta u

534

Ph l c A

V d P.18. Cho k N v dy s (an ) th a mn i u ki n . . . . a0 = 0 ; a1 = 1 ; an+2 = 4an+1 an . Ch ng minh r ng an . 3k n . 3k . pn q n v i p := 2 + 3, q := 2 3, pq = 1. L i gi i. D th y an = 2 3 pn q n 3 pn q n p3n q 3n +3 C 3an (4a2 + 1) = 12 = = a3n , n. n 2 3 2 3 2 3 T a1 = 1, a2 = 4 v an+3 = 15an+1 4an suy ra n u n khng chia h t cho 3 th an cng khng chia h t cho 3. t n = 3s .t v i (t; 3) = 1. Theo trn, at khng chia h t cho 3. Ta c
s1

an = a3st =

3a3s1t (4a2s1 t 3

+ 1) = = 3 al
k=0

(4a2k t + 1) := 3s .at.M. 3

Do n N, s (4n2 + 1) khng chia h t cho 3 nn at .M khng chia h t cho 3. V y s m c a 3 trong khai tri n chnh t c c a n b ng s m c a 3 trong khai . . tri n chnh t c c a an . V y n . 3k n = 3s .t, (s k) an = 3s .L, (L . k . N) an . 3 . V d P.19. Ch ng minh r ng t n t i q R sao cho s 2
22 2q

, (n l n

nng ln ly th a) l s nguyn t v i m i n N . (Ta k hi u [x] l ph n nguyn c a s th c x.) L i gi i. G i l t p cc s nguyn t v k hi u 2n (q) := 2


22
2 q

(n l n nng

ln ly th a). Ta s xy d ng dy cc o n th t n = [an ; bn ] th a mn i u ki n 1 2 3 sao cho v i m i q n , t n t i pn pn = [2n (q)]. 3 . Khi [21(q)] = 2 v i m i q 1. V i n = 1, ch n 1 = 1; 2 Gi s c o n n v v i m i q n , pn = [2n (q)] . Ta c pn = [2n (q)] pn Nhng v i m 2n (q) < pn + 1 2pn 2n+1 (q) < 2.2pn .

2 th gi a m v 2m lun t n t i t nh t m t s nguyn t , ni

P-3. M t s v d p d ng

535

cch khc, t n t i pn+1 , sao cho 2pn

pn+1

2.2pn . Ta d ng o n

1 ))) . n+1 = log2(log2 ( (log2 (pn+1 )))); log2 (log2 ( (log2 pn+1 + 2
n l n l y hm h p n l n l y hm h p

C n+1 n v pn+1

1 [2n+1 (q)] = pn+1 , q 2 n+1 . V y ta xy d ng c dy cc o n th t (n ) th a mn yu c u t 2n+1 (q) < pn+1 +

ra. Do = n N. V d Zeta.

n = nn t n t i q . V i s q ta c [2n (q)] v i m i
n=1

P.20. Chu i

(z) := 1 +

1 1 1 + z + + z + c g i l hm z 2 3 n

S d ng hm Zeta hy ch ng minh 1 1 1 (2) = 1 + 2 + 2 + + 2 + := 2 3 n


+

n=0

2 1 = n2 6

(1)

L i gi i. 6 . Euler xu t pht t ba nh n xt sau: 1. N u a th c P (x) = a0 xn + a1xn1 + + an1 x + an cc nghi m x1 ; x2; ; xn R th P (x) = an 1 2. sin x = x x x1 1 x x 1 . x2 xn (1.1) (1.2) (a0 = 0, an = 0) c

x3 x5 x2n1 + + (1)n1 + . 3! 5! (2n 1)! sin x 3. Hm s c v s nghi m thu c R l xn = n, (n N). x M r ng cng th c (1.1)7 , Euler vi t x sin x = 1 x n n=1
6 7

x 1+ n

=
n=1

x2 n2 2

(1.3)

C a chnh L.Euler G n 100 nm sau, nh ton h c ng i c K. Weierstrass (1815-1897) m i ch ng minh c s m r ng ny l ng n!

536

Ph l c A

H s c a x2

v ph i c a (1.3) l 1 1 1 1 2 2 2 2 = 2 (2). 2 2 n 1 1 sin x l = x 3! 6


+

(1.4)

T (1.2), h s c a x2 trong khai tri n c a T v t (1.4) ta c

1 1 1 (2) = 1 + 2 + 2 + + 2 + := 2 3 n

n=0

2 1 = n2 6

Ch P.4. V hm Zeta, t gi a th k XIX, nh ton h c ng i c Georg Friedrich Bernhard Riemann (1826-1866) a ra gi thi t r ng m i nghi m 1 c a hm Zeta u l s ph c v c d ng zk = + iyk . Cho n nay, gi thi t 2 Riemann v n l bi ton m , cha gi i quy t c. B ng my tnh i n t , ng i ta tm c vi tri u nghi m c a hm Zeta v chng u c d ng nh gi thi t Riemann. Tuy nhin i v i Ton h c i u cha ph i l ch ng minh! V d P.21. Hy tnh xc su t m t phn s c l y b t k l phn s t i gi n. L i gi i.8 Ta ch n trong s cc phn s d ng m v i 1 m; n N , trong n N N l s cho tr c. C t t c N 2 phn s nh v y. G i f (N ) l s

cc phn s t i gi n trong s N 2 phn s . Khi , xc su t p c n tm l f (N ) . p = lim N + N 2 S phn s c t v m u c th rt g n cho 2 (t v m u cng ch n) l N2 1 1 N N = nn cc phn s nh v y s chi m = 2 ph n c a t ng s 2 2 4 4 2 1 cc phn s . Do , s phn s m khng t i gi n c cho 2 s chi m 1 2 2 ph n. Tng t , l ng phn s khng t i gi n c cho 3, 5, 7 s chi m tng ng
8

C a nh ton h c ng i Nga P.L.Chebyshev (1821-1894).

538

Ph l c B

1 1 1 ; 1 2 ; 1 2 ph n. G i q l c s nguyn t l n nh t 2 3 5 7 c a N th s phn s khng t i gi n trong s N 2 phn s ni trn l l 1 N2 1 1 22 1 1 1 1 2 . 32 q

Hi n nhin, khi N + th q +. Ta c p = lim 1 f (N ) = lim 1 2 q+ N + N 2 2 1 1 1 1 2 32 q =


q

1 . q2

tnh p ta xt

1 1 v p d ng khai tri n c a , c p 1x 1 = p 1 1 1 1 1 2 1 2 2 3
+

1 1 1 2 q

=
q j=0

1 . q 2j

L p lu n tng t nh

v d P.5 v s d ng k t qu c a v d P.20 ta c

1 1 6 1 1 2 = 1 + 2 + 2 + + 2 + = p = 2 0, 6079. p 2 3 n 6

Ph l c B

H ng l c h i quy v h ng l c tu n hon
Q-1 Ma tr n ly linh

Ma tr n ly linh v ma tr n tu n hon l cc v n c c p n. Trong chng ny, chng ta s nghin c u, khai thc m t ng d ng c a chng; ch ng h n nh n u ma tr n c ng ng trong cc h sinh h c l ma tr n lu linh hay tu n hon th dng i u c a h khi th i gian ra v cng s d dng nh n c nh tnh ch t c bi t c a cc ma tr n ny. M t khc, s d ng khai tri n Jordan chng ta c th tm c cng th c nghi m t ng minh v m t m t php ch ng minh m i v tnh n nh nghi m c a h ng l c (c r i r c v lin t c). nh ngha Q.5. Ma tr n vung A c g i l ma tr n ly linh n u t n t i s nguyn dng p sao cho Ap = 0 ( y 0 l ma tr n khng). a th c c trng c a ma tr n c nh ngha b i A () = det(I A). nh l Q.9. Cho A l m t ma tr n vung c n n trn tr ng b t k. Th th, A l ma tr n ly linh n u v ch n u A () = n .
539

540

Ph l c B

Ch ng minh. N u a th c c trng c a ma tr n A c d ng n th p d ng nh l Caley - Hamilton ta c An = 0. V y A l ma tr n lu linh. ch ng minh ph n o l i c a nh l ta nh n xt r ng v i tr ng k b t k lun t n t i tr ng K l m r ng c a tr ng k sao cho trong K m i a th c v i h s trong k c nghi m, t c K l tr ng ng i s . V th , khng m t tnh t ng qut, ta gi s tr ng cho l tr ng ng i s . K hi u l m t gi tr ring c a ma tr n lu linh A ng v i vc t ring v c a A. Khi Av = v. Theo gi thi t A l ma tr n ly linh nn t n t i s nguyn dng p > 1 sao cho Ap = 0. Do Apv = p v = 0. Nhng vc t ring v khng th b ng 0 nn p = 0. Suy ra = 0. V y a th c c trng c a A ph i c d ng n . nh l c ch ng minh. Nh n xt Q.3. Nh n xt r ng, n u k l tr ng s th c R ho c tr ng s ph c C th ta c php ch ng minh khc. Th t v y, v k l khng gian Banach nn theo nh l c a Gelfand, bn knh ph (A) = sup{| |: (A)} = lim || An || n .
n
1

M A l ma tr n ly linh nn t n t i s nguyn dng p > 1 sao cho Ap = 0. Do v y (A) = 0 nn = 0. V y a th c c trng c a A ph i c d ng n . K t h p nh l ny v i nh l Caley - Hamilton ta c H qu Q.2. N u A l m t ma tr n ly linh c (n n), th ta c An = 0. Nh n xt Q.4. H qu ny ni r ng n u ta c n ki m tra tnh lu linh c a m t ma tr n n n th ch c n tnh n lu th a th n c a n l . N u t i lu th a n m v n cha nh n c ma tr n 0 th ma tr n ch c ch n khng th l ma tr n lu linh c. Hn n a ta c n ch r ng t ng cng nh tch c a hai ma tr n lu linh khng nh t thi t ph i l lu linh. Th t v y xt hai ma tr n lu linh (2 2) sau y A= 0 1 0 0 v B= 0 0 1 0 .

Q-1. Ma tr n ly linh

541

Ta c A2 = B 2 = 0, do A v B l cc ma tr n ly linh. Nhng c t ng A+B = 0 1 1 0 v tch AB = 1 0 0 0

khng l ma tr n lu linh v (A + B)2 = I (ma tr n n v ) v (AB)2 = AB. (Cng c th tnh tr c ti p c a th c c trng c a A + B l 2 1 v a th c c trng c a AB l 2 nn chng khng th l lu linh). M t khc nh n th y r ng n u hai ma tr n lu linh A v B l t a giao hon v i nhau (AB = BA) th r rng c t ng v tch c a chng l lu linh. o l i ta c hai m nh quan tr ng sau y: M nh Q.1. N u A, B v A + B l cc ma tr n ly linh c (2 2) th ta c AB = BA. T , AB v BA l cc ma tr n ly linh. Ch ng minh. Theo nh l Q.9 ta c A2 = B 2 = (A + B)2 = 0. V v y, AB + BA = 0, do AB = BA. T suy ra, (AB)2 = ABAB = AABB = 0, do AB l ma tr n ly linh. Tng t ta thu c BA l ma tr n ly linh. M nh c ch ng minh M nh Q.2. N u A, B v AB, BA l cc ma tr n ly linh c (2 2) th A + B l ma tr n ly linh v ta cng thu c AB = BA. Ch ng minh. Ta c (A + B)2 = A2 + AB + BA + B 2 = AB + BA. T , (A + B)4 = (AB + BA)2 = (AB)2 + ABBA + BAAB + (BA)2 = 0. i u ny ch ng t A + B l ma tr n ly linh v nh m nh trn ta thu c AB = BA. M nh c ch ng minh. Nh n xt Q.5. i v i nh ng ma tr n lu linh c l n hn 2 2 th m nh Q.1 v Q.2 khng cn ng. Ta l y cc v d nh sau:

542

Ph l c B

0 2 2 tra c A, B, A + B l cc V d Q.22. V i A =

0 0 2 ma

0 0 v B = 0 tr n lu linh nhng ma 0 0 0 0 4 2 0 4 4

0 2 1 0 0 1 . D ki m 0 0 0 tr n

AB = khng l ma tr n lu linh v (AB)3 = 0 1 V d Q.23. V i A = 0 0 0 0 c AB, BA, A, B l cc ma

0 0 0 0 32 16 0 32 32

= 0. 0 0 1 tr 0 0 0 0 . D ki m tra 0 0 n

0 1 v B= 0 tr n lu linh nhng ma 0 1 0 0 0 1 1 0 0

A+B = khng l ma tr n lu linh v

(A + B)2 = I. Nh n xt Q.6. Ma tr n lu linh xu t hi n trong l thuy t h ng l c nh m t h ng l c h i quy n gi n nh t. N u xu t pht t m t vc t b t k trong khng gian n chi u th h th ng lun quay v g c to sau khng qu n b c. Ti p theo ta c p n m t s khi ni m v tnh ch t c a ma tr n tu n hon.

Q-2

Ma tr n tu n hon

nh ngha Q.6. Ma tr n vung U c g i l ma tr n tu n hon n u t n t i s nguyn dng k > 1 sao cho U k = I ( y I l ma tr n n v ). Ma tr n tu n hon l v d n gi n cho h ng l c tu n hon. Sau p b c h th ng c a ta tr v tr ng thi ban u. y cng l chu k c a h ng

Q-2. Ma tr n tu n hon

543

l c. Cc ma tr n tu n hon u l cc ma tr n n a n (xem [?], p.613)). Nh c l i r ng ma tr n n a n l cc ma tr n c h vc t ring y (t c l chng ng d ng v i ma tr n cho). Ch ng h n xt ma tr n U = cos(2/p) sin(2/p) sin(2/p) cos(2/p)

ta c ngay U p = I nn U l ma tr n tu n hon. y l m t php quay quanh g c to v i gc


2 . p

R rng l sau p b c ta quay v v tr ban u. M t

l p cc v d h p d n khc l cc ma tr n hon v . Nh ng ma tr n ny dng bi u di n cc nhm i x ng. c th hn nh ng v n ny ta k hi u V l khng gian vc t n chi u trn tr ng s ph c v i c s l {v1 , v2 , , vn }. K hi u Sn l nhm i x ng v i cc ph n t l cc hon v c a t p h p

{1, 2, , n}. Tng ng v i m i hon v ta thnh l p nh x tuy n tnh P nh sau P vj = v(j) v i j = 1, 2, , n. Ma tr n c a nh x tuy n tnh P trong c s {v1 , v2 , , vn } cng c k hi u l P v c tn l ma tr n hon v . V m t tr c quan, cc ma tr n hon v l cc b ng s vung m trong m i dng m i c t c ng m t s 1, cc s cn l i u b ng 0. Ch ng h n P(1,2) = 0 1 0 1 0 0 0 0 1 v P(1,3,2) = 0 1 0 0 0 1 . 1 0 0

v i (1, 2) l hon v i ch 1 v i 2, cn (1, 3, 2) l vng xch a 1 vo 3, 3 vo


2 3 2 cn 2 th tr v . Ta c P(1,2) = P(1,3,2) = I v a th c c trng c a P(1,2) l

P(1,2) = (2 1)(1) cn a th c c trng c a P(1,3,2) l P(1,3,2) = (3 1). By gi ta s nghin c u chi ti t a th c c trng c a cc ma tr n tu n hon. B Q.1. N u U p = U q = I, trong p v q l cc s nguyn t cng nhau th U = I. Ch ng minh. V p v q l hai s nguyn t cng nhau, nn t n t i cc s

544

Ph l c B

t nhin m v n sao cho pm = nq + 1. Do , U mp = U nq+1 . Theo gi thi t, ta c v tri l I v v ph i l U . B c ch ng minh. nh l Q.10. Cho A l m t ma tr n c n n trn tr ng s h u t Q v i cc gi tr ring khc nhau trn tr ng s ph c C. Gi thi t thm l t n t i s nguyn t p sao cho Ap = I. Th th a th c c trng c a A ph i c d ng p 1 ho c p1 + p2 + + 1. Suy ra p = n ho c p = n + 1. Ch ng minh. Cho l m t gi tr ring c a A. Th th p = 1. M t khc cc gi tr ring c a A u phn bi t nn a th c c trng c a A ph i l th a s c a a th c p 1. Khi phn tch a th c p 1 ra th a s nguyn t trn tr ng s h u t Q ta c p 1 = ( 1)(p1 + p2 + + 1). V y a th c c trng c a A ch c th l m t trong hai d ng trn. Ta ch ng minh xong. Nh n xt Q.7. i u ki n cc gi tr ring c a A ph i phn bi t l v cng quan tr ng khng th b c. V d ma tr n n v I tho mn t t c cc i u ki n khc c a nh l ny m khng c a th c c trng nh hai d ng trn. H qu Q.3. N u p l s nguyn t l n hn 2 th t nh t m t trong hai s {cos(2/p), sin(2/p)} ph i l s v t . Ch ng minh. Ta xt ma tr n sau U= cos(2/p) sin(2/p) sin(2/p) cos(2/p) .

Khi U p = I. N u c hai s {cos(2/p), sin(2/p)} l s h u t , ta s d ng nh l 1.2 v nh n c p = 2 ho c 3. Theo gi thi t c a ta p l s nguyn

Q-2. Ma tr n tu n hon

545

t l n hn 2. V i p = 3 ta c ngay sin(2/3) = ch ng minh xong.

3/2 l s v t . H qu c

By gi ta xt a th c c trng c a cc ma tr n hon v . Tr c h t nh n xt r ng vc t v = v 1 + v2 + + vn l vc t ring c a m i ma tr n hon v ng v i gi tr ring b ng 1. M t khc n u hon v khng thay i v tr c a j th vj s l vc t ring c a ma tr n P ng v i gi tr ring b ng 1. Nh v y n u c nh k i m th a th c c trng c a P s chia h t cho ( 1)k . C th hn ta c B Q.2. Gi s Sn l m t vng xch di p. Th th a th c c

trng () c a ma tr n hon v P l (p 1)( 1)np . Ch ng minh. Ta s li t k t t c cc vc t ring ( c l p tuy n tnh) c a ma tr n P . Khng m t tnh t ng qut, gi s l vng xch (1, 2, , p). Khi {vp+1 , , vn } l n p vc t ring c l p tuy n tnh tng ng v i gi tr ring 1. By gi t = cos(2/p) + i sin(2/p) l cn b c p c a 1 v uj = jp v1 + j(p1)v 2 + + j vp v i j = 1, 2, , p. Khi P uj = jpv 2 + j(p1) v3 + + j v1 = j uj . B i nh l quen thu c c a VanderMonde ta c h vc t ring j j = 1, 2, , p l c l p tuy n tnh. Suy ra a th c c trng c a ma tr n hon v P l ( )( 2) ( p )( 1)np = (p 1)()np . B c ch ng minh. nh l Q.11. Gi s Sn c bi u di n d i d ng tch c a k vng xch r i nhau 1, 2, , k . Gi s pi l di c a i (v i i = 1, 2, , k) v

546

Ph l c B

q = n (p1 + p2 + + pk ). Th th a th c c trng () c a ma tr n hon v P l (p1 1)(p2 1) (pk 1)( 1)q . Ch ng minh. B ng quy n p theo k v s d ng b 1.2, ta c nh l ng v i k = 2. Khng m t tnh t ng qut, ta gi s 1 l vng xch (1, 2, , p1 ) v gi s 2 l vng xch (p1 + 1, p1 + 2, , p1 + p2 ). Tr c h t ta c n p1 p2 = q vc t ring c l p tuy n tnh c a P tng ng v i gi tr ring 1 l

vp1 +p2 +1 , , vn . By gi gi s 1 = cos(2/p1 ) + i sin(2/p1 ) l nghi m ph c th p1 c a 1 ( p1 = 1) v gi s 2 = cos(2/p2 ) + i sin(2/p2 ) l nghi m ph c th p2 c a 1 ( p2 = 1). Nh ta c th vi t cc vc t ring c a P tng
j ng v i 1 v 2 trong

= 1, 2, , p1 v j = 1, 2, , p2 . V th a th c

c trng () c a P c d ng () = (p1 1)(p2 1)( 1)np1 p2 . nh l c ch ng minh. Ti p theo ta ta nghin c u khng gian vc t tuy n tnh nh chu n k chi u V trn tr ng s ph c C. Ma tr n ly ng l ma tr n vung U c (k k) sao cho U I l ma tr n ly linh. R rng, a th c c trng c a ma tr n ly ng U l ( 1)k . V v y, bn knh ph c a ma tr n ly ng l 1. r ng, chu n c a cc ma tr n ly linh c th r t l n, m c d bn knh ph c a chng l 0. Ta nh ngha eAt = I + tn t A + + An + . 1! n!

R rng, chu i ny h i t v i chu n c a ma tr n. T nh ngha, d th y (eAt ) = AeAt , N u A v B l cc ma tr n vung giao hon th e(A+B)t = eAt eBt,

Q-2. Ma tr n tu n hon

547

N u l m t gi tr ring c a A th et l gi tr ring c a eAt , N u U 2 = I th eU t = I cosh t + U sinh t. N u N l ma tr n ly linh c (k k) th eN t = I + l ma tr n ly ng. Ta hy xt cc v d sau, N u U= 0 1 1 0 th eU t = cosh t sinh t sinh t cosh t cos t sin t sin t cos t (U 2 = I); tk1 t N + + N k1 1! (k 1)!

N u

U=

0 1 1 0

th

eU t =

(U 2 = I).

Nh c l i r ng, m i ma tr n A u c th bi u di n duy nh t d i d ng A = S + N , trong S l ma tr n n a n, N l ma tr n ly linh v SN = NS (khai tri n Jordan c ng tnh). Ta c nh l quen thu c sau m php ch ng minh n c th th y d dng nh s d ng khai tri n ny. nh l Q.12. Nghi m c a h u(t) = Au(t) v i t > 0 c d ng u(t) = eAt u(0). Hn h a, n u A = S + N l khai tri n Jordan c ng tnh c a A, trong S l ma tr n n a n c k k c k vc t ring c l p tuy n tnh v 1, v2 , , vk tng ng v i cc gi tr ring 1 , 2 , , k (khng nh t thi t khc nhau), th nghi m t ng qut c a h c d ng u(t) = 1e1 t eN tv1 + 2 e2t eN tv2 + + k ek t eN tvk N u Re (j ) < 0 v i t t c j = 1, 2, , k, th lim u(t) = 0. (4.2) v i t 0. (4.1)

548

Ph l c B

Ch ng minh. T SN = NS ta c e(S+N )t = eSt eN t t ta c (4.1). Ta ch ng minh(4.2). r ng l n c a || eN tvj || l gi tr c a a th c (t bi n s ) b c k 1 khng i. Do , lim | ej t | || eN tvj ||= 0,

v suy ra (4.2). Ch r ng nghi m c a h x(t) = y(t) y(t) = 0 c d ng x(t) = a + bt y(t) = b Ma tr n A c a h ny l ma tr n ly linh v ch c m t vc t ring ( c l p tuy n tnh). r ng eAt = I + tA = 1 t 0 1

v ta lun c nghi m c a h u(t) = Au(t) v i t > 0 l u(t) = eAtu(0). Ti p theo, ta ch ng minh tnh n nh nghi m c a h ng l c r i r c b ng cch dng khai tri n Jordan nhn tnh. Nh c l i r ng t t c cc ma tr n kh ngh ch A u c th bi u di n (duy nh t) d i d ng tch (giao hon c) c a m t ma tr n n a n S v m t ma tr n ly ng U (khai tri n Jordan nhn tnh). Gi tr ring c a S chnh l gi tr ring c a A. Ta c nh l quen thu c sau m php ch ng minh n c th th y d dng nh s d ng khai tri n ny. nh l Q.13. Nghi m c a h un+1 = Aun v i n = 0, 1, , c d ng un = An u0 . Hn n a, n u A = SU l khai tri n Jordan nhn tnh c a A (A c gi thi t l ma tr n kh ngh ch), trong S l ma tr n n a n c (k k) c

Q-2. Ma tr n tu n hon

549

k vc t ring c l p tuy n tnh v 1 , v2, , vk tng ng v i cc gi tr ring 1 , 2 , , k (khng nh t thi t khc nhau), th nghi m t ng qut c a h c d ng un = 1 n U n v1 + 2 n U n v2 + + k n U n vk v i n = 0, 1, 1 2 k N u | j |< 1 v i t t c j = 1, 2, , k th
n

(4.3)

lim un = 0.

(4.4)

Ch ng minh. Do v1 , v2 , , v k l cc vc t ring c l p tuy n tnh trong khng gian k chi u, nn chng t o thnh c s c a khng gian ny. V v y, v i vc t u0 cho tr c, ta c u0 = 1 v1 + 2v 2 + + k vk . Thay vo cng th c un = An u0 ta c ngay un = An u0 = U n S n (1 1 + 2 2 v 2 + + k k vk ) = 1 n U n v 1 + 2 n U n v 2 + + k n U n v k . 1 2 k N u gi tr tuy t i c a nh hn th | n |= (1 + a)1 ti n t i 0 r t nhanh khi n ti n t i v cng. Cn chu n c a ma tr n lu ng
k1

U = (I + N ) =
r=0

n(n 1) (n r + 1) r N r!

s b ch n trn b i a th c
k1

p(n) =
r=0

|| N ||r n(n) (n r + 1) r!

c b c l k 1 (c nh) c a n. Hm s m c l n r t nhi u so v i hm a th c. Ni m t cch c th hn n u p(n) l a th c cn a l m t s dng th | p(n) | = 0. n (1 + a)n lim

550

Ph l c B

V v y ta c limn un = 0. nh l c ch ng minh trong tr ng h p A l ma tr n kh ngh ch. N u A khng kh ngh ch ta c th phn tch khng gian vc t cho thnh t ng tr c ti p c a hai khng gian con b t bi n l V1 v V2 = {v : Av = 0}. H n ch c a nh x tuy n tnh A trn khng gian con V1 l kh ngh ch nn ta c th p d ng k t qu v a ch ng minh lu n limn un = 0. r ng ma tr n ly linh A= 0 1 0 0 . trn k t

c duy nh t m t vc t ring ( c l p tuy n tnh) v1 = 1 0

nn nghi m t ng qut c a h un+1 = Aun v i n = 0, 1, 2, , khng c d ng (4.3) trong nh l trn. Hn n a, u0 v u1 khng nh t thi t l 0 v un = 0 v i t t c n > 1.

Ti li u tham kh o
[1] R. Courant, G.Robins, Ton h c l g? (ti ng Nga), NXB Matxcva, 1967 . [2] A.V. Dorofeeva, Gio trnh Ton cao c p cho khoa Tri t h c i h c (ti ng Nga), NXB MGU, Matxcva, 1971. [3] Titu Andreescu, Dorin Andrica, Birkhuser, 2006. a [4] I.M. Yaglom, 1963. [5] S.I. Xvarcburd, Izbrannye voproksy matematiki Fakultativnyi kurs 10, Moskva, 1963. [6] G.Polya, G.Szege, Cc nh l v bi t p c a gi i tch, (ti ng Nga), Nh xu t b n Mir, Moscow, 1973 [7] D.Shkliarsky, N.Chentsov, I.Iaglom, The USSR Olympiad Problem book, Dover Publications, New York, 1994. [8] Martin Aigner, Gunter M. Ziegler, Proofs from the Book, Third Edition, Springer, 2003 [9] Arthur Engel, Problems solving strategies, Springer 1998
551

cc tr ng

Complexnumbers from A to Z . . .,

S ph c v ng d ng trong hnh h c (ti ng Nga), Moskva,

552

TI LI U THAM KH O

[10] Alexeev A., nh l Abel qua cc bi ton v l i gi i (ti ng Nga), Nh xu t b n MCCME, 2001 [11] P.S.Modenov, Problems in Geometry, Mir publishers 1981 [12] H. Dorrie, 100 Great Problems of Elementary Mathematics, Their history and solutions, Dover Publications, Newyork 1965 [13] B.J.Venkatachala, Functional Equations, A problem Solving Approach, Prism Books, 2002 [14] Zvezdelina Stankova, Complex numbers in Geometry, Berkeley Mathematical circle. [15] Ross Honsberger, Mathematical Gems III, MAA Publications 1985 [16] Nguy n C nh Ton, Hnh h c cao c p, Nh xu t b n Gio d c, 1979. [17] L H i Chu, Thi v ch ton Qu c t , Nh xu t b n tr , 2001.

[18] on Qunh, S ph c v i Hnh h c ph ng, Nh xu t b n Gio d c, 1998. [19] Titu Andresscu, Complex Numbers from A to Z, Birkhauser, 2000. [20] Nguy n Vn M u (ch bin), M t s chuyn s h c ch n l c, Nh xu t b n Gio d c, 2008 [21] Nguy n Vn M u (ch bin), Chuyn hnh h c v cc v n lin quan, Nh xu t b n Gio d c, 2008 [22] Nguy n Vn M u (Ch bin), Tr nh o Chi n, Tr n Nam Dng, Nguy n ng Ph t, M t s chuyn ch n l c v a th c v p d ng. Nh xu t b n Gio d c, 2008. [23] Nguy n Vn M u, a th c i s v phn th c h u t , Nh xu t b n Gio d c, 2007 (ti b n l n th hai).

TI LI U THAM KH O

553

[24] i s 10, Nh xu t b n Gio d c, 1969. [25] Gi i tch 12, Nh xu t b n Gio d c, 2009. [26] Bl. Sendov, A. Andreev and Kjurkchiev, Numerical Solution of Polynomial Equations (Part 2, Vol. VIII trong b sch Handbook of Numerical Analysis, Eds., P. G. Ciarlet and Lions), Nh xu t b n Elsevier Science, 1994. [27] Ch bin: P. C. Aleksandrov, A. I. Markusevich, A. Ia. Khinchin, T i n ton s c p, Vi n Hn lm khoa h c gio d c Lin bang Nga, Nh xu t b n sch k thu t - l thuy t, Moskva, 1951 (ti ng Nga), trang 356 - 379. [28] Nguy n H u i n, a th c v ng d ng. Nh xu t b n Gio d c, 2005. [29] Ng Vi t Trung, L thuy t Galoa, Nh xu t b n i h c Qu c gia H N i, 2005. [30] T Duy Ph ng, Phng trnh b c ba v cc h th c trong tam gic, Nh xu t b n Gio d c, 2004. [31] Eric W. Weisstein, CRC Consise Encyclopedia of Mathemtics, CRC Press LLC, USA, 1999. [32] A. Sveshnikov, A. Tikhonov, The Theory of Function of a complex variable, Mir Publishers, 1973. [33] Nguy n Th y Thanh, C s l thuy t hm bi n ph c, NXB HQGHN, 2007. [34] Nguy n Th y Thanh, Bi t p ton cao c p, NXB HQGHN, 2007.

You might also like